Download NEET PG 2012 Question Paper with Answers

Download NEET PG (The National Eligibility cum Entrance Test Postgraduate) 2012 Question Paper with Answers

1.Whichmuscleisinsertedintothefloorof
theintertubercularsulcusofthehumerus?
a)Latissimusdorsi
b)Teresmajor
c)Pectoralismajor
d)Deltoid
CorrectAnswer-A
Ans.is'a'i.e.,Latissimusdorsi
Theshaftofhumerusiscylindricalintheupperhalfandtriangularon
cross-sectioninthelowerhalf.Theupperparthasintertubercular
sulcus(bicipitalgroove)anteriorly.Bicipitalgroovecontainslong
headofbicepswithitssynovialsheathandanascendingbranchof
anteriorcircumflexhumeralartery.Middlethirdofposteriorsurface
ofshaftofthehumerushasaspiralgroove(radialgroove)which
containsradialnerveandprofundabrachivessels.
Attachmenttoshaftsare?
*Insertionsofpectoralismajor(onlaterallipofbicipitalgroove),
latissimusdorsi(onfloorofbicipitalgroove),teresmajor(ondeltoid
tuberosity)andcoracobrachialis(mediallyonmidshaft).
*Originofbrachialis(anteriorsurface);Lateralandmedialheadof
triceps(posteriorsurface);pronaterteres(frommedial
supracondylarridge);brachioradialis(fromlateralsupracondylar
ridge)andECRL(fromlateralsupracondylarridge).

2.Atwhatleveldoesthetracheabifurcates
?
a)UpperborderofT4
b)LowerborderofT4
c)27.5cmfromtheincisors
d)LowerborderofT5
CorrectAnswer-B
Ans.is'b'i.e.,LowerborderofT4[RefBDCSth/eVolume1p.
267]
Tracheabifurcatesatcarina,attheleveloflowerborderofT,orT4-
T5discspace.

3.Cricoidcartilageliesatwhichvertebral
level?
a)C3
b)C6
c)T1
d)T4
CorrectAnswer-B
Ans.is'b'i.e.,C6[RefBDC5I'VeVol.III,p.237]

4.Whichofthefollowingistrueabout
vertebraldevelopment-
a)Thenotochordformstheannulusfibrosus
b)Thesclerotomeformsthenucleuspulposus
c)Thesclerotomesurroundsthenotochordonly
d)Thesclerotomesurroundsthenotochordandtheneuraltube
CorrectAnswer-D
Ans.is'd'i.e.,Thesclerotomesurroundsthenotochordandthe
neuraltube
[RefLangman'sembryology10th/ep.140]
Developmentofvertebralcolumn
Thehumannervoussystemdevelopsfromneuroectoderm.
Duringdevelopment,behindtheneuroectodermliesthemesoderm
(paraxialmesoderm)thatenclosesthenotochord(aderivativeof
endoderm).
Thisparaxialmesodermgiverisetosomites.
Somitesfurtherdifferentratedinto:
Dermatomyotome:-Giverisetoskeletalmusclesanddermis.
Sclerotomes:-Giverisetovertebralcolumn.
Sclerotorneswhichsurroundnotochordstartsprojectingposteriorly
(dorsally)tosurroundneuraltubeandforms.
Ventralsclerotomes:-Giverisetovertebralbodyandannulus
fibrosus,Lateralsclerotomes:-Giverisetovertebralarch(pedicle
andlamina).
Dorsalsclerotomes:-Giverisetospinousprocess.Thenotochord
formsthenucleuspulposus.

5.Whichofthefollowingisnotacongenital
anomaly?
a)Amastia
b)Polymastia
c)Polythelia
d)Mastitis
CorrectAnswer-D
Ans.is'd'i.e.,Mastitis[RefHumanembryologybyInderbir
Singh8th/ep.103]
Developmentalanomaliesofthemammaryglands:

1. Amastia:Theglandmaybeabsentononeorbothsides.
2. Athelia:Absenceofnipple
3. Polytheliaandpolymastia:Supernumerarybreastsandnipples
alongthemilkline.
4. Invertedorcraternipple
5. Microormacromastia

6.Rectumdevelopsfrom
a)Cloaca
b)Hindgut
c)Allantoicremnants
d)Urogeitalsinus
CorrectAnswer-A:B
Ans.is`b>a'i.e.,Hindgut>Cloaca[Ref:InderbirSinghhuman
embryologySth/ep.149]
Postallantoicpartofhindgutisthedilatedendodermalcloaca,which
isseparatedfromthesurfacebycloacalmembrane.
Urorectalseptumdividesendodermalcloacaeinto
Anteriorpart,knownasprimitiveurogenitalsinus,whichdevelops
intourinarybladderandurethra.
Posteriorpart,knownasprimitiverectum,whichgivesrisetolower
partofrectumandupperpartofanalcanal.

7.Skeletalderivativeof2?'pharyngealarch-
a)Malleus
b)Incus
c)Stapes
d)Maxilla
CorrectAnswer-C
Ans.is`c'i.e.,Stapes

8.Allarederivedfromectodermexcept-
a)Hypophysis
b)Retina
c)Spinalcord
d)Adrenalcortex
CorrectAnswer-D
Ans.is'd'i.e.,Adrenalcortex[RefInderbirSinghHuman
EmbryologySth/ep.300]
Thecellsoftheadrenalcortexarisefromthecoelomicepithelium
(mesoderm).
Thecellsofthemedullaarederivedfromtheneuralcrestcells
(ectoderm).
Theanteriorpituitaryisderivedfromthesurfaceectodermliningthe
oralcavityasitisanoutgrowthfromtheRathke'spouch.
Theposteriorpituitaryisactuallyacontinuationofthehypothalamus
andsoitisderivedfromtheneuralectoderm.?
Thespinalcordandbrainarederivedfromtheneuralectoderm.
Theretinaisalsoderivedfromtheneuralectoderm.

9.Developmentoflabiamajoraisfrom-
a)Urogenitalsinus
b)Mullerianduct
c)Genitalridge
d)Genitalswelling
CorrectAnswer-D
Ans.is'd'i.e.,Genitalswelling[Ref:InderbirSinghHuman
EmbryologySth/ep.256]
Embryogical
Fatein
Fateinmale
structure
female
Genitalswelling Labiamajora
Scrotum
Genitalfold
Labiaminora Ventralaspectof
penis,penile
urethra
Genitaltubercle Clitoris
Glanspenis

10.Blastocystmakescontactwith
endometriumon?
a)<3days
b)5-7days
c)8-11days
d)15-16days
CorrectAnswer-B
Ans.is'b'i.e.,5-7days[Ref:TextbookofHumanembryology-
286]
Contactofblastocystwithendometriumoccursatthetimeof
implantation.
Implantationoccursataround6-7days.

11.Haploidnumberofchromosomesisseen
in?
a)Spermatogonia
b)Primaryspermatocytes
c)Secondaryspermatocyte
d)None
CorrectAnswer-C
Ans.is'c'i.e.,Secondaryspermatocyte[RefGanong23'd%p.
403,404]
Diploidmeansthatcellcontains46chromosomes(diploidof23)and
haploidmeansthatcellcontains23chromosomes.
Developmentofspermisasfollows:-
Spermatogonia(Diploid-46)Primaryspermatocyte(Diploid-46)
Primaryspermatocyte(Diploid-46)isof2types-
1. SecondaryspermatocyteSpermatid(Haploid-23)
2. Secondaryspermatocyte(Haploid-23)Spermatid(Haploid-23)
Similarlyinoogenesis,haploidnumberisseeninsecondaryoocyts.

12.Spermchromosomefasteris-
a)Xchromosome
b)Ychromosome
c)Bothsame
d)None
CorrectAnswer-B
Ans.is'b'i.e.,Ychromosome[RefPricipleofmedicalphysiology-
51]
ThehumanYchromosomeissmallerthanXchromosome.
Hence,thespermscontainingYchromosomesarelighterandswim
fasterupthefemalegenitaltract,reachingtheovumearlierthanthe
Ybearingsperms.
Thisprobablycontributestothefactthattheglobalmalebirthrateis
slightlyhigherthenthefemalebirthrate.

13.Allarederivedfromectodermexcept?
a)Lens
b)Eustachiantube
c)Brain
d)Retina
CorrectAnswer-B
Ans.is'b'i.e.,Eustachiantube[RefHUMANEMBRYOLOGY
editedbyKrishnaGarg,2"dlep.56,133]
Eustachiantubeoriginatesfromthefirstpharyngealpouchi.e.
Endoderm.
Endoderm
Ectoderm
Mesoderm
Epitheliumof
Brain
LN&Spleen
wholeg.i.t.
NeuralCrest
Mesenchyme
Resp.tract
AdrenalMedulla
Mesothelium
(Eustachian
Pharyngealclefts
Pharyngealaches
tube)
Oligodendrocytes
CVS,blood,
Pharyngeal
Lens(fromsurfaceE-)
BMDuramater
pouches
Irismuscles(Sphincter& TrigoneofUB
Liver&GB
dilatorpupillae)
Monocytedery(Ex-
Urethra
Epitheliumofcornea,
Microglia)
UB
conjunctivaouter
Cillarybody&iris
Lowerpartof
Lids
stroma(except
vagins
Renalpigmentepithelium epithelium)
Ducts&acimiof Sensoryretina
Sclera,choroid,
pancreas
MembranousLabyrinth
vitreous
Mostendocrinal
Stromaofcornea
glands(except
Lids(Muscles)Adrenal
adr.medulla&
CortexBonyorbit
pituitarywhich

CortexBonyorbit
pituitarywhich
areectoderm)

14.Whichofthefollowingisderivedfrom
endoderm?
a)Gallbladder
b)Lens
c)Spleen
d)Lymphnodes
CorrectAnswer-A
Ans.is'a'i.e.,Gallbladder[RefEmbryologyGarg2"/evarious
pages]

15.Developmentofperitonealcavityisfrom?
a)Mesenchyme
b)Intraembryoniccoelom
c)Ectoderm
d)Endoderm
CorrectAnswer-B
Ans.is'b'i.e.,Intraembyoniccoelom[RefEmbryologybyIndu
Khuran2"dlep.96]
Allbodycavitiesdevelopfromintraembryoniccoelum.These
cavitiesareperitonealcavity,pleuralcavityandpericardialcavity.
Theperitoneumdevelopsultimatelyfromthemesodermofthe
trilaminarembryo.
Lateralplatemesodermsplitstoformtwolayersseparatedbyan
intraembryoniccoelom.
Thesetwolayersdeveloplaterintothevisceralandparietallayers
foundinallserouscavitiesincludingperitoneum.Andthepotential
spaceofintraembryoniccoelombetweenthesetwolayersbecome
thebodycavitieslikeperitonealcavity,pleuralcavityandpericardial
cavity.
Asembryodevelops,thevariousabdominalorgansgrowintothe
abdominalcavityfromstructuresintheabdominalwall.Inthis
processtheybecomeenvelopedinalayerofperitoneum,i.e.
viscerallayer.
Peritonealfoldsdevelopfromtheventralanddorsalmesenteryof
theembryo.
Peritoneum,pericardiumandpleuradevelopfrom-,Mesoderm
(lateralplatemesoderm)
Peritonealcavity,pericardialcavityandpleuralcavitydevelopfrom

Intraembryoniccoelom.

16.Cytotrophoblastsinvades?
a)Dparietalis
b)Dbasalis
c)Dcapularis
d)None
CorrectAnswer-B
Ans.is'b'i.e.,Dbasalis[RefHumanEmbryologybyRaniKumarp.
37]
Afterembeddingofblastocytintheendometrialstroma,the
trophoblastdifferentiatesinto-
Cytotrophoblast
Syncytiotrophoblast
Syncytiotrophoblastinvadesuterineepithelialcells.
CytotrophoblastinvadesDeciduabasalis(Dbasalis)afterpassing
throughoverlyingsyncytotrophoblast.


17.Coronarysinusdevelopsfrom?
a)Truncusarteriosus
b)Conus
c)Sinusvenosus
d)AVcanal
CorrectAnswer-C
Ans.is'c'i.e.,Sinusvenosus[RefEssentialsofhuman
embryologyp.873]
TheCoronarysinusdevelopsfromtheSinusvenosussegmentof
embryonicheart.Lefthornofsinusvenosusretrogressestoform
partofcoronarysinus

18.Whichisderivedfromwolfianduct?
a)Appendixoftestis
b)Uterus
c)Appendixofepididymis
d)Hydatidofmargagni
CorrectAnswer-C
Ans.is'c'i.e.,Appendixofepididymis[RefIBsingh8r"/e-260]
Appendixofepididymisdevelopsfrommesonephric(wolffian)duct.
Appendixoftestis(alsocalledhydatidofmargagni)anduterus
developfromparamesonephricduct.

19.Arteryof2ndpharyngealarchis?
a)Maxillaryartery
b)Stapedialartery
c)Subclavianartery
d)Commoncarotidartery
CorrectAnswer-B
Ans.is'b'i.e.,Stapedialartery[RefGarg2"/e-223;Langman's-
185]
2nd(hyoid
Stapedialartery
arch):
3rd:-
Commoncarotid/internalcarotid
4th:-:
Right4thaorticarch,subclavianartery,Left4thaortic
arch:aorticarch
6th:-
Rightaorticarch:pulmonaryartery,Left6thaortic
arch:Pulmonaryartstxandductusarteriosus

20.Fossaovalisisaremnantof?
a)Septumprimum
b)Septumsecundum
c)Ductusarteriosus
d)Ductusvenosus
CorrectAnswer-A
Ans.is'a'i.e.,Septumprimum[RefBDC5th/eVolume1p.247;
IBS7"Vep.265;Garg2"/ep.215]
Fossaovalisisaremnantofforamenovale.Flooroffossaovalisis
remnantofseptumprimum.

21.Ychromosomeis?
a)Metacentric
b)Submetacentric
c)Acrocentric
d)None
CorrectAnswer-C
Ans.is'c'i.e.,Acrocentric
Eachchromosomehastwoarms:-

1. parm(theshorteroftwo)
2. qarm(longerone
Dependingonthepositionoftherejunction(centromere)they
canbeclassifiedinto:
1.Metacentric
Thecentromereisinthemiddlesothatthetwoarmsof
chromosomesarealmostequal.
Thesechromosomesare:chromosomes1,2,3andX-
chromosome.
2.Submetacentric
parmisshortandqarmislong.Thesechromosomesare:4,5,6,
7,8and9
3.Acrocentricp
(short)armissoshortthatitishardtoobserve,butstillpresent.
Therearesixacrocentricchromosomes:13,14,15,21,22andY
chromosomes

22.Theouterlayeroftheblastocystforms?
a)Primitivestreak
b)Yolksac
c)Embryoproper
d)Trophoblast
CorrectAnswer-D
Ans.is'd'i.e.,Trophoblast[RefSinghIB,HumanembryologyStilep.
38]
Blastocystpossessesaninnercellmasscalledembryoblastwhich
subsequentlyformstheembryoproper,andanouterlayerofcells,
ortrophoblast,whichhelpstoprovidenutritiontotheembryo.

23.Inferiorparathyroiddevelopsfromwhich
arch?
a)1st
b)2nd
c)3rd
d)4th
CorrectAnswer-C
Ans.is'c'i.e.,3rd[RefLangman's11th/ep.269]

24.Mesodermalinorigin?
a)Astrocytes
b)Oligodendrocytes
c)Ependymalcells
d)Microglialcells
CorrectAnswer-D
Ans.is'd'i.e.,Microglialcells[RefI.B.singhp.319-321]
Microglialcellsarethenervoussystemcounterpartofthemonocyte
macrophagesystem.
Theuniquefeatureaboutmicrogliaisthatunlikeotherneuroglial
cellsitisnotdevelopedfromneuroectoderm.
Thesecellsaremesodermalinorigin.

25.Fossaovalisclosesbecauseoffusionof
?
a)Septumprimum+Endocardialcushion
b)Septumsecundum+Endocardialcushion
c)Septumprimum+Septumsecundum
d)None
CorrectAnswer-C
Ans.is'c'i.e.,Septumprimum+Septumsecundum[Ref
Essentialsofhumanembryology--213-215]
Afterbirth,theforamenovaleclosesbyfusionofseptumprimum
withseptumsecundum.

26.Whichofthefollowingisatraction
epiphysis?
a)Tibialcondyles
b)Trochanteroffemur
c)Coracoidprocessofscapula
d)Headoffemur
CorrectAnswer-B
Ans.is'b'i.e.,Trochanteroffemur[RefBDChandbookofgeneral
anatomy3r4/ep.34]


27.Glandsareclassifieddependingupontheirmodeofsecretion.Sebaceous
glandisanexampleofwhichofthefollowingtypeofgland?
a)Eccrine
b)Apocrine
c)Holocrine
d)Paracrine
CorrectAnswer-C
Sebaceousglandsareholocrineglands.Holocrineglandsarethoseinwhichthecellafter
fillingwithsecretoryproductdiesandisexpelledalongwithitscontent.
Eccrineglandsarethoseinwhichsecretoryproductisexpelledbyexocytosis.Egsweat
glandsinvolvedinthermoregulationandreceivingcholinergicsympatheticinnervation.
Apocrineglandsarethoseinwhichthesecretoryproductaccumulatesintheapical
cytoplasmandisexpelledoutbypinchingoftheapicalplasmamembrane.Egmammary
glandandapocrinetypeofsweatglandsthatareactiveafterpubertyandarefoundinthe
skinofaxillaandaroundgenitalorgans.
Ref:ClinicalAnatomy:(aProblemSolvingApproach)ByKulkarnipage9

28.Corporaamylaciaeisseenin-
a)Thymus
b)Lymphnode
c)Spleen
d)Prostate
CorrectAnswer-D
Ans.is'd'i.e.,Prostate

29.Corporaarenaceaisseenin?
a)Prostrate
b)Pineal
c)Seminalvesicle
d)Breast
CorrectAnswer-B
Ans.is'b'i.e.,Pineal[RefTextbookofhumanhistology-323]
Corporaarenacea(orbrainsand)arecalcifiedstructuresinPineal
glandandotherareasofbrainsuchaschoroidplexus.

30.Nutrientarteryruns?
a)Towardsmetaphysis
b)Awayfrommetaphysis
c)Awayfromepiphysis
d)None
CorrectAnswer-A
Ans.is'a'i.e.,Towardsmetaphysis[RefTextbookofgeneral
anatomyp.80]
Nutrientartery
Itentersthemiddleoftheshaftthroughanutrientforamen,runs
obliquelythroughthecortex,andthendividesintoascendingand
desendingbranchesthatruntowardsmetaphysis.
Eachbranchsubdividesintoanumberofsmallerparallelvessels
whichenterthemetaphysisandformhair-pinloops.
Theloopsanostomosewithepiphyseal,metaphysealandperiosteal
arteries.
Therefore,metaphysisisthemostvascularzoneofthelongbone.
Thenutrientarterysuppliesthemedullarycavityandinner-twothird
ofcorticalboneofdiaphysisandmetaphysis

31.Multi-unitsmoothmusclepresentatall
except?
a)Bloodvessels
b)Iris
c)Gut
d)Ductusdeferens
CorrectAnswer-C
Ans.is'c'i.e.,Gut[RefBDCgenralanatomy4"/ep.96;
Fundamentalofhumenanatomyp.6]

32.VasaVasorumofascendingaortaarises
from?
a)Leftcoronaryartery
b)Anteriorinterventricularartery
c)Posteriorinterventricularartery
d)Leftatrium
CorrectAnswer-A
Clinicalanatomy
"Bothcoronaryarteriessupplyvasavasorumofascendingaorta"
"Coronaryarteriesarevasavasorumofascendingaorta"
Vasavasorumofascendingaortaandarchofaortaarisefrom:
Coronaryarteries(attheirostia).
Brachiocephalictrunk.
Bronchialartery.
Vasavasorumofdescendingaortaarisefrom':
Intercostalarteries(thoracicpart).
Lumbarandmesentericarteries(abdominalpart).

33.Renalpapillaopensinto-
a)Cortex
b)Pyramid
c)Minorcalyx
d)Majorcalyx
CorrectAnswer-C
Ans.is'c'i.e.,Minorcalyx
Eachkidneyhastwodistinctzones:

1. Theoutercortex
2. Theinnermedulla
-Themedullacomprisesabout10renalpyramids.Theirapicesform
renalpapillaewhichindenttheminorcalyces.
-Thecortexisdividedintotwoparts
Corticalarchesorcorticallobules,whichformcapsoverthebases
ofthepyramids.
Renalcolumns,whichdipinbetweenthepyramid.
Eachpyramidalongwithoverlyingcorticalarchformsalobeofthe
kidney.
Therenalsinusisaspacethatextendsintothekidneyfromthe
hilus.
Itcontains:
Branchesoftherenalartery.
Tributariesoftherenalvein.
Renalpelvis:Pelvisdividesinto2to3majorcalyces,andthese,in
turn,divideinto7-13minorcalyces.Eachminorcalyxendsinan
expansionwhichisindentedbyonetothreerenalpapillae

34.1stcarpometacarpaljointis?
a)Pivot
b)Hinge
c)BallandSocket
d)Saddle
CorrectAnswer-D
Saddle

35.Structurepassesthroughuppertriangular
space:
a)Profundabrachii
b)Anteriorcircumflexhumeralartery
c)Posteriorcircumflexhumeralartery
d)Circumflexscapularartery
CorrectAnswer-D
UpperQuadrangularspace
Ithasthefollowingboundaries:
?theteresmajorinferiorly
?thelongheadofthetricepslaterally
Forthesuperiorborder,somesourceslisttheteresminor,while
otherslistthesubscapularis.
Itcontainsthescapularcircumflexvessels.

36.Followingadeepcutoverlyingthe
hypothenareminence,itisobservedthat
thepatientcannotholdasheetofpaper
betweenthe2ndand3rddigits.Whichof
thefollowingnervesismostlikely
damaged?

a)Deepbranchofulnarnerve
b)Deepbranchoftheradialnerve
c)Superficialbranchofulnarnerve
d)Mediannerve
CorrectAnswer-A
Ans.is'a'i.e.,Deepbranchofulnarnerve[RefBDC5thle
VolumeIp.110-111]
Inabilitytoholdpapersheetsignifiesinabilitytoadductthefingers.It
isnothingelsebutcardtest.
Deepbranchofulnarnervesuppliesthepalmarinterosseiwhich
controltheadductionbetweenfingers.

37.Whichistrueaboutsynovialjoint?
a)Stabilityisinverselyproportionaltomobility
b)Hyalinecartilagecoversarticularsurfaceofallsynovialjoints.
c)Metacarpo-phalangealjointisahingejoint
d)"Cartilageusuallydividesthejointintotwocavities".
CorrectAnswer-A
Ai.e.Stabilityisinverselyproportionaltomobility

38.Apatientisfoundtohaveamelanomaoriginatingintheskinoftheleft
forearm.Afterremovalofthetumorfromtheforearm,allaxillarylymphnodes
lateraltothemedialedgeofthepectoralisminormuscleareremoved.Which
axillarynodeswouldnotberemoved?

a)Apicallymphnodes
b)Centrallymphnodes
c)Laterallymphnodes
d)Pectorallymphnodes
CorrectAnswer-A
ThelymphnodeslateraltotheLATERALedgeofthepectoralis
minorarethelateralaxillarynodes.Pectoraloranterioraxillary
nodesaredeeptothelateraledgeofthepectoralismajormuscle.
Centralaxillarynodesarefounddirectlyunderpectoralisminor,
whilesubscapularorposterioraxillarynodesareadjacenttothe
subscapularismuscle.Theapicalaxillarynodesaremedialtothe
medialedgeofpectoralisminorandsoitwon'tberemoved.
level
Partofaxilla
Lymphnodes
I
Lowaxilla
Lymphnodeslateraltothe
lateralborderofpectoralis
minor(centralgroup,
lateralgroup,anterior
group,posteriorgroup)
II
Midaxilla
Lymphnodesbetweenthe
lateralandmedialborders
ofpectoralisminor.plus
interpectorallymphnodes
III
Apicalaxilla
Lymphnodesmedialto
medialmarginofpectoralis
minor(including

subclavicular,
infraclavicularorapical).

39.Whichmusclesteadiestheclavicleduring
movementofshoulder?
a)Pectoralismajor
b)Latissimusdorsi
c)Subclavius
d)Serratusanterior
CorrectAnswer-C
Ans.is'c'i.e.,Subclavius
(Ref:Clinicalanatomyj'd/ep.1367)
Subclaviussteadiestheclavicleduringmovementsofshoulder.

40.Duringheeltouchphaseofwalking
pressureincalfcompartmentis?
a)Morethanrestingpressure
b)Lessthanrestingpressure
c)Nochangeinpressure
d)Firstrisesandthenfalls
CorrectAnswer-C
Ans.is'c'i.e.,Nochangeinpressure
Inheelstrike,theankleisdorsiflexedandthemainmuscleactingon
ankleistibialisanterior.
Musclesofposteriorcompartmentofleg(Gastrocnemius-soleus)are
attheirnormallength.So,thepressureinposteriorcompartment
doesnotchange.
Ontheotherhand,InpushoffphaseGastrocnemius-soleusare
contractedwhichresultsinincreasedpressureofposterior
compartment(wheneverthereiscontractionofmusclesofa
compartment,pressureofthatcompartmentisincreased).

41.Thenutrientarterytothefemuris?
a)Profundafemorisartery
b)Femoralartery
c)Poplitealartery
d)Middlecircumflexfemoralartery
CorrectAnswer-A
Ans.is'a'i.e.,Profundafemorisartery
NutrientArteries
Thefemurissuppliedby2ndperforatingbranchofprofundafemoris
artery.
Tibiaissuppliedbyabranchofposteriortibial
Thefibulaissuppliedbytheperonealarterywhichisabranchof
posteriortibialartery.
Thehumerusissuppliedbyabranchofprofundabrachiiartery
Radius/ulnaaresuppliedbytheanteriorinterosseousarterywhichis
abranchoftheulnarartery.

42.Inwalking,gravitytendstotiltpelvisandtrunktotheunsupportedside,major
factorinpreventingthisunwantedmovementis?
a)Adductormuscles
b)Quadriceps
c)Gluteusmaximus
d)Gluteusmediusandminimus
CorrectAnswer-D
Thegluteusmediusmuscleabductsandmediallyrotatesthefemur
atthehipjoint.Inaddition,thegluteusmediusholdsthepelvis
secureoverthestanceleg,preventingpelvicdropontheopposite
swingsideduringgait.Thesuperiorglutealnerve(L4,L5,S1)
innervatesthismuscle.
Theactionofthegluteusminimusmuscleisthesameasthatofthe
gluteusmedius--itabductsthefemuratthehipjoint,holdingthe
pelvissecureoverthestancelegandpreventingpelvicdroponthe
oppositeswingsideduringgaitandhipmedialrotation.Theinferior
glutealnerve(L5,S1,S2)innervatesthismuscle.

43.Siteofinjectioningluteus?
a)Inferomedial
b)Superomedial
c)Superolateral
d)Superomedial
CorrectAnswer-C
Ans.is'c'i.e.,Superolateral
level Partofaxilla
Lymphnodes
I
Upperarm(Deltoid)
5cmdistaltotheacromionor
4cmproximaltotheinsertionof
deltoid.
Thisistopreventinjurytocircumflex
humeralnerve.
II
Glutealregion
Upperouter(superolateral)quadrant.
Thisistoavoiddamagetosuperior
andinferiorglutealvesselsand
sciaticnerve.
III
Thigh(lateralaspect) Infant:-Upperlateralquadrantof
(vastuslateralis)
thighbelowGT
Adult:-Middlethirdoflateralaspect.

44.Lockingofkneejointcanbecausedby:
a)OsgoodSchlatter
b)Loosebodyinkneejoint
c)Tuberculosisofknee
d)aandbboth
CorrectAnswer-D
Ans.is.d.aandbboth
Lockingofkneejoint(i.e.jointheldinflexion)isseeninmeniscus
tear,loosebody(d/tosteochondralfracture)andfracturesoftibial
spine
MechanismofLocking
Normallythemedialmeniscusoratleastitsanteriormovableportion
glidesslightlybackwardstowardstheinteriorofjointasthekneeis
flexed.
Ifthetibiaisatthesametimeabducted(valgus)andthemedial
compartmentofthekneethusopenedup,themobilityofthe
meniscusisstillfurtherincreased.
Suddenmedialrotationoffemuronthefixedtibiaforcesthemedial
meniscustowardsbackofjointandcausesmedialligamentto
becometautanditmayundergovarietyoftransverseoroblique
tear.
Theinnerfragmentslipsintotheinteriorofthejointandwhen,
extensionisattemptedandthekneebeginstoscrewhome'the
fragmentisnippedbetweenthecondylesandthejointis'locked'i.e.
heldinflexion.

45.Whichstructure(s)passesbehindthe
inguinalligament:
a)Femoralbranchofgenitofemoralnerve
b)Femoralvein
c)Psoasmajor
d)All
CorrectAnswer-D
Ai.e.Femoralbranchofgenitofemoralnerve;Bi.e.Femoralvein;C
i.e.Psoasmajor

46.Deltoidligamentisattachedtoallexcept:
a)Medialmalleolus
b)Medialcuneiform
c)springligament
d)sustentaculumtali
CorrectAnswer-B
Ans.Bi.e.Medialcuneiform
Medialcollateralligament(deltoidligament)attachestothemedial
malleolusofthetibiaandthenavicular,talus,andcalcaneusbones.
Thisligamentpreventsmedialdistraction(eversion)andexcessive
rangeofmotion.Itissubdividedintofourparts:
Tibionavicularpartattachesthemarginoftheplantar
calcaneonavicularligament(springligament).
Tibiocalcanealpartattachestothesustentaculumtaliofthe
calcaneusbone.
Posteriortibiotalarpartattachestothemedialsideandmedial
tubercleofthetalus.
Anteriortibiotalarpartattachestothemedialsurfaceofthetalus.

47.Trueaboutpopliteusareallexcept?
a)Flexorofknee
b)Intracapsularorigin
c)Suppliedbytibialnerve
d)Causeslockingofknee
CorrectAnswer-D
Popliteus
Popliteusisadeepmuscleofposteriorcompartmentofleg.
Featuresofpopletiusare-
Origin
Lateralsurfaceoflateralcondyleoffemur,originisintracapsular.
Outermarginoflateralmeniscusofknee.
Insertion
Posteriorsurfaceofshaftoftibiaabovesolealline.
Nervesupply
Tibialnerve
Action
Ulockskneejointbylateralrotationoffemurontibiapriorflexion.
Accessoryflexorofknee.

48.Tibialnerveinjury/palsycauses:
a)Dorsiflexionoffootatanklejoint
b)Planterflexionofthefootatanklejoint
c)Lossofsensationofdorsumoffoot
d)Paralysisofmusclesofanteriorcompartmentofleg
CorrectAnswer-A
Ai.e.Dorsiflexionoffootatanklejoint
*Tibialnerve(thelargercomponentofsciaticnerve)suppliesall
musclesofposteriorcompartmentofleg(gastrocnemius,soleus,
plantaris,popliteus,tibialisposterior,FHL&FDL)plantiflexingthe
ankle&foot.Therefore,tibialnerveinjuryresultsinlossofplanter
flexionalongwithcalcaneo-valgusattitudeoffoot(iedorsiflexion&
eversiond/tunopposedanteriorcompartmentmuscles).
-Deepperoneal(fibular)nervesuppliesallanteriorleg
compartmentmuscles(tibialisanterior,EHL,EDL,EDB,PT)
dorsiflexingtheankle&foot.
-Soleissuppliedbymedial&lateralplantarandsuralbranchesof
tibialnerveQanteriorly(frommedialtolateral)andcalcaneal
branchesoftibialnerve0posteriorly(i.e.overheel).
-ItthenpassesalonglateralsideoffootandlittletoeQsupplying
theoverlyingskin.
-Anterolateralaspectofleg,anddorsumoffootincluding2"-
4thzvebspacesQaresuppliedbysuperficialperonealnerve.
-Whereasdeepperoneal(fibular)nervesupplies1stwebspace(i.e.
adjacentsidesofgreatandsecondtoes)Q

49.Nervesupplyofcervicalesophagus?
a)Vagus
b)Leftrecurrentlarynagealnerve
c)Rightrecurrentlarynagealnerve
d)Alloftheabove
CorrectAnswer-D
Ans.isd'.,Alloftheabove
[RefClinicalanatomy3'd/ep.2891
Nervesupplyofesophagus:
Esophagusissuppliedbybothparasympatheticandsympathetic
fibers.
A)Parasympatheticsupply
Itprovidessensory,motorandsecretomotorsupplytoesophagus.
Completeparasympatheticinnervationisprovidedbyvagus
nerve:
-
1. Cervicalesophagus:Throughboth(right&left)recurrentlaryngeal
nerve.
2. Upperthoracicesophagus:Throughleftrecurrentlaryngealnerve
andbydirectbranchesfromvagusnerve.
3. Lowerthoracicesophagus:Throughesophagealplexus.
B)Sympatheticsupply
Itprovidessensoryandvasomotorsupply.
ItisprovidedbyTstoTcspinalsegments.

50.Apatientwithnativeaorticvalvedisease
camewithrighthemiparesis.Whatwill
youdotopreventfurtherstroke?

a)Antiplateletonly
b)Anticoagulantonly
c)Bothantiplateletandanticoagulant
d)Onedoseoflowmolecularweightheparinsub-cutaneously
followedbydualantiplatelettherapy
CorrectAnswer-A
Answer-A.Antiplateletonly
*Aspirinistheonlyantiplateletagentthathasbeenproven
elfectivefortheacutetreatmentofischemicstroke;useofaspirin
within48hoarsofstrokeonsetreducedbothstrokerecurrencerisk
andmortalityminimally.
*Asprinistheonlyantiplateletagentthathasbeenproveneffecfive
fortheacutetreatmentofischemicstroke;thereareseveral
antiplateletagentsprovenforthesecondarypreventionofstroke.
*Twolayertrials,theInternationalStrokeTrial(IST)andthe
ChineseAcuteStrokeTrial(CAST),foundthattheuseofaspirin
within48hoursofstrokeonsetreducedbothstrokerecurrencerisk
andmortalityminimally.

51.Allformboundariesoftriangleof
auscultationexcept
a)Trapezius
b)Latissmusdorsi
c)Scapula
d)Rhomboidmajor
CorrectAnswer-D
Ans.is'd'i.e.,Rhomboidmajor
Triangleofauscultationhasthefollowingboundaries
Superiorlyandmedially,bytheinferiorportionoftheTrapezius.
Inferiorly,bytheLatissimusDorsi.
Laterally,bythemedialborderofthescapula.
ThesuperficialfloorofthetriangleisformedbytheSerratusanterior
andthelateralportionoftheerectorspinaemuscles.
Deeptothesemusclesaretheosseousportionsofthe6"and7th
ribsandtheinternalandexternalintercostalmuscles.
Typically,theTriangleofAuscultationiscoveredbytheScapula.
Tobetterexposethetriangleandlistentorespiratorysoundswitha
stethoscope,patientsareaskedtofoldtheirarmsacrosstheirchest,
mediallyrotatingthescapulae,whilebendingforwardatthetrunk,
somewhatresemblingafetalposition.

52.Whichofthefollowingcomplicationsis
notseeninmitralvalveprolapse?
a)Stroke
b)Infectiveendocarditis
c)Mitralstenosis
d)Ventriculararrhythmia
CorrectAnswer-C
Answer-C.Mitralstenosis
SurfacemarkingofthemitralvalveisBehindsternalendofleft
4thcostalcartilage.
Infectiveendocarditis
Mitralinsuficiency(mitralregurgitation)
Strokeorothersystemicinfarctresultingfromembolismofleaflet
thrombi
Arrhythmias

53.IVCfilterisusedinfollowingexcept-
a)Toreducessymptoms
b)Negligiblesizeofemboli
c)Toprolonglife
d)Topreventprogressofnativebloodvesseldisease
CorrectAnswer-B
Answeris'b'i.e.Negligiblesizeofemboli

54.Waldeyer'sfascialies?
a)Infrontofthebladder
b)Behindtherectum
c)Betweenbladderanduterus
d)Betweenuterusandrectum
CorrectAnswer-B
Ans.is'B'i.e.,Behindtherectum
*ThecaveofRetziusisafat-filledretropubicspacethatallowsfor
theaccommodationofadistendedbladder.
*TheWaldeyer'sfascia,alsoknownasthepresacralfascia,
accommodatesthedistendedrectum.
*ThepouchofDouglasisafoldofperitoneumbetweentheuterus
andtherectum.
*ThepouchofDunnisafoldofperitoneumbetweenthebladder
andtheuterus.
*ThefasciaofDenonvillierliesbetweenthebladderinfrontand
rectumbehind.

55.Rightcoronaryarterysuppliesall,except?
a)Anterior2/3ofventricularseptum
b)SAnode
c)AVnode
d)LBB
CorrectAnswer-A
Rightcoronaryarterysupplies
*SAnode(in65%),
*wholeconductingsystem(AVbundle,bundleofhis,partofleft
bundlebranch)exceptRBBandpartofleftbranchofAVbundle,
*posteriorpartofventricularseptum,
*mostofrightventricleexceptsmallpartadjoininganterior
interventriculargrooveandsmallpartofleftventricleadjoining
posteriorinterventriculargroove.
Leftcoronaryarterysupplies
*mostoftheleftatrium,
*mostoftheleftventricleincludingapex,
*smallpartofrightventricleodjoininganteriorinterventricular
septum,
*anterior2/3ofventricularseptum,
*RBB,LBB
*SAnodein35%ofcases.

56.Thethoracicductcrossesfromtheright
totheleftatthelevelof
a)T12vertebra
b)T6vertebra
c)T5vertebra
d)T2vertebra
CorrectAnswer-C
Ci.e.T5vertebrae
Thoracicductbeginsascontinuationoftheupperendofthe
cisternachylinearthelowerborderofT12vertebraandentersthe
thoraxthroughtheaorticopeningofdiaphragm(atT12).
ItthenascendsthroughtheposteriormediastinumandatT5level
crossesfromrightsidetotheleftsideandascendsalongleftmargin
ofoesophagustoentertheneck

57.Whichofthefollowingstatementregardingloweresophagealsphincteris
TRUE?
a)Ithasnotonicactivity
b)Ithasatonewhichisprovidedbythesympatheticsystem
c)Relaxesonincreasingabdominalpressure
d)Relaxesaheadoftheperistalticwave
CorrectAnswer-D
Duringswallowingwhentheperistalticwavesweepdownthe
esophagusthegastroesophagealsphincterrelaxessothatthebolus
offoodcanpassintothestomach.
DistanceoftheLowerEsophagealSphincterfromtheupper
incisorsis37.5cm
theloweresophagealsphincteristonicallyactivebutrelaxeson
swallowing.ThistonicactivityoftheLESbetweenmealsprevents
refluxofgastriccontentsintotheesophagus.itstoneisunderneural
controlbytheparasympatheticnervoussystem.ContractionofLES
iscausedbythereleaseofacetylcholinefromthevagusand
releaseofNOandVIPfrominterneuronsinnervatedbyothervagal
fiberscausesittorelax.
Ref:FundamentalsofHumanPhysiologyByLauraleeSherwood
page447.Ganong'sReviewofMedicalPhysiology,24eCHAPTER
27.

58.Thetransversesinusispresentposterior
towhichstructures?
a)Rightatrium
b)Leftatrium
c)Upperpulmonaryartery
d)Aorta
CorrectAnswer-D
Ans.'D'i.e.,Aorta
Pericardialsinuses
*Ontheposteriorsurfaceoftheheart,thereflectionoftheserous
pericardium(epicardium)aroundlargeveinsformsarecesscalled
theobliquesinus.
*Obliquesinusisboundedanteriorlybytheleftatrium,and
posteriorlybytheparietalpericardiumandesophagus.
*Thetransversesinusisashortpassagethatliesbetweenthe
reflectionoftheserouspericardium(epicardium)aroundarterial
(aortaandpulmonarytrunk)andvenousendsofthehearttube.
*Thetransversesinusisboundedanteriorlybyascendingaortaand
pulmonarytrunk,posteriorlybySVC,andinferiorlybytheleftatrium

59.Sympatheticsupplytotheheartarisesfromwhichofthefollowingspinal
segments?
a)T1toT5
b)T2toT6
c)T3toT7
d)T4toT8
CorrectAnswer-A
Sympatheticnervecellssupplyingtheheartarelocatedinthe
intermediolateralhornofspinalsegmentsT1toT5.
Sympatheticsupplyiscardio-acceleratory,andonstimulation,they
increasetheheartrate,andalsodilatecoronaryarteries.
Bothparasympatheticandsympatheticnervesformthesuperficial
anddeepcardiacplexuses,thebranchesofwhichrunalongthe
coronaryarteriestoreachthemyocardium.

60.Surfacemarkingoftheobliquefissureof
thelungincludeallexcept
a)T3
b)5thrib
c)7thrib
d)6thcostalcartilage
CorrectAnswer-C
Ans.is'c'i.e.,7thrib
Surfacemarkingoffissures
Theobliquefissurecanbedrawnbyjoining:
Apoint8cmlateralto3rdthoracicvertebrae.
Anotherpointonthe5thribinthemidaxillaryline.
Athirdpointonthe6thcostalcartilage7.5cmfromthemedian
plane.

61.Leveloflowerborderoflungatmid
axillarylineis
a)6thrib
b)8thrib
c)10thrib
d)12thrib
CorrectAnswer-B
Bi.e.8thrib

62.Tributaryofcoronarysinus?
a)Anteriorcardiacvein
b)Thebesionvein
c)Smallestcardiacvein
d)Greatcardiacvein
CorrectAnswer-D
Ans.(D)Greatcardiacvein
Coronarysinus
Itopensintheposteriorwallofrightatrium,intheposteriorpartof
coronarysulcus.
ItopensintherightatriumbetweenIVCandtricuspidorifices.
CoronarysinusisguardedbyThebesianvalve(Thebesianvalve
(incompletesemilunarvalve)guardstheorificeofcoronarysinus.)
Tributariesofcoronarysinusare:
Greatcardiacvein:-Liesintheanteriorinterventriculargroove.Left
marginalvein
drainsintoit.
Middlecardiacvein:-Liesintheposteriorinterventriculargroove.
Posteriorveinofleftventricle.
Smallcardiacvein:-
Itliesintheposteriorpartofcoronarysulcus
withRCA.Rightmarginnalveinmaysometimesopenintosmall
cardiacvein,moreoften,however,rightmarginalveinopens
directlyintorightatrium.
Obliqueveinofleftatrium(veinofmarshall):-
Itiscontinuousabove
withligamentofIVC.Thesetwostructuresareembryological
remnantsofleftcommoncardinalvein(ductofcuvier).

63.BoundaryoftheKoch'striangleisnotformedby?
a)Tricuspidvalvering
b)Coronarysinus
c)Tendonoftodaro
d)Limbusfossaovalis
CorrectAnswer-D
Koch'sTriangleisatriangleenclosedbytheseptalleafletofthe
tricuspidvalve,thecoronarysinus,andthemembranouspartofthe
interatrialseptum.
Koch'striangle:WalterKarlKoch(1880?1962)wasadistinguished
Germansurgeonwhodiscoveredatriangular-shapedareainthe
rightatriumoftheheartthatmarkstheatrioventricularnode(known
asKoch'striangle).
Thethreesidesofthetrianglearedefinedbythefollowingstructures
withintherightatrium:Theostiumofthecoronarysinus,posteriorly;
Theanteriorportionofthetricuspidvalveannulus;and
ThetendonofTodaro(atendinousstructureconnectingthevalveof
theinferiorvenacavaostiumtothecentralfibrousbody),posteriorly.
Importance:
Usedasananatomicallandmarkforlocationoftheatrioventricular
nodeduringelectrophysiologyproceduressuchaspacingor
ablation.

64.Azygosveindrainsinto:
a)Leftbrachiocephalicvein
b)Inferiorvenacava
c)Superiorvenacava
d)Rightbrachiocephalicvein
CorrectAnswer-C
Theazygosveinendsbyjoiningtheposterioraspectofthesuperior
venacava
TheAzygosVein
Theazygosveinconnectsthesuperiorandinferiorvenaecavae,
eitherdirectlybyjoiningtheIVCorindirectlybythehemiazygosand
accessoryhemiazygosveins.
Theazygosveindrainsbloodfromtheposteriorwallsofthethorax
andabdomen.
Itascendsintheposteriormediastinum
Itiscoveredanteriorlybytheoesophagusasitpassesposteriorto
therootoftherightlung.
ItthenarchesoverthesuperioraspectofthisroottojointheSVC.
Inadditiontotheposteriorintercostalveins,theazygosvein
communicateswiththevertebralvenousplexuses.
Thisveinalsoreceivesthemediastinal,oesophageal,andbronchial
veins.

65.

Analvalveisfoundinwhichpartofanal
canal?

a)Upper
b)Middle
c)Lower
d)Atanus
CorrectAnswer-A
Ans.is'a'i.e.,Upper

66.Mainsupportofuterusisfrom-ligament:
a)Cardinal
b)Broad
c)Round
d)Pubocervical
CorrectAnswer-A
Cardinal

67.Allofthefollowingarteriesarethebranchesofcoeliactrunk,EXCEPT?
a)Leftgastricartery
b)Rightgastricartery
c)Splenicartery
d)Hepaticartery
CorrectAnswer-B
Branchesofthecoeliactrunkareleftgastricartery,splenicartery,
andcommonhepaticartery.Thecoeliactrunkarisesfromthe
abdominalaorta,immediatelybelowtheaortichiatusofthe
diaphragmattheT12vertebrallevel.Rightgastricarteryisabranch
ofcommonhepaticartery.
Branchesofcommonhepaticarteryare:
Properhepaticartery
Rightgastricartery
Gastroduodenalartery:rightgastroomentalartery,superior
pancreaticoduodenalarteryarebranchesofgastroduodenalartery.
Cysticartery
Branchesofsplenicartery:
Leftgastroepiploicartery
Shortgastricbranches
Pancreaticbranches

68.Whichofthefollowingstructuresseenin
thecavernoussinus?
a)MaxillarydivisionofVnerve
b)MandibulardivisionofVnerve
c)Internalcarotidartery
d)Facialnerve
CorrectAnswer-C
Ans.c.Internalcarotidartery
Contentsofthecavernoussinus
Structuresinthelateralwallofthesinus
Oculomotor(lll)nerve
Trochlear(lV)nerve
Ophthalmic(1stdivisionofV)nerve
Trigeminalganglion
Internalcarotidartery
Abducent(Vl)nerve

69.Anteriorrelationsoftherightkidneyare
allexcept?
a)Liver
b)4thpartofduodenum
c)Hepaticflexure
d)Adrenalgland
CorrectAnswer-B
Ans.is'b'i.e.,4thpartoftheduodenum
Theposteriorsurfaceofbothkidneysisrelatedtothediaphragm,
medialandlateralarcuateligament,psoasmajor,quadratus
lumborum,transversusabdominis,subcostalvessels,subcostal
nerve,iliohypogastricnerve,andilioinguinalnerve.
Inaddition,therightkidneyisrelatedtothe12thribandtheleft
kidneyisrelatedto11thand12thribs.
Themedialborderofeachkidneyisrelatedtothesuprarenalgland
abovethehilusandureterbelowthehilus.
Thelateralborderoftherightkidneyisrelatedtotherightlobeofthe
liverandhepaticflexureofthecolon.Ontheleftside,itisrelatedto
spleenanddescendingcolon.

70.Pancreasdivisumindicateswhichofthe
following?
a)Duplicationofthepancreas
b)Failureoffusionofdorsal&ventralpancreaticbuds
c)Formationofmorethantwopancreaticbuds
d)Formationofonlyonepancreaticbud
CorrectAnswer-B
Answer-.is'b'i.e.,Failureoffusionofdorsal&ventral
pancreaticbuds
[RefInderbirSinghHumanEmbryology8thlep.
168]
Anomaliesofpancreaticdevelopmentmaybe:
1.Annularpancreas:-Twocomponentsoftheventralbudfailto
fuseandgrowinoppositedirectionaroundtheduodenumandmeet
thedorsalpancreaticduct.
2.Pancreaticdivisum(dividedpancreas):-Ventralanddorsalbuds
failtofusewitheachother.
3.Inversionofpancreaticduct:-Themainpancreaticductis
formedbytheductofdorsalbud,i.e.accessoryductislargerthan
themainductandthemaindrainageofpancreasisthroughthe
minorduodenalpapilla.
4.Accessorypancreatictissue:-Maybefoundas
*Wallofstomach,duodenum,jejunumorileum.
*Meckel'sdiverticulum

71.'ObstructionofInferiorvenacava'
presents:
a)Paraumblicaldilatation
b)Thoraco-epigastricdilatation
c)Oesophagusvaries
d)Haemorrhoides
CorrectAnswer-A:B
A.ParaumbilicusveindilationandBi.e.Thoraco-epigastricdilation
Thethoracoepigastricveinisuniqueinthatitdrainstoboth
theSuperiorVenaCava(SVC)andtotheInferiorVenaCava(IVC).
Hence,itservesasananastomoticcaval-cavallinkbetweenthe
two.Furthermore,thethoracoepigastricveinisconnectedto
theportalveinviatheparaumbilicalveinandtherebyservesasa
portocavalanastomosisaswell.
Whenapatientexperiencesportalhypertension,therecanbe
congestion(backup)ofbloodthatentersintothecavalsystemvia
thethoracoepigastricvein.Whenthisoccurs,therecanbean
externallyvisibledilationoftheparaumbilical(andperhapseventhe
thoracoepigastricveins)whichleadstotheappearanceof"Caput
Medusae"

72.Rightovarianarteryisabranchof?
a)Abdominalaorta
b)Rightinternaliliac
c)Commoniliac
d)Externaliliac
CorrectAnswer-A
Ans.'a'i.e.,Abdominalaorta[RefBDC5th/eVolume.3p.343]
Lateralbranches:
*Inferiorphrenicartery
*Middlesuprarenalartery
*Renalartery
*Testicular/ovarianartery

73.Structureimmediatelyposteriorto
pancreatichead?
a)Rightrenalvein
b)Splenicartery
c)Inferiormesentericvein
d)Coeliactrunk
CorrectAnswer-A
Ans.is'a'i.e.,Rightrenalvein[RefBDC5`Vevolume2p.306]
Terminalpartofrightrenalveinisposteriortoheadofpancreatic

74.Whichofthesebestdescribestherenal
angle?
a)Theanglebetweenthelattissimusdorsiandthe12thrib
b)Theanglebetweentheerectorspinaeandtheiliaccrest
c)Theanglebetweenthe12thribandtheerectorspinae
d)Theanglebetweenthe12thribandtherectusabdominis
CorrectAnswer-C
Answer-.is'C'i.e.,Theanglebetweenthe12thribandthe
erectorspinae

Theanglebetweenthelowerborderofthe12thribandtheouter
borderoftheerectorspinaeisknownastherenalangle.
Overliesthelowerpartofthekidney.
Tendernessinthekidneyiselicitedbyapplyingpressureoverthis
area.

75.Renalveinthrombosisisassociatedwith
allofthefollowingexcept:
September2011

a)Trauma
b)Sicklecellanemia
c)Nephroticsyndrome
d)Dehydration
CorrectAnswer-B
Ans.B:Sicklecellanemia
ConditionsassociatedwithRVTare:Trauma,extrinsiccompression
(lymohnodes,aneurysm),invasionbyrenalcellcarcinoma,
dehydration(infants),nephriticsyndromeandPregnancy/oral
contraceptives
RVT:
Acutecasesoccursinchildrenandpresentswithsuddenlossof
renalfunction
Gradualthrombosisoccursinelderlyandonlymanifestationmaybe
recurrentpulmonaryemboliordevelopmentofhypertension
Definitivediagnosiscanbedonethroughselectiverenalvenography
withvisualizationoftheoccludingthrombus
Treatmentoptionsconsistsofanticoagulationandthrombectomy

76.Inpatientswithpenileinjury,Colle's
fasciapreventsextravasationofurinein?
a)Ischiorectalfossa
b)Perineum
c)Abdomen
d)None
CorrectAnswer-A
Ans.is'a'i.e.,Ischiorectalfossa[RefBDC5th/eVolumeHp.211]

77. Duringincision&drainageofischiorectal
abscess,whichnerveis/are
affected/injured:
a)Superiorrectalnerve
b)Inferiorrectalnerve
c)Superiorglutealnerve
d)Inferiorglutealnerve
CorrectAnswer-B
Bi.e.Inferiorrectalnerve
Throughaposteriorhorseshoeshapedrecessbothischiorectal
fossaeareconnectedbehindtheanalcanal;
soaunilateralabscess
maybecomebilateral.
Duringdissectionofischiorectalfossa,inferiorrectal,pudendal,
posteriorscrotalorlabialnerve&vesselsalongwithperforating
branchesofS2-S3andperinealbranchesofS4nervemayget
damaged.

78.Righthepaticveindrainswhichsegment
oftheliver?
a)I
b)II
c)IV
d)VII
CorrectAnswer-D
Ans.is'd'i.e.,VII
[RelGray's4fh/ep.1163-1167;Sabaton18h/ep.15841
Segmentalanatomyoftheliver:
Basedonthedistributionofportalveinandhepaticvein,Couinaud
dividedeachphysiological(functional)lobeofliverinto
4segmentseachandhenceliverisdividedinto8segments.
Thephysiologicalleftlobeiscomposedof4segmentsdesignatedI
toIVandissuppliedbyleftbranchofhepaticartery,Ieftbranchof
portalveinanddrainedbylefthepaticductandlefthepaticvein.
ThephysiologicalrightlobeconsistsofsegmentV,VI,VIIandVIII
andissuppliedbyrighthepaticartery,rightbranchofportalveinand
drainedbyrighthepaticductandrighthepaticvein.

79.Allofthefollowingarestructures
associatedwithpterygopalatinefossa,
EXCEPT:

a)Pterygopalatineganglion
b)Midthirdofmaxillaryartery
c)Maxillarynerve
d)Greaterpetrosalnerve
CorrectAnswer-B
Thepterygopalatinefossaistheregionbetweenthe
pterygomaxillaryfissureandthenasalcavity.
*Thefossaaccommodatesbranchesofthemaxillarynerve[cranial
nerve(CN)V-2],thepterygopalatineganglion,theterminal
branches
ofthemaxillaryartery,andgreatersuperficialpetrosal
nerve.

80.Portalveinisformedbyunionofwhichofthefollowingveins?
a)Superiormesentericvein&Splenicvein
b)Superiormesentericvein&inferiormesentericvein
c)Inferiormesentericvein&Splenicvein
d)inferiormesentericvein&Hepaticvein
CorrectAnswer-A
PortalveinisformedbytheunionofSuperiormesentericvein(SMV)
andsplenicveinposteriortotheneckofpancreas.Theinferior
mesentericveindrainsintothesplenicvein.
*Thehepaticportalveinpassposteriortothefirstpartofduodenum,
inthefreeedgeoflesseromentum.
*Attheportahepatis,itdividesintorightandleftbranches
supplyingtherightandleftlobesoftheliver.
*Withinthesinusoidsoftheliver,hepaticportalbloodand
oxygenatedbloodfromthehepaticarterymixtogetherandcome
intocontactwiththehepatocytes,wheremetabolitessuchas
productsofdigestionareexchanged.
*Bloodfromthesinusoidsemptiesintohepaticveinsdrainingthe
liverandinturndrainintoIVC,andbloodisreturnedtoheart.

81.Treatmentofanincidentallydetected
Appendicularcarcinoidmeasuring2.5cm
is:
September2002

a)Righthemicolectomy
b)Limitedresectionoftherightcolon
c)Totalcolectomy
d)Appendicectomy
CorrectAnswer-A
Ans.Ai.e.Righthemicolectomy

82.Neurovascularplaneinanterior
abdominalwall-
a)Betweenextobliqueandinternaloblique
b)Betweenint.obliqueandtransversusabdominis
c)Belowtransversusabdominis
d)Aboveext.oblique
CorrectAnswer-B
Ans.is'b'i.e.,Betweenint.obliqueandtransverses
abdominis
[Ref:Humananatomy5th/p.73]
Themusclesoftheanteriorabdominalwallconsistofthreebroad
thinsheetsthatareaponeuroticinfront,fromexteriortointeriorthey
areexternaloblique,internalobliqueandtransversus.
Thenerveandaccompanyingintercostalvesselsliebetweenthe
internalobliqueandtrannsverseabdominis.i.e.neurovascular
plane.

83.Celiacplexusblockallthefollowingis
trueexcept?
a)Relievedpainfromgastricmalignancy
b)Causehypotention
c)Canbeusedtoprovideanesthesiaforintraabdominalsurgery
d)Canbegivenonlybyretrocrural(classic)approach
CorrectAnswer-D
Ans.is'd'i.e.,Canbegivenonlybyretrocrural(classic)approach
Celiacplexusblockcanbedonebyfollowingthreeapproaches
:
Retrocrural(classic)approach,anterocruralapproachand
splanchnicnerveblock.
Seeexplanation-4ofsession-8ofAnaesthesiaofAllIndia2014-15
patternofthisbook.

84.Structurecrossingdorsalsurfaceof
ischialspineareA/E:
a)Internalpudendalvessel
b)Pudendalnerve
c)Obturatornerve
d)Nervetoobturatorinternus
CorrectAnswer-C
C.i.e.Obturatornerve
Psoasmajor,iliacus&pectineusmuscles,femoralvesselsand
nerve,femoralbranchofgenitofemoralnerve,lateralcutaneous
nerveofthighandlymphatics
passbelowinguinalligament.
'PIN'structuresi.e.Pudendalnerve,InternalPudendalvessels,
Nervetoobturatorinternuscomeoutofgreatersciaticforamen,
crossthedorsalsurfaceofischialspine&enterintolessersciatic
foramen.


85.Inbladderinjury,painisreferredtoall
except?
a)Upperpartofthigh
b)Lowerabdominalwall
c)Flank
d)Penis
CorrectAnswer-C
Ans.is'c'i.e.,Flank[RefB.D.C.VolII6th/ep.375;Clinical
Anatomy-912]
Painfibersfrombladderpassthroughbothparasympatheticand
sympatheticpathwayandenterT11-L2andS2-S4cordsegments.
Hencereferredpainisfeltinthelowerpartofanteriorabdominal
wall(hypogastrium),upperpartoffrontofthighs,scrotumorlabium
majus,penisorclitoris,andperineum.

86.StructurenotseenatL3level?
a)Iliacvessels
b)Aorta
c)Coeliactrunk
d)IVC
CorrectAnswer-C
Ans.is'C'i.e.,Coeliactrunk
CoeliactrunkisatT12-L1level.
ThetransversesectionatthelevelofL3showslowerabdominal
organs.

87.Spleenextendsfrom?
a)5thto9thrib
b)9thto11thrib
c)2ndto5thrib
d)11thto12thrib
CorrectAnswer-B
Ans.is'b'i.e.,9thto11thrib[RefBDCVol.IISthIep.431]
Surfacemarkingofspleen
1. Itismarkedontheleftsideoftheback,withitslongaxis
correspondingwiththatofthe10shrib.
2. Theupperbordercorrespondstoupperborderofrib,andthelower
bordertothelowerborderofthe11"'rib.
3. Medialendlies4-5cmfromthemidline,andthelateralendonthe
midaxillaryline.

88.Whichofthefollowingistrueabout
coeliacplexusblock?
a)LocatedretroperitoneallyatthelevelofL3
b)Usuallydoneunilaterally
c)Usefulforthepainfulconditionsoflowerabdomen
d)Mostcommonsideeffectisdiarrheaandhypotension
CorrectAnswer-D
Answer-D(Mostcommonsideeffectisdiarrheaandhypotension)
CeliacPlexusBlock:
LocatedretroperitoneallyatthelevelofLl
Usuallydonebilaterallv
Usefulforthepainfulconditionsofupperabdomen
Mostcommonsideeffectisdiarrheaandhvpotension

89.Ovarianfossaisformedbyallexcept?
a)Obliteratedumbilicalartery
b)Internaliliacartery
c)Ureter
d)Roundligamentofovary
CorrectAnswer-D
Eachovaryliesinovarianfossaonlateralpelvicwallwhichis
bounded:?

1. Anteriorly:Obliteratedumbilicalartery
2. Posteriorly:Ureterandinternaliliacartery

90.Allarebranchesoftheinferiormesenteric
arteryexcept?
a)Leftcolic
b)Sigmoidalartery
c)Middlerectal
d)Superiorrectal
CorrectAnswer-C
Ans.'C'i.e.,Middlerectal
Inferiormesentericarterybranches:
1. Leftcolicartery:Itdividesintodescendingandascendingbranches.
2. Sigmoidalarteries:Theseare2or3innumberandsupply
descendingandsigmoidcolon.
3. Superiorrectalartery:Itisacontinuationofinferiormesenteric
arteryinthelesserpelvisandanastomoseswithbranchesofthe
middleandinferiorrectalarteries.

91.Lymphaticdrainageoflateralwallof
nose
a)Submandibularnodes
b)Retropharyngealnodes
c)Deepcervicalnodes
d)Alloftheabove
CorrectAnswer-D
Ans.is'd'i.e.,Alloftheabove
LymphaticdrainageofnoseisBYAnteriorhalfofnasalcavity(Both
septumandlateralwall)-sSubmandibularnodes
Posteriorhalfofnasalcavity(Bothseptumandlateral
wall)Retropharyngealnodesandupperdeepcervicalnodes.

92.Bartholinglandsituatedin?
a)Superficialperinealpouch
b)Deepperinealpouch
c)Inguinalcanal
d)Ischiorecalfossa
CorrectAnswer-A
Superficialperinealpouch

93.Lymphaticdrainageofcervixisto
a)Iliaclymphnodes
b)Paraaorticlymphnodes
c)Superficialinguinallymphnodes
d)Deepinguinallymphnodes
CorrectAnswer-A
A.i.e.Iliaclymphnodes

94.Maximumcontributiontotheflooroforbit
isby:
a)Maxillary
b)Zygomatic
c)Sphenoid
d)Palatine
CorrectAnswer-A
Ans.A.Maxillary
Themaxillaearethelargestofthefacialbones,otherthanthe
mandible,andjointlyformthewholeoftheupperjaw.Eachbone
formsthegreaterpartofthefloorandlateralwallofthenasalcavity,
andoftheflooroftheorbit
"Orbitalsurfaceofmaxillaissmoothandtriangular,andforms
mostoftheflooroftheorbit"
Alsoknow:
Maxillaisalsothemostcommonlyfracturedboneoforbitalfloor.
Thefloor(inferiorwall)isformedbytheorbitalsurfaceofmaxilla,the
orbitalsurfaceofZygomaticboneandtheorbitalprocessofpalatine
bone
Thesevenbonesthatarticulatetotheorbitare
1. Frontalbone
2. Lacrimalbone
3. Ethmoidbone
4. Zygomaticbone
5. Maxillarybone
6. Palatinebone
7. Sphenoidbone

95.Posteriorcommunicatingarteryabranch
of
a)Internalcarotid
b)Externalcarotid
c)Middlecerebral
d)Posteriorsuperiorcerebellar
CorrectAnswer-A
Ai.e.Internalcarotid

96.Numberofvertebraeisusuallyconstantin
a)Cervical
b)Thoracic
c)Lumbar
d)Sacral
CorrectAnswer-A
Ans.a.CervicalAuthorpleaseprovide
*IllustratedEncyclopediaofHumanAnatomicVariation:OpusV:
SkeletalSystems:Vertebralcolumn;NumericalVariationin
VertebralColumnbyRonaldA.Bergman,PhD;AdelK.Afifi,MD,
MS;RyosukeMiyauchi,MD.
*Theusualgroupingformulaof7cervical,12thoracic,5lumbar,5
sacral,and4coccygealvertebraeisfoundinonlyabout
*ThecervicalregionisreportedtobethemostconstantQ,the
coccygealthemostvariable20%ofindividualsstudied.
*Thenumberofelementsofthevertebralcolumnhasbeenreported
tovarybetween32and35.Additiontoagroupisfrequentlyseen,
whichoccursthroughthereductioninnumberofvertebraeofan
adjacentgroup,thetotalnumberbeing
*Thelocationofsuchavertebraispredominatelyattheendsofthe
columnandatthelevelsoftransitionbetweenitsregions.Thus,
sacralizationofthefifthlumbar,lumbar-likearticularprocessesinthe
elevenththoracic,andthoraciccostalfacetsontheseventhcervical
areobserved.
*unchanged.Inthisvariation,thevertebraaddedisusually
intermediateinformbetweentheadjacentgroups.
*Thelevelsoftransitionmaybeshiftedcephalad,resultingin23
mobilevertebrae,orshiftedcaudad,resultingin25presacral

vertebrae.Suchvariationsmayoccurin2-11%ofthepopulation
*Thenumberofvertebraecomprisingthesacrummaybeincreased
tosix,resultingfromthefusionofthefirstcoccygeal(50%inwhites,
30%inNegroes)or,lessoften,ofthelastlumbar(sacralization)(8%
inwhites,11%inNegroes);oritmaybeincreasedtoseven,resulting
fromthefusionofthefirstcoccygealandthelastlumbar(4%in
whites,1.5%inNegroes).Thenumbermaybereducedtofour,
apparentlybythelumbarizationofthefirstsacral(0.4%inwhites,
1.5%inNegroes).
Numberofvertebrae
*Mostconstant:Cervicalregion?
*Mostvariable:coccygealregion?

97.Ophthalmicarteryisabranchof?
a)CavernouspartofICA
b)CerebralpartofICA
c)MCA
d)Facialartery
CorrectAnswer-B
Thecommoncarotidarteriesbifurcateatthelevelofthethyroid
cartilageintotheexternalandinternalcarotidarteries.Theexternal
carotidarterysendsbranchestotheneckandface,whereasthe
internalcarotidarteryascendstothebaseoftheskull,enteringthe
carotidcanal.Uponexitingthecarotidcanal,theinternalcarotid
arterycourseshorizontallyovertheforamenlacerumandentersthe
cavernoussinusand,afterturningsuperiorly,dividesintoitsterminal
branches.
Internalcarotidartery
Itisthemainarterysupplyingstructuresinsidethecranialcavityand
orbit.Itisdividedinto4parts:?
*Cervicalpart:-Itextendsfromupperborderofthyroidcartilageto
thebaseofskull.Thispartgivesnobranch.
*Petrouspart:-Itliesinbonycarotidcanalinthepetroustemporal
bone.Itgivestwobranches(i)Caroticotympanic,and(ii)pterygoid.
*Cavernouspart:-Itrunsthroughthemedialwallofcavernous
sinus.Itgivesthreebranches:(i)Meningealbranch,(ii)
hypophysealbranchand(iii)cavernousbranch.
*Cerebralpart:-Itisrelatedtoinferiorsurfaceofcerebrum.Itgives
followingbranches:(i)Ophthalmicartery,(ii)posterior
communicatingartery,(iii)anteriorchoroidalartery,(iv)anterior
cerebralarteryand(v)middlecerebralartery.


98.Whatistrueaboutchordatympani?
a)Postganglionicsympathetic
b)Preganglionicsympathetic
c)Preganglionicparasympathetic
d)Postgalglionicparasympathetic
CorrectAnswer-C
Chordatympaniarisesfromintratemporalpart(infallopioncanal)of
facialnerve.
*Itcarriespreganglionicsecretomotorfibers(notpostganglionic)to
submandibularandsublingualglands.
*Itjoinslingualnerveininfratemporalfossa.
*Itcarriestastesensationsfromanterior2/3oftongue.

99.Eustachiantubeopensinmiddleearin?
a)Floor
b)Anteriorwall
c)Superiorwall
d)Posteriorwall
CorrectAnswer-B
Ans.is'b'i.e.,Anteriorwall
Themiddleearisshapedlikeacube.
Whenseeninthecoronalsection,thecavityofthemiddleearis
biconcave.
Theboundariesofthemiddleearareasfollows:
1.Roofortegmentalwall
Separatesthemiddleearfromthemiddlecranialcavity.
Itisformedbyathinplateofbonecalledtegmentympani.
2.Floororjugularwall
Formedbyathinplateofbonewhichseparatesthemiddleearfrom
thesuperiorbulboftheinternaljugularvein.
Theflooralsopresentsthetympaniccanaliculuswhichtransmitsthe
tympanicbranchoftheglossopharyngealnerve.
3.Anteriororcarotidwall
Theuppermostpartbearstheopeningofthecanalofthetensor
tympani.
Themiddleparthastheopeningoftheauditorytube.
Theinferiorpartofthewallisformedbyathinplateofbonewhich
formstheposteriorwallofthecarotidcanal.Thisplateseparatesthe
middleearfromtheinternalcarotidartery.
4.Posteriorormastoidwall
Superiorly,istheaditusthroughwhichtheepitympanicrecess

communicateswiththemastoidantrum.
Belowitisthefossaincudiswhichlodgestheshortprocessofthe
incus.
Belowitisthepyramidgivingattachmenttothetendonofstapedius.
Verticalpartofthefacialcanalforfacialnerve
5.Lateralormembranouswall
Tympanicmembranealongwiththetympanicringandsulcus.
6.Medialorlabrynthinewall
Promontory-roundedbulgeproducedbythefirstturnofthe
cochlea.
Ovalwindow-itisposterosuperiortothepromontory.Itisclosedby
thefootplateofthestapes.
Horizontalpartofthefacialcanal-runsjustabovetheovalwindow.
Roundwindow-posteroinferiortothepromontory.Itisclosedbythe
secondarytympanicmembrane.
Processuscochleariformis-formsapulleyforthetendonofthe
tensortympani.
Prominenceofthelateralsemicircularcanal-abovethefacialcanal.

100.Rouvierenodesaresituatedin?
a)Nasopharynx
b)Oralcavity
c)Retropharynx
d)Clavicularnodes
CorrectAnswer-C
Ans.is'c'i.e.,Retropharynx
TheRouviere'snodeisthemostsuperiorofthelateralgroupofthe
retropharyngeallymphnodes,andisfoundatthebaseoftheskull.
Thekrewse'snodesarelymphnodessituatedinthejugular
foramen.Enlargementofthesenodescompressoncranialnerves
IX,XandXI,causingjugularforamensyndrome.

101.Passavantsridgeisformedby?
a)Palatoglossus
b)Superiorconstrictor
c)Salpingopharyngeus
d)Palatopharyngeus
CorrectAnswer-D
Ans.is'd'i.e.,Palatopharyngeus
Pharynxhastwogroupofmuscles:?
*Intrinsicmuscles:-Stylopharyngeous,salpingopharyngeous,
palatopharyngeous.
*Extrinsicmuscles:-Superiorconstrictor,middleconstrictor,inferior
constrictor.
-Allmusclesofpharynxaresuppliedbycranialaccessorythrough
branchesofvagusviapharyngealplexusexceptstylopharyngeus
whichissuppliedbyglossopharyngeal.
-Inferiorconstrictormusclehastwoparts:-(i)Thyropharyngeous
withobliquefibres,and(ii)Cricophatyngeouswithtransversefibres.
-Betweenthesetwopartsofinferiorconstrictorexistsapotential
gapcalledKillan'sdehiscence.Itisalsocalledthegatewaytotear
asperforationcanoccuratthissiteduringesophagoscopy.Itisalso
thesiteforherniationofpharyngealmucosaincaseofpharyngeal
pouch.
-UpperfibersofpalatopharyngeusconstitutethePassavant's
muscle
whichoncontractionraisesaridgecalledPassavant'sridge
onposteriorwallofnasopharynx.

102.Lowerlimitofretropharyngealspaceis
at?
a)C7
b)Bifurcationoftrachea
c)4thesophagealconstriction
d)None
CorrectAnswer-B
Ans.is'b'i.e.,Bifurcationoftrachea
Retropharyngealspaceisdividedintotwolateralspaces(spaceof
gillette)byafibrousband.
Retropharyngealspaceislimitedabovebythebaseofskulland
belowwherethealarfasciafuseswithbuccopharyngealfasciaatthe
levelofT4andcarina(bifurcationoftrachea).

103.Killiandehiscenceisin?
a)Superiorconstrictor
b)Inferiorconstrictor
c)Middleconstrictor
d)None
CorrectAnswer-B
Ans.is'b'i.e.,Inferiorconstrictor
Inferiorconstrictormusclehastwoparts:-(i)Thyropharyngeouswith
obliquefibres,and(ii)Cricopharyngeouswithtransversefibres.
Betweenthesetwopartsofinferiorconstrictorexistsapotentialgap
calledKillan'sdehiscence.Itisalsocalledthegatewaytotearas
perforationcanoccuratthissiteduringesophagoscopy.Itisalsothe
siteforherniationofpharyngealmucosaincaseofpharyngeal
pouch.

104.Sensorysupplyofthepalmisfrom
whichnerves-
a)MediannerveandRadialnerve
b)Radialnerveandulnarnerve
c)UlnarnerveandMediannerve
d)MusculocutaneousnerveandRadialnerve
CorrectAnswer-C
Ans.is'c'i.e.,UlnarnerveandMediannerve
(RelBDC5'h/eVoLIp.108-111)
OnPalmside:
Lateral2/3ofthepalmandlateralthreeandhalffibersMedian
nerve.
Medial1/3ofthepalmandmedialoneandhalffingersUlnar
nerve.

105.

Whichofthefollowingisnotthepartof
ethmoidbone?

a)Aggernasi
b)Cristagalli
c)Uncinateprocess
d)Inferiorturbinate
CorrectAnswer-D
Ans.d.Inferiorturbinate
Inferiorturbinateisnotthepartofethmoidbone.
`Lateralnasalwallhas3bonyprojectionscalledasturbinatesor
conchae.Frombelowupwards,theyareinferior,middleand
superiorturbinates.Theinferiorturbinateisaseparatebone,while
restoftheturbinatesarepartofethmoidallabyrinths.'
Theaggernasiaircells,arethemostanteriorethmoidalaircells,
lyinganterolateralandinferiortothefrontoethmoidalrecessand
anteriorandabovetheattachmentofthemiddleturbinate.Theyare
locatedwithinthelacrimalboneandthereforehaveaslateral
relationstheorbit,thelacrimalsacandthenasolacrimalduct.'
Thecristagalliisamedianridgeofbonethatprojectsfromthe
cribriformplateoftheethmoidbone.Itiswherethefalxcerebri
attachesanteriorlytotheskull.Theolfactorybulbslieoneitherside
ofthecristagalliontopofthecribriformplate.'
Intheethmoidbone,acurvedlamina,theuncinateprocess,projects
downwardandbackwardfromthispartofthelabyrinth;itformsa
smallpartofthemedialwallofthemaxillarysinus,andarticulates
withtheethmoidalprocessoftheinferiornasalconcha.'

106.Whichofthefollowingisnotsuppliedby
theanteriordivisionofmandibularnerve
(V3)?

a)Temporalis
b)Medialpterygoid
c)Lateralpterygoid
d)Masseter
CorrectAnswer-B
Bi.e.Medialpterygoid
Temporalis,messeterandlateralpterygoidmusclesQaresupplied
byanteriordivisionofmandibularnervewhereasmedialpterygoid
muscleQ
issuppliedbythemaintrunkofmandibularnerve.

107.Nucleusambiguusisnotassociatedwith
whichcranialnerve:
a)X
b)XI
c)IX
d)XII
CorrectAnswer-D
Ans.D:XII
NucleusAmbiguus
Function:
*Motorinnervationofipsilateralmusclesofthesoftpalate,pharynx,
larynxandupperesophagus.
Pathway:
*Axonsofmotorneuronsinthenucleusambiguuscoursewiththree
cranialnerves:C.N.IX(glossopharyngeal),C.N.X(vagus),C.N.XI
(therostralorcranialportionofspinalaccessory)toinnervate
striatedmusclesofthesoftpalate,pharynx,larynxandupper
esophagus.
Deficits:
*Lesionofnucleusambiguusresultsinatrophy(lowermotor
neuron)andparalysisofinnervatedmuscles,producingnasal
speech,dysphagia,dysphonia,anddeviationoftheuvulatowardthe
oppositeside(strongside).
*NoaffectionoftheSternocleidomastoidorTrapezius.These
musclesareinnervatedbycellsintherostralspinalcord(caudal
portionC.N.XI).

108.

WhichofthefollowingisNOTabranchof1stpartofmaxillaryartery?
a)Middlemeningealartery
b)Accessorymeningealartery
c)Inferioralveolarartery
d)Greaterpalatineartery
CorrectAnswer-D
Branchesofmaxillaryartery:

Themaxillaryarteryconsistsofthreeparts;mandibularpart,
pterygoidpart,pterygopalatinepart.
Branchesofmandibularpart:
Inferioralveolarartery
Middlemeningealartery
Deepauricularartery
Anteriortympanicartery
Occasionallyanaccessorymeningealbranch.
Branchesofpterygoidpart:
Massetericartery
Deeptemporalbranches
Pterygoidbranches
Buccalartery
Branchesofpterygopalatinepart:
Posteriorsuperioralveolarartery
Infraorbitalartery
Descendingpalatineartery
Greaterpalatineartery
Lesserpalatineartery
Sphenopalatineartery

Lateralposteriornasalarteries
Posteriorseptalbranches

109.Allofthefollowingaretrueabout
locationofoticgangliaexcept:
a)Inferiortoforamenovale
b)Lateraltotensorvelipalatini
c)Lateraltomandibularnerve
d)Anteriortomiddlemeningealartery
CorrectAnswer-C
Ans:C.Lateraltomandibularnerve
(RefGray41/ep552.40/ep543)
Mandibularnervelieslateraltooticganglion.
Ie.,Ganglionliesmedialtomandibularnerve.
Oticganglion:
Small,oval,flatreddish-greyganglion.
Situatedjustbelowforamenovate.
Peripheralparasympatheticganglionlocatedintheinfratemporal
fosse.
Functionallyassociatedwithglossopharyngealnerve&innervates
parotidglandforsalivation.
Connectedtochordatympaninerve&tonerveofpterygoidcanal.
Pathwaysprovideanalternatepathwayoftastefromanteriortwo-
thirdsoftongue.

110.

Cranialpartofaccessorynervesuppliesallpalatalmuscles,EXCEPT?
a)Palatoglossus
b)Palatopharyngeus
c)Tensorvelipalatini
d)Tensorvelitympani
CorrectAnswer-C
Thecranialrootoftheaccessorynerveissmallerthanthespinal
root.Itexitstheskullthroughthejugularforamenandunitesfora
shortdistancewiththespinalroot.Itsfibersinnervatethe
pharyngealandpalatalmuscles,excepttensorvelipalatini.
Becausethecranialpartofaccessorynerve(CNXI)leavesthe
jugularforamenasjoiningtheCNX,itissometimesconsideredpart
oftheplexusaswell
Thetensorvelipalatiniissuppliedbythemedialpterygoidnerve,a
branchofmandibularnerve,thethirdbranchofthetrigeminalnerve
-theonlymuscleofthepalatenotinnervatedbythepharyngeal
plexus

111.Passavantridge?
a)Superiorconstrictorandpalatopharyngeus
b)Inferiorconstrictorandpalatopharyngeus
c)Superiorconstrictorandpalatoglossus
d)Inferiorconstrictorandpalatoglossus
CorrectAnswer-A
Passavantridge
Nearthesuperiormarginofpharynx,afewfibresofsuperior
constrictorblendwithabandofmusclefibresbelongingtothe
palatopharyngeusmuscle.
Thesefusedfibresformabandorringaroundtheposteriorwalland
sidewallsofthenasopharyngealisthumus.
Whenthesoftpalateiselevatedthismusclebandappearsasa
ridgeisknownaspassavant'sridge.

112.Whichofthefollowinglaryngeal
cartilageishyaline?
a)Epiglottis
b)Comiculate
c)Cricoid
d)Cuneiform
CorrectAnswer-C
Ans.is'c'i.e.,Cricoid
Hyalinecartilages
Elasticcartilages(donotossify)
Thyroidcartilage
Epioglottis
Cricoidcartilage
Corniculate
Basalpartofarytenoidcartilage Cuneiform
Processesofarytenoid

113.Little'sareais?
a)Anteroinferiorlateralwall
b)Anteroinferiornasalseptum
c)Posteroinferiorlateralwall
d)Posteroinferiornasalseptum.
CorrectAnswer-B
Ans.is'b'i.e.,Anteroinferiornasalseptum
Little'sareaissituatedintheanteriorinferiorpartofnasalseptum,
justabovethevestibule.
Woodruff'splexusissituatedintheposteriorinferiorpartoflateral
wall

114.Maxillarybonedoesnotarticulatewith:
a)Ethmoid
b)Sphenoid
c)Frontal
d)Lacrimal
CorrectAnswer-B
Ans:B.Sphenoid
(RefGray's41/ep484,40/ep473-476)
Maxillarybonedoesnotarticulatewithsphenoid.
Articulationofmaxilla:
Eachmaxillaarticulateswithninebones:
Twoofcranium:Frontal&ethmoid.
Sevenoftheface:Nasal,zygomatic,lacrimal,inferiornasal
concha,palatine,vomer&adjacentfusedmaxilla.
Sometimesarticulateswithorbitalsurface&withlateralpterygoid
plateofsphenoid.

115.Primaryandsecondarypalatesare
dividedby
a)Greaterpalatineforamen
b)Canineteeth
c)Alveolararch
d)Incisiveforamen
CorrectAnswer-D
D.i.e.Incisiveforamen
Theincisiveforamenisdividinglandmarkbetweentheprimary&
secondarypalateQ;
andanterior&posteriorcleftdeformities

116.Parotidductopensoppositeto:
a)Upper1stmolar
b)Upper2ndmolar
c)Upper2ndpremolar
d)Upper1stpremolar
CorrectAnswer-B
Ans.B:Upper2ndmolar

117.Multiplesinusesfrominfectionofgreat
toeismainlycausedby:
September2007

a)Tuberculosis
b)Actinomycetes
c)Trichosporum
d)Histoplasmosis
CorrectAnswer-B
Ans.B:Actinomycetes
Mycetomaisachroniclocalizedgranulomatousinflammatorylesion
commoninthetropicsandsub-tropics.
MultiplesiAuses,softtissueswellinganddischargeofcoloured
granulescharacterizeamycetomafoot.Chronicsinusesdischarging
granulesarecausedby(1)Eumycetes(truefungi),(2)
Schizomycetes,whichincludes
Actinomycosis(aerobichigherfilamentousbacteria)and
Botryomycosis(bacterialinfection).
-ActinomycosisisoftencausedbyActinomaduraorNocardia
specieswhereinitgainsentryafterpenetratingtrauma(splinter,
gravelorthornprick).
Youngmalesarecommonlyaffectedand75%ofthelesionsoccurin
thelowerlimbs.
Thediseaseslowlyprogressesfromanoduletomultiplesinuses
dischargingcolouredgranules.Involvementofboneislatefollowing
dermalandsofttissuespread.
-Botryomycosis(amisnomer),achronicsuppurativeinfectionis
causedbybacteriainvolvingskinorviscera.Thisrarecondition
mimicsadeepmycoticinfectionandisoftencausedby

mimicsadeepmycoticinfectionandisoftencausedby
StaphylococcusorPseudomonasspeciesofbacteria.Thefeetand
handsarecommonlyaffectedandpenetratingtraumamaybea
predisposingfactor.Itusuallypresentsasanodule,sinusorulcer,
whichisinitiallylocalized.
Thediagnosisofactinomycosisisbasedonclinicalfindings,
demonstrationofcharacteristicgranulesandcultureoftheorganism
fromadeeptissuebiopsy.
Immunologicalstudies(counterimmunoelectrophoresisandELISA)
arealsoused.Botryomycosisisdiagnosedbasedonapositive
gram'sstainandculture;negativefungalculturesanddemonstration
ofthecharacteristicbotryomycoticgranulesathistopathologyfroma
deepbiopsy.
Inendemicareas,subcutaneousswellingwithsinusesshouldbe
consideredasmycetomaunlessprovedotherwise.Thedifferential
diagnosisincludeschronicosteomyelitis,tuberculosisandchronic
abscesses.

118.Risoriusisamuscleof?
a)Mastication
b)Deglutition
c)Facialexpression
d)Eyemovement
CorrectAnswer-C
Ans.is'c'i.e.,Facialexpression[RefBDCVol.III,5th/ep.57]
Therisorius(alsorisoriusmuscle,latin:musculusrisorius)isa
muscleoffacialexpressionlocatedlaterallytothemouthopening,
whichpullstheangleofthemouthlaterally.
*Origin
-Therisoriusoriginatesfromthemassetericfascia.
*Insertion
-Therisoriusinsertsintotheskinoftheangleofthemouth.
*Action
-Uponactivationtherisoriuspullstheangleofthemouthlaterally.
-Contractionsoftherisoriusmuscleproducefacialexpressionof
pleasureandlaughter.
*Innervation
-Therisoriusisinnervatedbythebuccalbranchofthefacialnerve
(CNVII).
*Bloodsupply
-Therisoriusismainlysuppliedbythesuperiorlabialbranchofthe
facialartery.

119.Musclecausingflexionofhip?
a)Bicepsfemoris
b)Psoasmajor
c)Gluteusmaximus
d)TFL
CorrectAnswer-B
Ans.is'b'i.e.,Psoasmajor

120.

Thethyrocervicaltrunkisabranchofwhich
partofsubclavianartery?

a)1st
b)2nd
c)3rd
d)None
CorrectAnswer-A
Ans.'a'i.e.,1st
Branchesofthesubclavianartery:?
1. 1stPartVertebralartery,internalthoracicartery,thyrocervical
trunk,andonleftsidecostocervicaltrunk.
2. 2ndPartOnrightsidecostocervicaltrunk.
3. 3rdPartDorsalscapularartery.

121.Biphasicactionpotentialofmixednerve
except?
a)Allornonephenomenon
b)Twoormorepositivepeaks
c)Refractoryperiod
d)Recordedonsurface
CorrectAnswer-B
Ans.is'b'i.e.,Twoormorepositivepeaks
Actionpotentialwhenrecordedbyputtingtwoelectrodesonthe
surfaceofaneuron(insteadofputtingonsurfaceandone
intracellularly),showsabiphasicresponse,i.e.Biphasicaction
potential.
Asthewaveofdepolarizationreachedthefirstelectrode,this
electrodebecomesnegativeandanupwarddeflection(Peak)is
recorded.
Vagus,Glossopharyngeal,Facialaremixednerve.

122.Nucleusfasciculatusisseenin?
a)Frontallobe
b)Medulla
c)Temporallobe
d)Midbrain
CorrectAnswer-B
Ans.is'b'i.e.,Medulla[RefQuantitativeHumanphysiology:An
introductionp.327]
Ithasnotbeenmentionedinanytextbook.
Butaccordingtotheabovementionedreferencenucleus
fasciculatusistheothernameofnucleuscuneatus.
"Thesensoryfibersofdorsalcolumntravelintracts,fasciculus
gracilisandfasciculusCuneatusintheCordandthesefibersmake
synapseswithsecondorderneuronsinthenucleusgracilisandthe
nucleusfasciculatus".--QuantitativeHumanphysiology.
Nucleusgracilisandnucleusfasciculusarefoundinthemedulla.

123.Whichofthefollowingisanoperculated
sulcus?
a)Calcarine
b)Collateral
c)Lunate
d)Central
CorrectAnswer-C
Ans.is'c'i.e.,Lunate
Operculatedsulcusseparatesbyitslipsintotwoareasand
containsathirdareainthewallsofthesulcuse.g.lunatesulcusis
anoperculatedsulcus,separatingthestriateandparastriateareas.
AxialPosteriorpartofcalcarinesulcus
LimitingCentralsulcusAnteriorpartofcalcarinesulcus
OperculatedLunatesulcus
CompleteCollateralsulcus,Anteriorpartofcalcarinesulcus

124.Allthefollowingarecharacteristicsof
oculomotornerveexcept:
a)Carriesparasympatheticnervefibres
b)Suppliesinferiorobliquemuscle
c)Entersorbitthroughtheinferiororbitalfissure
d)Causesconstrictionofpupil
CorrectAnswer-C
Ci.e.Entersorbitthroughtheinferiororbitalfissure
Oculomotornerveentersorbitthroughthesuperiororbitalfissure.
Thevisceralmotorcomponentcontrolsparasympatheticinnervation
(nervesrelatedtoinvoluntaryactions)oftheciliarymusclesand
constrictorpapillae,aidinginaccommodationandpupillarylight
reflexes.
TheIIIcranialnervesuppliesallextraocularmusclesexceptLateral
rectusandsuperiorobliquemuscle.

125.TrueaboutCorpuscallosum:
a)Unitefarareaoftwosidesofbrain
b)Connecttwofrontallobe
c)Unitetwohemisphere
d)All
CorrectAnswer-D
Ai.e.Unitefarareaoftwosidesofbrain;Bi.e.Connecttwofrontal
lobe;Ci.e.Unitetwohemisphere

126.Whatarethecellularcontentsofcerebellarcortex?
a)Corticalcells
b)Glomuscells
c)Principlecells
d)Intercalatedcells
CorrectAnswer-A
Ans.ACorticalcells.
CELLSINCEREBRALCORTEX:
Corticalcells:
Majorityarepyramidalcells
Pyramidalcells-"Sinequanon"forcerebralcortex
Axonsofpyramidalcellsleavethecortex
Formsdescendingtract(e.g.Corticospinal,Corticobulbaretc).

127.Allarelateralbranchesofabdominal
aorta,EXCEPT
a)Righttesticularartery
b)Leftrenalartery
c)Inferiormesentericartery
d)Middlesuprarenalartery
CorrectAnswer-C
C.i.e.Inferiormesentericartery
OvarianorTesticulararteryislateralbranchofabdominalaortaand
uterinearteryisabranchofinternaliliacartery(anteriordivision).
Lateralbranchesofabdominalaortaare-Inferiorphrenic,Middle
Suprarenal,Renal&Gonadal(testicularorovarian)arteries.
Mnemonic-"InferiorMSRuinGonads"

128.Chamberlainslineis?
a)Palatetoocciput
b)Palatetotemporal
c)Palatetoforamenmagnum
d)Palatetoparietal
CorrectAnswer-C
Ans.is'c'i.e.,Palatetoforamenmagnum[Ref:Atlasof
radiographicmeasurement]
TheChamberlainlineisdrawnfromtheposteriorsurfaceofthehard
palatetothetipoftheopisthion(posterioraspectoftheforamen
magnum).
Itisusedtomeasurethedistanceofhowmuchtheodontoidtip
extendsabovethisline.Ifthetipofthedensextends>3mmabove
thislinethenithelpstorecognizethepresenceofbasilar
invagination(acraniocervicaljunctionabnormalitywherethetipof
thedensprojectupintotheforamenmagnum)

129.Osseocartilagenousjunctionispresent
at?
a)Nasion
b)Rhinion
c)Radix
d)Columella
CorrectAnswer-B
Ans.is'b'i.e.,Rhinion[RefTextbookofgeneralanatomyp.10]
NasionThedepressionatthejunctionofnosewithforehead.
RhinionThepointlocatedattheosseocartilagenousjunction
overthedorsumofthenose.
RadixJunctionbetweenthefrontalboneandnasalbone.
ColumellaColumnbetweenthenostrilsatthebaseofthenose..

130.Primordialgermcellsarederivedfrom:
a)Neuralcrest
b)Genitalridge
c)Somatopleuriticmesoderm
d)Yolksac
CorrectAnswer-D
Formationofprimordialgermcells
Structuresderivedfromneuralcrestareneuronsofspinalposterior
(dorsal)nerverootganglia,neuronsofsensorygangliaofthe5to
10thcranialnerves,neuronsandsatellitecellsofsympathetic
gangliaetc.
Intheregionwheretestesistodevelop,thegerminalepithelium
getsthickenedandisknownasgenitalridge.
Thecellsofgerminalepitheliumproliferateandformssexcords
whichgetsconvertedintomedullarycordsandfinallygetscanalized
toformseminiferoustubules
Chorionisformedbytheparital/somatopleuricextraembryonic
mesoderm(ontheinside)andtheoverlyingTrophoblast
Thecellsoftheovariesandthetestes,fromwhichgermcellsare
formed,arebelievedtobesegregatedearlyinthelifeoftheembryo.

131.WhichofthefollowingdoesNOT
stimulateperipheralchemoreceptors:
a)Hypoxia
b)Hypocapnia
c)Acidosis
d)Lowperfusionpressure
CorrectAnswer-B
Bi.e.Hypocapnia

132.Peripheralandcentralchemoreceptors
maybothcontributetotheincreased
ventilationthatoccursasaresultof
whichofthefollowing?

a)Adecreaseinarterialoxygencontent
b)Adecreaseinarterialbloodpressure
c)Anincreaseinarterialcarbondioxidetension
d)Adecreaseinarterialoxygentension
CorrectAnswer-C
Thecentralchemoreceptorslocatedonorneartheventralsurfaceof
themedullacauseanincreaseinventilationinresponsetoan
increaseinPacO2andtoalesserextenttoadecreaseinarterialpH
becausethebloodbrainbarrierisrelativelyimpermeableto
hydrogenions.
Theperipheralchemoreceptorsinthecarotidbodiescausean
increaseinventilationinresponsetoanincreaseinPacO2a
decreaseinarterialpH,andadecreaseinPaO2.Neitherthecentral
chemoreceptorsnorthecarotidbodiesarestimulatedbyadecrease
inarterialbloodpressureorO2content.

133.Transpulmonarypressureisthedifferencebetween:
a)Thebronchusandatmosphericpressure
b)Pressureinalveoliandintrapleuralpressure
c)Atmosphereandintrapleuralpressure
d)Atmosphereandintraalveolarpressure
CorrectAnswer-B
Transpulmonarypressureisthepressuredifferencebetween
alveolarpressureandintrapleuralpressure.Beforethestartof
inspirationorattheendofexpirationitisabout+5cmH2O.Positive
transpulmonarypressurekeepsthealveoliopen.
Intrapleuralpressureisthepressurebetweentwolayersofpleura.
Itisabout-5cmH2Obeforethestartofinspirationorattheendof
expiration.
Alveolarpressureisthepressurewithintheterminalairspaces.It
isthesumofpleuralpressureandelasticrecoilpressureofthelung.
Itisatmosphericbeforethestartofinspirationorattheendof
expiration.
Transthoracicpressureisthepressuredifferencebetween
alveolarpressureandpressureatthebodysurface.
Ref:FundamentalsofRespiratoryPhysiologyByASChakrabarty,
Page32

134.Differenceintheamountof02inspired
andCO2expired?
a)20ml/min
b)50ml/min
c)75ml/min
d)100ml/min
CorrectAnswer-B
Ans.B.50ml/min
250mlofO2entersthebodyperminuteand200mlofCO2is
excreted.

135.Bohreffectisdescribedas:
September2009
a)DecreaseinCO2affinityofhemoglobinwhenthepHofblood
rises
b)DecreaseinCO2affinityofhemoglobinwhenthepHofblood
falls
c)DecreaseinO2affinityofhemoglobinwhenthepHofblood
rises
d)DecreaseinO2affinityofhemoglobinwhenthepHofblood
falls
CorrectAnswer-D
Ans.D:DecreaseinO2affinityofhemoglobinwhenthepHofblood
falls
ThedecreaseinO2affinityofhemoglobinwhenthepHofbloodfalls
iscalledtheBohreffectandiscloselyrelatedtothefactthat
deoxygenatedhemoglobin(deoxyhemoglobin)bindsH+more
activelythandoesoxyhemoglobin.ThepHofbloodfallsasitsCO,
contentincreases,sothatwhenthePCO2rises,thecurveshiftsto
therightandtheP5,,rises.

136.Truestatementrelatingtocomplianceof
lung:
a)Increasedbysurfactant
b)Decreasedinemphysema
c)Atheightofinspirationcomplianceisless
d)Itcanbemeasuredbymeasuringintrapleuralpressureat
differentlungvolume
e)None
CorrectAnswer-A:C:D
Ans.(A)Increasedbysurfactant(C)Atheightofinspiration
complianceisless(D)Itcanbemeasuredbymeasuring
intrapleuralpressureatdifferentlungvolume
[Ref:Ganong25th/629-32,24th/629-33;Guytonllth/473-75;A.K.
Jain5th/437]
Lungcompliance:
Measuredbymeasuringintrapleuralpressureatdifferentlung
volume.
Animportantfactoraffectingthecomplianceofthelungsisthe
surfacetensionofthefilmoffluidthatlinesthealveoli.
Deficiencyofsurfactant-lesscompliance;moresurfactant-more
compliance.
Compliancedecreaseswiththeinflationofthelungsasmore
pressureisrequiredtodistendthealreadydistendedlung.
Thecurveisshifteddownwardandtotheright(complianceis
decreased)bypulmonarycongestionandinterstitialpulmonary
fibrosis;pulmonaryfibrosisisaprogressivediseaseofunknown

causeinwhichthereisstiffeningandscarringofthelung.
Thecurveisshiftedupwardandtotheleft(complianceisincreased)
inemphysema.

137.Venousbloodwithhighhematocritis
seenin?
a)RBChighchloride
b)PlasmahighNa
c)PlasmahighHCO3
d)RBChighK
CorrectAnswer-A
Ans.A.RBChighchloride
Hematocritofvenousbloodisnormally3%greaterthanthatof
arterialblood.

138.
Whichofthefollowingis/areeffectof
increased2,3-DPGonoxygen-
hemoglobindissociationcurve?
a)edaffinityofheamoglobintooxygen
b)edaffinityofhaemoglobintooxygen
c)Leftshiftofoxygen-hemoglobindissociationcurve
d)Rightshiftofoxygen-hemoglobindissociationcurve
e)Nochangeinoxygen-hemoglobindissociationcurve
CorrectAnswer-B:D
Ans.(B)edaffinityofhaemoglobintooxygen(D)Rightshift
ofoxygen-hemoglobindissociationcurve
[Ref:Ganong25th/ep.610-41;Guyton's12'h/ep.j56-57;AKlain
6'h/ep.430]
*Oxygen-hemoglobindissociationcurveis2,3DPGinRBC.
*DPGisanoptionalby-productoftheglycolyticpathway.
*DPGbindswithdeoxygenatedhemoglobinbutnotwithoxygenated
hemoglobin.
*RaisedDPGconcentrationreleasesoxygenfromoxyhemoglobin
byshiftingthefollowingreversiblereactiontotheright.
Mechanism:
*OnemoleculeofDPGbindswithonemoleofdeoxyhemoglobin.
*HenceanincreaseinDPGconcentrationshiftstheoxygen-
hemoglobindissociationcurvetotheright.
*Thus2,3DPGcausesdelivery(unloading)ofO,tothetissues.
*Fetalhemoglobinhasconsiderablylessaffinityfor2,3-DPGthan
doesadulthemoglobinthereforefetalhemoglobinhasagreater

affinityforoxygenthanadulthemoglobin.
Inhumanblood,theaffinityoffetalhemoglobinfor2,3-DPGisonly
about4O%thatofadulthemoglobin.
Thismakesfetalhemoglobinbehaveasif2,3-DPGlevelsarelow.

139.Stabilityofalveoliismaintainedby?
a)Lungcompliance
b)Negativeintrapleuralpressure
c)Increaseinalveolarsurfaceareabythesurfactant
d)Residualairinalveoli
CorrectAnswer-C
Ci.e.Increaseinalveolarsurfaceareabythesurfactant

140.Whichofthefollowingdefinesvital
capacity?
a)Airinlungafternormalexpiration
b)Maximumairthatcanbeexpiratedafternormalinspiration
c)Maximumairthatcanbeexpiratedaftermaximuminspiration
d)Maximumairinlungafterendofmaximalinspiration
CorrectAnswer-C
Ans.C.Maximumairthatcanbeexpiratedaftermaximum
inspiration
Vitalcapacity(VC):
4700ml.
Amountofairthatcanbeexhaledwithmaximumeffortafter
maximuminspiration(ERV+TV+IRV).
Usedtoassessstrengthofthoracicmusclesaswellaspulmonary
function.

141.HerringBreuerreflexisanincreasein?
a)Durationofinspiration
b)Durationofexpiration
c)Depthofinspiration
d)Depthofexpiration
CorrectAnswer-B
Ans.is'b'i.e.,Durationofexpiration
TheHering-Breuerinflationreflexisanincreaseinthedurationof
expirationproducedbysteadylunginflation,andtheHering-Breuer
deflationreflexisadecreaseinthedurationofexpirationproduced
bymarkeddeflationofthelung.

142.Dailypancreaticsecretion?
a)1.5L
b)2.5L
c)5.0L
d)10L
CorrectAnswer-A
Ans.A.1.5L

143.Microcirculationconsistsof?
a)Capillaries
b)Capillariesvenulesandarterioles
c)Aorta
d)Arteriesandveins
CorrectAnswer-B
Ans.,B.Capillariesvenulesandarterioles
Themicrocirculationconsistsofarteriolestovenules,i.e.arterioles,
capillariesandvenules.

144.Gasexchangeintissuestakesplaceat?
a)Artery
b)Capillary
c)Vein
d)Venules
CorrectAnswer-B
Ans.B.Capillary
Siteofgasexchange=Capillaries

145.Allofthefollowingstatementsabout
thirdHeartsound(S3)aretrue,except:
a)Occursduetorapidfillingoftheventriclesduringatrialsystole
b)SeenininConstrictivePericarditis
c)SeeninAtrialSeptalDefect(ASD)
d)SeeninVentricularSeptalDefect(VSD)
CorrectAnswer-A
AnswerisA(Occursduetorapidfillingoftheventriclesduring
atrialsystole)
Thirdheartsoundoccursattheendofearlyrapidfillingphaseofthe
ventriclebutnotatthetimeofatrialsystole.Theheartsound
associatedwithventricularfillingduringatrialsystoleisthefourth
heartsound(S4)
FourthHeartsoundoccursinassociationwithaneffectiveatrial
contraction()(Itispresumablycausedbyin-rushofbloodintothe
ventricleswhentheatriacontractsandhenceitisalsocalledthe
'AtrialHeartSound)
PathologicalThirdHeartSound(S3)maybeassociatedwith
ASDandVSD
`ApathologicalS3isoftenpresentinlargelefttorightshuntsdueto
highflowacrossthemitralvalvewithVSDorpatentductus
arteriosusandwithhighflowacrossthetricuspidvalvewithASD.
Thepresenceofthissoundintheseconditionsdoesnotimply
congestiveheartfailure,andsuchpatientsmaymaintainnormal
myocardialcontractilityforyearsaftertheS3isdetected'-'Hurst:
TheHeart'11th/271
CongenitalHeartDiseasesassociatedwithLoudS3
VentricularseptalDefect(VSD)()

PatentDuctusArteriosus(PDA)Q
AtrialSeptalDefect(ASD)Q
PathologicalThirdHeartSound(S3)maybeassociatedwith
ConstrictivePericarditis
Constrictivepericarditisischaracterixsticallyassociatedwith
pericardialknockwhichisadistinctformofthirdheartsound(S3)
`PericardialknockisS3thatoccursearlier(0.1to0.12afterA2)
andishigherpitchedthannormal.Itspresencedependsupon
therestrictiveeffectsoftheadherentpericardiumwhichhalts
diastolicfillingabruptly'


146.Durationof2"heartsoundis?
a)0.15sec
b)0.12sec
c)0.08sec
d)0.1sec
CorrectAnswer-B
Ans.B.0.12sec

147.Trueaboutvolumereceptorsareall,EXCEPT:
a)Theyarelowpressurereceptors
b)Theyprovideafferentsforthirstcontrol
c)Theyarelocatedincarotidsinus
d)Theymediatevasopressinrelease
CorrectAnswer-C
Thelow-pressurebaroreceptorsarelocatedinthevenaecavaeand
thepulmonaryveins,andintheatria.Theyarealsocalledvolume
receptors.Thesereceptorsrespondtochangesinthewalltension,
whichisproportionaltothefillingstateofthelowpressuresideof
circulation(below60mmHg).Theirimpulsesregulatethesecretion
ofantidiuretichormone(ADH/Vasopressin),reninandaldosterone.
Thelow-pressurebaroreceptorshavebothcirculatoryandrenal
effects;theyproducechangesinhormonesecretion,resultingin
profoundeffectsontheretentionofsaltandwater;theyalso
influenceintakeofsaltandwater.

148.WhichoneofthefollowingistheCORRECTstatementregardingcoronary
bloodflow?
a)Coronarybloodflowisdirectlyrelatedtoperfusionpressureand
inverselyrelatedtoresistance
b)Coronarybloodflowisinverselyrelatedtoperfusionpressure
anddirectlyrelatedtoresistance
c)Coronarybloodflowisdirectlyrelatedtoperfusionpressureand
alsotoresistance
d)Coronarybloodflowisinverselyrelatedtobothpressureand
resistance
CorrectAnswer-A
Coronarybloodflowisdirectlyrelatedtotheperfusion
pressure(aorticdiastolicpressure)andthedurationof
diastole.
Becausecoronaryflowdropstonegligiblevaluesduring
systole,thedurationofdiastolebecomesalimitingfactorfor
myocardialperfusionduringtachycardia.
Coronarybloodflowisinverselyproportionaltocoronary
vascularresistance
.Resistanceisdeterminedmainlybyintrinsic
factors--includingmetabolicproductsandautonomicactivity--and
byvariouspharmacologicagents.Damagetotheendotheliumof
coronaryvesselshasbeenshowntoaltertheirabilitytodilateandto
increasecoronaryvascularresistance.
Ref:KatzungB.G.(2012).Chapter12.Vasodilators&theTreatment
ofAnginaPectoris.InB.G.Katzung,S.B.Masters,A.J.Trevor(Eds),
Basic&ClinicalPharmacology,12e.

149.Einthovenslaw-
a)I+III=II
b)I-III=II
c)I+II+III=0
d)I+III=avL
CorrectAnswer-A
Ans.A.I+III=II
Einthoven'sLawstatesthatiftheelectricalpotentialsofanytwoof
thethreebipolarlimbelectrocardiographicleadsareknownatany
giveninstant,thethirdonecanbedeterminedmathematicallyby
simplysummingthefirsttwo(butnotethatthepositiveandnegative
signsofthedifferentleadsmustbeobservedwhenmakingthis
summation).
ThusthesumofthevoltagesinleadsIandIIIequalsthevoltagein
leadII.

150.MechanismbywhichAchdecreases
heartrateisby:
a)Delayeddiastolicdepolarization
b)Increaseinplateau
c)Decreasepreload
d)Increaseafterload
CorrectAnswer-A
Ans:A.Delayeddiastolicdepolarization
Acetylcholinedecreasesheartrateprimarilybyinhibitingthe
spontaneousdepolarizationofcellsinSAnode;alsoknownas
diastolicdepolarization.Thisisachievedbyinhibitionofthefunny
currentintheSAnode.
Effectofacetylcholineoncardiovascularsystem
Heartrate
Achinhibitsfunnycurrentgenerationinthe
decreases
pacemakercellsofSAnode
AVconduction
AchblocksLtypecalciumchannelsintheAV
decreases
node
Atrialcontraction Atriumissuppliedbycholinergicfibersmorethan
decreases>
theventricles.Achopenspotassiumchannels
ventricular
anddecreasescyclicAMPinthemyocardialcells.
contraction
Achincreasescalciuminendothelialcells,which
Vasodilation
stimulatescalciumdependentENOSand
releasesNOwhichcausesvasodilation.

151.Meanarterialpressureiscalculatedas:
a)(SBP+2DBP)/3
b)(DBP+2SBP)/3
c)(SBP+3DBP)/2
d)(DBP+3SBP)/2
CorrectAnswer-A
Ai.e.(SBP+2DBP)/3


152.Calculatetheejectionfractionfromthe
givenvolumepressurecurve:
a)40%
b)50%
c)55%
d)60%
CorrectAnswer-D
Ans:D.60%
(RefGunong25/ep54024/ep542)
Ejectionfractioncalculatedfromthegivenvolumepressurecurveis
60%.
PressureVolumeLoop:
ab:isovolumetriccontraction
bc:Ventricularcontractionduringsystole
cd:Isovolumetricrelaxation.
Calculation:
End-DiastolicVolume(EDV)(Pointa)=130mL

EndSystolicVolume(ESV)(Pointd)=50mL
StrokeVolume(SV)=EDV?ESV=80mL
EjectionFraction=SV/EDV=80/130=0.6i.e.%EF=60%.


153.Preloadmeasures?
a)Endsystolicvolume
b)Enddiastolicvolume
c)Peripheralresistance
d)Strokevolume
CorrectAnswer-B
EnddiastolicvolumeREF:Guyton'sphysiology22ndeditionpage
111,http://en.wikipedia.org/wiki/Preload_%28cardiology%29
"Forcardiaccontraction,thepreloadisusuallyconsideredtobethe
end-diastolicpressurewhentheventriclehasbecomefilled"
Quantitatively,preloadcanbecalculatedas
WhereLVEDP=Leftventricularenddiastolicpressure,LVEDR=
Leftventricularenddiastolicradius(attheventricle'smidpoint),and
h=thicknessoftheventricle.ThiscalculationisbasedontheLawof
Laplace.

154.Strokevolumeisincreasedby?
a)Increasedend-diastolicandend-systolicvolumes
b)Decreasedend-diastolicandend-systolicvolumes
c)Increasedend-diastolicvolumeanddecreasedend-systolic
volume
d)Decrasedend-diastolicvolumeandincreasedend-systolic
volume
CorrectAnswer-C
Ans.is'c'i.e.,Increasedend-diastolicvolumeanddecreasedend-
systolicvolume
Thestrokevolumeistheamountofbloodpumpedoutbyleft
ventricleineachstroke.
Strokevolumeisgivenbythedifferencebetweenend-diastolic
ventricularvolume(thevolumeofbloodintheleftventricleatthe
endofdiastole;normal120inl)andend-systolicventricularvolume
(thevolumeofbloodattheendofsystole;normal50ml).
Strokevolume(70ml)=End-diastolicventricularvolume(120ml)-
End-systolicventricularvolume(50ml)


155.Theplateauphaseofthisgraphisdue
to:
a)Themovementoffewersodiumionsacrossthecellmembrane
b)Thecalciumchannelsremainingopenlongerthanthesodium
channels
c)Theincreasedmembranepermeabilitytopotassiumion
d)Adecreaseintheamountofcalciumdiffusingacrossthe
membrane
CorrectAnswer-B
Ans:B.)Thecalciumchannelsremainingopenlongerthanthe
sodiumchannels.
Actionpotentialofcardiacmuscles:


Phase2/"Plateauphase":
*Calciumchannelsopenandfastpotassiumchannelsclose.
*Abriefinitialrepolarizationoccurs.
Whyplateau-shaped?
*Actionpotentialthenplateausasaresultof,
-Increasedcalciumionpermeability
-Decreasedpotassiumionpermeability
.
Eventsduringphase2:
*Thevoltage-gatedcalciumionchannelsopenslowlyduringphases
1and0,andcalciumentersthecell.Potassiumchannelsthenclose,
andthecombinationofdecreasedpotassiumioneffluxand
increasedcalciumioninfluxcausestheactionpotentialto
plateau
Noteonotherphases:
1.Phase0/Depolarization:
*Fastsodiumchannelsopen.
*Whenthecardiaccellisstimulatedanddepolarizes,themembrane
potentialbecomesmorepositive.
*Voltage-gatedsodiumchannels(fastsodiumchannels)open
andpermitsodiumtorapidlyflowintothecellanddepolarizeit.

*Themembranepotentialreachesabout+20millivoltsbeforethe
sodiumchannelsclose.
2.Phase1/"InitialRepolarization":
*Fastsodiumchannelsclose.
*Cellularrepolarizationstarts,andpotassiumionsleavethecell
throughopenpotassiumchannels.
3.Phase3/"RapidRepolarization":
*Calciumchannelscloseandslowpotassiumchannelsopen.
*Theclosureofcalciumionchannelsandincreasedpotassiumion
permeability.
*Thispermitspotassiumionstorapidlyexitthecell,ends
theplateauandreturnsthecellmembranepotentialtoitsresting
level.
4.Phase4/"Restingmembranepotential":
*Averagesabout"-90millivolts"

156.Vasoconstrictioninskin?
a)Sympathetic
b)Parasympathetic
c)Whealandflare
d)Warmclimate
CorrectAnswer-A
Ans.A.Sympathetic
Sympatheticstimulationactingviaalpha1and2arevasoconstrictor
totheskinarterioles.

157.Normalinterstitialpressureis?
a)10to15mmHg
b)-5to0mmHg
c)20to30mmHg
d)-10to-20mmHg
CorrectAnswer-B
Ans.B.-5to0mmHg
Normalinterstitialfluidhydrostaticpressure(orsimplyinterstitial
pressure)isusually-1mHg.
Howeveritvariesaccordingtotissuesandrangesfrom-5mmHgto
-1mmHg.
Itisslightlysubatmosphericinmostofthetissues.

158.Striatumdamageaffectsprimarily?
a)Proceduralmemory
b)Shorttermmemory
c)Longtermmemory
d)Explicitmemory
CorrectAnswer-A
Ans.A.Proceduralmemory
Proceduralmemoryisatypeofimplicitmemorythatenablesusto
carryoutcommonlyleanedtaskswithoutconsciouslythinkingabout
them,e.g.,ridingabike,tyingashoeorwashingdishes.
Proceduralmemorylikelyusesadifferentpartofbrainthanepisodic
memory-withbraininjuryyoucanloseone'sabilitywithoutlosing
other.
That'swhyapersonwhohasexperiencedamnesiaandforgetmuch
abouthisorherpersonallifeoftenretainsproceduralmemoryi.e.,
howtodriveacaroruseaforketc.
Striatum(apartofbasalganglia)isresponsibleforprocedure
memory.

159.Somatosensorycortexlesionwillcause
?
a)Pain
b)Temperature
c)Localization
d)Vibration
CorrectAnswer-C
Ans,C,Localization
Corticallesionsdonotabolishallthesomaticsensations.Cortical
anesthesiamainlyinvolvelossofproprioceptionandtactile
sensations(finetouch,twopointdiscrimination,astereognosisor
stereoanesthesia.
Painandtemperatureareleastaffected,buttheyarepoorly
localization.

160.Settingposturebeforeplanned
movement?
a)Premotorcortex
b)Motorcortex
c)Frontal
d)Supplementarymotorcortex
CorrectAnswer-A
Ans.A.Premotorcortex
Thepremotorcortexfunctionisstillincompletelyunderstood,butit
maybeconcernedwithsettingpostureatthestartofaplanned
movementandwithgettingthelndividualpreparedtomove.
Itismostinvolvedincontrolofproximallimbmusclesneededto
orientthebodyformovement.

161.Allthefollowingfeaturesareseeninneuronsfromdorsalrootganglia,
EXCEPT:
a)Theyaremultipolar
b)Theycontainlipofuscingranules
c)Theyhavecentrallylocatednuclei
d)Theyarederivedfromneuralcrestcells
CorrectAnswer-A
Dorsalrootganglionconsistofsensoryneuronswhichare
pseudounipolarandhavenosynapticconnectionsintheganglion.
Theyareclassifiedaspseudounipolarbecauseitlacksdendrites
andhasasingleaxonthatbifurcateintoacentrallyefferentbranch
thatfunctionsasadendritetocarryafferentsensorysignals.

162.Neuronsinsympatheticgangliaare?
a)Unipolar
b)Bipolar
c)Pseudounipolar
d)Multipolar
CorrectAnswer-D
Ans,d.Multipolar
Ventral,lateralanddorsalhornsofspinalcordandsympathetic
chaingangliacontunmultigolarneuron,whereasdorsalrootganglia
containspseudounipolarneurons.

163.Pt.isabletorecognisepersonbyname
butnotbyface.Lesionisin?
a)Postparietalregion
b)Occipital
c)Frontallobe
d)Temporallobe
CorrectAnswer-D
Ans.D.Temporallobe
Animportantpartofthevisualinputgoestotheinferiortemporal
lobe,whererepresentationsofobjects,particularlyfacesarestored.
Inhumans,storageandrecognitionoffacesismorestrongly
representedintherightinferiortemporallobeinright-handed
individuals.

164.CSF/plasmaglucoseratiois?
a)0.2-0.4
b)0.6-0.8
c)1.2-1.6
d)1.6-2.2
CorrectAnswer-D
Ans.D.1.6-2.2

165.Burningpainiscarriedbywhichtypeof
fibres?
a)Aalpha
b)Adelta
c)Abeta
d)C
CorrectAnswer-D
Ans.D.C
Painistransmittedviatwofibertypes:
*ThinlymyelinatedAdeltafibers(2-5mindiameter)whichconduct
atratesof12-30m/s.
*UnmyelinatedCfibers(0.4-1.2mindiameter)whichconductatlow
ratesof0.5-2m/s.
-Thermoreceptorsalsospanthesetwoflbertypes.
-ColdreceptorsareondendriticendingsofAdeltafibersandC
fibers,whereaswarmth(heat)receptorslueonCfibers.

166.Amansleptwithheadoverforearm,next
morninghecomplainsoftingling,
numbnessoverforearm.Itiscausedby
?

a)SensitivitytohypoxiaisA>B>C
b)SensitivitytopressureisA>B>C
c)SensitivitytohypoxiaisC>B>A
d)SensitivitytopressureisB>A>C
CorrectAnswer-C
Ans.C.SensitivitytohypoxiaisC>B>A
TypeCfibers(TypeIVLloyd&Hunt)areleastsusceptibleto
hypoxia'

167.Transducinisaproteinfoundin:
a)Glomerulus
b)Retina
c)Skeletalmuscle
d)Adrenalmedulla
CorrectAnswer-B
RetinaREF:Ganong22ndedition,chapter8,
http://en.wikipedia.org/wiki/Transducin
Transducin(alsocalledGt)isaheterotrimericGproteinthatis
naturallyexpressedinvertebrateretinarodsandcones.
Mechanismofaction:HeterotrimericTransducin(alpha-beta-gamma
subunits)isactivatedbyaconformationalchangeinrhodopsindue
totheabsorptionofaphotonbyrhodopsin'sactivegroupretinal.
ActivationcausestheGDPboundtothealphasubunittobe
exchangedwithGTPfromsolutionandresultsinactivatedalpha
dissociatingfrombeta-gamma.
ActiveTransducin-alphathencausescyclicGMP
Phosphodiesterasetoincreaseitsactivity,therebyloweringthe
concentrationofcGMP,anintracellularsecond-messenger
molecule.DecreaseincGMPconcentrationleadstotheclosureof
cGMP-regulatedNa+andCa2+ionchannelsandahyperpolarized
membranepotential.Thischainofsignalingeventsisalsocalled
"thevertebratephototransductioncascade"

168.Lesionofpreopticnucleusof
hypothalamuscauses?
a)Hyperphagia
b)Hyperdypsia
c)Hyperthermia
d)Hyperglycemia
CorrectAnswer-C
Ans.C.Hyperthermia
Preopticnucleusisconcernedwithregulationofbodytemperature.
Therefore,itslesionwillresultindefectivetemperature

169.Shiveringiscontrolledby:
September2012,March2013
a)Dorsomedialnucleus
b)Posteriorhypothalamus
c)Perifornicalnucleus
d)Lateralhypothalamicarea
CorrectAnswer-B
Ans.Bi.e.Posteriorhypothalamus
Shivering/Shuddering
Itisabodilyfunctioninresponsetoearlyhypothermiainwarm-
bloodedanimals.
Whenthecorebodytemperaturedrops,theshiveringreflexis
triggeredtomaintainhomeostasis.
Musclegroupsaroundthevitalorgansbegintoshakeinsmall
movementsinanattempttocreatewarmthbyexpendingenergy.
Shiveringcanalsobearesponsetoafever,asapersonmayfeel
cold,thoughtheircoretemperatureisalreadyelevated.
Locatedintheposteriorhypothalamusnearthewallofthethird
ventricleisanareacalledtheprimarymotorcenterforshivering.
Thisareaisnormallyinhibitedbysignalsfromtheheatcenterinthe
anteriorhypothalamic-preopticareabutisexcitedbycoldsignals
fromtheskinandspinalcord.

170.

Sodiumchannelsaremaximuminwhichpart
ofneuron?

a)Soma
b)Axonhillock
c)Dendrites
d)Axon
CorrectAnswer-B
Ans.is'b'i.e.,Axonhillock
Inamotorneuron,theaxonhillockandtheinitialsegmentofaxon
havethelowestthresholdforexcitation.
Thisisbecausetheyhaveamuchhigherintensityofvoltagegated
sodiumchannel.

171.Thedistancebywhichtwotouchstimuli
mustbeseparatedtobeperceivedas
twoseparatestimuliisgreatestat?

a)Thelips
b)Thepalmofthehand
c)Thebackofscapula
d)Thedorsumofthehand
CorrectAnswer-C
Ans.C.Thebackofscapula
Themagnitudeoftwopointdiscriminationthresholdsvariesfrom
placeofplaceonthebodyissmallestwheretouchreceptorsare
mostabundant.
stimulationpointsonthebackmustbeseparatedbyatleast65mm
beforetheycanbedistinguishedasseparate,whereasonthe
fingertipstwostimuliassrecognizediftheyareseparatedbyaslittle
as2mm.

172.Purkinjefibresareinhibitoryfor?
a)Deepcerebellarnuclei
b)Climbingfibre
c)Basketcells
d)Spinocerebellartracts
CorrectAnswer-A
Ans.A.Deepcerebellarnuclei
Aftercomplexinhibitingandexcitatoryinteractionsofvariousfibers
andcellsinthecortex,theoutputofcerebellarcortex,isprojectedto
deepcerebellarnucleibyaxonsofpyramidalcells(onlyoutputcells
ofcerebellarcortex).
TheoutputofthePurkinjecellsisinhibitoryinthedeepcerebellar
nuclei.
However,theoutputofdeepcerebellarnucleitothebrainstemand
thalamusisalwaysexcitatorybecause,besideinhibitoryinputsof
purkinjecells,deepcerebellarnucleialsoreceiveexcitatoryinputs
fromafferentmossyandclimbingfiberswhichusuallyaremore
prominent.

173.Neurotransmitterinvolvedin
nigrostriatialpathwayis?
a)Serotonin
b)Dopamine
c)Cholinergic
d)Adrenergic
CorrectAnswer-B
Ans.B.Dopamine
Dopaminergicnigrostriatalprojectionfromthesubstantianigrapars
compacta(sNpc)tothestriatumCaudatenucleusandputrrnen)and
correspondingGABAergicproiectionfromstriatumtosubstantia
nigraparsreticulate(SNPR).
Dopamineisthemajorneurotransmitterinsubstantianigra.

174.Lossofproprioception&finetouch?
a)Anteriorspinothalamictract
b)Lateralspinothalamictract
c)Dorsalcolumn
d)Corticospinaltract
CorrectAnswer-C
Ans.C.Dorsalcolumn

175.Trueaboutspinocerebellartractis?
a)Equilibrium
b)Smoothensandcoordinatesmovement
c)Learninginducedbychangeinvestibuloocularreflex
d)Planningandprogramming
CorrectAnswer-B
Ans.B.Smoothensandcoordinatesmovement
Spinocerebellumisconcernedwithsmootheningandcoordinationof
movements.
Itachievesthisbygettingamomenttomomentreportby
spinocerebellartract.

176.Vomitingcentreissituatedinthe:
September2008
a)Hypothalamus
b)Midbarin
c)Pons
d)Medulla
CorrectAnswer-D
Ans.D:Medulla
Vomitingisbelievedtobecontrolledbytwodistinctbraincentres--
thevomitingcentreandthechemoreceptortriggerzone--both
locatedinthemedullaoblongata.
Thevomitingcentreinitiatesandcontrolstheactofemesis,which
involvesaseriesofcontractionsofthesmoothmusclesliningthe
digestivetract

177.Chemoreceptorsarelocatedinwhich
area?
a)Medulla
b)Archofaorta
c)Bifurcationofcarotidartery
d)Alloftheabove
CorrectAnswer-D
Ans.is'd'i.e.,AIIoftheabove
Centralchemoreceptors-VentralSurfaceOfMedulla.
Peripheralchemoreceptors-Atbifurcationofcommoncarotidartery
(carotidbody)andarchofaorta(aorticbodies)

178.Saltytaseisdueto?
a)Ca+2channels
b)Na+channels
c)G-protein
d)H+channels
CorrectAnswer-B
Ans,B.Na+channels
Salty-tastingsubstancesdepolarizetastecellsbyactivating
amiloride-sensitiveNa+channels.

179.Restingmembranepotentialinnerve
fibre
a)Isequaltothepotentialofventricularmusclefibre
b)Canbemeasuredbysurfaceelectrodes
c)IncreasesasextracellularK+increases
d)DependsuponK+equilibrium
CorrectAnswer-D
Di.e.Dependsonpotassiumionequilibrium

180.Epinephrinereducesinsulinby?
a)Alphaactionpredominantly
b)Betaactionpredominantly
c)Alphaandbeta
d)Muscarinicreceptors
CorrectAnswer-A
Ans.A.Alphaactionpredominantly
Epinephrinedecreasesinsulinreleaseviaalpha-2actionas
pancreaticbeta-cellshasalpha-2receptors.

181.Adrenalineeffectsinsulinby?
a)Stimulationofalphacells
b)Stimulationofbetacells
c)Stimulationofdeltacells
d)Stimulationofgcells
CorrectAnswer-B
Ans.B.Stimulationofbetacells
Adrenalinedecreasesinsulinsecretionbyactingonalpha-2
receptorsonbeta-cellsofpancreas.

182.Smoothmusclephysiologydifferent
fromskeletalmuscle
a)K+requiresforcontraction
b)Ca2+requiredforcontraction
c)Troponinisabsent
d)Myosinisrequiredforcontraction
CorrectAnswer-C
Ans.is'c'i.e.,Troponinisabsent
*Troponinisabsentinsmoothmusclebutrequiredinskeletal
musclesforcontraction.
*Ca+andMyosinarerequiredbybothsmoothmusclesandskeletal
muscles.
*K+hasnodirectroleinthecontractionofsmoothmuscleand
skeletalmuscle.
Smoothmusclecontraction
*Insmoothmuscle,thereisnotroponin.Thereforecalciuminitiates
contractionthroughamechanismdifferentfromthatemployedby
skeletalmuscle.Smoothmusclecontainsacalcium-bindingprotein
calledcalmodulin.Anincreaseincytoplasmic(sarcoplasmic)
calciumleadstoitsbindingtocalmodulin.Thecalcium-calmodulin
complexactivatesmyosinkinase,alsocalledmyosinlightchain
kinase(MLCK).MLCKisaphosphorylasewhichphosphorylatesa
lightchainbelongingtomyosinchain,oftencalledcross-bridge
phosphorylation.
Thephosphorylatedmyosinheadinteractswith
actin,i.e.,cross-bridgingofmyosinwithactin.Thecross-bridging
leadstocontraction.
*Thereversalofcontractileresponsedependsonareductionin

sarcoplasmiccalciumbycalciumpumpsinthesarcoplasmic
reticulummembraneandsarcolemmawhichpumpcalciumintothe
sarcoplasmicreticulumandextracellularfluidrespectively.
Reductionincytoplasmic(sarcoplasmic)calciumlevelactivatesan
enzymemyosinphosphatase.Myosinphosphatase
dephosphorylatesthemyosinhead,therebyterminatingtheactin-
myosininteractionandproducesrelaxation.
So,calciuminfluxhastwofunctionsinsmoothmuscles:-
*Generationofactionpotential
*Essentialforcontractioninitiation(Excitationcontractioncoupling)
-Itisinterestingtoknowthatalthoughdephosphorylationofmyosin
headisnecessaryforrelaxation,thedephosphorylationofmyosin
doesnotnecessarilyleadtotherelaxationofthesmoothmuscle.
Thisisduetothelatchingeffect,i.e.,myosinbridgesholdontothe
actinfilamentlikealatch.Duetothissmoothmusclecontracts,and
itcanmaintainitscontractionforaprolongedperiod(sustained
contraction)foralongperiodwithminimalexpenditureofenergy
Characteristicfeatureofsmoothmuscle.
-Aswithskeletal,muscleATPisrequiredwhichishydrolyzedby
myosinheadwhichhasATPaseactivity(Inasimilarwayasin
skeletalmuscle).

183.Motormarchisseenin?
a)Axontemesis
b)Neurotemesis
c)Neuropraxia
d)Nerveregeneration
CorrectAnswer-A
Ans.A.Axontemesis
Motormarch(sequentialreinervationofmusclesfromproximalto
distal)isseeninaxontemesis.
InneuroPraxiathereisnoanatomicdisruption,somotormarchisnot
seen.
Inneurotemesisthereisnorecovery,thusnomotormarch'

184.Whenthetensioninamusclefibreis
maximum,itslengthiscalledas?
a)Equilibriumlength
b)Optimumlength
c)Initiallength
d)None
CorrectAnswer-B
Ans.B.Optimumlength
Uptoalimitgreatertheinitiallength(i.e.,lengthatrelaxed-state)
greateristheforceofcontraction.
I.e.,thereisanoptimallength,atwhichtheforcegeneratedbya
muscleismaximal'

185.Tetaniccontractionisdueto
accumulationof?
a)Na+
b)Ca+
c)K+
d)Cl?
CorrectAnswer-B
Ans.B.Ca+
Tetaniccontractionisaboutfourtimesthetwitchtension,
Followingtheoryhasbeenputforwardforthishighertension
generatedduringmuscletetanus:-
Duringasingletwitch,theamountofCa+2isreleasedinto
sarcoplasmisnotenoughtoproducetetanictension.
Whenthemuscleisstimulatedinrapidsuccession,Ca+2comesout
intothesarcoplasmwitheachstimulusandthereisaprogressive
accumulationofCa+2,inthesarcoplasm.
TetanictensionisreachedwhensarcoplasmicCa+2levelsreach
theirmaximum.

186.WhichofthefollowingisNOTTRUEaboutCSF?
a)RemovalofCSFduringduraltapcausesintenseintracranial
headache
b)Normallycontainnoneutrophils
c)Formedbyarachnoidvilliwithintheventricles
d)pHislessthanthatofplasma
CorrectAnswer-C
TheCSFisformedinthechoroidplexusesandtheremainderisformedaround
bloodvesselsandalongventricularwalls.
CSFfillstheventriclesandsubarachnoid
space.Inhumans,thevolumeofCSFisabout150mLandtherateofCSFproductionis
about550mL/d.ThustheCSFturnsoverabout3.7timesaday.ThecompositionofCSFis
essentiallythesameasthatofbrainextracellularfluid(ECF),whichinlivinghumansmakes
up15%ofthebrainvolume.pH(7.31-7.34)slightlylessthanplasma(7.35-7.45).CSF
containnoneutrophils.Itsdifferentialcountis:Lymphocyte-60-70%,Monocytes-30-50%,
Neutrophils-None.
Ref:BarrettK.E.,BarmanS.M.,BoitanoS.,BrooksH.L.(2012).Chapter33.Circulation
throughSpecialRegions.InK.E.Barrett,S.M.Barman,S.Boitano,H.L.Brooks(Eds),
Ganong'sReviewofMedicalPhysiology,24e.

187.Duringacclimitisation,decreased
sweatingisduetodownregulationof?
a)Epinephrinereceptors
b)Norepinephrinereceptors
c)Acetylcholinereceptors
d)Dopaminereceptors
CorrectAnswer-C
Ans.C.Acetylcholinereceptors
Examinerjustwantstoknowthereceptorswhicharepresentin
sweatgland.
Sweatingisunderthecontrolofsympatheticsystem.Butthe
neurotransmitterisacetylcholinewhichactsoncholinergic
sympatheticreceptors.

188.Substancethatiscompletelyreabsorbed
fromthekidney?
a)Na+
b)K+
c)Urea
d)Glucose
CorrectAnswer-D
Ans.D.Glucose
Onanormaldietmaximumsolutesarereabsorbedalmost
completely:-Glucose,Aminoacids&HCO3-:100%

189.Substratewhichisbothsecreted&
filtered?
a)UricAcid
b)Glucose
c)Urea
d)Na+
CorrectAnswer-A
Ans,A.UricAcid
SubstanceswhicharebothsecretedandfilteredareK+,uricacid
andcreatinine.

190.ANPactsatwhichsite?
a)Glomerulus
b)LoopofHenle
c)PCT
d)Collectingduct
CorrectAnswer-D
Ans.D.Collectingduct
Theatrialnatriureticpeptides(ANPs)arepresentasgranulesinthe
atrialmusclecells.
ANPisreleasedinresponsetoincreasedplasma
Na+concentration.
Ingeneral,theeffectsofANParephysiologicallyantagonisticto
thoseofangiotensinII.
ANPcausesnatriuresisduetoincreasedGFRbyrelaxingmesangial
cellsofglomerulus.
ANPdecreasestheNa+reabsorptionfromthedistaltubuleand
collectingduct.
ANPdecreasesthesecretionofrenin,aldosterone,andADH.
Inaddition,NPalsorelaxesthevascularsmoothmuscleinarterioles
andvenulesandthereforelowersBP.

191.AsfluidcomesdownthePCT,whatis
true?
a)Concentrationofureafalls
b)ConcentrationofHCO3-falls
c)ConcentrationofNa'increases
d)Concentrationofinulindecreases
CorrectAnswer-D
Ans.D.Concentrationofinulindecreases

192.Whichofthefollowingismostimportant
insodiumandwaterretention?
a)Renninangiotensinsystem
b)ANP
c)BNP
d)Vasopressin
CorrectAnswer-D
Ans.D.Vasopressin
DespiteitseffectonNa+andwaterreabsorption,aldosteroneisa
weakregulatorofbodyNa,andwaterbalance,themajorregulator
beingthe'thirst-ADH'mechanism.
Aldosteroneisthesoleregulatorofexternalpotossiumbalance".

193.Cephalicphaseofgastricsecretion?
a)20%
b)70%
c)10%
d)100%
CorrectAnswer-A
Ans.A.20%
Cephalicphaseaccountsfor20%ofgastricacidsecretionand
gastricphaseaccounts70-80%ofgastricacidsecretion.

194.Cephalicphaseofgastricsecretion?
a)Onfoodenteringstomach
b)Onfoodenteringintestine
c)Onseeingfood
d)Onstress
CorrectAnswer-C
Ans.C.Onseeingfood
Thecephalicphaseofgastricsecretionoccursevenbeforefood
entersthestomach,especiallywhileitisbeingeaten.
Itresultsfromthesight,smell,thought,ortasteoffood,andthe
greatertheappetite,themoreintenseisthestimulation.

195.Gastricsecretionis:
a)Inhibitedbycurare
b)Stimulatedbynoradrenaline
c)Increasedbystomachdistention
d)Stimulatedbyanincreaseintonicactivity
CorrectAnswer-C
Ci.e.Increasedbystomachdistension

196.Whichinhibitsgastricsecretion?
a)Secretin
b)Insulin
c)HighgastricpH
d)Calcium
CorrectAnswer-A
Ai.e.Secretin
GastrinisahormonewhichisproducedbyG-cellsinthelateralwall
ofglandsintheantralportionofgastricmucose.
Itisalsofoundin
pancreaticisletsinfetallife,gastrinomasofpancreas,andpituitary
glandQ,
hypothalamus,medullaoblongataandinvagus&sciatic
nerves.

197.Motilinsecretiondecreasedin?
a)Thirsty
b)Starving
c)Ingestedmeal
d)Interdigestivestate
CorrectAnswer-C
Ans.C.Ingestedmeal
Whenamealisingested,secretionofmotilinissuppresseduntil
digestionandabsorptionarecomplete.

198.WhichismaximallyabsorbedfromGIT?
a)Pentose
b)Hexose
c)Diasaccharide
d)Polysaccharide
CorrectAnswer-B
Ans.B.Hexose
Hexosesarerapidlyabsorbedacrossthewallofthesmallintestine.

199.Pancreaticlipasehydrolysesester
linkageoftriacidglyceridesatposition?
a)1&2
b)1&3
c)2&3
d)Only3
CorrectAnswer-B
Ans.B.1&3
Mostfatdigestionthereforebeginsintheduodenum,pancreatic
lipasebeingoneofthemostimportantenzymesinvolved.
Thisenzymehydrolyzesthe1-and3-bondsofthetriglycerides
(triacylglycerols)withrelativeeasebutactsonthe2-bondsatavery
lowrate,sotheprincipalproductsofitsactionarefreefattyacids
and2-monoglycerides(2-monoacylglycerols).
Itactsonfatsthathavebeenemulsified.

200.Gastricsecretionsareessentialfor
absorptionof-
a)Cobalmin
b)Fat
c)Thiamine
d)Folicacid
CorrectAnswer-A
Ans.A.Cobalmin
VitaminB12isabsorbedintheileum.
Thisvitaminbindstointrinsicfactor,aproteinsecretedbythe
stomach,andthecomplexisabsorbedacrosstheilealmucosa.

201.EffectofaceytylcholineonLES?
a)Contraction
b)Relaxation
c)Noeffect
d)Contractionfollowedbyrelaxation
CorrectAnswer-A
Ans.A.Contraction
Thisbarrieragainstrefluxoftheharmfulgastricjuice(pepsinand
HCDisstrengthenedwhenthesplincterpressureisraisedby-
1. Acetylcholine
2. Adrenergicagonists
3. Gastrin
4. Motilin
5. Somatostatin
6. SubstanceP
7. Histamine
8. PGF2-alpha.
9. Proteinrichfood
10. Highintraabdominalpressure.

202.Ptylinissecretedby?
a)Gastricgland
b)Salivarygland
c)Duodenalgland
d)Pancreaticgland
CorrectAnswer-B
Ans.B.Salivarygland
Salivaryamylaseisalsocalledptylinwhichissecretedinsalivaby
salivarygland.

203.PGsinsemenaresecretedby?
a)Prostate
b)Seminalvesicle
c)Sperms
d)Testes
CorrectAnswer-B
Ans.B.Seminalvesicle

204.17OHsteroid?
a)Androgen
b)Progesterone
c)Estrogen
d)None
CorrectAnswer-B
Ans.B.Progesterone

205.GLUT2ispresentmainlyin?
a)Betacellsofpancreas
b)Placenta
c)Skeletalmuscle
d)Cardiacmuscle
CorrectAnswer-A
Ans.A.Betacellsofpancreas

206.Glucosetransporterinvolvedininsulin
stimulatedglucosetransportisGLUT?
a)1
b)2
c)3
d)4
CorrectAnswer-D
Ans,D.4

207.Insulinisessentialforglucoseentryin?
a)Muscle
b)Corticalneurons
c)Renaltubularcells
d)Betacellsofpancreas
CorrectAnswer-A
Ans.A.Muscle
Glucoseenterscellsbyfacilitateddiffusionor,intheintestineand
kidneys,bysecondaryactivetransportwithNa+.
Inmuscle,adipose,andsomeothertissues,insulinstimulates
glucoseentryintocellsbyincreasingthenumberofglucose
transporterslnthecellmembranes.

208.Wolff?Chaikoffeffectisdueto?
a)SuppressionofTSHsecretion
b)DecreasediodinationofMIT
c)DecreasedT3toT4conversion
d)Iodineintake
CorrectAnswer-D
Ans.D.Iodineintake
Iodineisthefastestactingthyroidinhibitor.
Themostimportantactionisinhibitionofhormonerelease(thyroid
constipation),butallfacetsofthyroidsynthesismaybeaffected.
Excessiodideinhibitsitsowntransportinthyroidcellsandmayalter
theredoxpotentialofcells,thusinterferingiodinationreducedT3/T4
synthesis(Wolff?Chaikoffeffect).

209.Majoradrenalandrogenis?
a)Testosterone
b)11-hydroxyderivativeofandrostenedione
c)17-ketosteroiddehydroepiandrosterone
d)Cortisol
CorrectAnswer-C
Ans.C.17-ketosteroiddehydroepiandrosterone
Themajoradrenalandrogenisthel7-ketosteroid
dehydroepiandrosterone,althoughandrostenedioneisalso
secreted.
Themajorandrogenoftestisistestosterone.

210.DiurnalvariationofACTHdependson?
a)Suprachiasmaticnucleus
b)Supraopticnucleus
c)Ventrolateralnucleus
d)Thalamus
CorrectAnswer-A
Ans.A.Suprachiasmaticnucleus
ThebiologicclockresponsibleforthediurnalACTHrhythmis
locatedinthesuprachiasmaticnucleiofthehypothalamus.

211.Hypothalamusincreasesreleaseofall
hormonesfromthepituitaryexcept?
a)TSH
b)FSH
c)CRH
d)Prolactin
CorrectAnswer-D
Ans,D.Prolactin
Formostoftheanteriorpituitaryhormones.
ltisthereleasinghormonethatareimportantbutprolactinismainly
undertheinhibitorycontrol.

212.1stresponsetohypoglycemia?
a)Decreasedinsulin
b)Increasedglucagon
c)Increasedcortisol
d)Increasednorepinephrine
CorrectAnswer-A
Ans.A.Decreasedinsulin
Decreasedinsulinisthefirstresponsefollowedsuccessivelyby
increaseinglucagon,epinephrine,cortisolandGH.

213.Velocityofhumansperm-
a)1-3mm/min
b)4-6mm/min
c)6-9mm/min
d)10-13mm/min
CorrectAnswer-A
Ans.A.1-3mm/min
Humanspermmoveataspeedof-3mm/minthroughthefemale
genitaltract.
Spermsreachuterinetubes30-60minutesaftercopulation.

214.Acidophilssecrete
a)GH
b)TSH
c)ACTH
d)FSH
CorrectAnswer-A
Ans.A.GH
Acidophilsoftheanteriorpituitarysecretesgrowthhormone
andprolactin.


215.Vasopressinissecretedby
a)Supraoptic
b)Preoptic
c)Paraventricular
d)Posteriornucleus
CorrectAnswer-A
Ans.A.Supraoptic
ADHisformedprimarilyinthesupraopticnuclei,whereasoxytocinis
formedprimarilyintheparaventricularnuclei".

216.Growthhormoneleveldecreasedin
a)Hypoglycemia
b)Fasting
c)Sleep
d)Exercise
CorrectAnswer-C
Ans.C.Sleep
REMsleepdecreasesGHsecretionwhereasSlow-wavesleep(N3
ofNREM)causesanincreaseinGHsecretion.

217.Pituicytesareseenin?
a)Anteriorlobe
b)Posteriorlobe
c)Intermediatelobe
d)All
CorrectAnswer-B
Ans.B.Posteriorlobe
Pituicytesareglialcellsofposteriorpituitary.

218.Sizeofsperm?
a)100microns
b)65nm
c)100nm
d)65microns
CorrectAnswer-D
Ans.D.65microns

219.cAMPactionmediatesallexcept?
a)Glucagon
b)Folliclestimulatinghormone
c)Leutinizinghormone
d)Estrogen
CorrectAnswer-D
Ans.D.Estrogen

220.InThyroidfollicleforhowlongThyroxine
isstored?
a)2-3weeks
b)2-3days
c)2-3months
d)2-3years
CorrectAnswer-C
Ans.C.2-3months
Thyroidhormonesarestoredinthefolliclesinanamountsufficient
tosupplythebodywithitsnormalrequirementsofthyroidhormones
for2to3months.

221.Ghrelinfalseis?
a)Producedbystomachcells
b)Increasedappetite
c)Isrelatedtoregulationofthyroidhormone
d)Stimulatesgrowth
CorrectAnswer-C
Ans.C.Isrelatedtoregulationofthyroidhormone
Ghrelinissecretedprimarilybythestomachandappearstoplayan
importantroleinthecentralcontroloffoodintake.
Italsostimulatesgrowthhormonesecretionbyactingdirectlyon
recePtorsinthepituitary.
Itisalsoproducedinthehypothalamusandhasmarkedgrowth
hormone-stimulatingactivity.

222.IsotopeusedtomeasureRBCvolumeis
?
a)Cr51
b)H3
c)D20
d)I135
CorrectAnswer-A
Ans.A.Cr51
BloodcellvolumeismeasuredbyCr51labeledRBC.

223.Glucoseistransportedinrenaltubular
cellsby
a)Ksymport
b)Kantiport
c)Naantiport
d)Nacotransport
CorrectAnswer-D
Ans.D.Nacotransport
Sodium-dependentglucosetransporters,SGLTIandSGLT2,are
responsibleforthesecondaryactivetransportofglucoseinthe
intestineandrenaltubules.
GlucoseandNa+bindtothesodium-dependentglucosetransPorter
(SGLT)2intheapicalmembrane,andglucoseiscarriedintothecell
asNa+movesdownitselectricalandchemicalgradient.

224.Watertravellingfromextracellularto
intracellularisby?
a)Co-transport
b)Diffusion
c)Filtration
d)Activetransport
CorrectAnswer-B
Ans.B.Diffusion
Bestanswerofthisquestionisnonebecausetransportofwater
(solvent)acrossthecellmembraneiscausedbyosmosis.
Allthegivenoptionsarethetransportprocessesofsoluteacrossthe
cellmembrane(notforsolvent).
Howeveramongthegivenoptions,diffusionistheclosestone:
"Diffusionofsolventtowardsanareawherethereishigher
concentrationofsoluteiscalledosmosis".

225.Osmolarityisdefinedas?
a)Numberofosmoleperlitre
b)Numberofosmoleperkg
c)Weightofsoluteperlitre
d)Weightofsolventperlitre
CorrectAnswer-A
Ans.A.Numberofosmoleperlitre
Osmolarity>Numberofosmoleperlitre.
Osmolality>Numberofosmoleperkg.

226.Insensiblewaterlossperdayis?
a)100ml
b)300ml
c)700ml
d)1000ml
CorrectAnswer-C
Ans.C.700ml
Insensiblewaterloss-Thereiscontinuouslossofwaterby
evaporationfromrespiratorytractanddiffusionthroughtheskin,
whichtogetheraccounts700ml/dayofwaterlossundernormal
conditions.
Theinsensiblewaterlossfromskin(diffusionthroughskin)occurs
independentlyofsweatingandaccounts350mlofwaterlossper
day.
So,intheabsenceofsweating,waterlossfromtheskinis350
ml/day

227.Whichofthefollowingincreasesparticle
diffusionacrossthecellmembrane?
a)Increasingsizeofparticle
b)Decreasinglipidsolubilityofsubstance
c)Increasinglipidsolubilityofsubstance
d)Decreasingsizeofopeningincellmembrane
CorrectAnswer-C
Ans.C.Increasinglipidsolubilityofsubstance
Simplediffusionisfavoredbysmallsize,lipidsolubilityandabsence
ofpolarity(non-polarsubstance)andcharge(neutralmolecule)
throughathin,largemembranewheretheconcentrationgradientis
more.

228.MechanismofheatlossinmodernX-ray
tubeis
a)Radiation
b)Evaporation
c)Conduction
d)Convection
CorrectAnswer-A
Ans.A.Radiation
ThemechanismofheatlossinthemodernX-raytubeisradiation.
ClassicalX-raytube:
Theoutertubeismadeupofglass.
Cathode-tungstenfilament.
Ananodeisthetargetandismadeoftungsten.
Themechanismofheatlossisconduction.
Theatomicnumberoftungstenis74.
ModernX-ray
tube:
Theoutertubeismadeupofstainlesssteel
Cathode-tungsten+thorium
Anode-tungsten+10%rhenium.Ananodeisrotatingandthe
mechanismofheatlossisradiation.

229.Mostpotentantioxidant?
a)VitA
b)VitK
c)VitE
d)VitC
CorrectAnswer-C
Ans.C.VitE
VitaminE(tocopherol)isthemostinpotentantioxidantinthebody,
actinginthelipidphaseofmembranesprotectingagainsttheeffects
offreeradicals.

230.Inmoderateexcercisetherespiratory
rateisincreasedduetoresponseof?
a)Propioceptionreceptorinthejoints
b)PCO2inarterialblood
c)PO2inarterialblood
d)J-receptorstimulation
CorrectAnswer-A
Ans.A.Propioceptionreceptorinthejoints
Inmoderateexercisetheabruptincreaseinventilationatthestartof
exerciseisduetopsychicstimuliandafferentimpulsesfrom
proprioceptorsinmuscles,tendonsandjoints.

231.Myosinfilamenthasafixedlengthof
a)0.16nm
b)1.6micrometers
c)16nm
d)l.6mm
CorrectAnswer-B
Ans.B.1.6micrometers
Thetotallengthofeachmyosinfilamentisuniform,almostexactly
1.6micrometers.

232.Plasmamembraneismainlycomposed
of?
a)Cholesterol
b)Carbohydrate
c)Phospholipid
d)Protein
CorrectAnswer-D
Ans.D.Protein

233.Onweightbasis,themembranecontains
proteinandlipidintheratioof?
a)1:2
b)1:1
c)2:1
d)4:1
CorrectAnswer-B
Ans.B.1:1
Thecellmembranecontainsproteinsandlipidsina50:50ratio.This
referstotheratiooftheirmassesandnotnumbers".
Itisworthnotingherethatabovedescribedpercentageisintermsof
areaoccupied.However,intermsofmasses,thecellmembrane
containsproteinsandlipidsina50:50ratio.

234.Whichofthefollowingisthecauseofnonshiveringthermogenesisin
adults?
a)Noradrenaline
b)Thyroidhormone
c)Musclemetabolism
d)Brownfatbetweentheshoulders
CorrectAnswer-A
Nonshiveringthermogenesisreferstoincreaseinmetabolicratethatisnotaresultof
muscleactivity.Itappearstobeelicitedthroughsympatheticstimulationandcirculating
catecholamines.Epinephrineandnorepinephrinewhicharereleasedincreasesmetabolic
activityandheatgeneration.
Ref:GuytonandHall-TextbookofMedicalPhysiology,10thEdition,Pages821,828-829;
MedicalPhysiology:PrinciplesforClinicalMedicineByRodneyA.Rhoades,4thEdition,
Page568;FundamentalsofHumanPhysiologyByLauraleeSherwood,Page489

235.Thethicknessofendometriumatthe
timeofimplantationis:
a)3?4mm
b)20?30mm
c)15?20mm
d)30?40mm
CorrectAnswer-A
Ans.isNone/ai.e.3-4mm
"7heEndometriumisinthesecretoryphasecorrespondingto20-
21daysofcycle"atthetimeofimplantation.
"Afterovulation,theendometriumnowdemonstratesacombined
reactiontoestrogenandprogesteroneactivity.Mostimpressiveis
thattotalendometrialheightisfixedatroughlyitspreovulatory
extent(5-6mm)despitecontinuedavailabilityofestrogen."
Readingtheabovetextitisclearthatendometriumis-5-6mm
thickatthetimeofimplantation,whichisnotgivenintheoption.Still
ifyouhavetomarkoneansweroption'a'i.e.3-4mmbeingclosest
couldberight.
ExtraEdge
Implantationoccurs7-9daysafterovulation?.
Inhuman,theblastocystburrowsintheuterinecavitytillwholeofit
lieswithinthethicknessofendometrium.Thisiscalledasinterstitial
implantation.
Afterimplantationoftheembryotheuterineendometriumiscalled
thedecidua".
Deciduabasalis-Thepartofdeciduawheretheplacentaistobe
formed.
DeciduaCapsularis-Thepartofthedeciduathatseparatesthe
embryofromtheuterinelumen.DeciduaParietalis-Thepartofthe

embryofromtheuterinelumen.DeciduaParietalis-Thepartofthe
decidualiningrestoftheuterinecavity.
Attheendofpregnancy,thedeciduaisshedoffalongwithplacenta
andmembranes.

236.Digestiveenzymesare
a)Hydrolases
b)Oxidoreductases
c)Dehydrogenases
d)Ligases
CorrectAnswer-A
Ans.is'a'i.e.,Hydrolases[RefHarper's29thlep.518-19]
Alldigestiveenzymesarehydrolases.

237.Enzymesthatmoveamoleculargroup
fromonemoleculetoanotherareknown
as-

a)Ligases
b)Oxido-reductases
c)Transferases
d)Dipeptidases
CorrectAnswer-C
Ans.isC.i.e.,Transferase[RefChatterjea8th/ep.123;Harper
28th/ep.52]
TransferasesCatalyzetransferofC-N-,orP-containinggroupfrom
onesubstratetoanother,

238.Glucose-6-phosphatedehydrogenase
need-
a)NAD
b)NADP
c)FAD
d)FMN
CorrectAnswer-B
Ans.is'b'i.e.,NADP[Ref:Harper29th/ep.197]
NAD*-linkeddehydrogenasesPyruvatedehydrogenase,isocitrate
dehydrogenase,malatedehydrogenase,a-ketoglutarate
dehydrogenase,glutamatedehydrogenase,glyceraldehyde-3-P
dehydrogenase,lactatedehydrogenase,p-hydroxyacylCoA
dehydrogenase,glycerol3-Pdehydrogenase(cytoplasmic).
NADP'-linkeddehydrogenasesGlucose-6-Pdehydrogenase,6-
Phosphogluconatedehydrogenase,3-ketoacylreductase,Enoyl
reductase,gulonatedehydrogenase.
FAD-linkeddehydrogenasesSuccinatedehydrogenase,fattyacyl
CoAdehydrogenase,glycerol-3Pet*hydrogenase(mitochondrial).

239.WhichofthefollowingisNADPlinked
a)G6PD
b)APDH
c)a-ketoglutaratedehydrogenaes
d)None
CorrectAnswer-A
Ans.is'a'i.e.,G6PD[RefHarper28"lep.175]
Amongstthegivenoptions,onlyG6-PD(glucose-6-phosphate
dehydrogenase)isNADPlinkedenzyme.

240.AldehydedehydrogenaserequiresNAD
as?
a)Cofactor
b)Apoenzyme
c)Coenzyme
d)None
CorrectAnswer-C
Ans.isC.i.e.,Coenzyme[RefReadbelow]
Thecompleteenzyme,i.e.proteinpart(apoenzyme)withitsnon-
proteinpartiscalledHoloenzyme.
Alcoholdehydrogenaseistheenzyme(protein)partofcomplete
enzyme(Holoenzyme).Thus,alcoholdedydrogenaseitselfis
apoenzyme.
Itrequiresnon-proteinpartNAD,whichisanorganiccomponent.
So,NADactsascoenzymeforalcoholdehydrogenase.

241.Apoenzymeis?
a)Cofactor
b)Coenzyme
c)Proteinmoiety
d)None
CorrectAnswer-C
Ans.is'c'i.e.,Proteinmoiety[RefHarper2815/ep.52]
Thecompleteenzyme(Holoenzyme)ismadeofproteinportion
(apoenzyme)andcofactor/coenzyme.

242.Inxanthineoxidasecofactoris?
a)Selenium
b)Zn
c)Molybdenum
d)Mg
CorrectAnswer-C
Ans.is'c'i.e.,Molybdenum[Ref:Essentialbiochemistryp.786]
Twoimportantenzymesusingmolybdenumarexanthineoxidase
andsulfiteoxidase.

243.Whatareisoenzyme-
a)Physicallysameformsofdifferentenzymes
b)Physicallydisinctformsofsameenzyme
c)Formsofsameenzymethatcatalyzedifferentreactions
d)Formsofdifferentenzymethatcatalyzesamereactions
CorrectAnswer-B
Ans.is'b'i.e.,Physicallydisinctformsofsameenzyme
Isoenzymesarethephysicallydistinceformsofthesameenzyme.
Theycatalyzethesamechemicalreactionorreactionsbutdiffer
fromeachotherstructurally?,electrophoretically?and
immunologically.
Isoenzymespossessquaternarystructure,andaremadeupoftwo
orthreedifferentsubunit?(multimericQ).
Thesubunitshaveslightlydifferentprimarystructures.Isoenzymes
catalyzethesamereactionandactonsamesubstrate?,butwith
differentKm?andVmaxvalues,i.e.,isozymeshavedifferent
kinetics?.
Theisoenzymescanbeseparatedfromeachotherby
electrophoretic,chromatographicorimmunochemicaltechniques.
Separationandquantitationofisoenzymescangiveinformationof
greatdiagnosticimportanceasthetissuedistributionofisoenzymes
isquitespecific

244.Enzymecausingcovalentbondcleavage
withouthydrolysis?
a)Lyase
b)Ligase
c)Hydrolase
d)Transferase
CorrectAnswer-A
Ans.is'a'i.e.,Lyase[RefClassificationofenzymesfromyour
notes]
Cleavagebyhydrolysis(additionofwater)Hydrolases
Cleavagewithouthydrolysis(withoutadditionofwater)Lyases

245.Enzymesactby?
a)Increaseinactivationenergy
b)Decreaseinactivationenergy
c)Shiftequilbriumconstant
d)None
CorrectAnswer-B
Ans.is'b'i.e.,Decreaseinactivationenergy
Inachemicalreaction,thesubstratehastobeconvertedtoahigher
energyform(calledtransitionform)beforeitcanformthereaction
products.
Thetransitionstateisstructurallyanintermediatebetweenthe
substrateandtheproduct,andrepresentsthehighestenergy
arrangementofatoms.
Therefore,itisunstable;onceformed,itdecomposesalmost
immediatelytoformthereactionproduct.
So,thishighenergyintermediateactsasenergybarrier,separating
thesubstratesandtheproducts.
Thisbarrier,calledthefreeenergyofaction,istheenergydifference
betweentheenergyofthesubstratesandhighenergy
intermediates.
Inotherwords,initiallysomeenergymustbeputintothesubstrate
forconversionintotransitionstate(high-energyintermediate);thisis
thefreeenergyofactivation.
Theenzymesspeedupthechemicalreactionbyloweringthe
magnitudeoftheactivationenergybarrier,i.e.,freeenergyof
activation.

246.Kmvalueisdefinedas:
a)SubstrateconcentrationatVmax/2
b)SubstrateconcentrationoftwiceVmax
c)SubstrateconcentrationofthriceVmax
d)SubstrateconcentrationofonethirdVmax
CorrectAnswer-A
Ans.A.SubstrateconcentrationatVmax/2

247.ThepredominantisoenzymeofLDH
occuringinliverinjuryis?
a)LDH-1
b)LDH-2
c)LDH-4
d)LDH-5
CorrectAnswer-D
Ans.is'd'i.e.,LDH-5[Ref:Essentialsofbiochemistryp.756]
PredominantforminliverisLDH5.

248.Specificactivityofenzymeis?
a)limo'ofenzymepergramofsubstrate
b)Enzymeunitspermgofprotein
c)Conc.ofsubstratetransformedperminute
d)None
CorrectAnswer-B
Ans.is'b'i.e.,Enzymeunitspermgofprotein[RefLippincott's
3"1/ep.


249.Typeofinhibitionofaconitaseby
Transaconitateis?
a)Non-competitive
b)Competitive
c)Allosteric
d)None
CorrectAnswer-B
Ans.is'b'i.e.,Competitive[RefEssentialsofBiochemistryp.
685]


enzymes
competitiveinhibitors
1
Lactatedehydrogenase
Oxamate
2
Aconitase
Transaconitate
3
Succinate
MalanateQ
dehydrogenase
4
HMG-CoAreductase
HMG,Lovastatin
5
Dihydrofolatereductase
Amethopterin,Methotrexate
6
Xanthineoxidase
Allopurinol
7
Alcoholdehydrogenase
Ethanol
8
Carbonicanhydrase
Acetazolamide
9
Digoxin
Na-KATPase
10
5-Fluorouracil
Thymidylatesynthase

250.Functionalplasmaenzymeis?
a)Fibrinogen
b)LDH
c)SGOT
d)SGPT
CorrectAnswer-A
Ans.is'a'i.e.,Fibrinogen[Refharper's28th/ep.59]
Clotingfactors(includingfibrinogen)arefunctionalplasmaenzymes.

251.Substratelevelphosphorylationisby
a)Pyruvatekinase
b)Phosphofructokinase
c)Hexokinase
d)ATPsynthase
CorrectAnswer-A
Ans.'A'Pyruvatekinase
SubstrateLevelPhosphorylation-
InGlycolysis-
Phosphoglyceratekinase
Pyruvatekinase
InCitricAcidCycle-
Succinatethiokinase

252.Substratelevelphosphorylationoccurinstepcatalysedbywhichofthe
followingenzymeinTCAcycle?
a)Isocitratedehydrogenase
b)Malatedehydrogenase
c)Aconitase
d)Succinatethiokinase
CorrectAnswer-D
SuccinatethiokinaseistheenzymethatgeneratesATPdirectlyby
substrate-levelphosphorylation.InKrebscyclesuccinatethiokinase
catalyzetheconversionofSuccinylCoAintosuccinate.Inthisstep
GDPisphosphorylatedtoGTP.GTPcanthenbeconvertedtoATP
byreactingwithanADPmolecule.
Substrate-levelphosphorylationisatypeofmetabolismthatresults
intheformationandcreationofadenosinetriphosphate(ATP)or
guanosinetriphosphate(GTP)bythedirecttransferanddonationof
aphosphoryl(P03)grouptoadenosinediphosphate(ADP)or
guanosinediphosphate(GDP)fromaphosphorylatedreactive
intermediate.
Inglycolysissubstratelevelphosphorylationoccurintwosteps:
Conversionof1,3BPGto3Phosphoglyceratecatalyzedby
Phosphoglyceratekinase
ConversionofPhosphoenolpyruvatetopyruvatecatalyzedby
Pyruvatekinase
Ref:TextbookofBiochemistryByDMVasudevan,3rdEdition,
Page195

253.InETC,cyanideinhibits?
a)ComplexI
b)CytochromeCoxidase
c)ComplexIV
d)ComplexIII
CorrectAnswer-B:C
Ans.is'b'i.e.,CytochromeCoxidase&'c'i.e.,ComplexIV
[RefHarper29th/ep.127,28th/ep.108,109;Vasudevan6thlep.
234]
*ComplexI:-Barbiturates(amobarbital),piercidinA,rotenone,
chlorpromazine,guanithidine.
*ComplexII:-Carboxin,TTFA,malonate.
*ComplexIII:-Dimercaprol,BAL,actinomycinA,Naphthyloquinone.
*ComplexIV(cytochromecoxidase):-Carbonmonoxide(CO),
cyanide(CN-),H2S,azide(N')

254.NADPHviaglycerophosphateshunt
makeshowmanyATP?
a)1
b)2
c)3
d)4
CorrectAnswer-B
Ans.is>b'i.e.,2[Ref:Harper29"/ep.129-130]
Inglycerophosphateshuttle,themitochondrialenzymeislinkedto
respiratorychain(ETC)viaaflavoprotein,Soonly1.5molofATP
areproduced(Accordingtooldercalculations,2ATPmolofATPare
produced).
Inmalateshuttle,themitochondrialenzymeislinkedtoETCvia
NAD,so2.5molofATPareproduced(accordingtoolder
calculations3molofATPareproduced).

255.Enzymeinvolvedinoxidative
phosphorylation?
a)Pyruvatekinase
b)SuccinylCoAthiokinase
c)NADHdehydrogenase
d)None
CorrectAnswer-C
Ans.is'c'i.e.,NADHdehydrogenase[RefHarper29'/ep.126-
128,Vasudevan6thlep.234]
Oxidativephosphorylationtakesphasealongtheelectrontransport
chain(respiratorychain),wheretheATPissynthesizedindirectly
fromcreationofaproteingradientandmovementofprotiensacross
innermitochondrialmembranehelpsinformationofATP.
Theproteingradientiscreatedbylargechangeinfreeenergydueto
transportofelectroninETC.ElectronsentertheETCviaNAD'or
FAD.
ComplexI(NADH-CoQreductase)catalyzesthetransferof
electronfromNADHtocoenzymeQ(CoQ).{NADH-CoQreductase
isalsocalledNADHdehydrogenase.}.

256.Amongthefollowing,themaximum
redoxpotentialisfor:
a)NADH/NAD
b)Succinate/Fumarate
c)Ubiquinone
d)Fe+3/Fe+2
CorrectAnswer-D
Ans.Di.e.Fe+3/Fe+2
Becauseelectronstendtoflowspontaneouslyfromcarriesoflower
redoxpotential(E?)tocarriesofhigherredoxpotential.Theorder
(sequence)ofelectroncarriersinETCofmitochondria(andsothe
increasingorderofredoxpotential)is
Substrate-->NADP+/NADPH-->NADVNADH-->NADH
dehydrogenase(FMN)/NADHdehydrogenase(FMNH2)-->
FAD/FADH2-->UbiquinoneorCoenzymeQ-->Fe3+/Fee*in
cytochromesb-->C-->a-+a3-->02

257.InETCNADHgenerates-
a)1ATPs
b)2ATPs
c)3ATPs
d)4ATPs
CorrectAnswer-C
Ans.is'>c'i.e.,3ATPs[Ref:Harper28'"/ep.106,Vasudevan
5thiep.231]
TheenergyliberatedofsiteI(complexI)isusedtosynthesize1
ATPmolecule,atsiteII(complexIII)isusedtosynthesize1ATP
moleculeandatsiteIII(ComplexIV)isusedtosynthesize1/2ATP
molecule.
Thus,when1NADHmoleculeenterstherespiratorychain,it
produce2.5moleculesofATP?.WhenImoleculeofFADH2
enterstherespiratorychainonly1.5moleculesofATPare
produced?
assiteIofenergyliberationisbypassed.
Note:PreviouslyitwasassumedtheNADHproduces3ATPsand
FADgenerates2ATPs.Recentexperimentsshow-thattheseold
valuesareoverestimatesandNADHproduces2.5ATPs?and
FADH2produces1.5ATPs?.


258.Inglycolysis,inorganicphosphateis
usedreaction,catalyzedby?
a)Enolase
b)Pyruvatekinase
c)Glyceraldehyde-3-pdehydrogenase
d)Aldolase
CorrectAnswer-C
Ans.is'c'i.e.,Glyceraldehyde-3-pdehydrogenase[RefHarper's
250/ep.171]


259.Trueaboutglycolysis?
a)HexokinaseproduceATP
b)1cycleproduces2ATP
c)Itproducesdirectly2moleculesoflactate
d)Aldolaseproducesirreversiblepolymerization
CorrectAnswer-C
Ans.is'c'i.e.,Itproducesdirectly2moleculesoflactate[Ref
Harper29th/ep.170-177]
Eachcycleofanaerobicglycolysisproduces2moleculesoflactate
(lacticacid)whereasinaerobicconditionitproduces2moleculesof
pyruvate.
Optionbistrickyone.Eachglycolyticcycleproduces4ATPs(not
2ATPs).Butoutofthese4ATPs,2areusedinthecycleitself.
Therefore,thereisnetgainof2ATPs.So,:-
1. Eachcycleofglycolysisproduces-4ATPs.
2. Eachcycleutilizes2ATPs
3. Netgainineachcycle-2ATPs

260.Enzymesofglycolysisarefoundin:
a)Cytosol
b)Cellmembrane
c)Mitochondria
d)Ribososmes
CorrectAnswer-A
Alloftheenzymesofglycolysisarefoundinthecytosol.
Ref:Harper28thedition,chapter18.

261.Theratelimitingstepinglycolysisis
catalyzedby?
a)Pyruvatekinase
b)Enolase
c)Glucokinase
d)Phosphofructokinase
CorrectAnswer-A:C:D
Ans.is'd>a&c'i.e.,Phosphofructokinase>Pyruvatekinase&
Glucokinase[RefLippincott's5thlep.99]
Phosphofructokinase,glucokinaseandpyruvatekinasearerate
limitingenzymesofglycolysis.However,phosphofructokinaseis
themostimportantone.

262.NumberofATPmoleculesandNADH
formedineachcycleofglycolysis?
a)4ATP,2NADH
b)2ATP,2NADH
c)4ATP,4NADH
d)2ATP,4NADH
CorrectAnswer-A
Ans.is'a'i.e.,4ATP,2NADH
Enegeticsofglvcolysis
Duringglycolysis2ATPareutilizedand4ATPareproducedat
substratelevel.2reducingequalentsNADH'areproducedand
reoxidizedbyelectrontransportchain,togenerata5ATPmolecules
(2.5ATPperNADH'molecule).Thustotal9ATPmoleculesare
producedand2areutilized,i.e.,Thereisnetgainof7ATP
moleculesinaerobicglycolysis.
Inanaerobicconditions,thereoxidationofNADHbyelectron
transportchainispreventedandNADHgetsreoxidizedby
conversionofpyruvatetolactatebylactatedehydrogenase.Thus,in
anaerobicglycolysisonly4ATPareproducedatsubstratelevel.
Therefore,thereisnetgainof2ATPmoleculesinanaerobic
glycolysis.
Note:-PreviouscalculationsweremadeassumingthatNADH
produces3ATPsandFADH2generates2ATPs.Thiswillamountto
anetgenerationof8ATPsperglucosemoleculeduringglycolysis.
Recentexperimentsshowthattheseoldvaluesareoverestimates
andNADHproduces2.5ATPsandFADH2produces1.5ATPs.
Thus,netgenerationisonly7ATPsduringglycolysis.

263.Allareactivatedbyinsulinexcept?
a)Lipoproteinlipase
b)Pyruvatekinase
c)Hormonesensitivelipase
d)Acetyl-CoAcarboxylase
CorrectAnswer-C
Ans.isC.i.e.,Hormonesensitivelipase
Enzymes/Pathwaysactivatedbyinsulin
Glycolysis:PFK-1,Pyruvatekinase,glucokinase,PDH.
Glycogenesis:Glycogensynthase.
Lipogenesis:Acetyl-Co-carboxylase,Fattyacidsynthase.
Cholesterolsynthesis:HMG-CoAreductase.
Triglyceridesynthesis:Acyl-CoAglycerol-3-Ptransferase,glycerol
kinase.
Lipoproteindegradation:Lipoproteinlipase

264.Nonsensecodonsbringabout?
a)Elongationofpolypeptidechain
b)Pre-translationalmodificastionofprotein
c)Initiationofproteinsynthesis
d)Terminationofproteinsynthesis
CorrectAnswer-D
Ans.isD.i.e.,Terminationofproteinsynthesis[RefHarper
28"lep.353,354]
Stoporterminationornonsensecodons:-Threeofthe64possible
nucleotidetripletsUAAQ(amber0),UAGQ(Ochre)andUGAQ(opal)
donotcodeforanyaminoacid.Theyarecallednonsensecodons
thatnormallysignalterminationofpolypeptidechains0.Thus,though
thereare64possibletripletcodons,only61codesfor20amino
acids(asremainingthreearenon-sensecodons).

265.ATPisconsumedatwhichstepof
glycolysis
a)Enolase
b)Hexokinase
c)Pyruvatekinase
d)Isomerase
CorrectAnswer-B
Ans.is'b'i.e.,Hexokinase
ATPisconsumedatreactionscatalysedby-hexokinase,
phosphofructokinaseI.
ATPisproducedatreactionscatalyzedby->phosphoglycerate
kinase,pyruvatekinase.

266.MethioninecanentretheTCAcycleat
whichlevel?
a)Fumarate
b)Oxabacetate
c)Succinyl-CoA
d)Citrate
CorrectAnswer-C
Ans.is'c'i.e.,Succinyl-CoA

267.RatelimitingstepsinTCAcycle?
a)a?KetoglutarateSuccinylCoA
b)Citrate-Isocitrate
c)SuccinylCoA4Succinate
d)Succinate-Fumarate
CorrectAnswer-A
Ans.is'a'i.e.,alpha-Ketoglutarade-SuccinylCoA[RefHarper's
28th/ep.147,154]
RegulatingstepsinTCACyclearecatalyzedby:a)Citrate
Synthase:CatalyzescondensationofacetylCoA&oxaloacetateto
fromcitrate.
Isocitratedehydrogenase:Catalyzestheconversionofisocitrateto
a-ketoglutaratebydecarboxylation.
a-ketoglutaratedehydrogenase:Catalyzestheconversionofa-
ketoglutaratetosuccinylCoA

268.RatelimitingstepinTCAcycleis
catalyzedby-
a)a-ketoglotaratesynthase
b)Fumarase
c)Aconitase
d)Thiokinase
CorrectAnswer-A
Ans.is'a'i.e.,a-ketoglotaratesynthase[RefHarper28th/ep.
147,154]


269.Fluroacetateinhibits?
a)Citratesynthetase
b)Aconitase
c)Succinatedehydrogenase
d)Alphaketoglutaratedehydrogenase
CorrectAnswer-B
Ans.is'b'i.e.,Aconitase[RefHarper28thIep.145]

270.Inhibitorsofglycolysisareallexcept?
a)Fluoride
b)Fluoroacetate
c)Arsenite
d)Iodoacetate
CorrectAnswer-B
Ans.is'b'i.e.,Fluoroacetate[RefHarper's28`"/ep151]
FluoroacetateisaninhibitorofTCAcycle.

271.Thebiosynthesisoftheenzymepyruvate
carboxylaseisrepressedby?
a)Insulin
b)Glucagon
c)Cortisol
d)Epinephrine
CorrectAnswer-A
Ans.is'a'i.e.,Insulin[Ref:Harper28th/ep.168]
Gluconeogenesisisregulatedbyfourkeyenzymes:(i)Pyruvate
carboxylase;(ii)Phosphoenolpyruvatecarboxykinase;(iii)Fructose-
1,6-bisphosphatase;and(iv)Glucose-6-phosphatase.
Thehormoneglucagon,epinephrineandglucocorticoidsstimulate
gluconeogenesis,byinducingtheseenzymes.Conversely,insulin
inhibitsgluconeogenesisbyrepressingtheirsynthesis.

272.Wherecanglucose6phosphatenotbe
convertedtoglucose?
a)Muscle
b)Liver
c)Adiposetissue
d)Kidney
CorrectAnswer-A
Ans.is'a'i.e.,Muscle[Ref:Harper250lep.178-179]
Glucose-6-phosphataseisabsentinmusclestherefore,glucose-6-
phosphatecannotbedegradedtofreeglucoseinmuscles.
Moreover,glucose-6-phosphatecannotdiffuseoutofthemuscles.
Therefore,musclecannotprovideglucosetomaintainbloodglucose
level.Rather,muscleglycogenactsasasourceofenergy;the
glucose-6-phosphateenterstheglycolysistoproduceenergy.

273.Commonintermediatebetween
gluconeogenesisandfattyacid
synthesis?

a)Glucose-6-phosphate
b)Acetyl-CoA
c)Citrate
d)Succinyl-CoA
CorrectAnswer-C
Ans.is'c'i.e.,Citrate[RefHarper29thlep.188]
CitrateinGluconeogenesis
Gluconeogenesisinvolvesglycolysis,thecitricacidcycleandsome
specialreactions.
Citrateisanintermediarymetabolismofgluconeogenesis(through
TCAcycle).

274.Glycogensynthaseisactivatedby?
a)Insulin
b)Glucagon
c)Epinophrine
d)AMP
CorrectAnswer-A
Ans.is'a'i.e.,Insulin[RefHarper28th/ep.159-161,Lehinger
5th/ep.603,604]


275.Glycogenstoragediseasewhich
presentsaslysosomalstoragedisease?
a)Vongierke'sdisease
b)Pompesdisease
c)Mcardle'sdisease
d)Andersen'sdisease
CorrectAnswer-B
Ans.is'b'i.e.,Pompesdisease[RefLippincott's4th/ep.129-
131]
TypeIIglycogenstoragedisease(Pompesdisease)istheonly
glycogenstoragediseasethatisalysosomalstoragedisease

276.Aldolase-Bisinvolvedinmetabolismof
?
a)Galactose
b)Fructose
c)Sucrose
d)None
CorrectAnswer-B
Ans.is'b'i.e.,Fructose[RefHarper's28th/ep.179)

277.Inhumans,ascorbicacidcannotbe
synthesizedbecauseof?
a)DeficiencyofG6PD
b)Deficiencyofxylulosekinase
c)DeficiencyofL-gulonolactoneoxidase
d)Deficiencyofphosphoglucomutase
CorrectAnswer-C
Ans.is'c'i.e.,DeficiencyofL-gulonolactoneoxidase
Inman,otherprimatesandguineapigs,ascorbicacidcanbe
synthesizedduetoabsenceofL-glucanolactoneoxidase,an
enzymeusedinloweranimalstosynthesiseascorbicacidasa
byproductofuronicacidpathway(glucuronicacidcycle).

278.Glucoseoxidaseconvertsglucoseto?
a)Gluconicacid
b)Glucuronicacid
c)Iduronicacid
d)Galacticacid
CorrectAnswer-A
Ans.is'a'i.e.,Gluconicacid[Ref:Internet]
Oxidase-peroxidaseenzymesystemisusedtodetermineglucosein
urine&blood.
Glucoseoxidaseenzymeproduceshydrogenperoxide&gluconic
acidfromglucose.PeroxidasecatalysesthereactionofH202with
colourlesspotassiumiodidetobrowniodide.Thisproducesacolour
change,theintensityofwhichmayindicateglucoseconcentrationin
sometests(suchasBoehringer,Diastix).

279.Glucoseisconvertedtoglucuronateby?
a)Oxidationofaldehydegroup
b)Oxidationofterminalalcohol
c)Oxidationofboth
d)None
CorrectAnswer-B
Ans.is'b'i.e.,Oxidationofterminalalcohol[RefChatterjee8th/e
p.29]
Whenaldosesugarsareoxidizedtheymayformthreedifferent
sugaracid,dependinguponoxidationofaldehydegroup(atC-1)or
terminalalcoholgroup(atC-6).
1. Aldonicacid:-Oxidationofanaldosewithhypobromousacid
(HOBr)oxidisesonlyaldehydegroupandconvertittocarboxyl
grouptoformaldonicacid.Forexample,glucoseisoxidizedto
gluconicacid.
2. Saccharicacid:-Oxidationofaldoseswithnitricacidconvertboth
aldehydeandterminalprimaryalcoholgroupstocarboxylgroup,
formicsaccharicacid.Forexample,glucoseisoxidizeto
glucosaccharicacid.
3. Uronicacid:-Whenanaldoseisoxidizedinsuchawaythatthe
terminalprimaryalcoholisconvertedistocarboxylwithoutoxidation
ofaldehydegroup,auronicacidisproduce.Forexample,glucoseis
oxidizedtoglucuronicacid.

280.Increaseduricacidlevelsareseenin
whichglycogenstoragedisease?
a)TypeI
b)TypeII
c)TypeIII
d)TypeIV
CorrectAnswer-A
Ans.is'a'i.e.,TypeI[RefHarper29m/ep.339]
PurineoverproductionandhyperuricemiainvonGierkedisease
(glucose-6-phosphatasedeficiency)occurssecondarytoenhanced
generationofPRPPprecursor,i.e.ribose-5-phosphate(apentose).
Inglucose-6-phosphatasedeficiency,glucose-6-phosphatecannot
beconvertedtoglucose.Accumulatedglucose-6-phosphateisthen
metabolizedviaHMPshunt,whichinturngenerateslargeamounts
ofribose-5-phosphate,aprecursorofPRPP.Theincreased
synthesisofPRPPthenenhancesdenovosynthesisofpurine
nucleotides

281.Gulonatedehydrogenaserequires?
a)NADP
b)NAD
c)FAD
d)FMN
CorrectAnswer-A
Ans.is'a'i.e.,NADP[RepHarper29t5/ep.201-204]
Gulonatedehydrogenaseisanenzymeinglucuronicacidpathway
(uronicacidpathway)thatrequiresNADP'.

282.Allarereducingsugarsexcept-
a)Sucrose
b)Lactose
c)Glucose
d)Fructose
CorrectAnswer-A
Ans.A.Sucrose
Disaccharides
SugarUnits
Linkage
Trehalose(Sugarof
DGlucose+
1-------1linkage
insecthemolymph, DGlucose
yeastandfungi)
Sucrose(CaneSugar) DGlucose
1-------2
+DFructose
linkage

283.MutationinGLUT-2causes?
a)Dandywalkersyndrome
b)Fanconibeckersyndrome
c)Beckwithsyndrome
d)Menke'sdisease
CorrectAnswer-B
Ans.is'b'i.e.,Fanconibeckersyndrome[RefDiagnosticof
Endocrinefunctionischildrenandadolescentsp.271]
GLUT2isexpressedinpancreatic13-cells,hepatocytesandin
epithelialcellsofkidneyandintestine.
Fanconi-BickelsyndromeiscausedbyGLUT2mutation,adisease
characterizedbyproximalrenaltubulopathy,impairedglucose
homeostasisandhepatomegaly.

284.Aminosugarareformedforms?
a)Glucose-1-phosphate
b)Glucose-6-phosphate
c)Fructose-1-phosphate
d)Fructose-6-phosphate
CorrectAnswer-D
Ans.is'd'i.e.,Fructose-6-phosphate[RefDineshpuri3rdlep.
170]
Aminosugarsaredervativesofmonosaccharidesinwhichanamino
groupreplacesthe-OHresidueoncarbon-2ofhexose,suchas
glucose,galactoseandmannose,Thecorrespondingcompounds
areglucosamine,galactosamineandmannosamine,respectively.
Theaminoacidgroupisusuallyacetylated,e.g.N-
acetylglucosamineorN-actylgalactosamine.
Theaminosugarsarerequiredforthesynthesisofglycolipids,
glycoproteinsandproteoglycans.
Theyaresynthesizedfromfructose-6-phosphate

285.Differencebetweenganglioside&
cerebroside,allexcept?
a)Charge
b)PresenceofNANA
c)Presenceofcarbohydrate
d)Nativetissue
CorrectAnswer-C
Ans.is'c'i.e.,Presenceofcarbohydrate[RefLippincott's4th/e
p.208-210]
Bothgangliosideandcerebrosidecontaincarbohydrateasbothare
glycolipids.
NANAispresentinganglioside(notincerebroside).

286.1"acetylgroupdonorinfattyacid
synthesisis?
a)MalonylCoA
b)Palmitate
c)AcetylCoA
d)Citrate
CorrectAnswer-C
Ans.is'c'i.e.,AcetylCoA[Ref:Harper29"lep.216-217]
Acetyl-CoAactsasaprimertodonate1st2carbonatoms(C-15and
C-16)ofpalmitate.TheadditionofallthesubsequentC2unitsisvia
malonyl-CoA.
Propionyl-CoAactsasprimer(fordonatingfirst3carbonatoms)in
thesynthesisofodd-carbonnumberfattyacids.

287.Onlyvitaminthathelpincarbonfixation
?
a)Folicacid
b)Pantothenicacid
c)Niacin
d)Thiamine
CorrectAnswer-A
Ans.is'a'i.e.,Folicacid

288.Ratecontrollingenzymeoffattyacid
synthesis-
a)Thioesterase
b)Transacetylase
c)Acetyl-CoAcarboxylase
d)Ketacylsynthase
CorrectAnswer-C
Ans.is'c'i.e.,Acetyl-CoAcarboxylase[RefHarper29th/ep.217,
220]
Productionofmalonyl-CoAistheinitial?andratelimitingstepinfatty
acidsynthesis.
Acetyl-CoAneedstobeconvertedtoactivatedform,whichwillserve
asthedonorofcarbonunitstogrowingfattyacidchain.
Malonyl-CoA?)a3-carboncompoundissuchactivatedform.
Itisproducedbycarboxylationofacetyl-CoA,areactioncatalyzed
byacetyl-CoAcaroxylase?.Acetyl-CoAcarboxylaserequiresbiotin
asacofactor?.

289.Whichofthefollowingisrequiredfor
fattyacidsynthesis?
a)NADPH
b)NADH
c)FADH
d)None
CorrectAnswer-A
Ans.is'a'i.e.,NADPH[RefHarper29th/ep.216-217]
FattyacidsynthesistakesplaceincytosolQ.
Acetyl-CoAistheimmediatesubstrateforlipogenesisandsynthesis
alwaysendsinformationofplamiticacie.
Inhumans;liverandlactatingmammaryglandsarethemainorgans
forlipogenesisQ.
Althoughkidney,brain,lungsandadiposetissuearealsoinvolved,
toalesserextent.
CofactorrequirementsforfattyacidsynthesisareNADPH,ATP,
Mr1.2,biotinandHCO3-(asasourceofCO2).
Becausemostfattyacidshavemultiplesoftwocarbons,theyare
synthesizedfromsuccessiveadditionoftwocarbonunits,thedonor
ofwhichisacetyl-CoA.
So,thebasicbuildingblockisacetyl-CoA?whichisthesourceofall
thecarbonatomsofthefattyacidbeingsynthesized.

290.Citrateusedinfattyacidsynthesisuses
whichenzyme?
a)CitrateSynthase
b)ATPcitratelyase
c)Aconitase
d)Malicenzyme
CorrectAnswer-B
Ans.is'b'i.e.,ATPcitratelyase[RefHarper's28th/ep.193,196]
Glucoseistheprimarysubstrateforlipogenesisandacetyl-CoA
(immediatesubstrateforfattyacidsynthesis)isformedfromglucose
viaoxidationofpyruvatewithinthemitochondria.
However,acetyl-CoAcannotpenetrateinnermitochondrial
membrane.
Thereforeitistransferredintheformofcitrate.
Citrateisformedinthemitochondrialmatrixbythecondensationof
acetyl-CoAwithoxaloacetate(firstreactionincitricacidcycle).
Thencitrateistransportedintocytosolviathetricorboxylate
transporterinexchangewithmalate.
Incytosol,citrateiscleavedbyATP-citratelyasetooxaloacetate
andacetyl-CoA.

291.Whichofthefollowingisw-6fattyacid-
a)Cervonicacid
b)Linoleicacid
c)Alphalinolenicacid
d)Elaidicacid
CorrectAnswer-B
Ans.is'b'i.e.,Linoleicacid[RefHarper28th/ep.123]
AlphalinolenicacidLinoleicacidOleicacid
ClupandonicacidGamalinolenicacidNervonicacid
CervonicacidArachidonicacidElaidicacid

292.Linoleicacidis-
a)w-3fattyacid
b)w-6fattyacid
c)w-9fattyacid
d)Saturatedfattyacid
CorrectAnswer-B
Ans.isb'i.e.,w-6fattyacid[RefHarper28thlep.123]

293.Lipogenesisoccursin?
a)Liver
b)Skeletalmuscles
c)Myocardium
d)Lungs
CorrectAnswer-A:D
Ans.is'a>d'i.e.,Liver>Lungs
Fattyacidsynthesistakesplaceincytosol?.
Acetyl-CoAistheimmediatesubstrateforlipogenesisandsynthesis
alwaysendsinformationofpalmiticacid.

294.Whichofthefollowingismonoenoicacid
?
a)Arachidonicacid
b)Linoleicacid
c)Oleicacid
d)Linolenicacid
CorrectAnswer-C
Ans.C.Oleicacid
Inhumans;liverandlactatingmammaryglandsarethemainorgans
forlipogenesisQ.
Althoughkidney,brain,lungsandadiposetissuearealsoinvolved,
toalesserextent.
CofactorrequirementsforfattyacidsynthesisareNADPH,ATP,
Mn.2,biotinandHCO3-(asasourceofCO).
Becausemostfattyacidshavemultiplesoftwocarbons,theyare
synthesizedfromsuccessive

295.Whichofthefollowingfattyacidhas
maximumnumberofcarbonatoms?
a)Oleicacid
b)Linolenicacid
c)Arachidonicacid
d)Cervonicacid
CorrectAnswer-D
Ans.is'd'i.e.,Cervonicacid
Cervonicacidhas22carbonatoms,morethanoleicacid(18C),
linolenicacid(18C)andarachidonicacid(20C).
So,thebasicbuildingblockisacetyl-CoAQwhichisthesourceofall
thecarbonatomsofthefattyacidbeingsynthesized.

296.Chylomicronremnantsareassociated
with?
a)Apo-A
b)Apo-B100
c)Apo-E
d)Apo-C
CorrectAnswer-C
Ans.is'c'i.e.,Apo-E

297.Apoprotein-C?
a)Activateslipoproteinlipase
b)Inactivateslipoproteinlipase
c)Facilitatestriglyceridetransport
d)Alloftheabove
CorrectAnswer-D
Ans.is'd'i.e.,Alloftheabove[RefHarper28th/ep.213]
ApoC-IIisanactivatorofLPL,wherasapoA-IIandapoC-IIIactas
inhibitorsofLPL.
Asapo-CisacomponentofchylomicronsandVLDL,itfacilitates
transportofTGs.

298.Whichisnottrueofchylomicrons?
a)Lowestdesnsity
b)Max.contentisTGs
c)Max.contentischolesterol
d)Largestsize
CorrectAnswer-C
Ans.is'c'i.e.,Max.contentischolesterol[Ref:Harper29th/ep.
238,Chatterjea8thlep.445,446]


299.OxidisedLDLismoreathreogenic
because?
a)BindstoApoE
b)BindstoscavengerR
c)BindstoATPbinding
d)Accumulatesinmacrophages
CorrectAnswer-D
Ans.is'd'i.e.,Accumulatesinmacrophages[RefRobbin's8thle
p.837]
MacrophagesengulfLDLcholesterolandformfoamcellsformation
ofearliestlesion,i.e.fattystreak.
Macrophagesalsoformoxygenfreeradicalsthatcauseoxidationof
LDLtoyieldoxidizedLDL(modifiedLDL).

300.LCATdeficiencyincreasesthefollowing
a)HDL
b)LDL
c)VLDL
d)Chylomicron
CorrectAnswer-A
Answer:A.HDL
Lecithincholesterolacyltransferasedeficiency(LCATdeficiency)isa
disorderoflipoproteinmetabolism.
AdeficiencyofLCATcausesaccumulationofunesterified
cholesterolincertainbodytissues.Cholesteroleffluxesfromcellsas
freecholesterolandistransportedinHDLasesterifiedcholesterol.
LCATistheenzymethatesterifiesthefreecholesterolonHDLto
cholesterolesterandallowsthematurationofHDL.
LCATistheenzymethatesterifiesthefreecholesterolonHDLto
cholesterolesterandallowsthematurationofHDL.LCATdeficiency
doesnotallowforHDLmaturationresultinginitsrapidcatabolismof
circulatingapoA-1andapoA-2.TheremainingformofHDL
resemblesnascentHDL.

301.Hormonesensitivelipaseisinhibitedby?
a)Thyroidhomone
b)Insulin
c)GH
d)ACTH
CorrectAnswer-B
Ans.is`b'i.e.,Insulin[RefHarper29"Vep.246,247]

302.Lecithinehydrolysisyeilds?
a)Choline
b)Pyruvate
c)Glycine
d)None
CorrectAnswer-A
Ans.is'a'i.e.,Choline[RefHarper29th/ep.233]

303.Omegaoxidationoffathyacidsoccur?
a)ER
b)Mitochondria
c)Cytosol
d)None
CorrectAnswer-A
Ans.is'a'i.e.,ER[RefHarper's28th/ep.185,195]
Beta(a)Mitochondria
Alpha(a)Endoplasmicreticulum,Mitochandria
Omega(w)Microsomalsystem(smoothER)
a-oxidationofverylongchainFAPeroxisomes

304.Whatistrueregardingmediumchain
fattyacids?
a)Don'trequirepancreaticlipase
b)Notdepositedinadiposetissue
c)Diffusedirectlyintoportalcirculation
d)Alloftheabove
CorrectAnswer-D
Ans.is'd'i.e.,Alloftheabove[RefVasudevan6"p.160]
Metabolismofshortchainfattyacids(SCFAs)andmediumchain
fattyacids(MCFAs)isdrasticallydifferentfromlongchainfattyacids
(LCFAs).
SCFAsandMCFAsdonotrequirepancreaticlipaseandbilesalts
fordigestion,butrequiredforLCFAsdigestion.
SCFAsandMCFAsaredirectlyabsorbedfromintestineintoportal
circulation,whereasLCFAsaretakenbylymphatics,after
incorporationintochylomicrons.
SCFAsandMCFAsareoxidizedbyperipheralcellsandarenot
usedforstorage.LCFAsareesterifiedwithglyceroltoform
triacylglycerol,storageformoflipid

305.Whatwillyougivetostopchyluriain
diet?
a)SmallchainFA
b)MediumchainFA
c)LongchainFA
d)Omega3unsaturatedFA
CorrectAnswer-B
Ans.is'b'i.e.,MediumChainFA[RefHarrison's18thiep.294]
Mediumchainfattyacidsdirectlyentertheportalveinon
absorption,bypassingthelymphatics.Hence,theyareusedinchyluria

306.Inapersonfastingovernightwith
carnitinedeficiency,followingchemicals
increaseinquantityinblood?

a)Glucose
b)Fattyacids
c)Aminoacids
d)Ketonebodies
CorrectAnswer-B
Ans.is'b'i.e.,Fattyacids[Ref:Harper29th/ep.208-209,214]
Instarvation,thereisincreasedhydrolysisofTGs(ofadipose
tissues)intoglycerolandfattyacids.Fattyacidsarefurtheroxidized
by[3-oxidationinthemitochondria.
Carnitineisrequiredfortransportedofactivatedfattyacidinto
mitochondriafor3-oxidation.
Ifcarnitineisdeficient,fattyacidscannotbetransferredintothe
mitochondria,buttheyarecontinuouslyproducedduetohydrolysis
ofTGs(instarvationthereisdecreasedinsulintoglucagonratio,
whichstimulateshydrolysisofTGs).
Thusfreefattyacidlevelisincreasedasthereisincreased
productionbutnoutilization(p-oxidation).

307.Serumappearymilkywhitein?
a)IncreasedLDL
b)IncreasedHDL
c)IncreasedVLDL
d)IncreasedChylomicrons
CorrectAnswer-D
Ans.is'd'i.e.,IncreasedChylomicrons[RefHarrison18thlep.
3151]
"Thefastingplasmaisturbid,andifleftat4?C(39.2?F)forafew
hours,thechylomicronsfloattothetopandformacreamy
supernatant".
Chylomicronsarethelargestlipoproteinmoleculewithmaximum
lipidcontents.Therefore,theyhaveleastdensityandfloatonthetop
givingacreamy(milky)supernatant.

308.Adiposetissuefatmetabolismisdoneby
?
a)Lipoproteinlipase
b)Hormonesensitivelipase
c)Acidlipase
d)Acidmaltase
CorrectAnswer-B
Ans.is'bi.e.,Hormonesensitivelipase[Ref:Harper29th/echap.
16]
Adiposetissuefat(TGs)metabolismHormonesensitivelipase
Lipoproteins(VLDL&chylomicrons)TGsmetabolism-Lipoprotein
lipase

309.EnzymedeficientinTypeI
Hyperlipidemiais?
a)HMGCoAreductase
b)Lipoproteinlipase
c)Cholesterolacyltransferase
d)Peroxidase
CorrectAnswer-B
Ans.is'b'i.e.,Lipoproteinlipase[Ref:Harper2e/ep.232table
(26.1)]

310.Apo-Edificiencyisseenin
a)TpyeIhypolipoproteinemia
b)TpyeIIhypolipoproteinemia
c)TpyeIIIhypolipoproteinemia
d)TpyeIVhypolipoproteinemia
CorrectAnswer-C
Ans.is'c'i.e.,TpyeIIIhypolipoproteinemis[RefHarisonp.3149,
3150,Chatterjea8th/ep.45]

311.Whichofthefollowingisnotaffectedin
Abetalipoproteinemia?
a)LDL
b)VLDL
c)HDL
d)IDL
CorrectAnswer-C
Ans.is'c'i.e.,HDL[RefChatterjea8thlep.462,463&DineshPuri
3rdlep.248]
Hypolipoproteinemia
Inthisgroupofdisordersconcentrationofoneormorelipoproteins
inplasmaisdecreased.Thecommonestofthesedisorders
areabetalipoproteinemia,
hypobetalipoproteinemia
andhypoalphalipoproteinemia.
Abetalipoproteinemia:-Thereisdefectivesynthesisorsecretionof
apoprotein-B(apo-B)inintestineandliver.So,thereisdeficiencyof
apo-Bcontaininglipoproteinsi.e.chylomicrons?,VLDL?,IDL?
andLDL?.Asaresultextremelylow
plasmalevelsofcholesterol
andtriacylglycerols?occurs.HDLlevelsarenormal?asHDLdoes
notcontainapo-B.
Hypobetalipoproteinemia:-Thereisdecreasedsynthesisofapo-B
duetoapo-Bgenemutations.So,apo-Bcontaininglipoproteinsare
synthesizedatlowerrate.ThereisslightdecreaseinVLDL,IDLand
LDLlevels.HDLisnormal.Plasmacholesterolandtriglyceridesare
decreased.
Hypoalphalipoproteinemia(Tangierdisease):-Thereismarked
deficiencyofmajorlipoproteinHDL
(apo-A-Iandapo-A-II),

probablybecauseofacceleratedcatabolism.

312.InZellwegersyndrome,thereis?
a)Accumulationoflongfattyacids
b)Accumulationofshortchainfattyacids
c)Accumulationofverylongchainfattyacids
d)Accumulationofmediumchainfattyacids
CorrectAnswer-C
Ans.is'c'i.e.,Accumulationofverylongchainfattyacids[Ref
Harper29m/ep.554;Chatterjea6thlep.412]
ZellwegerSyndromeisarareinbornerrorofperoxisomalfatty
acidoxidation?
duetoabsenceoffunctionalperoxisomesinall
tissues.
Asaresult,thelongchainfattyacidsarenotoxidizedin
peroxisomesandaccumulateintissuesparticularlyinbrain,liver,
kidneyandmuscleandusuallyresultindeathbyagesix.

313.Autooxidationisseenin?
a)Cholesterol
b)Arachidonicacid
c)Stearicacid
d)Palmiticacid
CorrectAnswer-B
Ans.is'b'i.e.,Arachidonicacid[RefEssentialofbiochemistryp.
736]
Polyunsaturatedfattyacids(PUFAs)undergoperoxidation(auto-
oxidation).
Amongstthegivenoptions,onlyarachidonicacidisPUFA.

314.Whichofthefollowingisalipotropic
factor:?
a)Sphingomyelin
b)Histidine
c)Methionine
d)Bilirubin
CorrectAnswer-C
Ans.is'c'i.e.,Methionine[RefEssentialsofbiochemistry/p.761]
Lipotropicfactorsaresubstanceswhichpreventaccumulationof
fat(TGs)inliver.
Primarylipotropicfactorscholine,betaine,methionine,lecithine,
inositol.
Otherfactorswithsomelipotropicaction:-Vitamin-13,2,folic
acid,casein,glycine,essentialfattyacids,selenium,vitaminEand
serine.

315.Huntersyndromeisduetodeficiencyof
a)Betagalactosidase
b)Sphingomyelinase
c)IduronateSulfatase
d)Hyaluronidase
CorrectAnswer-C
Ans.is'c'i.e.,IduronateSulfatase

316.Non-Essentialaminoacidis-
a)Tyrosine
b)Phenylalanine
c)Lysine
d)Threonine
CorrectAnswer-A
Ans.A.Tyrosine

317.Indoleringispresentin?
a)Tryptophan
b)Tyrosine
c)Phenylalanine
d)Threonine
CorrectAnswer-A
Ans.is'a'i.e.,Tryptophan[Ref:Lehninger4thIep.80]
Someaminoacidscontainaspecialfunctionalgroupintheir
sidechainwhichprovidesomespecificfunctionstothatamino
acids.Theseare:-

Hydroxylgroupinserineand Guanidiniumin
Imidazolein
threonine
arginineQ
histidine
Amidegroupinaspargineand Benzenein
Phenolin
glutamine
phenylalanine
tyrosine
Indoleringin
Pyrrolidinein
Thioetherinmethionine
tryptophanQ
proline
13-Carboxylin
E-aminoin
Sulphydrylincysteine
glutamicacid
lysine
y-carboxylinglutamicacid

318.Xanthoproteicreactioninvolves-
a)Carbolicacid
b)H2SO4
c)HCL
d)Nitricacid
CorrectAnswer-D
Ans.'D'Nitricacid
Xanthoproteictest:Theringsystemsinphenylalanine,tyrosineand
tryptophanundergonitrationontreatmentwithconcentratednitric
acidwhenheated.Theendproductisyellowincolorwhich
isintensifiedinstrongalkalinemedium.Thisreactioncausesthe
yellowstainintheskinbynitricacid.

319.Taurineisbiosynthesizedby?
a)Arginine
b)Leucine
c)Valine
d)Cysteine
CorrectAnswer-D
Ans.is'd'i.e.,Cysteine[RefHarper250/ep.298-99]
Taurineissynthesizedfromcysteineby3enzyme-catalyzed
reactions:

1. Cysteineisoxidizedtocysteinesulfinate.
2. Cysteinesulfinateisdecarboxylatedtoformhypotaurine.
3. Hypotaurineisoxidizedtoformtaurine

320.Selenocysteineisassociatedwith?
a)Carbonicanhydrase
b)Catalase
c)Deiodinase
d)Transferase
CorrectAnswer-C
Ans.C.Deiodinase
Selenocysteineisconsideredas21standardaminoacid.
Itispresentattheactivesiteofsomeenzymesthatcatalyzeredox
reactions,e.g.thioredoxinreductase,glutathioneperoxidase,and
thedeiodinase(convertsthyroxintotriiodothyronine).
Biosynthesisofselenocysteinerequirescysteine,serine,ATPanda
specifict-RNA.
Serineprovidesthecarbonskeletonofselenocysteine.
Selenocysteinehasastructuresimilartocysteine,butcontainingthe
traceelementseleniuminplaceofsulfuratomofcysteine.

321.Methylmalonylaciduriaisseenin
deficiencyof?
a)VitB12
b)VitB6
c)VitC
d)Folicacid
CorrectAnswer-A
Ans.is'a'i.e.,VitB12[RefHarper29"Vep.537]
IsomerizationofmethylmalonylCoAtosuccinylCoA:
Inthisreaction,activeformofvitaminBI2isdeoxyadenosyl
cobalamine.
Propionyl-CoAisproducedascatabolicendproductofsome
alipathicaminoacidsandn-oxidationofoddchainfattyacids.
PropionylCoAisthenconvertedtosuccinylCoAthrough
methylmalonyl-CoA.
Thusmethylmalonyl-CoAisaccumulatedandexcretedinurineas
methylmalonicacid(methylmalonate)invitaminB12deficiency0,i.e.
methylmalonicacidurie

322.Carnitineissynthesisedfrom-
a)Lysine
b)Agrinine
c)Histidine
d)Choline
CorrectAnswer-A
Trimethyllysineand-butyrobetainehydroxylasesarerequiredfor
thesynthesisofcarnitine

323.Tyrsoineistheprecursorofallexcept?
a)Thyroxine
b)Melanin
c)Dopmine
d)Nicotinicacid
CorrectAnswer-D
Ans.is'd'i.e.,Nicotinicacid[Ref:Harper's28th/ep.266,268,
254]
Tyrosineisaprecursorofmanyimportantcompoundssuchas
catecholamines(epinephrineQ,norepinephrineQ),dopamine),
thyroxineQ,triiodothryonine,melanin0.


324.Whichofthefollowingenzymeisnot
usedbyliverinureacycle?
a)CPS-I
b)CPS-II
c)Arginase
d)Arginosuccinate
CorrectAnswer-B
Ans.is'b'i.e.,CPS-II[RefHarper29th/ep.277-278]
CarbmoylphosphatesynthaseII(CPSII)isinvolvedinpyrimidine
synthesis(notinureacycle).

325.Aminoacidusedbyliverinureacycle?
a)Glutamine
b)Glutamate
c)Aspartate
d)Fumarate
CorrectAnswer-C
Ans.is'c'i.e.,Aspartate[RefHarper29`"/ep.276-278]
Thesourceoftwonitrogenatomsofurea->onefromammonia?and
onefromaminogroupofaspartate?.SourceofcarbonisCO,Q.
Thusoutofallaminoacidsinvolvedinureacycleaspartateis
consumed,whilethereisnonetlossorgainofornithine,citrulline,
arginosuccinateorarginine.Thereisproductionoffumarateasby
product?.
3moleculesofATPareconsumed(2infirstreactionand1inthird
reaction).However,4highenergyphosphatebondsareutilizedas
3rdATPisconvertedtoAMP+PPi.

326.Hydrolysisoccursatwhichstepofurea
cycle?
a)Cleavageofarginine
b)FormationofArginosuccinate
c)Formationofcitrulline
d)Formationofornithine
CorrectAnswer-A
Ans.'A',Cleavageofarginine
Ureasynthesisisa5stepcyclicprocess,with5distinct
enzymes.Thefirst2enzymesarepresentinmitochondriawhilethe
restarelocalizedinthecytosol
Step1.FormationofCarbamoylPhosphate-Onemoleculeof
ammoniacondenseswithCO2inthepresenceoftwomoleculesof
ATPtoformcarbamoylphosphate.Itiscatalyzedbycarbamoyl
phosphatesynthetase-I(CPS-I).
Step2.FormationofCitrulline-Thecarbamoylgroupistransferred
totheNH2groupofornithinebyornithinetranscarbamoylase.
Step3.FormationofArgininosuccinate-Onemoleculeofaspartic
acidaddstocitrullineformingacarbontonitrogenbondwhich
providesthe2ndnitrogenatomofurea.Argininosuccinate
synthetasecatalyzesthereaction.
Step4.FormationofArginine-Argininosuccinateiscleavedby
argininosuccinatelyase(argininosuccinase)toarginineand
fumarate.Theenzymeisinhibitedbyfumarate.Thefumarate
formedmaybefunneledintotheTCAcycletobeconvertedto
malateandthentooxaloacetatetobetransaminatedtoaspartate.
ThustheureacycleislinkedtotheTCAcyclethroughfumarate.

Step5.FormationofUrea-Thefinalreactionofthecycleisthe
hydrolysisofargininetoureaandornithinebyarginase.

327.Urea&Kreb'scyclearelinkedat?
a)Arginine
b)Ornithine
c)Oxaloacetate
d)Fumarate
CorrectAnswer-D
Ans.is'd'i.e.,Fumarate[RefHarper250/ep.276-277]
Fumarateisreleasedduringureacycle,whichisanintermediateof
Kreb'scycle,thuslinkingthetwo.

328.Whichwillactivatecarbomoylphosphate
synthaseI?
a)Alanine
b)N-acetylglutamate
c)Ornithine
d)None
CorrectAnswer-B
Ans.is'b'i.e.,N-acetylglutamate
Carbamoylphosphatesynthase-I(CPS-I),amitochondrialenzyme,
catalyzestheformationofcarbamoylphosphate?bycondensationof
CO2andammonia.TwomoleculesofATParerequiredforthe
reaction.CPS-IistheratelimitingenzymeofureacycleQ.Itisan
allostericenzymeandallostericallyactivatedbyN-acetylglutamate

329.TransaminationofAlanineresultsin
formationof?
a)Oxaloacetate
b)Pyruvate
c)Aspartate
d)Arginine
CorrectAnswer-B
Ans.is'b'i.e.,Pyruvate

330.Cofactorsforglutamatedehydrogenase?
a)NAD+
b)FAD
c)FMN
d)FADH2
CorrectAnswer-A
Ans.'A'NAD+
AnaerobicDehydrogenasesaretheenzymesthatcatalyzethe
removalofhydrogenfromasubstratebutoxygencannotactasthe
hydrogenacceptor.They,therefore,requirecoenzymesas
acceptorsofthehydrogenatoms.
Whenthesubstrateisoxidized,thecoenzymeisreduced.
NAD+isderivedfromnicotinicacid,amemberofthevitaminB
complex.TheNAD+linkeddehydrogenasesare-
Glyceraldehyde-3-phosphatedehydrogenase
Isocitratedehydrogenase
Malatedehydrogenase
Glutamatedehydrogenase
BetahydroxyacylCoAdehydrogenase
Pyruvatedehydrogenase
Alpha-ketoglutaratedehydrogenase

331.Hyperammonemiatype-1isdueto
deficiencyof
a)Arginase
b)Arginosuccinatelyase
c)Arginosuccinatesynthase
d)CPS-1
CorrectAnswer-D
Ans.is'd'i.e.,CPS-1[RefDineshPuri3'/ep.275]
Disorderscausedbygeneticdefectsofureacycleenzymes
Hyperammonemiatype-IHyperammonemiatype-IICitrullinemia
ArgininosuccinicaciduriaArgininemia
Defectiveenzyme
Carbamoylphosphatesynthase-IOrnithinetranscarbamoylase
ArgininosuccinatesynthaseArgininosuccinatelyaseArginase
Productsaccumulated
AmmoniaAmmoniaCitrullineArgininosuccinateArginine.

332.Neonataltyrosenemiaisdueto
deficiencyof?
a)Tyrosinase
b)Fumarylacetoacetatehyroxyase
c)Hydroxyphenylpyruvatehydroxylase
d)Tyrosinetransminase
CorrectAnswer-C
Ans.is'c'i.e.,Hydroxyphenylpyruvate
hydroxylase
[RefHarper's28th/ep.266,268]
Tyrosinemia
Itisadefectinmetabolismoftyrosine.Itmaybeoffollowing
types.

1. Tyrosinemiatype-I(tyrosinosis/hepatorenalsyndrome):-Itisdueto
defectinfumarylacetoacetatehydroxylasedeficiency.Patientswith
chronictyrosinosisarepronetodevelopcirrhosisandhepatic
carcinoma?.Thereiscabbagelikeodorinacutetyrosinosis.
2. Tyrosinemiatype-II(Richer-Hanhartsyndrome):-Itisdueto
deficiencyoftyrosinetransaminaseQ(tyrosineaminotransferase).
3. Neonataltyrosinemia:-Itisduetodeficiencyof
hydroxyphenylpyruvatehydroxylase.

333.Separationofproteinsbytheirmass?
a)Electrophoresis
b)Saltingout
c)SDS-PAGE
d)Ionexchangechromatography
CorrectAnswer-C
Ans.is'c'i.e.,SDS-PAGE[RefVasudevan6"diep.600]

334.Twosamechargedproteinscanbe
separatedbyallexcept-
a)Agarose
b)DEAECellulose
c)Sephadex
d)Noneofthese
CorrectAnswer-B
Ans.is'b'i.e.,DEAECellulose[RefEssentialsofBiochemistryp.
670,795]
DEAEcellulosechromatography(anionexchange)separates
moleculesbasedonmolecularcharge.Therefore,itcannotseparate
twoproteinswithsamecharge.
Agarose(sepharose)anddextran(sephadex)areusedingel
filtrationchromatography,whichisbasedonmolecularsize.Thus,
theycanseparateproteinsofsamecharge.

335.250nmlightisabsorbedby?
a)Arginine
b)Alanine
c)Tyrosine
d)Histidine
CorrectAnswer-C
Ans.is'c'i.e.,Tyrosine
Aminoacidsdonotabsorbvisiblelightandthusarecolourless.
However,aromaticaminoacidsQ(tryptophan,tyrosine,
phenylalanine)absorb
high-wavelength(250-290nm)UVlight.
Trypytophanhasthegreatestabsorptionmaximainthisregion
thanothertwoaromaticaminoacids.
Thus,aromaticaminoacidsareresponsibleforUVabsorption
ofmostproteins0,
maximumabsorptionbeingat280rim.

336.Inglycolysis,inorganicphosphateis
usedreaction,catalyzedby?
a)Enolase
b)Pyruvatekinase
c)Glyceraldehyde-3-pdehydrogenase
d)Aldolase
CorrectAnswer-C
Ans.is'c'i.e.,Glyceraldehyde-3-pdehydrogenase

337.AboutDenaturationofprotein,whichis
true?
a)Biologicalpropertypersists
b)Primarystructurelost
c)Alwaysirreversible
d)Mostlyrendersproteininsoluble
CorrectAnswer-D
Ans.is'd'i.e.,Mostlyrendersproteininsoluble[RefLippincott's
4thIep.
57]
Thetermdenaturationreferstodisruptionofhigherorder
(secondary,tertiaryandquaternary)structureofprotein.
Allnon-covalentbondsthatmaintainhigherorderstructureare
disrupted,butpeptidebond(covalentbond)remainsintact.
Thus,theprimarystructureisnotalteredduringdenaturation,
i.e.,aminoacidsequenceisnotaltered,
butdenaturationmay
completelydisruptsecondary,tertiaryandquaternary
structure,e.g.,
denaturatedoligomericproteinsdissociatedinto
subunits,eachwitharandamcoilformation.
Denaturationisalwaysaccompaniedbyalossofbiological
function,e.g.,enzymesareinactivated
andantibodiesfailtoact
withantigens.
Denaturationisgenerallyirreversible,e.g.,boiledeggdoesnot
regainitsoriginalformwhenkeptin
cold.
Denaturedproteinsarelesssolubleandinmanycases
theyprecipitate.

338.Ochronosisisduetoaccumulationof?
a)Homogentisicacid
b)Phenylpyruvate
c)Xanthurenate
d)Glyoxylate
CorrectAnswer-A
Ans.is'a'i.e.,Homogentisicacid[Ref:Harper29th/ep.287-289]
Alkaptonuria
Itisduetodeficiencyofhomogentisateoxidase.Asa
resulthomogentisicacid(homogentisate)isexcreted
execessivelyinurine.
Therearethreeimportantcharacteristic
featuresinalkaptonuria?
1. UrinebecomesdarkafterbeingexposedtoairQ.Itisdue
spontaneousoxidationofhomogentisateintobenzoquinone
acetate,whichpolymersetoformblack-brownpigment
alkaptonwhichimpartsacharacteristicblack-browncolourto
urine.

2. Alkaptondepositionoccursinsclera,ear,nose,cheeksand
intervertebraldiscspace.Aconditioncalledochronosis.There
maybecalcificationofintervertebraldiscs
Q.
3. Onchronosisarthritisaffectingshoulder,hips,knee.
4. Benedict'stestisstronglypositiveinurineandsoistheferric
chloride(Pea)testQ.Benedict'sreagentgivesagreenishbrown
precipitatewithbrownishblacksupernatent.Fehling'sreagent
(FeC13)givesbluegreencolour.

339.Trueaboutalkaptonuria?
a)DeficiencyofTyrosinase
b)Urineisblack
c)Banedicttestisnotuseful
d)Fecl,testgivesredcolour
CorrectAnswer-B
Ans.is'b'i.e.,Urineisblack[RefNelson18thiep.1812]

340.EnzymedeficientinIsovalericacidemia
a)IsovalerylCoAdehydrogenase
b)Phenylalaninehydroxylase
c)Arginase
d)None
CorrectAnswer-A
Ans.is'a'i.e.IsolvalerylCoAdehydrogenase
-Isovalericacidemiaisduetothedefectinthemetabolismof
leucine.
-TheenzymedefectiveisisovalerylCo-Adehydrogenase.
-Acharacteristicodorofsweatyfeetispresent.
-Vomiting,acidosis,andcomafollowtheingestionofexcessprotein.
Accumulatedisovaleryl-CoAishydrolyzedtoisovalerateand
excreted.

341.Whichofthefollowingmetabolitesis
involvedinglycogenolysis,glycolysis
andgluconeogenesis?

a)Galactose-l-phosphate
b)Glucose-6-phosphate
c)Uridinediphosphoglucose
d)Fructose-6-phosphate
CorrectAnswer-B
Ans.is'b'i.e.,Glucose-6-phosphate[RefHarper28thlep.166,
158]
Glucose-6-phosphateisthemetabolitewhich:?
1. Joinsglycolysiswithglycogenesisandglycogenolysis
2. Joinsglycogenolysistopentosephosphatepathway(PPP)
3. Isinvolvedinglycolysis,glycogenesisandgluconeogenesis,
glycogenolysis,gluconeogenesis,PPP

342.Whycitricacidcyclecalledamphibolic
pathway?
a)Bothexergonicandendergonicreactionstakesplace
b)Metabolitesareutilizedinotherpathways
c)Itcanproceedbothinforwardandbackwarddirection
d)Sameenzymescanbeusedinreversedirections
CorrectAnswer-A
Ans.is'a'i.e.,Bothexergonicandendergonicreactionstakes
place
[Ref:Harper29th/ep.151,166]
Citricacidcycleiscalledamphibolicpathwaybecauseitactsasa
linkbetweenanabolic(endothermic)andcatabolic(exothermic)
pathways.

343.Instarvation,nitrogeniscarriedfrom
muscletoliverandkidneyby:
a)Alanine
b)AsparticacidandSerine
c)Glycine
d)Asparagines
CorrectAnswer-A
Ai.e.Alanine
Instarvation,alanineandglutaminearequantitativelythemost
importantgluconeogenicaminoacidsQ.
Soalaninecarriesnitrogen
frommuscletoliver&kidneyforfurthermetabolism&energy
productionduringstarvation.

344.Allofthefollowingoccurin
mitochondriaexcept
a)Citricacidcycle(Kreb'scycle)
b)Glycogenolysis
c)Fattyacidoxidation
d)Electrontransportchain
CorrectAnswer-B
Bi.e.Glycogenolysis

345.HMG-CoAinlivermitochondriais
inhibitedby?
a)Insulin
b)Glucagon
c)Glucocorticoid
d)Epinephrine
CorrectAnswer-A
Ans.is'a'i.e.,Insulin
Thisquestionisalittletricky.
Effectofinsulinonketogenesis
Ketogenesisisusuallyassociatedwithexcessivefattyacidoxidation
(lipolysis)whichprovidesthesubstrate(acetyl-CoA)forketogenesis.
Thus,factorswhichinhibitlipolysiswillinhibitketogenesis(andalso
productionofHMG-CoA).
Insulinisanantilipolytichormone.Itinhibitslypolysisandtherefore
ketogenesis.ThusitdecreasesthesynthesisofHMGCoAin
ketogenesis.
So,insulininhibitsaswellasstimulatesproductionofHMG-CoA.
Thenwhyisinsulintheanswerofthisquestion?
Herecomesthetrickypartofthisquestion.Readthequestion
carefullyexaminerhasaskedaboutHMG-CoAproductioninliver
mitochondria.
Ketogenesisoccursinmitochondria.
CholesterolsynthesisoccursincytosolandsmoothER.
ThusinsulininhibitsHMG-CoAproductioninmitochondriabut
stimulatesitincytosol.

346.Organwhichcanutilizeglucose,FAand
ketonebodiesis-
a)Liver
b)Brain
c)Skeletalmuscle
d)RBC
CorrectAnswer-C
Ans.is'c'i.e.,Skeletalmuscle[RefHarper28th/ep.141]
Skeletalmusclescanutilizeglucose,glycogen,fattyacidsand
ketonebodies.
Livercannotutilizeketonebodies.
Brainanderythrocytesareexclusivelydependentonglucoseexcept
inprolongedstarvationwherebrainutilizesketonebodies
predominantly

347.Majorsourceofenergyforbrainin
fasting/starvation?
a)Glucose
b)Glycogen
c)Fattyacids
d)Ketonebodies
CorrectAnswer-D
Thereisnostoredfuelinthebrain,butitutilized60%oftotalenergy
underrestingconditions.
Glucoseisvirtuallythesolefuelforthebrain,exceptinprolonged
starvingwhenketonebodiesarethemajorsource.
Fattyacidsdonotserveasfuelforthebrain,becausetheyare
boundtoalbumininplasma;hencecannotcrosstheblood-brain
barrier.

348.Fattyacidmetabolismgives?
a)AcetylCoA
b)MalonylCoA
c)Ketonebodies
d)Cholesterol
CorrectAnswer-A
Ans.is'a'i.e.,AcetylCoA[RefHarper250/ep.208-210]
Acetyl-CoAhasaspecialcentralrole.Acetyl-CoAisthecommon
degradationproductofglucose(byglycolysisQandPDHcomplex),
fattyacidsandketogenicaminoacids.
Itsacetylgroupcanbeutilizedinsynthesisoffatty
acidsQ,
cholesterolQandothersteroidsQ,ketonebodiesQ;orcanbe
oxidizedviaTCA(citricacidcycle).
Infedstateacetyl-CoAisoxidisedviaTCAcycleandusedfor
synthesisoffattyacidsandcholesterolQ,whereasinstarvationitis
usedtosynthesizeketonebodiesQ

349.Whichoneofthefollowingstatements
concerninggluconeogenesisiscorrect?
a)Itoccursinmuscle
b)Itisstimulatedbyfructose2,6-bisphosphate
c)ItisinhibitedbyelevatedlevelsofacetylCoA
d)Itisimportantinmaintainingbloodglucoseduringthenormal
overnightfast.
CorrectAnswer-D
Ans.is'd'i.e.,Itisimportantinmaintainingbloodglucose
duringthenormalovernightfast
[RefMark'sBasicmedical
biochemistry
p.566]
Duringfasting,manyofthereactionsofglycolysisarereversedas
theliverproducesglucosetomaintainbloodglucoselevels.This
processofglucoseproductioniscalledgluconeogenesis.
Duringovernightfast,bloodglucoselevelsaremaintainedbyboth
gluconeogenesisandglycogenolysis.However,afterapproximately
30hoursoffasting,liverglycogenstoresaremostlydepleted.
Subsequently,gluconeogenesisistheonlysourceofbloodglucose.

350.Chemiosmoticcouplingofoxidation
phosphorylationisrelatedto?
a)FormationofATPatsubstratelevel
b)ATPgenerationofpumpingofproton
c)ATPgenerationofpumpingofneutron
d)ATPformationbytransportof0,
CorrectAnswer-B
Ans.is'b'i.e.,ATPgenerationofpumpingofproton[RefHarper
29'diep.125-127]
Chemoiosmotictheory
Itstatesthat"freeenergyofelectrontransport"isconservedby
pumpingprotonsfrommitochondrialmatrixtotheintermembrane
space,soastocreateanelectrochemicalprotongradientacrossthe
innermitochondrialmembrane,withoutersideofmembraneis
positivelychargedascomparedtoinside.Electrochemicalpotential
ofthisgradientisusedtosynthesizeATPbyATPsynthase

351.Whichofthefollowingistrue?
a)Glucokinasehashighaffinityforglucose
b)Hexokinasehaslowaffinityforglucose
c)Glucokinasehaslowaffinityforglucose
d)Hexokinaseisinducedbyinsulin
CorrectAnswer-C
Ans.is'c'i.e.,Glucokinasehasloweffinityforglucose[Ref
Harper28`"/ep.151-152]
Glucokinase,isspecificforglucose.IthashighKinQ(i.e.,lowaffinity
forglucose),highVmaxandunlikehexokinase,itisnotinhibitedby
glucose-6-phosphateQ.Asithaslowaffinityforglucose(highkm),it
comesintoplayonlywhenintracellularglucoseconcentrationis
high.Itisinducedbyfeedingandinsulin.Glucagoninhibits
glucokinase.
Functionofhexokinaseistoprovideglucose-6-phosphateata
constantrate,accordingtheneedsofcells,i.e.,functionof
hexokinaseistoprovideconstantglucoseutilizationbyalltissuesof
bodyevenwhenbloodsugarislow.Functionofglucokinaseinthe
liveristoremoveglucosefrombloodafterameal,providing
glucose-6-phosphateinexcessofrequirementforglycolysissothat
itcanbeusedforglycogensynthesisandlipogenesis.

352.Lipogenesisisstimulatedby?
a)Insulin
b)Glucagon
c)Epinephrine
d)Corticosteroids
CorrectAnswer-A
Ans.is'a'i.e.,Insulin[RefDineshPuri3rdiep.318]

353.Anapleroticreactioniscatalyzedby?
a)Pyruvatecarboxylse
b)Enolase
c)Pyruvatekinase
d)G6PD
CorrectAnswer-A
Ans.is'a'i.e.,Pyruvatecarboxylse[RefDineshPuri3`diep.177]
Conversionofpyruvatetomalatebythecytoplasmicmalic-enzyme.
MalatecanthenenterthemitochondrionasasubstratefortheTCA
cycle.
Pyruvatemayreactwithaspartateorglutamateintransaminase
reactions,producingtheTCAcycleintermediatesoxalocetateanda-
ketogluarate,respectively.
SeveralglycogenicaminoacidsmayserveassourceofTCA
intermediates

354.Substancewithhighestthermogenic
effect?
a)Fat
b)Proteins
c)Carbohydrate
d)Allarethesame
CorrectAnswer-B
Ans.is`b'i.e.,Proteins[RefProgressinobesityresearch-397]
Thermogeniceffect(thermiceffect)offoodreferstotheincreasein
metabolicratethatoccursafteringestionofparticularfood.
Thisresultsinanincreaseintheamountofheatgeneratedbythe
body.
Proteinisatthetopofthehierarelyofmacronutrients,asforasthe
thermogeniceffectinconcerned.About25-30%ofenergyis
consumedtodigesttheprotein.
Carbohydratescomeinthemiddleandlipids(fats)areinthirdplace.

355.Whichofthefollowingisnotthesource
ofcytosolicNADPH?
a)Isocitratedehydrogenase
b)ATPcitratelyase
c)Malicenzyme
d)G6PD
CorrectAnswer-B
Answer.B.ATPcitratelyase
NADPHisacofactorusedinanabolicreactions,suchaslipidand
nucleicacidsynthesis,whichrequireNADPHasareducingagent.
ThemajorsourceofNADPHinanimalsandothernon-
photosyntheticorganismsisthepentosephosphatepathway.
Thekeyenzymesintheseprocessesare:NADP-linkedmalic
enzyme,NADP-linkedisocitratedehydrogenase,NADP-linked
glutamatedehydrogenaseandnicotinamidenucleotide
transhydrogenase.

356.RQisleastin?
a)Brain
b)RBC
c)Adipose
d)Heart
CorrectAnswer-D
Ans.is'd'i.e.,Heart[RefReadbelow]
Respiratoryqnotient(RQ)of:

1. Carbohydrateis1
2. Fatis0.70
3. Proteinis0.82
Undernormalconditionthemajorfuelofheartisfattyacids,while
otherthreeorgans(giveninoptions)utilizeglucose.
ThusRQvalueisminimumforheart.

357.Overnightfastingwhatoccurs?
a)Glucosedecreases
b)FFAincreases
c)Increasedgluconeogenesis
d)Increasedbeta-hydroxybutyrate
CorrectAnswer-C
Ans.is'c'i.e.,Increasedgluconeogenesis[RefVasudevan&hie
p.85,86,Harper28th/ep.240,241]
Inovernightfastingglucoselevelismaintaineddueto
glycogenolysisandgluconeogenesis.
FFAandketonebodies(p-hydroxybutyrate)startsrisingafter2-3
days,i.e.inlaterpartofinitialstageofprolongedstarving

358.WhichofthefollowingisaFatSoluble
vitamin?
a)Thiamine
b)Niacine
c)VitaminA
d)Ribaflavin
CorrectAnswer-C
Ans.is'c'i.e.,VitaminA[RefHarper's296/ep.335]

359.Ascorbicacidisrequiredforsynthesisof
?
a)Phenylserine
b)Homoserine
c)Hydroxylysine
d)Selenocysteine
CorrectAnswer-C
Ans.is'c'i.e.,Hydroxylysine[RefHarper's29th/ep.540]
Hydroxylationofprolineandlysineresiduetakesplaceduringpost-
translationalmodificationinroughER.
Theenzymecatalyzingthereactionsareprolylhydroxylase(for
proline)andlysylhydroxylase(forlysine).
Boththeseenzymesaredioxygenases?usingmolecularoxygen(02)
andcofactorforboththeseenzymesisvitaminC(ascorbicacid)?.a-
Ketoglutatrateisacoreductant,whichisoxidizedtosuccinate.

360.Whichofthefollowingvitaminsdoesnot
participateinoxidativedecarboxylation
ofpyruvatetoacetylCoA?

a)Thiamine
b)Niacine
c)Riboflavin
d)Biotin
CorrectAnswer-D
Ans.is`d'i.e.,Biotin

361.VitaminKisrequiredfor:
March2005
a)Chelation
b)Transamination
c)Carboxylation
d)Noneoftheabove
CorrectAnswer-C
Ans.C:Carboxylation
ThefunctionofvitaminKinthecellistoconvertglutamatein
proteinstogamma-carboxyglutamate(gla).Withinthecell,vitaminK
undergoeselectronreductiontoareducedformofvitaminK(called
vitaminKhydroquinone)bytheenzymevitaminKepoxidereductase
(orVKOR).
AnotherenzymethenoxidizesvitaminKhydroquinonetoallow
carboxylationofGlutamatetogammacarboxyglutamate;this
enzymeiscalledthegamma-glutamylcarboxylaseorthevitaminK-
dependentcarboxylase.
Thecarboxylationreactionwillonlyproceedifthecarboxylase
enzymeisabletooxidizevitaminKhydroquinonetovitaminK
epoxideatthesametime;thecarboxylationandepoxidation
reactionsaresaidtobecoupledreactions.

362.Pantothenicacidisneededfordonating
thefollowingmoiety?
a)Acetyl(oracyl)CoA
b)Carboxyl
c)Hydroxyl
d)Amino
CorrectAnswer-A
Ans.is'a'i.e.,Acetyl(oracyl)CoA[RefHarper29th/ep.540]
Pantothenicacidfunctionsascoenzymebyprovidingbuildingblock
ofcoenzymeAandACP
1. Coenzyme-Aparticipatesinreactionsofcitricacidcycle,fattyacid
oxidation,acetylation,andcholesterolsynthesis.
2. ACPtakespartinfattyacidsynthesis

363.Mostpotentantioxidant?
a)VitA
b)VitK
c)VitE
d)VitC
CorrectAnswer-C
Ans.is'c'i.e.,VitE[Ref:Harper29thiep.532,541,543]
Amongstgivenoptions,vitaminA,EandCareanti-oxidants.
However,VitaminE(tocopherol)isthemostimportantantioxidantin
thebody,actinginthelipidphaseofmembranesprotectingagainst
theeffectsoffreeradicals.

364.Coenzymeformofpyridoxineis?
a)ADP
b)NAD
c)PLP
d)FAD
CorrectAnswer-C
Ans.is'c'i.e.,PLP[RefGuyton12thiep.854]

365.ThecofactorvitaminB12isrequiredfor
thefollowingconversion:
a)DopaminetoNorepinephrine
b)PropionylCoAtomethylmalonylCoA
c)MethylmalonylCoAtosuccinylCoA
d)Cysteinetohomocysteine
CorrectAnswer-C
Ans:C.MethylmalonylCoAtosuccinylCoA
(Ref:Harper30/ep550,558,28iep346)
VitaminB12asCofactorfor:
MethylmalonylCoAmutase-Isomerizationofmethylmalonylco-Ato
succinylco-A.
Methioninesynthase-Methylationofpyrimidineringtoform
thymine.
Homocysteinemethyltransferase-Methylationofhomocysteineto
methionine
Metabolismofdiol.
Inbacteriaforinterconversionofglutamate&beta-methylaspartate?

366.VitaminB1isrequiredforwhichreaction
a)Transamination
b)Oxidativedecarboxylation
c)Carboxylation
d)Alloftheabove
CorrectAnswer-B
Ans.is'b'i.e.,Oxidativedecarboxylation[RefHarper29th/ep.
534]
*Activeform(coenzymeform)ofthiamineisthiamine
pyrophosphate(TPP),alsocalledthiaminediphosphate(TDP).
*TPPactsascoenzymefor
-Oxidativedecarboxylation:-Pyruvatedehydrogenase,a-
ketoglutaratedehydrogenase,brached-chainketoacid
dehydrogenase.
-TransketolaseinPPP.

367.Pyridoxinedeficiencyleadstoaltered
metabolismof?
a)Phenylalanine
b)Tryptophan
c)Methionine
d)Tyrosine
CorrectAnswer-B
Ans.is'b'i.e.,Tryptophan[RefDineshpuri3rdlep.378]
Tryptophanloadtestisusedforpyridoxin.
Inpyridoxin(vitaminB6)deficiency,xanturenicacidexcretionis
increasedaftergivingtryptophanloaddose.

368.Thepyruvateutilizationintissuesis
decreasedin?
a)Perniciousanemia
b)Scurvy
c)Beriberi
d)Pellagra
CorrectAnswer-C
Ans.is'c'i.e.,Beriberi[RefHarper29th/ep.534]
Pyruvateutilizationisdecreasedinthiaminedeficiency.Beriberiis
duetothiaminedeficiency.
Inthiaminedeficiency,pyruvatecannotbeconvertedtoacetyl-CoA
asthiaminepyrophosphateisacoenzymeforpyruvate
dehydrogenasewhichcatalyzestheconversionofpyruvateto
acetyl-CoA.Hence,excessofpyruvateismetabolizedtolactateby
lactatedehydrogenase

369.Whichofthefollowingrequiresvitamin
BIZ?
a)Serinetolysine
b)Homocysteinetomethionine
c)Serinetoglycine
d)Glutaminetoglutamate
CorrectAnswer-B
Ans.is'b'i.e.,Homocysteinetomethionine[Ref:Harper2967ep.
537-539]
ConversionofhomocysteintomethionineInthisreaction,active
formismethylcobalamine.Thisistheonlyreactionwhichrequires
bothvitaminB,2(asmethylcobalamine)andfolicacid(asN5-methyl-
Hdolate).Thereactioniscatalyzedbytheenzymecobalamin-
dependentmethioninesynthasealsocalled5-
methyltetrahydrofolate-homocysteinemethyltransferase

370.Whichmicronutrientdeficiencycauses
anemia?
a)Copper
b)Molybdenum
c)Selenium
d)Flurine
CorrectAnswer-A
Ans.is'a'i.e.,Copper
Coppercontainingproteinceruloplasminisnecessaryfortransport
ofironintheFerricformacrossmembranes
CopperisanintegralcomponentofALAsynthase,whichis
necessaryforhemesynthesis
Copperhelpsintheuptakeofironacrossnormoblasts

371.Adeninephosphoribosyltransferaseis
involved?
a)Denovopurinesynthesis
b)Purinedegradation
c)Salvagesynthesisofpurinenucleotides
d)None
CorrectAnswer-C
Ans.is'c'i.e.,Salvagesynthesisofpurinenucleotides[Ref
Harper29thie
p.334-335]
Salvagepathwayofpurinenucleotidesynthesis
Freepurinebases(adenine,guanineandhypoxanthine)andpurine
nucleosidesareformedincellsduringthemetabolicdegradationof
nucleicacidsandnucleotides.Thesefreepurinebasesandpurine
nucleosidesarereusedintheformationofpurinenucleotides.This
iscalledsalvagepathway(salvagemeanspropertysavedfrom
loss).Salvagesynthesisrequiresfarlessenergythandenovo
synthesis.

372.Uricacidisformedby?
a)Catabolismofproteins
b)Catabolismofketones
c)Catabolismofpurines
d)Catabolismfopyrimidines
CorrectAnswer-C
Ans.is'c'i.e.,Catabolismofpurines

373.Beta-alanineisderivedfrom?
a)Adenosine
b)Guanosine
c)Thymine
d)Uracil
CorrectAnswer-D
Ans.is'd'i.e.,Uracil[Ref:Harper29illep.339,340]

374.Purineareformedby?
a)Asparticacid,glycine,uricacid
b)Aspartate,glycine,
c)Aspartate,glutamate
d)Aspartate,glycine,glutamine
CorrectAnswer-D
Ans.is'd'i.e.,Aspartate,Glycine,Glutamine[RefHarper29thie
p.332]
AminoacidsinvolvedinpurinesynthesisGlycine,aspartate,
glutamine.
AminoacidsinvolvedinpyrimidinesynthesisGlutamine,aspartic
acid(aspartate).

375.Whichofthefollowingisnotinvolvedin
synthesisofpyrimidines?
a)Glutamine
b)CO
c)Asparticacid
d)Glycine
CorrectAnswer-D
Ans.is'd'i.e.,Glycine[RefHarper29th/ep.336-337]
AminoacidsinvolvedinpurinesynthesisGlycine,aspartate,
glutamine.
AminoacidsinvolvedinpyrimidinesynthesisGlutamine,aspartic
acid(aspartate)

376.WatsoncrickmodelisforwhichDNA?
a)BDNA
b)ADNA
c)CDNA
d)ZDNA
CorrectAnswer-A
Ans.is'a'i.e.,BDNA[Ref:Harper250/ep.344]
DNAistherepositoryofgeneticinformation.DNAislocatedin
nucleus.DNAisalsopresentinmitochondria

377.

Theanticodonregionisanimportantpartof
the

a)r-RNA
b)m-RNa
c)t-RNa
d)hn-RNa
CorrectAnswer-C
Ci.e.t-RNA

378.Shinedalgarnosequenceisrelatedto?
a)Transcription
b)Translation
c)DNAreplication
d)None
CorrectAnswer-B
Ans.is'b'i.e.,Translation
ShinedalgarnosequenceinprokaryotesandKozakconsensus
sequencesineukaryoteshelpsininitiationofproteinsynthesis
(Translation)
Inprokaryotes,asequenceofnucleotidebasesonmRNAknown
asShine-Dalgarnosequence(SDsequence)facilitatesthe
bindingofmRNAtothepreinitiationcomplex.SDsequenceisa
purine-richsequenceofnucleotidebases,whichislocated-6to-10
bpfromAUGcodon?.
InEukaryotes,'Kozakconsensus'sequencesurroundsAUG
(initiationcodon)anddeterminestheinitiatingcodonofmRNA.

379.Nucleotideconsistsofallexcept?
a)Sugar
b)Phosphate
c)Fattyacid
d)Base
CorrectAnswer-C
Ans.is'c'i.e.,Fattyacid[RefHarper's28"/ep.286]
Nucleotidesaremonomericunitsofnucleicacids.Theyarerequired
forsynthesisofnucleicacid.
Eachnucleotideismadeupof:(i)Anitrogenousbase,(ii)A
sugar(pentosesugar)
and(iii)Aphosphategroup(phosphoric
acid).
Nitrogenousbasecombineswithasugarto
formnucleosides.Thenucleosidecombineswithphosphoricacidto
formanucleotide.

380.CentralDogmaofmolecularbiology
indudesallexcept?
a)Transcription
b)Translation
c)RNAreplication
d)DNAreplication
CorrectAnswer-C
Ans.is'c'i.e.,RNAreplication
DNAstoresgeneticinformation:-
Informationaboutaminoacid
sequenceofalltheproteinsispresentintheformofgenesinDNA.
TheentiregeneticmaterialpresentintheDNAofanorganismis
knownasgenome.TheimportantroleofDNAintransferof
informationinlivingcellsiscalledcentraldogmaofmolecular
biology.
Accordingtothecentraldogma,informationflowsfrom
DNAtoRNAtoprotein.

381.WhatisthefunctionofDNAligase?
a)Unwinding(denaturation)ofdsDNAtoprovideanssDNA
template
b)Sealsthesinglestrandnickbetweenthenascentchainand
Okazakifragmentsonlaggingstrand
c)InitiationofDNAsynthesisandelongation
d)InitiatessynthesisofRNAprimers
CorrectAnswer-B
Ans.is'b'i.e.,Sealsthesinglestrandnickbetweenthenascent
chainandOkazakifragmentsonlaggingstrand
[RefHarper
2501ep.367]


382.Primasefunctionsas?
a)JoiningDNAfragments
b)SynthesiningsmallRNAfragmentsduringDNAsynthesis
c)SynthesisingsmallRNAfragmentsduringtranslation
d)UnwindingofDNA
CorrectAnswer-B
Ans.is'b'i.e.,SynthesiningsmallRNAfragmentsduringDNA
synthesis
[RefHarper's29th/ep.366,367]

383.Whatisattatchedto3'endofmRNAafter
transcription?
a)PolyAtail
b)CCA
c)Intron
d)7-methylguanosine
CorrectAnswer-A
Ans.is'a'i.e.,PolyAtail[Ref:Harper's29`h/ep.392]
MammalianmRNAmoleculescontaina7-methylguanosinecap
structureattheir5'terminal,andmosthaveapoly(A)tailatthe3'
terminal.
ProkaryoticmRNAisfunctionalimmediatelyuponsynthesis,i.e.
prokaryoticprimarytranscriptofmRNAisfunctional.
Thusitdoesnotrequirepost-transcriptionalmodification.
InEukaryotestheprimarytranscriptofmRNAisthehnRNA
(hetrogeneousnuclearRNA).
AftertranscriptionhnRNAisextensivelymodifiedtoformfunctional
mRNA.

384.WhichRNAisusedinRNAsplicing?
a)mRNA
b)SmallnuclearRNA
c)SmallcytosolicRNA
d)tRNA
CorrectAnswer-B
Ans.is'B'i.e.,SmallnuclearRNA[RefHarper29th/ep.378,390]
SplicesomeSplicesomeisanassemblymadeupofsmallnuclear
RNA(snRNA),someproteinsandhnRNA.snRNAcombineswith
proteinstoformsmallnuclearribnonucleoproteinparticles(snRNPs
orsnurps)thatmediatesplicing.ItissnRNAcomponentofsnurps
thatcatalyzessplicing?.SnurpsareU,,U,,U3,U4,U5andU6

385.Whichofthefollowingisaribozyme?
a)Peptidyltransferase
b)Elongationfactor2
c)Primase
d)RNApolymerase
CorrectAnswer-A
Ans.is'a'i.e.,Peptidyltransferase[RefHarper29thIep.405]
SomeRNAmoleculeshaveintrinsiccatalyticactivity.
Theactivityoftheseribozymesofteninvolvesthecleavageof
nucleicacid.
TwoimportantRNAenzymesorribozymesare:?
Thepeptidyltransferasethatcatalyzespeptidebondformationon
theribosomeandRibozymesinvolvedintheRNAsplicing.

386.Primerfunctionisin?
a)Transcription
b)Translation
c)InitiationofDNAreplication
d)TerminationofDNAreplication
CorrectAnswer-C
Ans.is'c'i.e.,InitiationofDNAreplication[RefHarper's29thlep.
366,367]
DNAsynthesiscannotcommencewithdeoxyribonucleotides
becauseDNApolymerasecannotaddamononucleotidetoanother
mononucleotide.
Thus,DNApolymerasecannotinitiatesynthesisofcomplementary
DNAsynthesisstrandofDNAonatotallysinglestrandedtemplate.
Forthis,theyrequireRNAprimer,whichisashortpieceofRNA
formedbyenzymeprimase?(RNApolymerase?)usingDNAasa
template.
RNAprimeristhenextendedbyadditionofdeoxyribonucleotides.
Lateron,theribonucleotidesoftheprimerarereplacedbydeoxyribo
nucleotides.

387.Chargaff'srulestatesthat?
a)A=T,G=C
b)A=G,T=C
c)A=C,G=T
d)Anycombinationpossible
CorrectAnswer-A
Ans.is'a'i.e.,A=T,G=C[Ref:Harper29th/ep.344&28thlep.302;
Essentialsofbiochemistryp.915]
ChargaffsrulestatesthatinDNAofallspeciesquantitiesofpurines
isthesameasthatofpyrimidines,i.e.A+G=T+C

388.Inglycolysis,NADHisproducedat?
a)Pyruvatekinase
b)Enolase
c)Glyceraledehyde-3-P-dehydrogenase
d)PFK-1
CorrectAnswer-C
Ans.is'c'i.e.,Glyceraledehyde-3-P-dehydrogenase[RefHarper
28th/ep.151-152]
Reducingequivalent(NADH)productioniscatalyzedby:
Glyceraldehyde3-phosphatedehydrogenase

389.Reversetranscriptioninvolves?
a)RNAdependentDNAsynthesis
b)DNAdependentRNAsynthesis
c)DNAdependentDNAsynthesis
d)RNAdependentRNAsynthesis
CorrectAnswer-A
Ans.is'a'i.e.,RNAdependentDNAsynthesis[RefHarper29thle
p.348]
SynthesisofRNAfromDNAiscalledtranscription.
Intranscription,RNAissynthesizedbyRNApolymerase.RNA
polymeraseisalsocalledDNAdependentRNApolymerase
becauseitisdependentonDNA(non-codingstrand)forRNA
synthesis.
Reversetranscription,asthenamesuggests,isthereverseof
transcriptioni.e.synthesisofDNAformRNA.
InreversetranscriptionDNAissynthesizedbyreversetranscriptase.
ReversetranscriptaseisalsocalledRNAdependentDNA
polymerasebecauseitisdependentonRNAforDNAsynthesis.

390.Enzymerequireforcuttingthestrand
DNAsynthesis?
a)DNApolymerase
b)DNAligase
c)Topoisomerase
d)Helicase
CorrectAnswer-C
Ans.is'c'i.e.,Topoisomerase[RefLippincott's5thiep.400,401]
AsthetwostrandsofDNAareseparatedaproblemisencountered,
i.e.appearanceofpositivesupercoils(supertwists)intheregionof
DNAaheudofreplicationform.
Theaccumulationofpositivesupercoilinginterfereswithfurther
unwindingofthedoublehelix.
Tosolvethisproblem,thereisagroupofenzymescalledDNA
topoisomeraseswhichareresonsibleforremovingsupercoilsinthe
helix.
DNAtopoisomerasesarenickandsealenzymes,i.e.theyhaveboth
nuclease(strand-cutting)andligase(strand-resealing)activities.

391.Whichdoesnotplayaroleinprotein
synthesis?
a)Exon
b)Intron
c)m-RNA
d)ATP
CorrectAnswer-B
Ans.is'b'i.e.,Intron[RefLippincott's5tvep.426]
Primarytranscriptcontainsintrons&exons.Splicingremoves
introns(segmentofgenethatisnotrepresentedinmaturem-RNA)
fromprimarytranscript.
SynthesisofproteinfrommRNAiscalled
translation.
Translationistheprocessbywhichribosomesconvert
theinformation(geneticcode)carriedbymRNAtothesynthesisof
newprotein.
Translationoccursinribosomes.Basicrequirementsfor
translationincludemRNA,tRNAs,ribosomes,energy(ATPand
GTP),enzymes,andspecificproteinfactorslikeinitiationfactors,
elongationfactorsetc.

392.Functionofexonuclease-
a)Polymerization
b)Proofreading
c)Chainelongation
d)Termination
CorrectAnswer-B
Ans.is'b'i.e.,Proofreading[RefDineshpuri3rd/ep.455,456]
Nucleasesreferstoanenzymethatcatalyzeshydrolysisof
phosphodiesterbondinanucleicacid.
Thenuclesesareoftwotypes:
*Endonucleases:Cleavetheinternalphosphodiesterbonds.
*Exonucleases:Cleavebondsatends.Someexonucleasescleave
onlyatthe3'end(the3'-exonucleaseactivity)whileothercleaveat
the5'end(the5'-exonucleaseactivity).
-31-exonucleaseactivityisresponsibleforproofreading.3'-
exonucleaseactivityispresentinDNApolymeraseI,IIandIII.
-51-exonucleaseactivityisresponsibleforerrorcorrectionin
damagedDNA.

393.Insicklecellanemia,translocationon
codon6isduetosubstitutionof?
a)Valineforglutamate
b)Glutamateforvaline
c)Isoleucineforvaline
d)Valineforisoleucine
CorrectAnswer-A
Ans.is'a'i.e.,Valineforglutamate[RefHarper's29th/ep.444]
Sicklecelldisease,iscausedbymutationofasinglebaseoutofthe
3x109inthegenome,aT-to-ADNAsubstitution,whichinturn
resultsinanA-to-UchangeinthemRNAcorrespondingtothesixth
codonofthe13-globingene.Thealteredcodonspecifiesadifferent
aminoacid(valineratherthanglutamicacid),andthiscausesa
structuralabnormalityofthep-globinmolecule.

394.Whichofthefollowingusuallyrequirea
RNAintermediatefor
cloning/replication?

a)Transposons
b)Plasmids
c)Phages
d)Cosmids
CorrectAnswer-A
Ans.is'a'i.e.,Transposons[Ref:Lippincot4th/ep.461&Harper
29tVep.436,437]
Transposons(Tn)aremobilesegmentsofDNAthatmoveinan
essentiallyrandommannerfromonesitetoanotheronthesameor
adifferentchromosome.
Movementismediatedbytransposase,anenzymeencodedbyTn
itself.Movementcanbe:-(i)direct,inwhichtransposasecutsout
andtheninsertsTnatanewsite,or(ii)replicative,inwhichtheTnis
copiedandthecopyinsertedelsewherewhiletheoriginremainsin
place.
Ineukaryotes,includinghumans,replicativetranspositionfrequently
involvesaRNAintermediate,inwhichcasethetransposoniscalled
aretrotransposoni.e.tronsposonsthatinvolveaRNAintermediate
arecalledretrotransposons

395.Chromosomalstudyisbestcarriedout
in?
a)Prophase
b)Metaphase
c)Telophase
d)Anaphase
CorrectAnswer-B
Ans.is`b'i.e.,Metaphase[Ref:Anderson10thlep.225,226,
Robbins8thie
p.158]
Methodofkaryotyping
Karyotypingisthestudyofcoromosone.
Dividingcellsarearrestedinmetaphasebyadditionofcolchicineor
colcemid(deacetylmethylcolchicine).
Subsequently,cellsareexposedtoahypotonicsolutiontoinduce
swellingofthecellforenhancingspreadingofthechromosomes.
Themetaphasecellsarethenfixedwithmethanol/glacialaceticacid
mixtureandstainedbyoneoftheseveralbandingtechniques.
Afterstainingchromosomesareanalysedunderamicroscopeand
photographed.
Finally,akaryotypeisconstructedbymanualorautomatedpattern.
Chromosomesarearrangedinpairsanddecreasingorderoflength

396.Terminationcodonis?
a)AUG
b)UAA
c)AUA
d)AGG
CorrectAnswer-B
Ans.is'b'i.e.,UAA
Initiationcodon.4AUG
Stopcodons(terminationcodonsornonsensecodons)UAA,
UGA,UAG

397.DNAmicroarraysallowdetectionofGene
mutationsusing?
a)PolymerasechainReaction
b)Cloning
c)SouthernBlotting
d)Hybridization
CorrectAnswer-D
Ans.is'd'i.e.,Hybridization[RefBiologybyRavenTatap.331]
DNAMicroarray(DNA-Chips)
DNAmicroarrayscontainthusands(500-5000)ofimmobilizedDNA
probes/sequences(fewdozentohundredsofnucleotidelong)from
knowngenesorganizedinanareanolongerthanamicroscope
slide.DNAsegments(fromDNAlibraries)areamplifiedbyPCRand
placedonsmallwellsinasolidpolystyreneplates,usingrobotic
devices.Uptomillionsuchspotsaredepositedinapredesigned
arrayonasurfaceareaofjustfewcm'.Analternatewayisto
synthesizeDNAdirectlyonthesolidsurfaceusingphotolithography.
C-DNA(obtaineddirectlyorfrommRNAofpatientsparticularcell
typeorstage)isaddedtoeachwell,hybridizeandfluorescenceis
assessedtoassessgenesbeingexpressedinthosecells/stage.
Thisisbasedonprinciplesofnucleicacidhybridizationlikesouthern
ornorthernblottestsbutallowssimultaneousstudyofmultiple
genesorentiregenomeratherthansingle.
Hybridizationmeansbindingofcomplementarystrandsofnucleic
acidaccordingtoWatson-Crickrules(i.e.A=TandG=Cbinding).
SouthernblotforDNAandNorthernblotforRNAallowsthe
study/detectionofsinglegenewhereasmicroarraytechnique(for
DNA&RNA)allowsdetectionofmultiplegenesorentiregenome.

SoitcanbeconsideredasmultipleSouthernorNorthernblot
analysisrunninginparallel.

398.Whichoffollowingisananalouge
guanosine?
a)Abacavir
b)Allopurinol
c)Bromodeoxyuridine
d)None
CorrectAnswer-A
Ans.is'a'i.e.,Abacavir[RefEssentialsofbiochemistryp.888]

399.IdentificationofindividedbytheirDNA
wasinventedby?
a)Shapiro
b)Lewis
c)Jeffreys
d)Pasture
CorrectAnswer-C
Ans.is'c'i.e.,Jeffreys[RefLippicott'sSthiep.83,474,Lehniger
5thie
p.319-21]
TheDNAfingerprintingwasfirstreportedin1984bySirAlec
Jeffreys?attheuniversityofleicesterinEngland.

400.Whichofthefollowingstatements
regardingmaturecytoplasmic
messengerRNAistrue?

a)TranscribedfromNuclearDNA
b)HasThiamineinplaceofUracil
c)SugarisDeoxyRibose
d)Itsmolecularweightismorethanhn-RNA
CorrectAnswer-A
Ans.is'a'i.e.,TranscribedfromNuclearDNA[Ref:Lippincott's
Biochemistry5thp.428;FundamentalsofCytogenetics&Genetics
(2010)
p.444]
MesengerRNA(mRNA)isformedbytheprocessof'transcription'
fromoneofthestrandsofdoublestrandednuclearDNA
andcarriesgeneticinformationfromthenuclearDNAtothecytosol
whereitisusedasatemplateforproteinsynthesis.
mRNAcomprisesonly5-10%oftotalcellularRNA.Itcarriesthe
information(massage)fromthenucleustotheribosome.mRNAis
synthesizedinthenucleusaslieterogenousRNA(hnRNA)',which
isprocessedintofunctionalmRNA.

401.Thefoldsincollagenisdueto?
a)Glycine
b)Alanine
c)Arginine
d)Histidine
CorrectAnswer-A
Ans.is'a'i.e.,Glycine[RefEssentialsofbiochemistyp.868]
Inordertoformatriple-helixapolypeptidechain(a-chain)must
containglycineaseverythirdresidueinthesequence.
Thisisbecauseonlytheglycineissmallenoughtobe
accommodatedinthelimitedspaceavailabledownthecentralcore
ofthetriplehelix.
Eachturnofpolypeptidechain(a-chain)containsthreeaminoacid
residues,andglycine(Gly)ispresentateverythirdpositions.
Thusglycineconstitutes33%ofthetotalaminoacidresidues.
Therepeatingaminoacidresidues,representedas(Gly-X-Y)n,isan
absoluterequirementforformationoftriplehelix.
XandYcanbeanyaminoacids,butmostofthetimeXisproline
(10%ofthetotalaminoacidresidues)andmostofthetimeYis
hydroxyproline.

402.WhichofthefollowingGAGisnot
sulphated?
a)Chondroitin
b)Dermatan
c)Keratan
d)Hyaluronicacid
CorrectAnswer-D
Ans.is'd'i.e.,Hyaluronicacid[Ref:Harper's29thlep.596]
AGlycosaminoglycan(GAG)isanunbranchedpolysaccharidemade
upofrepeatingdisaccharides,onecomponentofwhichisalwaysan
aminosugar(hencethenameGAG),eitherD-glucosamineorD-
galactosamine.
Theothercomponentoftherepeatingdisaccharide(exceptinthe
caseofkeratansulfate)isauronicacid,eitherL-glucuronicacid
(GlcUA)orits5'-epimer,L-iduronicacid(IdUA).
Proteoglycansareproteinsthatcontaincovalentlylinked
glycosaminoglycans.
Theproteinsboundcovalentlytoglycosaminoglycansarecalled
"coreproteins".
Withtheexceptionofhyaluronicacid,alltheGAGscontainsulfate
groups,eitheras0-estersorasN-sulfate(inheparinandheparan
sulfate).
Hyaluronicacidaffordsanotherexceptionbecausethereisnoclear
evidencethatitisattachedcovalentlytoprotein,asthedefinitionof
aproteoglycangivenabovespecifies.

403.Repeatitivechainsofglucosaminewith
uronicacidareseenin?
a)NANA
b)Heparansulphate
c)Keratansulphate
d)Noneofthese
CorrectAnswer-B
Ans.is`b'i.e.,Heparansulphate[RefDineshpuriYalep.31,32]
Proteoglycansaremadeupofcoreprotein,coretrisaccharideand
glycosaminoglycans.
Proteoglycans=Glycosaminoglycans+Coretrisaccharide+Core
protein.
Glycosaminoglycans(mucopolysaccharides)aremadeupof
repeatingdiasaccharideunits.Eachdiasaccharideunitcontains?

404.Acuteintermittentporphyriaisdueto
deficiencyof?
a)UroporphyrinogenIsynthase
b)UroporphyrinogenIIIsynthase
c)Ferrochelatase
d)ALAsynthase
CorrectAnswer-A
Ans.is'a'i.e.,UroporphyrinogenIsynthase[RefHarper29tVep.
313]


405.Myoglobincontains?
a)Iron
b)Copper
c)Zink
d)Selenium
CorrectAnswer-A
Ans.is'a'i.e.,Iron[RefHarper29th/ep.308,Vasudevan6th/ep.
242]
Hemoproteinsareproteinswhichhavehemeasprostheticgroup.
Importanthemoproteinsarehemoglobie,myoglobinQ,
cytochromesQ(cytochromeCQ,cytochromeP450Q),catalaseQ,
peroxidase,tryptophanpyrrolaseandnitricoxidesynthase.
Allhemoproteinscontainironasironisthecentralcomponentof
heme

406.Reversefoldingofproteinsiscarriedout
by?
a)Valine
b)Threonine
c)Chaperone
d)Aspartate
CorrectAnswer-C
Ans.is'c'i.e.,Chaperone[RefHarper's29th/ep.558,559]
Certainproteinsplayaroleintheassemblyorproperfoldingofother
proteinswithoutthemselvesbeingcomponentsofthelatter.
Suchproteinsarecalledmolecularchaperones.
MostchaperonesexhibitATPaseactivityandbindADPandATP.
Thisactivityisimportantfortheireffectonfolding

407.Ratelimitingenzymeinhemesynthesis
?
a)ALAsynthase
b)Hmgcoareductase
c)ALAdehydratase
d)Uroporphyrinogen1synthase
CorrectAnswer-A
Ans.is'a'i.e.,ALAsynthase[RefHarper's29'lep.309]
Hemesynthesistakesplaceinallcells,butoccurstogreatestextent
inbonemarrowandliver.Thefirststepinthesynthesisofhemeis
thecondensationofglycineandsuccinylCo-Atoform6-
aminolevulinicacid(6-ALA),whichoccursinmitochondria.This
reactioniscatalyzedby6-ALAsynthasewhichrequirespyridoxal
phosphate(PLP)ascofactor.Thisistheratelimitingstepinheme
synthesis.

408.Proteinglycosylationoccursin?
a)ER
b)Golgibodies
c)Mitochondria
d)Peroxisomes
CorrectAnswer-A:B
Ans.is'b>a'i.e.,Golgibodies>ER[RefHarper's29th/ep.549,
572]
TheendoplasmicreticulumandtheGolgiapparatusarethemajor
sitesinvolvedinglycosylationprocesses.
However,0-glycosylationoccursonlyintheGolgiapparatusandso
itistheorganellewherealltypesofglycosylationreactionscantake
place

409.Thisattachestoproteinbefore
destruction?
a)Ubiquitin
b)RNAseF
c)Zymase
d)Chaperone
CorrectAnswer-A
Ans.is'a'i.e.,Ubiquitin[RefHarper291Vep.560-561]
UbiquitinIsaKeyMoleculeinProteinDegradation
Therearetwomajorpathwaysofproteindegradationineukaryotes.
OneinvolveslysosomalproteasesanddoesnotrequireATP.The
otherpathwayinvolvesubiquitinandisATP-dependent.
Itplaysthemajorroleinthedegradationofproteins,andis
particularlyassociatedwithdisposalofmisfoldedproteinsand
regulatoryenzymesthathaveshorthalf-lives.

410.Weakestbondis?
a)Covalent
b)Hydrogen
c)Electrostatic
d)Vanderwall
CorrectAnswer-D
Ans.is'd'i.e.,Vanderwall[Ref:Harper28thlep.9;Basicsof
molecularbiologyp.786]
StrongestbondCovalent
Weakestbond-4Vanderwallsforces
Covalent(strongest)?>Electrostatic0(ionicorsaltlinkage)>
hydrogen>hydrophobic>Vanderwaal's(weakest)?

411.Proteinsegregationoccursin?
a)Golgiappparatus
b)Peroxisomes
c)ER
d)Mitochondria
CorrectAnswer-A
Ans.is'a'i.e.,Golgiappparatus[RefHarper29th/ep.549]
Golgiapparatusplaysamajorroleinsortingofproteins.

412.Whichofthefollowingtripeptide?
a)Glutathione
b)Angiotensin
c)Glucagon
d)Oxytocin
CorrectAnswer-A
Ans.is'a'i.e.,Glutathione[RefHarper28thlep.679-680]
AngiotensinIII Heptapeptide(6aminoacids)
AngiotensinII Octapeptide(8aminoacids)
Oxytocin
Nonapeptide(9aminoacids)
Bradykinin
Nonapeptide(9aminoacids)
Vasopressin Nonapeptide(9aminoacids)
Glucagon
Decapeptide(10aminoacids)
AngiotensinI Decapeptide(10aminoacids)

413.Shortestpeptide?
a)AngiotensinII
b)AngiotensinIII
c)Oxytocin
d)Vasopressin
CorrectAnswer-B
Ans.is'b'i.e.,AngiotensinIII[RefEssentialsof
biochemistry
p.627]

414.InFITCthecoloremittedafterbluelight
absorption?\
a)Yellowgreen
b)Orangered
c)Applegreen
d)Goldenbrown
CorrectAnswer-A
Ans.is'a'i.e.,Yellowgreen
Fluoresceinisothiocyanate(FITC)isaderivativeoffluoresceinused
inwide-rangingapplicationsincludingflowcytometry.FITCisthe
originalfluoresceinmoleculefunctionalizedwithanisothiocyanate
reactivegroup(-N=C=S),replacingahydrogenatomonthebottom
ringofthestructure.Thisderivativeisreactivetowardsnucleophiles
includingamineandsulfhydrylgroupsonproteins.
FITC(fluoresceinisothiocyanate)isafluorochromedyethatabsorbs
ultravioletorbluelightcausingmoleculestobecomeexcitedand
emitavisibleyellowgreenlight.Thisemissionceasesuponremoval
ofthelightcausingtheexcitation.

415.Oxidativedeaminationoccursin?
a)Cytoplasmofallcells
b)Mitochondriaofallcells
c)Cytoplasmofhepatocytes
d)Mitochondriaofhepatocytes
CorrectAnswer-D
Ans.is'd'i.e.,MitochondriaofHepatocytes[Ref:Harper29'/ep.
274]
Deaminationmeansremovalofaminogroupofaminoacidinthe
formofammonia.Thus,anaminoacidisconvertedtoaketoacid.
Deaminationcoupledwithoxidationiscalledoxidative
deamination.
Itoccursinthemitochondria.
Oxidativedeaminationoccursprimarilyintheliver(majororgan)and
kidney.


416.WhichisoformofLDHisraisedin
Anemia?
a)LDH5
b)LDH4
c)LDH3
d)LDH2
CorrectAnswer-D
Ans.is'd'i.e.,LDH2[RefChatterjea7th/ep.600-605,Harper
28th/e
p.59]
LDH-2isfoundinRBCsandisincreasedinmegaloblasticanemia.

417.Inapoptosis,cytochromeCactsthrough
-
a)Apaf1
b)Bcl-2
c)FADD
d)TNF
CorrectAnswer-A
Ans.is'a'i.e.,Apaf1
Oneoftheseproteinsiscytochromec,wellknownforitsrolein
mitochondrialrespiration.Inthecytosol,cytochromeCbindstoa
proteincalledApaf-1(apoptosisactivatingfactor-1),andthe
complexactivatescaspase-9.(Bc1-2andBcl-xmayalsodirectly
inhibitApaf-1activation,andtheirlossfromcellsmaypermit
activationofApaf-1).

418.Whenstemcellstransformstoformcells
characteristicofothertissues,the
processiscalledas-

a)De-differentiation
b)Re-differentiation
c)Trans-differentiation
d)Sub-differentiation
CorrectAnswer-C
Ans.is'c'i.e.,Trans-differentiation
Transdifferentiation
*Transdifferentiationtakesplacewhenanon-stemcelltransform
intoadifferenttypeofcell,orwhenanalreadydifferentiatedstem
cellcreatescellsoutsideitsalreadyestabilisheddifferentiation.
*Rememberveryimportantfactthatitisthenon-stemcellor
alreadydifferentiatedstemcell(i.e.maturecell)thatistransformed
intoothertypeofcell.Itisnotstemcellthatistransforming.
*Transdifferentiationisatypeofmetaplasia.
*Then,whatisthedifferencebetweentransdifferentiationand
metaplasia.
*InTransdifferentiationonlydifferentiatedstemcellistransformed
intoothercelltype,whileinmetaplasiaanyofthetwo,eitherstem
cellordifferentiatedcellcantransformintoothercelltype.
*So,alltransdifferentiationprocessesaremetaplasia,butnotall
metaplasiaaretransdifferentiation.oMostlikelyquestionhasbeen
wronglyframedhere,thereshouldbenon-stemcellinsteadofstem
cellinthequestion.Anywaysanswerremainsthesame,asnoother
optionisrelatedtothistypeoftransformation.


419.Cellsmostsensitivetohypoxiaare?
a)Myocardialcells
b)Neurons
c)Hepatocytes
d)Renaltubularepithelialcells
CorrectAnswer-B
Ans.is'b'i.e.,Neurons
Thesusceptibilityofatissuetohypoxiainfluencesthelikelihoodof
infarction.
Neuronsaremostsensitivetohypoxia(irreversiblechangesdevelop
in3-4minutes)followedbymyocardialcells(irrversiblechanges
developin20-40minutes).
Fibroblastsareamongstthemostresistentcellstohypoxia.

420.LinesofZahnoccurin-
a)Postmortemclot
b)Infarct
c)Embolus
d)Corallinethrombus
CorrectAnswer-D
Ans.is'd'i.e.,Corallinethrombus
ItisseeninPrimaryplateletthrombi.

421.Dohlebodies-
a)DilatedEndoplasmicReticuluminNeutrophils
b)Mitochondria
c)Golgiapparatus
d)Lysosomes
CorrectAnswer-A
Ans.is'a'i.e.,Dilatedendoplasmicreticuluminneutrophils
Dohlebodiesarelightbluegray,bosophilicinclusionsinthe
peripheralcytoplasmofneutrophils.oTheyarethoughttobe
remnantsoftheroughendoplasmicreticulumcontainingRNA.

422.whichofthefollowingdoesnotbelongto
thefamilyofselectin?
a)Pselectin
b)Lselectin
c)Aselectin
d)Eselectin
CorrectAnswer-C
Ans.is'c'i.e.,Aselectin
Leukocyteandendothelialadhesionmolecules
Leukocyteadhesionandtransmigrationareregulatedlargelyby
bindingofcomplementaryadhesionmoleculesontheleukocyte&
endothelialsurface,andbychemicalmediators.
Cytokinesaffecttheseprocessesbymodulatingtheexpressionof
adhesionmoleculessothatleukocytescanfirmlyadhereto
endothelium.
Therearefourmolecularfamiliesofadhesionreceptors.
1.Selectin
Selectinsfunctioninthecelltocellinteractioni.e.,adhesionof
leukocytestotheendothelium.
E-selectin(CD-62E)ispresentonendothelialcellsanditbindsto
sialyl-lewis.(asialylatedformofoligosaccharideonleukocytes).
EndothelialcellexpressionofE-selectionisahallmarkofacute
cytokine-mediatedinflammation.
P-selectin(CD-62P)ispresentonendotheliumandplateletsandit
bindstosialyl-Lewisonleukocytes.
L-selectin(CD-62L)ispresentonleukocytesanditbindstomucin-
likeglycoproteinG1yCAM-Iontheendothelium.
2.Immunoglobulinfamily

Presentonendothelium
ICAM-1(intracellularadhesionmolecule-1)bindsto132-integrins
(VLA-4)ofleukocytes.
Plateletendothelialcelladhesionmolecule(PECAMorCD-31)is
presentonbothendotheliumandleukocytes.Itisthemajor
adhesivemoleculefordiapedesis.
3.Integrins
Integrinspromotecell-cellorcell-matrixinteraction(incontrastto
selectinsthatpromoteonlycell-cellinteraction)i.e.,integrinshelpin
adhesionofleukocytestoendotheliumandadhesionofleukocytes
toanextracellularmatrixsuchasfibronectin,vitronectin,activated
compliment.
Presentonleukocytes
integrins(VLA-4)bindtoVCAM-1oftheendothelium.
P2-integrins(LFA-1andMAC-1)bindtoICAM-1ofthe
endothelium.
4.Mucinlikeglycoprotein
Theseglycoproteinsarepresentintheextracellularmatrixandon
thecellsurface.
AnexampleisHeparansulfate,thatservesasaligandforthe
leukocyteadhesionmoleculeCD-44.

423.Cell-matrixadhesionsaremediatedby?
a)Cadherins
b)Integrins
c)Selectins
d)Calmodulin
CorrectAnswer-B
Ans.is'b'i.e.,Integrins
Cell-matrixadhesions,areusuallymediatedbyintergrins
Cell-celladhesionsinAnchoringjunctionsaremediatedby
cadherins.

424.Followinginjurytoabloodvessel,
immediatehaemostasisisachievedby-
a)Fibrindeposition
b)Vasoconstriction
c)Plateletadhesion
d)Thrombosis
CorrectAnswer-B
Ans.is'b'i.e.,Vasoconstriction
Hemostasis
*Hemostasisisaphysiologicalprocesswherebybleedingishalted,
thusprotectingtheintegrityofthevascularsystemaftertissueinjury.
*Itisresponsibleforminimizingbloodloss.
*Itiscommonlyreferredtoasstoppageofbleeding.
Thehemostaticmechanismshaveseveralfunctions:?
*l.Maintainbloodinfluidstatewhilecirculatingwithinthevascular
system.
*Arrestbleedingatthesiteofinjurybyformationofhemostaticplug.
*Ensuretheremovalofhemostaticplugwhenhealingiscomplete.
Thecomponentsofnormalhemostasisinclude:?
-Bloodvessels(endothelium)
-Platelets
-Plasmacoagulationfactorsandtheirinhibitors.
-Fibrinolyticsystem.
Whenabloodvesselisinjured,severalstepsoccurinhemostasisat
thesiteofinjury:?
Vasoconstriction
*Afterinitialinjury,thereisabriefperiodarteriolarvasoconstriction,
minimizingvesseldiameterandslowingbleeding.

*Thisisduetoreflexneurogenicmechanismandaugmentedbya
potentendotheliumderivedvasoconstrictorendothelin.
*Howeverthiseffectistransient,bleedingwouldresumeiftherewill
noactivationofplateletandcoagulationsystem.
Primaryhemostasis
*Endothelialinjuryexposeshighlythrombogenicsubendothelial
extracellularmatrixtobind(adhere)withcollegenofECM.
*Bindingofplateletsactivatesthesecellsandplateletsrelease
secretorygranules.
*Thesesecretoryproducts(fromsecretorygranules)recruite
additionalplateletsto(plateletplug).
C.Secondaryhemostasis
*Coagulationsystemisactivatedandthrombinisgenerated.
*Thrombinconvertsfibrinogentofibrin.
*Ultimatelytheseeventsformsanireversiblyfusedmassof
platelets,thrombinRBCsandfibrinogen-->Definitivesecondary
hemostaticplug.
D.Repairofvesselsanddissolutionofclot
*Theclotattractsandstimulatesthegrowthoffibroblastandsmooth
musclecellswithinthevesselwall,andbeginsrepairprocess.
*Atthisstagefibrinolyticsystemisalsoactivated,resultingin
dissolutionoftheclot.

425.Maximumcollageninwoundhealingis
seenat-
a)Endoffirstweek
b)Endofsecondweek
c)Endofthirdweek
d)Endof2months
CorrectAnswer-B
Ans.is'b'i.e.,Endofsecondweek
Duringsecondweekthereiscontinuedaccumulationofcollagen
andproliferationoffibroblast.Maximumcollagenisseeninsecond
week.

426.MostpotentstimulatorofNaiveT-cells-
a)Maturedentriticcells
b)Folliculardentriticcells
c)Macrophages
d)B-cell
CorrectAnswer-A
MatureDendriticcells
MatureB-cellsandT-cellsbeforeantigenicexposurearecalled
naive-BandTcellsrespectively.Sequenceofeventsinactivationof
naiveTcells.
Immaturedendriticcellsintheepidermisarecalledlangerhanscell.
Theseimmaturedendriticcells(langerhanscells)capturethe
antigenintheepidermis.
Aftercapturingtheantigenthesecellssecretecytokines.
Thesecytokinescauselossofadhesivenessoflangerhanscells.
Langerhanscellsseparatefromeachotherandmigrateinto
lymphaticvessels.
Inlymphaticvessel,maturationoflangerhanscellstakesplace.
ThenthesematurelangerhansdendriticcellsreachtonaiveTcells
inthelymphnodesandpresentantigentothesecellsandactivate
them.

427.Commonvariable
hypogammaglobulinemiashows?
a)DecreasedBcellcount
b)IncreasedBcellcount
c)Complimentopsinization
d)Neutropenia
CorrectAnswer-C
Ans.is'c'i.e.,NormalBcells
Mostpatientswithcommonvariableimmunodeficiencyhavenormal
ornear-normalnumbersofBcellsinthebloodandlymphoidtissues.
TheseBcells,however,arenotabletodifferentiateintoplasma
cells.
Theclinicalmanifestationsarecausedbyantibodydeficiency.
Thefeaturecommontoallpatientsishypogammaglobulinemia,
generallyaffectingalltheantibodyclassesbutsometimesonlyIgG.

428.Amyloidosisismostcommonlyseenin?
a)MaturityonsetDM
b)TypeIDM
c)TypeIIDM
d)HTN
CorrectAnswer-C
Ans.is'c'i.e.,TypeIIDM
ThetwobestexamplesoflocalizedamyloidosisareAlzheimer's
diseaseandtype2diabetesmellitus.
Localizedamyloiddepositionresultsfromtheproductionofaunique
polypeptide,whichcontainsanamyloidogenicsequenceandis
capableofformingabeta-pleatedsheetstructurenecessaryfor
thesedepositstoaggregate.
Intype2diabetesitistheisletamyloidpolypeptide(IAPP)also
knownasamylin.
InAlzheimer'stheuniquepeptideisthebeta-amyloidprotein(A
beta).

429.WhichtypeofAmyloidosisiscausedby
mutationofthetransthyretinprotein?
a)FamilialMediterraneanfever
b)Familialamyloidoticpolyneuropathy
c)Dialysisassociatedamyloidosis
d)Prionproteinassociatedamyloidosis
CorrectAnswer-B
Ans.is'b'i.e.,Familialamyloidoticpolyneuropathy
Transthyretin(TTR)isnormalserumproteinthatbindsandtransport
thyroxineandretinol.
TTRcancausefollowingtypesofamyloidosis:?
MutantTTR:-MutationinTTRcancausesyndrome
offamilialamyloidoticpolyneuropathyorfamilial
amylodoticcardiomyopathy.
Wild(non-mutant)TTR:-ThereisnomutationofTTRandwildtype
TTRformsfibrilswhichresultsinsenilesystemicamyloidosis(senile
cardiacamyloidosis).


430.TruestatementaboutinheritenceofanX
linkedrecessivetraitis-
a)50%ofboysofcarriermotherareaffected
b)50%ofgirlsofdiseasedfatherarecarrier
c)Fathertransmitsdiseasetotheson
d)Mothertransmitsthediseasetothedaughter
CorrectAnswer-A
Ans.is'a'i.e.,50%ofboysofcarriermotherareaffected
allX-linkeddisordersareX-linkedrecessive.
AsmalehasonlyoneX-chromosome,themalewithaffectedgene
onX-chromosomewillalwaysmanifestthedisease.
Ontheotherhand,femalehas2X-chromosomes,heterozyogous
femalewillbecarrierbecauseofexpressionofnormalallelonthe
otherX-chromosome.
50%boysofthecarriermotherwillbeaffected.
FatherwillnottransmittthediseasetosonasboysdonotinheritX-
chromosomefromfather.

431.Neurofibromatosistrueall,except-
a)Autosomalrecessive
b)Associatedwithcataract
c)Scoliosis
d)Multiplefibroma
CorrectAnswer-A
Ans.is'a'i.e.,Autosomalrecessive
Neurofibromatosiscomprisesoftwodistinctdisorders-
*NeurofibromatosisI
*NeurofibromatosisII
*Thegenesforthesearelocatedondifferentchromosomes.
*Bothareinheritedinanautosomaldominantpattern.
*Theclassicalformofthediseasewithmultipleneuromasiscalled
NeurofibromatosisIandiscausedbyamutationofthegene
neurofibrominonchromosome17

432.ThetumorsuppressorgeneP53induces
cellarrestat-
a)Mphase
b)S-G2phase
c)G1-Sphase
d)Go-phase
CorrectAnswer-C
Ans.is'c'i.e.,G1-Sphase
FactorsactingatG,Spointare:?
Stimulatorofcycle:
CyclinD-CDK4(phosphorylateRB
gene);
cyclinE-CDK-2.
Inhibitorsofcycle:TheCip/kipfamily(p21,p27,p57);INK4a/ARF
family(p16INK4a,p14ARF,p16,p17,p18,p19

433.Maletomaletransmissionisseenin-
a)Autosomaldominantdiseases
b)Autosomalrecessive
c)X-linkeddominant
d)Mitochondrialdisease
CorrectAnswer-A
Ans.is'a'i.e.,Autosomaldominantdisease
Xchromosomeisnottransmittedfromfathertoson(optioncis
excluded)andmitochondrialdisordersarealwaysmaternally
inherited(optiondisexcluded).
Autosomalrecessivedisordercanbetransmittedfromfathertoson,
butonlyifthemotherisalsoaffectedoratleastsheiscarrier.Ifonly
maleisaffected,autosomalrecessivedisordercannotbe
transmittedtoson.
Autosomaldominantdisordercanbetransmittedinanydirection:-
Fromfathertosonordaughter.
Frommothertosonordaughter.

434.Femaleisaffected,maleisnot.Disease
isautosomaldominant,whatisthe
chanceinchildren?

a)50%affected
b)25%affected
c)75%affected
d)Allaffected
CorrectAnswer-A
Ans.is'a'i.e.,50%Affected
Ifoneoftheparent(motherorfather)isaffectedinautosomal
dominantdisorder,everychildhas50%ofchanceofhavingthe
diseaseand50%ofchancenothavingthedisease.

435.Overgrowthofaskinstructureata
localisedregion-
a)Hamartoma
b)Malignanttumor
c)Choriostoma
d)All
CorrectAnswer-A
Ans.is'a'i.e.,Hamartoma
Aberrantdifferentiationmayproduceamassofdisorganizedbut
maturespecializedcellsortissueindigenoustotheparticularsite,
referredtoasahamartoma.
Anectopicrestofnormaltissueiscalledachoristoma.egarestof
adrenalcellsunderthekidneycapsule.
Whenaneoplasm,benignormalignant,producesamacroscopically
visibleprojectionaboveamucosalsurfaceandprojects,itistermed
apolyp.

436.Radiationexposureduringinfancyhas
beenlinkedtowhichoneofthefollowing
carcinoma-

a)Breast
b)Melanoma
c)Thyroid
d)Lung
CorrectAnswer-C
Ans.is'c'i.e.,Thyroid
"Differentiatedthyroidcarcinomaparticularlypapillaryvariety
frequentlyfollowsaccidentalirradiationofthyroidininfancyand
childhood"-Bailey
Radiationinducedcancers
Radiationmayinducesomenon-lethalchangesinDNAsequences
whichmaycausemalignanttransformation

437.Li?Fraumenisyndromeisdueto
mutationofwhichgene-
a)P21
b)P53
c)P41
d)P43
CorrectAnswer-B
CancerarisesthroughaseriesofsomaticalterationsinDNAthat
resultinuncontrolledcelldivision.
Humancancershavefollowingimportantetiologicalfactors?
Geneticpredispositiontocancer
Non-hereditarypredisposingconditions
Geographicandenvironmentalfactors
Genetic(Hereditaryorinherited)predispositiontocancer
Alargenumberofcancershavehereditarypredispositions.
Geneticpredispositionmaybeofthreetypes.
AutosomaldominantInheritedcancersyndromes
Thisisthemostcommontypeofgeneticpredisposition.
Themechanisminvolvesuncontrolledcelldivisionduetogermline
mutationofcancersuppressorgene.
Gene
Inheritedpredisposition
RB
Retinoblastoma
p53
Li-fraumenisyndrome
pl6INK4A
Melanoma
Familialadenomatouspolyposis/colon
APC
cancer
NF-1,NF-2
Neurofibromatosis1and2
BRAC-1,BRAC-2
Neurofibromatosis1and2

BRAC-1,BRAC-2
Neurofibromatosis1and2
MEN1,RET
Multipleendocrineneoplasia
MSN2,MLH1,MSH
Heriditarynonpolyposiscoloncancer
6
PATCH
Nevoidbasalcellcarcinomasyndrome
2.DefectiveDNArepairsyndrome
Besidethedominantlyinheritedprecancerousconditions,agroupof
cancerpredisposingconditionsiscollectivelycharacterizedby
defectsinDNArepair.
Normally,iftheDNAdamageispresentitisrepairedatcell-cycle
checkpoints.
IfDNArepairmechanismisdefective,cellsreplicatewithdefective
DNAandmutationsorchromosomalbreaksaretransferedinthe
progenyofcellsthatcanleadtouncontrolledreplication.
Mostoftheseconditionsareinheritedasautosomalrecessive,e.g?
1. Xerodermapigmentosa
2. Fanconisyndrome
3. Bloomsyndrome
4. Ataxiatelangectasia
Oneconditioninthisgroupi
sautosomaldominantheriditorynonpolypidcoloncancer(HNPCC).
3.Familialcancers
Besidestheinheritedsyndromesofcancersusceptibility,some
cancersoccurathigherfrequencyincertainfamilieswithouta
clearlydefinedpatternoftransmission.
Thatmeans,thereisfamilialclusteringofcases,butroleofinherited
predispositionisnotclearforeachindividual.
Example-Breast,ovarian,andpancreaticcancers.

438.Regardingdesmoidtumourwhichisnot
correct?
a)Oftenseenbelowtheumbilicus
b)Unencapsulated
c)Morecommoninwomen
d)Highlyradiosensitive
CorrectAnswer-D
Ans.is`d'i.e.,Highlyradiosensitive
Desmoidtumourisanaggressivefibromatosisor
musculoaponeuroticfibromatosisisamonoclonalfibroblastic
proliferationarisinginmusculoaponeuroticstructures.
Histologicallythesetumoursareconsistofspindleshapedcellsin
collagenousmatrixandlackthepleomorphic,atypicalor
hyperchromaticnucleiofmalignancy.
AminorityofdesmoidtumoursareassociatedwithGardener
syndromeandmutationsoffamilialadenomatouspolyposis(FAP)
gene.
Mostspontaneousdesmoidtumoursareassociatedwithmutations
ofbeta-cateningene.

439.Carcinomaoflung,breastmarkeris-
a)CEA
b)AFP
c)11CG
d)CA-15-3
CorrectAnswer-A
Ans.is'a'i.e.,CEA
Carcinoembryonicantigenisamarkerincancerofcolon,pancreas,
lung,breast,andovary.

440.Telomerase-
a)RNApolymerase
b)Causescarcinogenesis
c)Presentinsomaticcells
d)Absentingermcells
CorrectAnswer-B
TelomeraseisaDNApolymerase(RNAdependentDNA
polymerase)whichispresentingermcellsandisabsentfrommost
somaticcells.
Thustelomeraseactivityandmaintenanceoftelomerelengthare
essentialforthereplicativepotentialincancercells.

441.7dayoldMIthemostsensitive
biochemicalmarker
a)TroponinIT
b)CPKMB
c)LDH
d)Myoglobin
CorrectAnswer-A
Ans.is'a'i.e.,TroponinIT
TroponinTorIisasensitivememberandreturntonormalafter7-10
days(seepreviousexplanations).

442.Vegetationsinlibmansacendocarditis
are?
a)Largeandfragile
b)Smallwartyalongthelineofclosureofvalve
c)Smallormediumsizedoneitherorbothsidesofvalve
d)Smallblandvegetations
CorrectAnswer-C
Ans.is'c'i.e.,Smallormediumsizedoneitherorbothsidesofvalve
SLE,mitralandtricuspidvalvulitiswithsmall,sterilevegetations,
calledLibman-Sacksendocarditisisoccasionallyencountered.
Thelesionsaresmallsingleormultiple,sterile,granularpink
vegetations
rangingfrom1to4mmindiameter.Thelesionsmaybe
locatedontheundersurfaces
oftheatrioventricularvalves,onthe
valvularendocardium,onthecords,oronthemuralendocardiumof
atriaorventricles.
Anintensevalvulitismaybepresent,characterizedbyfibrinoid
necrosisofthevalvesubstancethatisoftencontiguouswiththe
vegetation.
Subsequentfibrosisandseriousdeformitycanresultthatresemble
chronicRHDandrequiresurgery.

443.Atheromatouschangesofbloodvessels
affectsearlyin-
a)Kidney
b)Heart
c)Liver
d)Spleen
CorrectAnswer-B
Ans.is'b'i.e.,Heart
SitesofAtherosclerosis
Atheroscleroticplaquesdevelopprimarilyinelasticarteries(e.g.,
aorta,carotidandiliacarteries),andlarge&mediumsizedmuscular
arteries(e.g.,coronaryarteryandpoplitialarteries).
Indescendingorder,thevesselsmostcommonlyinvolvedare-
Abdominalaorta(mostcommon)Causinganeurysm
Coronaryarteries(heart)CausingMI
PoplitealarteriesCausingischemicgangreneoflowerlimbs.
InternalcarotidarteriesCausingstrokeaCircleofwillis
Vesselsusuallysparedare
Vesselsofupperextremities
Mesentricandrenalvessels,exceptattheirostia.

444.Whichoneofthefollowingsetsof
componentsofcigarettesmokeisa
causalagentofcoronaryarterydisease-

a)Tarandnicotine
b)Nicotineandcarbonmonoxideandtar
c)CarbonmonoxideandTar
d)Carbondioxide
CorrectAnswer-B
Ans.is`b'i.e.,Nicotineandcarbonmonoxideandtar
Mechanismsforsmoking-inducedCoronaryheartdisease
Carbonmonoxideinducesatherogenesis.
NicotinestimulationofadrenergicdriveraisingbothBPand
myocardialoxygendemand.
Lipidmetabolismwithfallinprotectivehigh-densitylipoproteins.
Note:
Tarisassociatedwithcarcinogenesis(notatherosclerosis).But
amongstthegivenoptionsoptionbisthebestanswerasitconsists
ofbothnicotineandCO.

445.AllistrueaboutGiantcellarteritis
except?
a)Involveslargetosmallsizedareteries
b)Granulomatousinflammation
c)Mostcommonlyinvolvedarteryisabdominalaorta
d)Segmentalnatureoftheinvolvement
CorrectAnswer-C
Ans.is'c'i.e.,Mostcommonlyinvolvedarteryisabdominalaorta
Itisthemostcommonformofsystemicvasculitisinadults,isan
acuteandchronic,oftengranulomatous,inflammationofarteriesof
largetosmallsize.
Itaffectsprincipallythearteriesinthehead-especiallythetemporal
arteries--butalsothevertebralandophthalmicarteriesandthe
aorta,whereitmaycausethoracicaorticaneurysm

446.Mostcommoncauseofdissecting
hematomaisbecauseof-
a)Hypertension
b)Marfan's
c)Iatrogenic
d)Kawasaki
CorrectAnswer-A
Ans.is'a'i.e.,Hyertension
Morethan90%ofdissectionsoccurinmenbetweentheagesof40
and60withantecedenthypertension.

447.PyogenicgranulomtrueA/E
a)Bacterialinfection
b)Bleeding
c)Benigntumour
d)Capillaryhemangioma
CorrectAnswer-A
Answer-A.Bacterialinfection
Pyogenicgranuloma(PG)orlabularcapillaryhemangiomaisa
benignvasculartumouroftheskinormucousmembrane
characterizedbyrapidgrowthandfriablesurface.
Angiogenicgrowthfactorssuchasvascularendothelialgrowth
factors(VEGE)anddecorin,transcriptionfactors,andsignal
transductionpathways(MAPK)areoverexpressedinpyogenic
granulomas

448.Cysticmedialnecrosisisseenin-
a)Marfanssyndrome
b)FriedrichsataxiaPattern
c)Downssyndrome
d)Kawasakidisease
CorrectAnswer-A
Ans.is'a'i.e.,Marfan'ssyndrome
Cysticmedialnecrosis(CMN)isadisorderoflargearteries,
characterizedbyanaccumulationofabasophilicgroundsubstance
inthemediawithcyst-likelesions.
Itisknowntooccurincertainconnectivetissuediseasessuchas
Marfansyndrome,Ehlers-Danlossyndrome,andannuloaortic
ectasia,whichusuallyresultfromdegenerativechangesintheaortic
wall.
TherelationshipsbetweenCMNandcongenitalheartdefectsaswell
asotherdisordershavebeenevidenced.Themechanismsarestill
controversial,eventhoughmanymolecularstudieshavebeen
conducted
itischaracterizedbyelastictissuefragmentationandseparationof
theelasticandfibromuscularelementsofthetunicamediabysmall
cleftlikespaceswherethenormalelastictissueislost;theseareas
arefilledwiththeamorphousextracellularmatrixofconnective
tissueandresemblebutarenottrulycysts.

449.Medialcalcificationisseenin-
a)Atherosclerosis
b)Arteriolosclerosis
c)Monckebergssclerosis
d)Dissectinganeurysm
CorrectAnswer-C
Ans.is'c'i.e.,Monckebergssclerosis
Firstseetypesofarteries
Basedontheirsizeandstructuralfeatures,arteriesaredividedinto
threetypes.
LargeorElasticarteries
Aorta
Itslargebranches,eg.-Innominate,Subclavian,commoncarotid,
iliac.
Pulmonaryarteries.
Mediumsizedormusculararteries
SmallbranchesofaortaCoronary,Renal
Smallarteriesandarterioles
Withinsubstanceofthetissue
Nowseetheiraffection?
Arteriosclerosis
Dontgetconfusewithatherosclerosis.
Arteriosclerosismeanshardeningofarteriesbythickeningandloss
ofelasticityofarterialwall.oTherearethreepatternsof
arteriosclerosis.
Atherosclerosis
Characterizedbyintimallesions,i.e.atheromas,whichprojectinto
vascularlumenandmayobstructit.oThisisthemostcommon

patternofarteriosclerosis.
Itinvolveselasticarteries,andlarge&mediumsizemuscular
arteries.
Arteriolosclerosis
Seeninarteriolesofpatienswithhypertensionanddiabetes.
Twoanatomicvariantsarecommon-->hyalineandhyperplastic
arteriolosclerosis.
Monkebergmedialcalcificsclerosis
Itisseeninsmallandmediumsizedmusculararteries.
Itisadegenerativeandapparentlynon-inflammatorydisease.
Mediaofthesearteriesbecomescalcified.
Itoccursinpatientsolderthan50years.

450.Visceralaneurysmismostcommonly
seenin
a)Splenic
b)Renal
c)Hepatic
d)Coronary
CorrectAnswer-A
Answer-A.Splenic
Mostcommonvisceralarteryaneurysmissplenicarteryaneurysm.
2ndmostcommonvisceralarteryaneurysmishepaticartery
aneurysm.

451.Raynaud'sphenomenonwhatchangeis
seeninvesselsinitialstage-
a)Nochange(Fibrinoid,Thrombosis)
b)Thrombosis
c)Fibrinoidnecrosis
d)Hyalinesclerosis
CorrectAnswer-A
Ans.is'a'i.e.,Nochange
Structuralchangesinthearterialwallsareabsentexceptlateinthe
course,whenintimalthickeningcanappear.


452.Pulmonaryinfarctionoccurswithall
except-
a)Saddleembolusatbifurcation
b)Blockageof2ndand3rdgenendarteries
c)Arteriolesareblocked
d)None
CorrectAnswer-D
Ans.is'd'i.e.,None
Themorphologicconsequencesofembolicocclusionofthe
pulmonaryarteriesdependonthesizeoftheembolicmassandthe
generalstateofthecirculation.
Largeembolimayimpactinthemainpulmonaryarteryoritsmajor
branchesorlodgeatthebifurcationasasaddleembolus.Sudden
deathoftenensues,owinglargelytotheblockageofbloodflow
throughthelungs.Deathmayalsobecausedbyacutefailureofthe
rightsideoftheheart(acutecorpulmonale).
Smallerembolicantraveloutintothemoreperipheralvessels,
wheretheymaycauseinfarction.Inpatientswithadequate
cardiovascularfunction,thebronchialarterialsupplycanoften
sustainthelungparenchymadespiteobstructiontothepulmonary
arterialsystem.Underthesecircumstances,hemorrhagesmay
occur,butthereisnoinfarctionoftheunderlyinglungparenchyma.
Onlyabout10%ofemboliactuallycauseinfarction.Althoughthe
underlyingpulmonaryarchitecturemaybeobscuredbythesuffusion
ofblood,hemorrhagesaredistinguishedbythepreservationofthe
pulmonaryalveolararchitecture;insuchcases,resorptionofthe
bloodpermitsreconstitutionofthepreexistingarchitecture.

453.Creolabodiesareseenin-
a)Bronchialasthma
b)Chronicbronchitis
c)Emphysema
d)Bronchiectatsis
CorrectAnswer-A
Ans.is'a'i.e.,Bronchialasthma
Creolabodiesareahistopathologicfindingindicativeofbronchial
asthma.
Foundinapatient'ssputum,theyareciliatedcolumnarcells
sloughedfromthebronchialmucosaofapatientwithasthma.
Othercommonfindingsinthesputumofasthmapatients
includeCharcot-Leydencrystals,Curschntann'sSpirals,and
eosinophils.


454.Bronchogenicsequestrationisseenin
whichlobe-
a)Leftlowerlobe
b)Rightupperlobe
c)Leftmiddlelobe
d)Leftupperlobe
CorrectAnswer-A
Ans.is'a'i.e.,Leftlowerlobe
Bronchogenicsequestrationreferstothepresenceofadiscrete
massoflungtissuewithoutanynormalconnectiontotheairway
system.
Intralobarsequestrationsarefoundmostfrequentlyintheposterior
basalsegmentoftheleftlowerlobe.
Bloodsupplytothesequesteredareaarisesnotfromthepulmonary
arteriesbutfromtheaortaoritsbranches.
Extralobarsequestrationsareexternaltothelung.Foundmost
commonlyininfantsasabnormalmasslesions,theymaybe
associatedwithothercongenitalanomalies.
Intralobarsequestrationsarefoundwithinthelungsubstanceand
areusuallyassociatedwithrecurrentlocalizedinfectionor
bronchiectasis.

455.Cysticfibrosisisassociatedwithall
except?
a)Infertility
b)Azoospermia
c)Nasalpolyps
d)Renalfailure
CorrectAnswer-D
Ans.isdi.e.,Renalfailure
Thickviscidplugsofmucusmayalsobefoundinthesmallintestine
ofinfants.Sometimesthesecausesmall-bowelobstruction,known
asmeconiumileus.
Chronicsinopulmonarydiseasemanifestedby:Persistent
colonization/infectionwithtypicalcysticfibrosis
pathogens,includingStaphylococcusaureus,nontypeable
Hemophilusinfluenzae,mucoidandnonmucoidPseudomonas
aeruginosa,Burkholderiacepacia;Chroniccoughandsputum
production,nasalpolyps.
Azoospermiaandinfertilityarefoundin95%ofthemaleswho
survivetoadulthood;congenitalbilateralabsenceofthevas
deferensisafrequentfindinginthesepatients.

456.InthestageofGreyhepatization-
a)WBC'sfillthealveoli
b)RBC'sfillthealveoli
c)Organismsfillthealveoli
d)Accumulationoffibrin
CorrectAnswer-D
Ans.is'd'i.e.,Accumulationoffibrin
Pathologicalchangesofbacterialpneumonia
A.Lobarpneumonia
Largeconfluentareaofthelungorentirelobesareconsolidated.
Thelowerlobesareaffectedmostcommonly.
Therearefourstagesoftheinflammatoryresponse(Laennec's
stages)?
Stageofcongestion(initialphase)
Theaffectedlobeisenlarged,heavy,darkredandcongested.
Cutsurfaceexudesblood-stainedfrothyfluid.
Thereisdilatationandcongestionofalveolarcapillaries.
Therearefewneutrophilsandnumerousbacteriainthealveolar
fluid.
Stageofredhepatization(earlyconsolidation)
Thetermhepatizationreferstoliver-likeconsistencyoftheaffected
lobeoncutsection.oTheaffectedlobeisredandfirm.
Theedemafluidofprecedingstageisreplacedbystrandsoffibrin.
Thereismarkedcellularexudateofneutrophilswithextravasationof
redcells.
Stageofgrayhepatization(lateconsolidation)
Theaffectedlobeisgrayishbrown,firmanddry.
Thefibrinstrandaredenseandmorenumerous.

Thereisprogressivedisintigrationofredcellsandneutrophils.
Themacrophagesbegintoappearintheexudate.
Theorganismsarelessnumerousandappearasdegenerated
forms.
Resolution
Thepreviouslysolidandfibrinousconstituentisliquefiedby
enzymaticaction.
Granularandfragmentedstrandsoffibrininthealveolarspacesare
seenduetoprogressiveenzymaticdigestion.
Thereisprogressiveremovaloffluidcontentaswellascellular
exudatefromtheairspaces,resultinginrestorationofnormallung
parenchymawithareation.
B.Bronchopneumonia
Patchyareasofredorgreyconsolidation,moreoftenmultilobarand
frequentlybilateralandbasal(lowerzones)becauseoftendencyof
secretionstogravitateintolowerlobes.
Thereissuppurativeexudate,consistingchieflyneutrophils,filling
bronchi,bronchiolesandadjacentalveolarspaces.
Alveolarseptathickenduetocongestedcapillariesandleucocytic
infiltration.

457.Markerofsmallcellcanceroflungis-
a)Chromogranin
b)Cytokeratin
c)Desmin
d)Vimentin
CorrectAnswer-A
Ans.is'a'i.e.,Chromogranin
Smallcellcarcinomacontains.
Polypeptidehormoree.g.PTHlikepeptide.
Neuroendocrinemarkers-4chromogranin,Synaptophysin,Leu7

458.Collapseoflungiscalled-
a)Emphysema
b)Bronchiactasis
c)Atelectasis
d)Bronchitis
CorrectAnswer-C
Ans.is'c'i.e.,Atelectasis
Atelectasis(collapse)
Atelectasisrefersto:-

1. IncompleteexpansionoflungsNeonatalatelectasis.or
2. CollapseofpreviouslyinflatedlungAcquiredatelectasis.
Thisresultsinareasofrelativelyairlesspulmonaryparenchyma.
Acquiredatelectasismaybedividedinto
Resorption(obstruction)atelectasis
Itisconsequenceofcompleteobstructionofanairway.
Withtimetrappedoxygenintheaffectedalveoliisresorbedand
collapseoccurs.
Resorptionatelectasisiscausedprincipallybyobstructiondueto
excessivesecretions(mucusplug)orexudateswithinsmall
bronchiolesasseeninBronchialasthma,chronicbronchitis,
Bronchiactasis
andforeignbodyaspiration.
Compressionatelectasis
Collapseoflungoccursduetoexternalperssureonlung.
Itisseenmostcommonlyinpatientswithcardiacfailurewho
developpleuraleffusionandinpatientswithneoplasticpleural
effusion.
Contractionatelectasis
Thecollapseisduetocontractureinthelungbecauseoffibroctic

changes.
Exceptforcontractionatelectasis,atelectasisisareversible
disorder,i.e.collapsedlungparenchymacanbere-expanded.

459.FrequencyofrenalinvolvementinHSP?
a)20-40%
b)>80%
c)40-60%
d)10%
CorrectAnswer-C
Ans.is'c'i.e.,40-60%
ThereportedincidenceofrenalinvolvementinHSPvaries
considerablybetweendifferentstudies.
Thismaybebecauseofthedifferentcriteriausedtodescribethe
involvement&thevariabilityofthelengthusedtofollowup.
IndifferentstudiesincidenceofpediatricrenalinvolvementinHSP
wasbetween20-56%andinadults50-78%

460.Mostimportantprognosticfactorof
wilmstumour-
a)Histopathology
b)Ploidyofcells
c)Age<1yr
d)Mutation,ofclpgene
CorrectAnswer-A
Ans.is'a'i.e.,Histopathology
Anaplasticnuclearchangeistheonlycriterionof"unfavourable"
histologyinWilm'stumorandallWilm'stumorlackingthisfeature
aredesignatedashaving"favourablehistology".
Anaplasticnuclearchangereflectsextremepolypoidy&isusually
apparentunderlowmagnification.
"Anaplastichistologyremainsacriticaldeterminantofadverse
prognosis.Evenanaplasiarestrictedtokidney
confersanincreasedriskofrecurrence&death,underscoringthe
needforcorrectlyidentifyingthishistologicfeatures."

461.Renalpapillarynecrosisisalmostalways
associatedwithoneofthefollowing
conditions:
March2004

a)Diabetesmellitus
b)Analgesicnephropathy
c)Chronicpyelonephritis
d)Poststreptococcalglomerulonephritis
CorrectAnswer-A
Ans.Ai.e.Diabetesmellitus
`Renalpapillarynecrosis,anaccompanimentofacutepyelonephritis
ismostoftenseenindiabeticsandischaracterisedbynecrosisof
renalpapillaeofoneorbothkidneyswithsharpeddemarcation
betweennecroticandlivingtissue'--Dorlands
Thuswhilepapillarynecrosisisafeatureofmorethanone
conditionsmentionedinthequestion,itismostcommonlyseenwith
diabetesmellitus.

462.Oncocyticcarcinomaarisesfrom-
a)Perivascular
b)Glomerulus
c)Loopofhenle
d)Collectingduct
CorrectAnswer-D
Ans.is'd'i.e.,Collectingducts
OncocyticorChromophoberenalcarcinomarepresents5%ofrenal
cellcancers.
Itiscomposedofcellswithprominentcellmembranesandpale
eosinophiliccytoplasm,usuallywithahaloaroundthenucleus.
Theyare,likethebenignoncocytoma,thoughttogrowfrom
intercalatedcellsofcollectingducts.oTheyhaveanexcellent
prognosiscomparedwiththatoftheclearcellandpapillarycancers.

463.
Mostcommoncauseofnephrotic
syndromeinadults?
a)Membranousglomerulonephritis
b)Minimalchangedisease
c)AcuteGN
d)FocalGN
CorrectAnswer-A
Ans.is'a'i.e.,Membranousglomerulonephritis
MostcommoncauseofNephroticsyndrome
Inadultsmembranousglomerulonephritis
Inchildrenminimalchangedisease(lipoidnephrosis)Causesof
Nephroticsyndrome
I. PrimaryGlomerulonephritis
V
Malignancy
Minimalchangedisease(mostcommonin
1.
1. Carcinomas
children)
2. MembranousGN(mostcommoninadults) 2. Myeloma
Hodgkin's
3. MembranoproliferativeGN
3. disease
4. Focalsegmentalglomerulosclerosis
VI Pregnancy
Toxaemiaof
5. FocalGN
pregnancy
VIICirculatory
6. IgAnephropathy
Disturbances
1.Renalvein
II Systemicdiseases
thrombosis
Constrictive
1. Diabetesmellitus
2. pericarditis

2. Amyloidosis
VIII.Hereditary
diseases
1.Alport's
3. SLE
disease
M. SystemicInfections
2. Fabry'sdisease
Nail-patella
1. Viralinfections(HBV,HCV,HIV)
3. syndrome
Bacterialinfections(bacterialendocarditis,
2.
syphilis,leprosy)
Protozoaandparasites(P.falciparum
3.
malaria,filariasis)
IV. HypersensitivityReactions
1. Drugs(heavymetalcompoundslikegold
andmercury,otherdrugs
likepenicillamine,trimethadioneand
tolbutamide,heroinaddiction)
2. Beestings,snakebite,poisonivy

464.Mostcommoncauseofnephritic
syndromeinadults?
a)Membranoproliferativeglomerulonephritis
b)FSGN
c)Membranousglomerulonephritis
d)None
CorrectAnswer-D
Ans.isNone
MostcommoncauseofNephriticsyndromeisrapidlyprogressive
glomerulonephritis(Poststreptococcalandnonstreptococcal
glomerulonephritis).
Membranousglomeruulonephritisisthemostcommoncauseof
nephroticsyndrome(notnephriticsyndrome)inadults.
Minimalchangediseaseisthemostcommoncauseofnephrotic
syndromeinchildren.
CausesofAcuteNephriticSyndrome
Primary
SystemicDisease
Glomerulonephritis
1.
AcuteGN
1.
SLE
Post-streptococcal
2.
Polyarteritisnodosa
Wegener's
Non-streptococcal
3.
granulomatosis
Henoch-Schonlein
2.
RapidlyprogressiveGN 4.
purpura
Membranoproliferative
3.
5 Cryoglobulinaemia
GN
4.
FocalGN
5.
IgAnephropathy

5.
IgAnephropathy

465.Podocytesareseenin-
a)Bowman'scapsule
b)Proximalconvolutedtubule
c)Distalconvolutedtubule
d)Collectingtubuleofthekidney
CorrectAnswer-A
Ans.is'a'i.e.,Bowman'scapsule
TheBowman'scapsule(capsulaglomeruli/glomerularcapsule)isa
cup-likesacatthebeginningoftubularcomponentofanephron.
Aglomerulusisenclosedinthesac(Bowman'scapsule).
FluidsfrombloodintheglomerulusarecollectedinBowman's
capsule(i.e.glomerularfiltrate)andfurtherprocessedalongthe
nephrontoformurine.
OutsidetheBowman'scapsuletherearetwopoles.
VascularpoleThesidewithafferentandefferentarterioles.
UrinarypoleThesidewithproximalconvulutedtubule.
InsidetheBowman'scapsule,thelayersareasfollows,frominside
tooutside?
Thinlayeroffenestratedendothelium
Glomerularbasementmembrane
Partoftheglomerulus
Visceralepithelium(Podocytes)
Parietalepithelium

466.Crescentsarederivedfrom-
a)Epithelialcells+fibrin+macrophage
b)Mesangium+fibrin+macrophage
c)Tubule+mesangiaum+fibrin
d)Mesangiaum+fibrin
CorrectAnswer-A
Ans.is'a'i.e.,Epithelialcells+fibrin+macrophage
RenalfeaturesinRPGN
Crescentformation-crescentsareformsby?
*CellsParietalepithelialcells+WBCs(monocytesand
macrophages)
*Fibrinstrandsareprominentbetweenthecellularlayersinthe
crescents.
-RuptureofGBM
-SubepithelialdepositsintypeIIRPGN
-LineardepositsintheGBMintypeIRPGN.

467.Transitionalcellcarcinomabladder
causedby-
a)Schistosomiasis
b)Ascariasis
c)Malaria
d)None
CorrectAnswer-A
Ans.is'a'i.e.,Schistosomiasis
Transitionalcellcarcinoma(TCC).Thisisthecommonestcancerof
thebladder.
Morethan90%ofbladdertumorsarisefromtransitionalepithelial
(urothelium)liningofthebladderincontinuitywiththeepithelial
liningoftherenalpelvis,ureters,andthemajorpartoftheurethra.
etiology:
Industrialoccupations:Workersinindustriesthatproduceaniline
dyes,rubber,plastic,textiles,andcablehaveahighincidenceof
bladdercancer.
Schistosomiasis:Thereisanincreasedriskofbladdercancer,
particularlysquamouscellcarcinoma,inpatientshavingbilharzial
infestation(Schistosomahaematobium)ofthebladder.
Dietaryfactors:Certaincarcinogenicmetabolitesoftryptophanare
excretedintheurineofpatientswithbladdercancer.
Locallesions:Severallocallesionsinthebladderpredisposetothe
developmentofbladdercancer.Theseincludeectopiavesicae
(atrophiedbladder),vesicaldiverticulum,leukoplakiaofthebladder
mucosaandurinarydiversioninthedefunctionalizedbladder.
Smoking:Tobaccosmokingisassociatedwitha2to3fold
increasedriskofdevelopingbladdercancer,probablydueto

increasedurinaryexcretionofcarcinogenicsubstances.
Drugs:Immunosuppressivetherapywithcyclophosphamideand
patientshavinganalgesic-abuse(phenacetin-)nephropathyhavea
highriskofdevelopingbladdercancer.

468.Starryskyappearanceisseenin-
a)Burkittslymphoma
b)CIL
c)DiffuselargeBcelllymphoma
d)ALCL
CorrectAnswer-A
Ans.is'a'i.e.,Burkittslymphoma
Burkittslymphoma
Itincludes(1)African(endemic)Burkittlymphoma,(2)sporadic
(nonendemic)Burkittlymphoma,and(3)asubsetofaggressive
lymphomasoccurringinindividualsinfectedwithHIV.
Theinvolvedtissuesareeffacedbyadiffuseinfiltrateof
intermediate-sizedlymphoidcellscontainingroundorovalnuclei
withcoarsechromatin,severalnucleoli,andamoderateamountof
faintlybasophilicoramphophiliccytoplasm.
Thenuclearsizeapproximatesthatofbenignmacrophageswithin
thetumor.
Ahighmitoticindexistypical,asisapoptotictumorcelldeath,
accountingforthepresenceofnumeroustissuemacrophageswith
ingestednucleardebris.
Thesebenignmacrophagesarediffuselydistributedamongthe
tumorcellsandhaveabundantclearcytoplasm,creatinga
characteristic"starrysky"pattern.
AllformsofBurkittlymphomaareassociatedwithtranslocationsof
thec-MYCgeneonchromosome8.Thepartnerinthetranslocation
isusuallytheIgHlocus(t(8;14))butmayalsobetheic(t(2;8))ory
(t(8;22))lightchainlocus.
EssentiallyallendemictumorsarelatentlyinfectedwithEBV.


469.BALLiSdueto-
a)Tcells
b)ImmatureBcells
c)ImmatureTcells
d)BothT&Bcells
CorrectAnswer-B
Ans.is'b'i.e.,ImmatureBcells
InBALL,precursorcellsarearrestedatstagesprecedingsurface
expressionoflg.Theleukemicblastsalmostalwaysexpressthepan
B-cellmoleculesCD19andCD10.
Inveryearlypre-BcellALL;CD19istheonlyBcell-specificmarker
present.
Earlypre-BALLisdistinguishedfromlatepre-BALLbytheabsence
ofcytoplasmicIgMheavychainintheformer.

470.Whatisthechromosomaltranslocation
inAMLM3-
a)T(18,21)
b)T(15,17)
c)T(8,21)
d)T(9,11)
CorrectAnswer-B
Ans.is'b'i.e.,T(15,17)
Class Chromosomalabnormalities
Mz
t(8:21)
M3
t(15:17)
M4
inv(16)

471.D.I.C.isseenin:
a)>Acutepromyelocyticleukemia
b)>Acutemyelomonocyticleukemia
c)>CMC
d)>Autoimmunehemolyticanemia
CorrectAnswer-A
Acutepromyelocyticleukemia[Ref.Harrison16'11/ep636]
Disseminatedintravascularcoagulationisassociatedwith
promyelocyticleukemia
Acutepromyelocyticleukemia
(AML-M3)constitutes5-10%ofall
casesofAML
Theleukemiccellsofthesetypeofanemiaarehypergranular.
Granulesoftheseleukemiccells(promyelocytes)contain
thromboplastinlikematerialresultinginwidespreaddisseminated
intravascularcoagulation.
Alsoknow
MajorityofM3casesdemonstrateareciprocaltranslocation
involvingchromosome15and17,t(15;17)

472.Helmetcellsarecharacteristicofanemia
of?
a)Hemolyticuremicsyndrome
b)Polysplenia
c)Spherocytosis
d)Acanthocytosis
CorrectAnswer-A
Ans.is'a'i.e.,Hemolyticuremicsyndrome
Schistocytes,helmentcells,trianglecells,burrcellsareseenin
microangiopathichemolyticanemia(MAHA).Changesinerythrocyte
morphology

473.Schistocyteis/arefoundin-
a)TTP
b)DIC
c)Severeirondeficiency
d)All
CorrectAnswer-D
Ans.isall
Schistocytes
Aschistocyteisdefinedasanirregularlyshapederythrocyte
fragmentcausedbymechanicaltraumaoranintrinsicabnormalityof
erythrocytes.
Schistocytes:MechanismsandDiseases
Shearingbyfibrinstrands:
Microangiopathichemolyticanemia
Disseminatedintravascularcoagulation
HemangiosarcomaoGlomerulonephritisoMyelofibrosis
Hemolyticuremicsyndrome
HyperspleenismTurbulentbloodflow
CongestiveheartfailureoValvularstenosis
Cavalsyndromeinheartwormdisease
HemangiosarcomaIntrinsicabnormalities
Chronicdoxorubicintoxicosis
evereirondeficiencyanemia
Pyruvatekinasedeficiency
Congenitalandacquireddyserythropoiesis
Microangiopathichemolyticanemiacanoccurin:-oExternalimpact
:Marchhemoglobinuria
Cardiachemolysis:Prostheticcardiacvalves

Fibrindepositinmicrovasculature:TTP,DIC,HUS

474.Lacunarcellsisseeninwhichtypeof
HodgkinsLymphoma:
September2010

a)Mixedcellularitytype
b)Lymphocytepredominant
c)NodularSclerosisType
d)Alloftheabove
CorrectAnswer-C
Ans.C:NodularSclerosisType
Reed-Sternbergcells(alsoknownaslacunarhistiocytesforcertain
types)aredifferentgiantcellsfoundonlightmicroscopyinbiopsies
fromindividualswithHodgkin'slymphoma(akaHodgkin'sdisease;a
typeoflymphoma),andcertainotherdisorders.Theyareusually
derivedfromBlymphocytes.

475.Russellbodiesarefoundin-
a)MultipleMyeloma
b)Gonadaltumor
c)Parkinsonism
d)Intracranialneoplasms
CorrectAnswer-A
Ans.is'a'i.e.,MultipleMyeloma
Plasmacelltumoursarecharacterizedbydysregulatedsynthesis
andsecretionofimmunoglobulin.
Thissometimesleadstointracellularaccumulationofintactor
partiallydegradedimmunoglobulins.Thisproducescertainvariants
ofplasmacellswhichare:
FlamecellsCharacterizedbypresenceoffieryredcytoplasm.
MOTTcellsCharacterizedbypresenceofmultiplebluegrapelike
cytoplasmicdroplets.
Besidesthesetherearecellscontainingvarietyofotherinclusions
including.
Fibrils
Russelbodies(cytoplasmic)
Crystallinerods.
Dutcherbodies(nuclear)

476.CDmarkerofhistiocytosisis-
a)CD1A
b)CD1B
c)CDIC
d)CD1D
CorrectAnswer-A
Ans.is'a'i.e.,CD1A
oCHIAisaTcellsurfaceantigenimportantindendriticcell
presentationofglycolipidsandlipopeptideantigens.oItisusedto
diagnoseLangerhanscellhistiocytosis.

477.Shapeofbirbeckgranulesis?
a)Tennisracket
b)Hockeystick
c)Bat
d)Ball
CorrectAnswer-A
Ans.is'a'i.e.,Tennisracket
Undertheelectronmicroscope,Birbeckgranuleshavea
pentalaminar,rodlike,tubularappearanceandsometimesadilated
terminalendresemblingtennis-racketappearance.

478.Theprimarydefectwhichleadstosickle
cellanemiais-
a)Anabnormalityinporphyrinpartofhemoglobin
b)Replacementofglutamatebyvalineinn-chainofHbA
c)AnonsencemutationintheI3-chainofHbA
d)Substitutionofvalinebyglutamateinthea-chainofHbA
CorrectAnswer-B
Ans.is'b'i.e.,Replacementofglutamatebyvalinein13chainof
HBA
Sicklecellanemia
Sicklecellanemiaisahereditaryhemoglobinopathy,i.e.,thetypeof
diseasecharacterizedbyproductionofdefectivehemoglobin.
Sicklecellanemiaresultsfrommutationin13-globingene.
Itiscausedbyapointmutationatthesixthpositionofthef3-globin
chainleadingtosubsitutionofavalineresidueforaglutamicacid
residueresultinginsicklehemoglobin(HbS).
Sicklecellanemiaisanautosomalrecessivedisorder.
oIfanindividualishomozygousforthesicklecellmutation,almost
allthehemoglobinintheredcellisHbS.oInheterozyogotes,only
about40%ofthehemoglobinisHbS,theremainderbeingnormal
hemoglobins.

479.Intravascularhemolysisoccursin-
a)Hereditaryspherocytosis
b)AcuteG6PD
c)Sicklecellds
d)bandc
CorrectAnswer-D
Ans.is'b'i.e.,AcuteG6PD;'c'i.e.,Sicklecelldis
Extravascularhemolysis
Hereditaryspherocytosis
Thalassemia
Sicklecellanemia
Autoimmunehemolyticanemia3Druginducedimmunehemolytic
anemia
G-6-PDdeficiency
Intravascularhemolysis
ParoxysmalnocturnalhemoglobinuriaoG-6-PDdeficiency
Clostridialtoxin
Falciparummalaria
Mechanicalinjurytoredcells
Defectivecardiacvalves
Thrombininmicrocirculation
Sicklecellanemia(minor)
Note:-InG-6-PDdeficiencybothextravascularandintravascular
hemolysisoccur.
InSicklecellanemia,usuallythereisextravascularhemolysisbut
intravascularhemolysiscanalsooccur.

480.Allofthefollowingprovideprotection
againstmalariaexcept-
a)Duffybloodgroup
b)Sicklecellanemia
c)Hereditaryspherocytosis
d)PNH
CorrectAnswer-D
Answer-D.PNH
PNHisahemolyticanemiacausedbyanacquiredintrinsicdefectin
thecellmembrane.
PNHresultsfromacquiredmutationthatinhibitsthesynthesisof
Glycosylphospatidylinositol(GPI).

481.PNHpatientswillbehavingdeficientsurfaceproteinsthatnormallyprotect
theredcellsfromactivatedcompliments.Whatarethetwodeficientsurface
proteins?

a)CD45andCD59
b)CD51andCD59
c)CD55andCD59
d)CD58andCD59
CorrectAnswer-C
ThedefinitivediagnosisofPNHisbasedonthedemonstrationthata
substantialproportionofthepatient'sredcellshaveanincreased
susceptibilitytocomplement(C),duetothedeficiencyontheir
surfaceofproteins(particularlyCD59andCD55).
Reference:HarrisonsPrinciplesofInternalMedicine,18thEdition,
Page884

482.Coldagglutininis-
a)IgG
b)IgM
c)IgA
d)IgD
CorrectAnswer-B
Ans.is'b'i.e.,IgM
Coldagglutininhemolyticanemia
ThisformofhemolyticanemiaiscausedbycoldagglutininIgM
antibodies.
AntibodiesreactwithRBCat0?to4?C.
Bothintravascularandextravascularhemolysismayoccur.
IgMantibodybindtoRBCandfixcomplementonRBCbefore
complementmediatedhemolysisoccur.Thistransientreactionwith
IgMdepositC3bonRBC.C3bactsasanopsoninisthatenhances
thephagocytosisofRBCsinmononuclearphagocyticsystemofliver
andspleenextravascularhemolysis.
HoweverifIgMremainsattachedtoRBC,sustainedactivationof
complementresultsinformationoflargeamountofmembrane-
attackcomplexthatdestroysRBCdirectly-->Intravascular
hemolysis.
Causesofcoldagglutininimmunohemolyticanemia:-Mycoplasma
infection,IMN,CMV,Influenza,HIV,Malignantlymphoma

483.Theanaemiaassociatedwithleukaemia
is?
a)Irondeficiency
b)Megaloblastictype
c)Myelophthisictype
d)Allofabove
CorrectAnswer-C
Ans.is'c'i.e.,Myelophthisistype
Myelophthisicanemia
Space-occupyinglesionsthatdestroysignificantamountofbone
marrowordisturbthemarrowarchitecture,depressitsproductive
capacity.Thisformofanemiaisreferredtoasmyelophthisic
anemia.
Characteristically,immatureerythroidandmyeloidprogenitors
appearsintheperipheralblood-->Leukoerythroblastosis.
Causesare
Metastaticcancer(M.C.cause)
Granulomatousdiseaseinfiltratingbonemarrow.
Myeloproliferativedisorders(suchasmyeloidleukemia)
Myelofibrosis

484.Maximumspherocytesisseenin-
a)Autoimmunehaemolyticanemia
b)VitB12deficiency
c)Aplasticanemia
d)None
CorrectAnswer-A
Ans.is'a'i.e.,Autoimmunehemolyticanemia
Mostcommoncauseofspherocytesisimmunehemolyticanemia.

485.InDIC,followingareseenexcept-
a)Fibrinogendecreased
b)Thrombocytopenia
c)NormalAPTT
d)PTelevation
CorrectAnswer-C
Ans.is'c'i.e.,NormalAPTT

486.Nottrueregardingfreshfrozenplasma?
a)Suppliesmajorcoagulationfactors
b)ABOmatchnotrequired
c)ShouldbeusedinreplacementoffactorsinDIC/trauma
d)Tobeusedwithin30minutesofhavingtrauma
CorrectAnswer-D
Ans.is'd'i.e.,Tobeusedwithin30minutesofhavingtrauma
Freshfrozenplasmacontainscomponentsofthecoagulation,
fibrinolyticandcomplementsystemsandotherproteins.
FFPcanbegivenwithoutbloodtype-matchedalthoughagglutination
reactionsarepossible,thoughveryrare.oThereisnomentionof
timeperiodwithinwhichFFPshouldbegivenaftertrauma.
TherisksofFFPincludediseasetransmission,anaphylactoid
reactions,alloimmunization,andexcessiveintravascularvolume,as
wellastransfusionrelatedacutelunginjuryandanincreasein
infections

487.Whatisthevenoushematocritlevelat
whichyouwilldiagnosepolycythemiain
anewborn?

a)55%
b)60%
c)65%
d)70%
CorrectAnswer-C
Definitionofpolycythemia:
a.Venoushaematocritofover65%.
b.Venoushaematocritofover64%at2hoursage.
c.Anumbilicalvenousorarterialhaematocritover63%ormore.
Themeanvenoushaematocritofterminfantsis53incordblood,60
at2hoursofage,57at6hoursofageand52at12-18hoursofage.
Asthehaematocritincreases,thereisincreasedviscosityand
decreasedbloodflow.Whenhaematocritincreasestomorethan
60%thereisafallinO2transport.
Definitionofhyperviscosity:Viscositygreaterthan14.6centiposeat
ashearrateof11.55asmeasuredbyaviscometer.(normalis1.4-
1.8centipoise).

488.Ecadheringenedeficiencyisseenin-
a)Gastricca
b)Intestinalca
c)Thyroidca
d)Pancreaticca
CorrectAnswer-A
Themajorityofgastriccancersarenothereditary,themutations
identifiedinfamilialgastriccancerhaveprovidedimportantinsights
intomechanismofcarcinogenesisinsporadiccancers.
GermlinemutationsinCDHI,whichencodesEcadherin,aprotein
thatcontributestoepithelialintercellularadhesionareassociated
withfamilialgastriccancers,whichareusuallyofdiffusetype.
MutationsinCDH1arepresentinabout50%ofsporadiccasesof
diffusegastriccancers,whileEcadherinexpressionisdrastically
decreasedintherest,oftenbymethylationoftheCDHIpromotor.
Lobularcarcinomaofbreastwhichalsotendstoinfilterateassingle
cells,&individualswithBRCA2mutationsareatincreasedriskof
developingdiffusegastriccancers.

489.MostcommonsiteofGISTis-
a)Ileum
b)Esophagus
c)Colon
d)Stomach
CorrectAnswer-D
Ans.is'd'i.e.,Stomach
?
50-
Stomach
--> 70%
?Small
20-
intestine
--> 30%
Colon&
5-
rectum
-3 15%
Esophagus ---> <5%

490.Pathologicalmanifestationofchronic
alcoholismincludeallofthefollowing
except-

a)Piecemealnecrosis
b)Balloningdegeneration
c)Microvesicularfattychanges
d)Centralhyalinesclerosis
CorrectAnswer-A
Ans.is'a'i.e.,Piecemealnecrosis
Steatosis(fattyliver)
?Microvesicularfattychange
?Laterchangestomacrovesicularfattychange
Alcoholichepatitis
?Hepatocellularnecrosis
?Ballooningdegeneration
?Mallorybodies
?Neutrophilicinfiltration
?Perivenularandperisinusoidolfibrosis-->centralhyaline
Alcoholiccirrhosis
?Nodularity
?Fibrosis

491.Thoriuminducedtumor
a)Angiosarcomaofliver
b)Renalcellcarcinoma
c)Lymphoma
d)Astrocytoma
CorrectAnswer-A
Answer-A.Angiosarcomaofliver
Thorotrost(thoriumdioxide)iscommonlyassociatedwithliver
neoplasm.
Themostcommonliverneoplasmassociatedwiththoriumexposure
Angiosarcomaoftheliver
Cholangiocarcinoma
Hepatocellularcarcinoma

492.Klatskintumoris-
a)Nodulartypeofcholangiocarcinoma
b)Fibrolamellarhepatocellularcarcinoma
c)Gallbladdercarcinoma
d)Hepatocellularcarcinoma
CorrectAnswer-A
Ans.is'a'i.e.,Nodulartypeofcholangiocarcinoma
oAccordingtotheirlocalization,cholongiocarcinomasareclassified
into
1.Intrahepatic(10-20%)
2.Extrahepatic(80-90%)
oTheextrahepaticformsincludeperihilartumorsknownasKlastkin
tumors,
whicharelocatedatthejunctionoftherightandlefthepatic
ductsformingthecommonhepaticduct,anddistalducttumors.
oAsubgroupofdistaltumorsariseintheimmediatevicinityof
ampullaofvater.Tumoroftheseregionalsoinclude
adenocarcinomaofduodenalmucosaandpancreaticcarcinomaand
arecollectivelyreferredtoasperiampullarycarcinomas.

493.Allofthefollowingareriskfactorsfor
carcinomagallbladder,EXCEPT-
a)Typhoidcarriers
b)Adenomatousgallbladderpolyps
c)Choledochalcysts
d)Oralcontraceptives
CorrectAnswer-D
Ans.isdi.e.,OralContraceptives
RiskfactorsforCaGallbladderare:
i)Gallstones
ii)Adenomatousgallbladderpolyps(particularlypolypslarger
than10mm)
iii)Calcified(porcelain)gallbladder
iv)Choleduochalcyst
v)Estrogens
vi)Anomalouspancreaticobiliaryductjunction
vii)Exposuretocarcinogens(azotoulene,nitrosamine)
viii)Typhoidcarriers
ix)Sclerosingcholangitis

494.Nutmegliverisseenin-
a)Rightsidedheartfailure
b)Leftsidedheart,failure
c)Increasedpulmonarypressure
d)Decreasedpulmonarypressure
CorrectAnswer-A
Ans.is'a'i.e.,Rightsidedheartfailure
oThecombinationofhypoperfusionandretrogradecongestion
(Chronicpassivecongestion)actssynergisticallytogenerate
centrilobularhemorrhagicnecrosis.Thelivertakesonavariegated
mottledappearancereflectinghemorrhageandnecrosisinthe
centrilobularregions,knowasnutmegliver.
oChronicpassivecongestionisseeninRHF.

495.Centralstellatescarisseenin
a)Focalnodularhyperplasia
b)Chronicregenerativehyperplasia
c)Hepatoblastoma
d)None
CorrectAnswer-A
Answer-A.Focalnodularhyperplasia
Focalnodularhyperplasiaisararetumor-likecondition
predominantlyfoundinwomenduringtothirdtothefifthdecadeof
life.
Mostcommonlyitisincidentallydiscoveredasasymptomaticmass.
Thepathognomonicmicroscopicfeature.
Acentralstellatescarseeninliverimagingstudiesisahighly
characteristicfeatureoffocalnodularhyperplasia.
ItisalsoseeninRenalcellcarcinoma(RCC).

496.Histopathologyofchronichepatitis-
a)Balloning
b)Councilmanbodies
c)Bridgingnecrosis
d)All
CorrectAnswer-C
Ans.is'c'i.e.,Bridgingnecrosis
.Portaltractsshowseverechronicinflammationwith
inflammatorycellsextendingintotheliverlobulesdisrupting
thelimitingplateofhepatocytes.
-Piecemealnecrosis-Portalfibrosis
.Bridgingnecrosis-Interfacehepatitis

497.Spongiformdegenerationofcerebral
cortexoccursin-
a)Creutzfeldt-Jakobdisease
b)Subacutesclerosingpanencephalitis
c)Fatalfamilialinsomnia
d)Cerebraltoxoplasmosis
CorrectAnswer-A
Ans.is'a'i.e.,Creutzfeldt-Jakobdisease
oOnmicroscopicexamination,inCreutzfeldt-Jakobdiseasethe
pathognomonicfindingisaspongiformtransformationofthe
cerebralcortexand,often,deepgraymatterstructures(caudate,
putamen);thisconsistsofamultifocalprocessthatresultsinthe
unevenformationofsmall,apparentlyempty,microscopicvacuoles
ofvaryingsizeswithintheneuropilandsometimesintheperikaryon
ofneurons.Inadvancedcases,thereissevereneuronalloss,
reactivegliosis,andsometimesexpansionofthevacuolatedareas
intocystlikespaces("statusspongiosus").
oKuruplaquesareextracellulardepositsofaggregatedabnormal
protein;theyareCongored-positiveaswellasPAS-positiveand
occurinthecerebellumincasesofGerstmann-Striiussler-Scheinker
syndrome;
theyarepresentinabundanceinthecerebralcortexin
casesofvariantCJD.
oInallformsofpriondisease,immunohistochemicalstaining
demonstratesthepresenceofproteinase-K-resistantPrPscin
tissue.
oFatalfamilialinsomniadoesnotshowspongiformpathology.
Instead,themoststrikingalterationisneuronallossandreactive
gliosisintheanteriorventralanddorsomedialnucleiofthethalamus;

neuronallossisalsoprominentintheinferiorolivarynuclei.

498.Whatisthehistologicalappearanceof
braininCreutzfeldt-Jakobdisease-
a)Neuronophagia
b)Spongiformchangeinbrain
c)Microabscesses
d)Demyelination
CorrectAnswer-B
Ans.is'b'i.e.,Spongiformchangeinbrain
oTheclassichistologicappearanceinCreutzfeldt-Jakobdiseaseis
spongiformchangeinthegraymatter:inallsixcorticallayersinthe
cerebralcortexorwithdiffuseinvolvementofthecerebellar
molecularlayer.Thesevacuolesappearglassyoreosinophilicand
maycoalesce.Neuronallossandgliosisarealsoseen.

499.Whichofthefollowingtumorsisnot
derivedfrommeninges-
a)Hemangioblastoma
b)Meningioma
c)Fibroustumor
d)Hemangiopericytoma
CorrectAnswer-A
Ans.is'a'i.e.,Haemangioblastoma
Meningealtumors
oMeningothelialtumor:Meningioma(mostcommonmeningeal
tumor).
oMesenchymalnon-meningothelialtumors:Meningealsolitary
fibroustumor,hemangiopericytoma,
meningealsarcoma,fibrous
histiocytoma,meningealmelanocytoma,(melanocytictumor).

500.Whichofthefollowingismostreliable
featureofmalignanttransformationof
pheochromocytoma?

a)Presenceofmitoticfigures
b)Capsularinvasion
c)Vascularinvasion
d)None
CorrectAnswer-D
Ans.isNone
"Becausebenignandmalignantpheochromocytomasmayhavean
identicalhistologicalappearance,theonlyabsolutecriterionof
malignancyismetastasis."
"Thediagnosisofmalignantpheochromocytomaismadeonlywhen
metastasisisdemonstrated."


501.Tumorthatfollowsruleof10is-
a)Pheochromocytoma
b)Oncocytoma
c)Lymphoma
d)Renalcellcarcinoma
CorrectAnswer-A
Ans.is'a'i.e.,Pheochromocytoma
oPheochromocytomasusuallysubscribetoaconvenient"ruleof
10s":-
10%ofpheochromocytomasariseinassociationwithoneof
severalfamilialsyndromes.
TheseincludetheMEN-2A&MEN-2B
syndromes,typeIneurofibromatosis,vonHippel-Lindausyndrome&
Sturge-Webersyndrome.
10%ofpheochromocytomasareextra-adrenal,occurringin
sitessuchasorganofZuckerkandl&carotidbody,wherethese
chromaffin-negativetumorsareusuallycalledparagangliomasto
distinguishthemfrompheochromocytomas.
10%ofnonfamilialadrenalpheochromocytomasarebilateral;
thisfiguremayriseto70%incasesthatareassociatedwithfamilial
syndromes.
10%ofadrenalpheochromocytomasarebiologicallymalignant,
althoughtheassociatedhypertensionrepresentsaserious&
potentiallylethalcomplicationofeven"benign"tumors.
10%ofadrenalpheochromocytomasariseinchildhood,usuallythe
familialsubtypes,andwithastrongmalepreponderance.The
nonfamilialpheochromocytomasmostoftenoccurinadultsbetween
40&60yearsofage,withaslightfemalepreponderance.

502.TrueaboutPsammomabodiesareall
except?
a)Seeninmeningioma
b)Concentricwhorledappearance
c)ContainsCalciumdeposits
d)Seeninteratoma
CorrectAnswer-D
Ans.is'd'i.e.,Seeninteratoma
oPsammomabodiesrepresentaprocessofdystrophiccalcification.
oSinglenecroticcellsmayconstituteseedcrystalsthatbecome
encrustedbythemineraldeposits.Theprogressiveacquisitionof
outerlayersmaycreatelamellatedconfigurations,called
psammomabodiesbecauseoftheirresemblancetograinsofsand.
oPsammomabodiesareseenin:papillarycancerofthyroid,
meningioma,papillaryserouscystadenocarcinomaofovary.


503.Inthymoma,allareseenexcept-
a)Hypogammaglobulinemia
b)Hyperalbuminemia
c)Redcellaplasia
d)MyaestheniaGravis
CorrectAnswer-B
Ansis(b)i.e.hyperalbuminemia
ThymomaisthemostcommonAnteriormediastinalmass.
Thymomasareseentobeassociatedwith
Myastheniagravis
Acquiredhypogammaglobulinemia
PureredcellaplasiaGrave'sds
-Perniciousanemia
-Dermatomyositis-polymyositis
-Cushingsyndrome


504.Orphanannieeyenucleiappearanceis
characteristicof-
a)Papillarycarcinomathyroid
b)Carcinomapituitary
c)Paraganglioma
d)Meningioma
CorrectAnswer-A
Ans.is'a'i.e.,Papillarycarcinomathyroid
oThenucleiofpapillarycarcinomacellscontainfinelydispersed
chromatin,whichimpartsanopticallyclearoremptyappearance,
givingrisetothedesignationgroundglassorOrphanAnnieeye
nuclei.

505.Mostcommonmalignancyoffallopian
tube
a)SCC
b)SerousCA
c)Teratoma
d)Chorioca
CorrectAnswer-B
Answer-B.SerousCA
Serous-49.5%-83.3%

506.Rokitanskiprotruberencesareseenin-
a)Mucinouscarcinoma
b)Teratoma
c)Epidermalcystoidsadenoma
d)Papillarycarcinoma
CorrectAnswer-B
Ans.is'b'i.e.,Teratoma
Teratoma
oTeratomasaredividedintothreecategories:
(1)Mature(benign):
Mostbenignteratomasarecysticandareknownasdermoid
cysts.
Theseneoplasmsarepresumablyderivedfromthe
ectodermaldifferentiationoftotipotentialcells.
Theyarebilateralin10%to15%ofcases.
Characteristically,theyareunilocularcystscontaininghair
andcheesysebaceousmaterial.Onsection,theyrevealathinwall
linedbyanopaque,gray-white,wrinkled,apparentepidermis.
Generally,inoneareaofthecystwall,asolidprominenceis
seenknownasRokitansky'sprotuberancewheretissueelements
suchastooth,bone,cartilage&variousotheroddtissuesare
present.
Onhistologicexamination,thecystwalliscomposedofstratified
squamousepitheliumwithunderlying
sebaceousglands,hairshafts,andotherskinadnexalstructures.In
mostcases,structuresfromothergermlayerscanbeidentified,
suchascartilage,bone,thyroidtissue,andotherorganoid
formations.


Dermoidcystsaresometimesincorporatedwithinthewallof
amucinouscystadenoma.
About1%ofthedermoidsundergomalignanttransformation
ofanyoneofthecomponentelements(butmostcommonly,
squamouscellcarcinoma).
(2)MonodermalorSpecializedTeratomas
Theraregroupoftumors,themostcommonofwhichare
strumaovariiandcarcinoid.
Theyarealwaysunilateral,althoughacontralateralteratoma
maybepresent.
Strumaovariiiscomposedentirelyofmaturethyroidtissue.
Interestingly,thesethyroidalneoplasmsmayhyperfunction,causing
hyperthyroidism.
Theovariancarcinoid,whichpresumablyarisesfrom
intestinalepitheliuminateratoma,mightinfactbefunctioning,
particularlyinlargetumors,producing5-hydroxytryptamineandthe
carcinoidsyndrome.
(3)ImmatureMalignantTeratomas
Theseareraretumorsthatdifferfrombenignteratomasin
thatthecomponenttissueresemblesthatobservedinthefetusor
embryoratherthanintheadult.
Thetumorisfoundchieflyinprepubertaladolescentsand
youngwomen.
Thesegrowrapidlyandfrequentlypenetratethecapsulewith
localspreadormetastases.
Onmicroscopy,therearevaryingamountsofimmaturetissue
differentiatingtowardcartilage,glands,bone,muscle,nerve,and
others.
Animportantriskforsubsequentextraovarianspreadisthe
histologicgradeoftumor,whichisbasedontheproportionoftissue
containingimmatureneuroepithelium.

507.Whichisnotariskfactorofendometrial
carcinoma-
a)Obesity
b)Smoking
c)Infertility
d)Tamoxifen
CorrectAnswer-B
Ans.is'b'i.e.,Smoking
Endometrialcarcinoma
Endometrialcarcinomaisthemostcommoninvasivecancerofthe
femalegenitaltractandaccountsfor7%ofallinvasivecancerin
women.
Thepeakincidenceisinthe55-to65-year-oldwoman.
Clinicopathologicalstudies&molecularanalysissupportits
classificationintotwomajorbroadcategories.
Type-ICarcinoma
Mostcommontype.
Majorityarewelldifferentiated&mimicproliferativeendometetrial
glands.
Theyareassociatedwith-obesity,diabetes,hypertension,infertility
andunopposedestrogenstimulation.Tamoxifenalsoincreasesthe
riskofendometrialcancer.
Endometrialhyperplasiaisaprecursortoendometroidcarcinoma.
MutationinPTENtumorsuppressorgenehavebeenseenin30-
80%ofendometeriodcarcinoma&20%patientswithendometrial
hyperplasia.
Additionalmolecularchangesthatarecommonaremicrosatellite
instability&mutationsinKRAS&betacateninoncogenes.

Type-HCarcinoma
TheseoccurinwomenadecadelaterthantypeIcarcinoma.
Theseusuallyariseinsettingofendometrialatropy.
Theyarepoorlydifferentiated.Themostcommonsubtypeisserous
carcinoma,clearcelltype&malignantmixedmulleriantumor.
Themostfrequentalterationdescribedismutationinp53tumor
suppressorgene.

508.Whichofthefollowinggermcelltumoris
malignant?
a)Leydigcelltumor
b)Sertolicelltumor
c)Seminoma
d)Dermoidcyst
CorrectAnswer-C
Ans.is'c'i.e.,Seminoma
o
Amongthegivenoptionsonlyoptionsc&daregermcelltumors.
Seminomaismalignantwhiledermoidcystisbenign.
Germcelltumors
oMalignant-Germinoma(seminoma,dysgerminoma),embryonal
carcinoma,endodermalsinustumor(yolksactumor),
choriocarcinoma,immatureteratoma.
Benign-Matureteratoma,dermoidcyst.

509.ERpositivestatusinCaBreastindicates
-
a)Prognosis
b)Etiology
c)Site
d)None
CorrectAnswer-A
Ans.is'a'i.e.,Prognosis
oEstrogenreceptor(ER)positivebreastcarcinomasaregenerally
slowgrowingandrespondwelltohormonaltreatment.
oGeneexpressionprolifing,whichcanmeasuretherelative
quantitiesofmRNAforessentiallyeverygene,hasidentifiedfive
majorpatternsofgeneexpressionininvasiveductalcarcinoma.
LuminalA
oThisisthelargestgroup(40-55%)whichhascharacteristicsof
normalluminalcells.
oThistypeisERpositiveandHER2/neunegative.
oThesecancersaregenerallyslowgrowingandrespondto
hormonaltreatment.Converselyonlyasmallnumberwillrespondto
standardchemotherapy
LuminalB
oThesetumorsareERpositive,hasahigherproliferativerateand
overexpressesHER2/neu--->Triple-positivecancer.
oTheycomposeamajorgroupofER-positivecancersthataremore
likelytohavelymphnodemetastasesandmayrespondtostandard
chemotherapy.
Normalbreastline
oTheseareERpositive,HER2/neunegativeandcharacterizedby

thesimilarityoftheirgeneexpressionpatterntonormaltissue.
Basalline
oTheseareERnegative,PRnegative,HER2/neunegativeTriple
negative.
oThesearecharacterizedbyexpressionofmarkerstypicalof
myoepithelialcells(e.g.,basalkeratins,Pcodherins,p63,orlaminin),
progenitorcells,orputativestemcells(e.g.cytokeratins5and6).
o
ManycarcinomaarisinginwomenwithBRACImutationsareofthis
type.
oTheseareaggressivetumors,frequentmetastasistovisceraand
braincanbeseen--->havepoorprognosis.
HER2positive
oTheseareERnegativeandoverexpressHER2/neuprotein.

510.Proliferatingbreastmassis-
a)Ductectasia
b)Fibroadenoma
c)Adenosis
d)Papilloma
CorrectAnswer-D
Ans.is'd'i.e.,Papilloma
Examplesincludeatypicalductalhyperplasia,atypicallobular
hyperplasia,andintraductalpapillomas.

511.Tadpolecellscommashapedcellson
histopathologyareseenin-
a)Trichoepithelioma
b)Spideroma
c)Rhabdomyosarcoma
d)Histiocytoma
CorrectAnswer-C
Ans.is'c'i.e.,Rhabdomyosarcoma
Histologyofrhabdomvosarcoma
Thediagnosticcellofrhabdomyosarcomaisrhabdomyoblast.
Rhabdomyoblastcontainseccentriceosinophilicgranularcytoplasm
richinthickandthinfilaments.oTherhabdomyoblastsmaybe?
i)Round
ElongatedTadpoleorStrapcells.
Ithasthreevariants
ii)Embryonal
Alveolar
iii)Pleomorphic
Rhabdomyoblastsarepositivefordesmin,MYOD1andMyogenin.

512.Glomustumorisseenin-
a)Retroperitoneum
b)Softtissue
c)Distalportionofdigits
d)Proximalportionofdigits
CorrectAnswer-C
Ans.is'c'i.e.,Distalportionofdigits
oItisabenignbutpainfultumorthatarisesfromthemodified
smoothmusclecellsoftheglomusbody,aspecializedarteriovenous
anastomosis
thatisinvolvedinthermoregulation.
oTheyaremostcommonlyfoundinthedistalportionofthedigits,
especiallyunderthefingernails.

513.Ewingssarcomaarisesfrom-
a)Gcells
b)Totipotentcells
c)Neuroectodermalcells
d)Neurons
CorrectAnswer-C
Ans.isci.e.,Neuroectodermalcells
oHistologically,Ewing'ssarcomahasacertainresemblanceto
primitiveneuroectodermalcells,andthetumorarisesfromsuch
cells.
oItisararemalignancyprimarilyaffectingchildrenandadolescents.
Itarisesmainlyinboneandlesscommonlyinsofttissues.
oInthese,thereisacharacteristicreciprocalchromosomal
translocation,whichleadstoanin-framefusionbetweentheEWS
geneandoneoftheETSfamilygenemembers.

514.Medulloblastomamostcommon
metastasisisto?
a)Lung
b)CNS
c)Liver
d)Spleen
CorrectAnswer-B
Ans.is'b'i.e.,CNS
oMedullablastomaspreadthroughCSFandfrequentlymetastasizes
todifferentlocationsinthebrainandspine.
oExtraneuralmetastasistotherestofthebodyisrare.

515.Mutationinmalignantmelanoma-
a)N-myc
b)CDK2A
c)RET
d)None
CorrectAnswer-B
Ans.is'b'i.e.,CDK2A
"Inmelanomacells,numbersofmutationsand/ordysregulated
expressionofB-RofN-Ras,CDK2A,MDM2,PTEN,p53havebeen
recognized".


516.Perivascularlymphocytes&microglial
nodulesareseenin-
a)Multiplesclerosis
b)HIVencephalitis
c)CMVmeningitis
d)Bacterialmeningitis
CorrectAnswer-B
Ans.is'b'i.e.,HIVencephalitis
oPerivascularinfiltrateoflymphocytes(andmacrophages)with
microglialnodulesisseeninHIVencephalitis.
oPerivascularinfiltrateoflymphocytes(andmonocytes)isalsoseen
inmultiplesclerosis.But,microglialnodules
arecharacteristicofHIVencephalitis.

517.Glomuscellsarefoundin-
a)Bladder
b)Brain
c)Chemoreceptors
d)Kidney
CorrectAnswer-C
Ans.is'c'i.e.,Chemoreceptors
Arterialchemoreceptorsconsistofglobularaggregationsof
chemoreceptivecells(glomuscells),andsupportivecells,separated
fromoneanotherbyfibroustissuesepta.
Inthesesetpaandbetweenglomuscells,numerouscapillariesand
nervefibersareseen.
Theglomuscellshavethestructureofendocrineaminehormone
secretingcells.

518.Commonesttumoroffaceinchidrenis-
a)Rhabdomyosarcoma
b)Sq.cellcarcinoma
c)Basalcellcarcinoma
d)Mixedparotidtumor
CorrectAnswer-A
Ans.is'a'i.e.,Rhabdomyosarcoma
Rhabdomyosarcoma
oThemostcommonsofttissuesarcomainchildrenis
rhabdomyosarcoma.
oThecommonsiteofinvolvementare:-
oHead&Neck(25%)2"dmostcommonExtremities
Genitourinary(22%)-->2ndmostcommon
Retroperitoneum

519.Mostcommonsiteformedulloblastoma
is-
a)Cerebellum
b)Pituitary
c)Cerebrum
d)Pinealgland
CorrectAnswer-A
Ans.is'a'i.e.,Cerebellum
oMedulloblastomaisaninfratentorialtumorandoccursexclusively
incerebellum.

520.TypeBadversedrugreaction-
a)Augmentedeffectofdrug
b)UnpredictableBizzarereaction
c)Effectseenonchronicuseofdrug
d)Delayedeffectofdrug
CorrectAnswer-B
Ans.is'b'i.e.,UnpredictableBizzarereaction
Typesofadversedrugreaction
1. TypesA(Augmented)reaction.
2. TypesB(Bizzare)reaction
3. TypesC(Chronic)reaction
4. TypesD(Delayed)effects
5. TypesE(Endingofuse)reaction

521.Dopamineallofthefollowingistrue
Except?
a)CausesincreaseinGIIsehemia
b)Positiveionotropic
c)Improvesrenalperfusion
d)CausesVasoconstriction
CorrectAnswer-A
Ans.is'a'i.e.,CausesincreaseinGIIschemia
oDopamineactsondopamine(Di&D2)andadrenergic(a1+a2+
[31)receptors,withnoactiononp,receptors.
oAtlowesteffectivedoseitstimulatesD1receptorsinrenal&
mesentricbloodvessels-->improvesrenalandmesentric
perfusion.
oAtmoderatelyhighdosesitactsas13,agonist-->positive
ionotropic.
oAthighdosesitactivatesa-adrenoreceptors-->vasoconstriction.

522.Allarealpha-blockerexcept?
a)Atenolol
b)Prazosin
c)Indoramine
d)Idazoxan
CorrectAnswer-A
Ans.is'a'i.e.,Atenolol

523.Whichofthefollowingpotassium
sparingdiureticalterscardiacmortality-
a)Spironolactone
b)Amiloride
c)Triamterene
d)Epleronone
CorrectAnswer-A
Ans.is'a'i.e.,Spironolactone
oAmongpotassiumsparingdiuretics,aldosteroneantagonists
(Spironolactone,eplerenone)reducemortalityinCHF.

524.Antiandrogenusedinheartfailure?
a)Carvedilol
b)Sampatrilat
c)Spironolactone
d)Abiraterone
CorrectAnswer-C
Ans.is'c'i.e.,Spironolactone
oSpironolactoneandeplerenonearethealdosteroneantagonists.
Theyareusedaspotassiumsparingdiuretics.Theirdiurecticeffect
isquitefeeble,butinCHFthesedrugsreducethemortality(at
doseslowerthandiureticdoses)byantagonizingtheeffectof
aldosterone(reversalofremodelling).Spironolactonealso
possessesanti-androgeniceffects.

525.Fosphenytoindifferentfromphenytoinin
whichofthefollowing-
a)Canbeusedinabsenceseizures
b)Canbemixedwithsaline
c)Canbegivenorally
d)Itisthedrugofchoiceformyoclonicseizures
CorrectAnswer-B
Ans.'b'i.e.,Canbemixedwithsaline
Whilephenytoincannotbeinjectedinadripofglucosesolution,
fosphenytoincanbeinjectedwithsalineandglucose.

526.Iodinecontentinamiodarone-
a)10-20%
b)20-40%
c)40-60%
d)60-80%
CorrectAnswer-B
Ans.is'b'i.e.,20-40%

527.Whichofthefollowingcausesincreased
renninonprolongeduse?
a)Clonidine
b)Enalapril
c)Methyldopa
d)Blocker
CorrectAnswer-B
Ans.is'bi.e.,Enalapril

528.Centrallyactingantihypertensivedrugis
?
a)Phenoxybenzamine
b)Methyldopa
c)Propanolol
d)Prazosin
CorrectAnswer-B
Ans.is'b'i.e.,Methyldopa
oCentrallyactingsympatholyticantihypertensivesareclonidine,
methyldopa,guanabenz,guanfacine,moxonidineandrilmenidine.

529.TrueregardingConivaptanis-
a)VasopressinAntagonist
b)V2selectiveaction
c)Givenorally
d)All
CorrectAnswer-A
Ans.is'a'i.e.,VasopressinAntagonist

530.Guanethidineisusedinthetreatmentofwhichofthefollowingcondition?
a)Thyrotoxicophthalmopathy
b)Ptosis
c)Bell'spalsy
d)Horner'ssyndrome
CorrectAnswer-A
Guanethidine(10%)eyedropsisusefulindecreasinglidretractioninThyrotoxicophthalmopathy.
Itisanadrenergicneuronblockerwhichactsbyinhibitingthereleaseofnoradrenalineinresponseto
nervestimulation.

531.5HT1agonistsusedas?(
a)Antianxietydrugs
b)Antipsychoticdrugs
c)GERD
d)Chemotherapyinducedvomiting
CorrectAnswer-A
Ans.is'a'i.e.,Antianxietydrugs
5-HTIAagonists(Buspiron,ipsapirone)actasantianxietydrugs.

532.Mechanismofactionoftheophyllinein
Bronchialasthmaincludeallofthe
followingExcept?

a)Phosphodiesteraseinhibition
b)Adenosinereceptorantagonism
c)Increasedhistonedeacetylation
d)Beta-2receptorstimulation
CorrectAnswer-D
Ans.is'd'i.e.,Beta-2receptorstimulation
Proposedmechanismsofactionoftheophylline
Phosphodiesteraseinhibition(Nonselective)
Adenosinereceptorantagonism(A1,A2)
IncreasedhistoneDeacetylaseactivity(redefficacyof
corticosteroids)
Inhibitionofintracellularcalciumrelease
Stimulationofcatecholaminerelease
InhibitionofNF-alphaBetatranslocationintothenucleus(nuclear
translocation)
Mediatorinhibition(Prostaglandins,TNF
alpha)

533.Omalizumabisusedintreatmentof:
a)Breastcarcinoma
b)Asthma
c)Rheumatoidarthritis
d)Noneoftheabove
CorrectAnswer-B
OmalizumabisablockingantibodythatneutralizescirculatingIgEwithoutbindingtocell-
boundIgE;itthusinhibitsIgE-mediatedreactions.
Thistreatmenthasbeenshowntoreducethenumberofexacerbationsinpatientswith
severeasthmaandmayimproveasthmacontrol.However,thetreatmentisvery
expensiveandonlysuitableforhighlyselectedpatientswhoarenotcontrolledonmaximal
dosesofinhalertherapyandhaveacirculatingIgEwithinaspecifiedrange.
Ref:Harrison'sprincipleofinternalmedicine17thedition,chapter248.

534.Efficacyofsalmeterolisincreasedifitis
givenalongwith-
a)Theophylline
b)Corticosteroid
c)Ipratropium
d)Sodiumcromoglycate
CorrectAnswer-B
Ans.is'b'i.e.,Corticosteroid
oConcurrentuseofinhaledsalmeterolwithinhaledglucocorticoid
produceseffectsequivalenttodoubledoseofthecorticoidalone.

535.Whichdrugdoesn'tincludeDMARD:
a)Chloroquine
b)Vincristine
c)Azathioprine
d)Leflunomide
CorrectAnswer-B
Ans.is'b'i.e.,Vincristine
oDiseasemodifyingantirheumaticdrugs(DMARDs):?
1. ImmunosuppresantsMethotrexate,azathioprine,cyclosporine
2. Sulfasalazine
3. Choloroquineorhydroxychloroquine
4. Leflunomide
5. Goldsod.thiomalate,Auranofin
6. d-Penicillamine

536.Whichofthefollowingdrugsactdirectly
withoutsexualstimulation?
a)Sildenafil
b)Tadalafil
c)Alprostadil
d)Testosterone
CorrectAnswer-C
Ans.C.Alprostadil
Alprastadil(pGEl)isdirectlyinjectedintocorporacavernusafor
erectiledysfunction'
Itactsbyincreasingarterialinflow"byvasodilationandreducing
outflowbycontractingthecarporalsmoothmusclethatoccludes
drainingvenules.

537.PGE2causeallexcept-
a)Waterretention
b)Uterinecontraction
c)Flushing
d)None
CorrectAnswer-A
Ans.is'a'i.e.,Waterretention
oPGE2causesincreaseinwaterexcretionbyinhibitingADHaction.

538.Dinoprostis-
a)PGEl
b)PGE2
c)PGF2alpha
d)PGI2
CorrectAnswer-C
Ans.is'c'i.e.,PGF2alpha
oDinoprost-PGF2alpha,intraamnioticallyformidtermabortion.
oDinoprostone-PGE2,intravaginallyformidtermabortion.

539.Whichenzymeisirreversiblyinhibitedby
aspirin?
a)Lipooxygenase
b)Cyclooxygenase
c)Thromboxanesynthase
d)Phospholipase
CorrectAnswer-B
Ans.is'b'i.e.,Cyclooxygenase

540.Indiabetesinsipidus,diureticshowing
paradoxicalantidureticactivity-
a)Thiazide
b)Triamterene
c)Spironolactone
d)Furosemide
CorrectAnswer-A
Ans.is'a'i.e.,Thiazide
ThiazidediureticsparadoxicallydecreaseurineoutputinDIdueto
formationofcAMPindistaltubuleseffectiveinbothcentraland
nephrogenicDI.

541.Sideeffectofthiazidediureticsareall
except?
a)Hyponatremia
b)Hypokalemia
c)Erectiledysfunction
d)Hypocalcemia
CorrectAnswer-A
Ans.is'A'i.e.,Hypocalcemia
Thiazidescausehypercalcemia(seeaboveexplanation).

542.Desmopressinispreferredover
vasopressinbecausedesmopressin-
a)Morepotent
b)MoreselectiveforVireceptor
c)Haslittlevasconstrictoractivity
d)aandc
CorrectAnswer-D
Ans.is'a'i.e.,Morepotent;'c'i.e.,Haslittlevasconstrictoractivity
oDesmopressinislongeracting
oDesmopressinisV2selective--)NoV,mediated
vasoconstriction.
oDesmopressinis12timesmorepotentthanvasopressin.


543.Whichofthefollowingisaselective
serotonin&norepinephrinereuptake
inhibitor?

a)Fluoxetine
b)Venlafaxine
c)Sertaline
d)Arnoxipine
CorrectAnswer-B
Ans.is'b'i.e.,Venlafaxine
oImportantSNRIsare:-Venalafaxine,Milnacipram,
desvenalafaxine,Duloxetine.

544.Allareneuralplateinducersexcept
a)Notochordappearance
b)HighBMP
c)FGFupregulation
d)Prechordalmesoderm
CorrectAnswer-B
Bi.e.HighBMP

545.Toxicdoseoflithium-
a)0.6
b)12
c)2.6
d)<0.6
CorrectAnswer-C
Ans.is'c'i.e.,2.6

546.Extrapyramidalsyndromelikeside
effectsareseenin-
a)Haloperidol
b)Clozapine
c)Tetracycline
d)Ketoconazole
CorrectAnswer-A
Ans.is'a'i.e.,Haloperidol
Drugscausingextrapvramidaleffects
oButyrophenones(Haloperidol)oMethyldopao
OCP'SoReserpine
oLevodopaoMetoclopromideo
PhenothiazinesoTricyclicAntidepressants

547.TrilenewhenusedwithSodalimecauses
?
a)Renaldamage
b)ARDS
c)Myocardialdepression
d)Hepatitis
CorrectAnswer-B
Ans.is'b'i.e.,ARDS
oFollowingagentsreactwithsodalime:
SevofluraneisdegradedbycontactwithCO2absorbant(sodalime)
inanaesthesiamachine,yieldingavinylethercalledCompoundA
whichcancauserenaldamage.
Trileneproducesphosgene(causingARDS)anddicholoroacetylene
(causesneurotoxicity)whenusedwithsodalime.

548.Cardiotoxicityofbupivacaine-
a)Depressedpacemakeractivity
b)Toxiccompounddamagingmyocardialcells
c)Depressedneuralcontrolonheart
d)VascularthrombosisandMyocardialischemia
CorrectAnswer-A
Ans.is'a'i.e.,Depressedpacemakeractivity
oLocalanaestheticsblockcardiacsodiumchannelsandthus
depressabnormalcardiacpacemakeractivity,excitability,and
conduction.Atextremelyhighconcentrations,localanaestheticscan
alsoblockcalciumchannels.

549.Shortactingnondelpolarizingblocker?
a)Rocurorium
b)Suxamethonium
c)Mivacurium
d)Pancuronium
CorrectAnswer-C
Ans.is'c'i.e.,Mivacurium

550.Whichisanintermediateactinginsulin?
a)Insulinlispro
b)Regularinsuin
c)NPHinsulin
d)Insulinglargine
CorrectAnswer-C
Ans.is'c'i.e.,NPHinsulin

551.Steroidingestedforlongtimeleadstoall
ofthefollowingexcept-
a)Avascularnecrosisofheadoffemur
b)Cataract
c)Glaucoma
d)Growthretardation
CorrectAnswer-C
Ans.is'c'i.e.,Glaucoma
Glaucomaoccursaftertopicaltherapy(notsystemic)

552.Leastglucocorticoidactionisseenwith
?
a)Fludrocortisone
b)Cortisone
c)Dexamethasone
d)Betamethasone
CorrectAnswer-B
Ans.is'b'i.e.,Cortisone
Leastpotentglucocorticoid-->Cortisone

553.Steroidwithmaxmineralocorticoid
activity?
a)Fludrocortisone
b)DOCA
c)Prednisolone
d)Triamsinolone
CorrectAnswer-A
Ans.is'a'i.e,Fludrocortisone

554.Intakeofexogenoussteroidcauses:
a)Addison'sdisease
b)Cushing'ssyndrome
c)Pheochromocytoma
d)Conn'ssyndrome
CorrectAnswer-B
AnswerisB(Cushing'ssyndrome):
ThemostcommoncauseofCushing'ssyndromeisiatragenic
administrationofsteroidsforavarietyofreasons.-Harrison


555.Whichofthefollowingisaselective
progesteronereceptormodulator-
a)Onapristone
b)Ulipristal
c)Nomegestrol
d)Toremifene
CorrectAnswer-B
Ans.is'b'i.e.,Ulipristal
oUlipristalisaSPRMapprovedforuseasanEmergency
Contraceptive.
oSPRM(selectiveprogesteronereceptormodulators):Asoprisnil,ul
ipristal,onapristone,mifepristone.

556.Tiboloneisa?
a)Naturalsteroidalestrogen
b)Naturalnon-steroidalestrogen
c)Syntheticsteroidalestrogen
d)Syntheticnon-steroidalestrogen
CorrectAnswer-C
Ans.C.Syntheticsteroidalestrogen
[RefKDT7Vep.306,311]
Syntheticestrogens
SteroidalEthinylestradiol,mestranol,tibolone.
NonsteroidalDiethylstilbestrol,hexestrol,dienestrol.

557.Finasterideisa:
a)5alphareductaseinhibitor
b)PDEinhibitor
c)Alphalablocker
d)Androgenreceptorblocker
CorrectAnswer-A
Finasterideisacompetitiveinhibitoroftheenzyme5-alphareductasewhichisresponsible
fortheconversionoftestosteroneintoamoreactivedihydrotestosteroneresponsibleforthe
androgenaction.
Whenusedinbenignprostatichypertrophy,itreducestheprostatesizeandincreasedpeak
urinaryflowrate.
Itisalsousedinmalepatternbaldnessandasapalliativetreatmentinprostaticcarcinoma.
Ref:KDTripathiTextbookofPharmacology,5thEdition,Page272

558.Whichofthefollowingisasynthetic
estrogen?
a)Estrone
b)Estriol
c)Estradiol
d)Diethylstibestrol
CorrectAnswer-D
Ans.is'd'i.e.,Diethylstibestrol
Syntheticprogesterones
1.Progesteronederivatives-->Medroxyprogesterone,Megestrol,
Dydrogesterone,Hydroxyprogesterone,Nomegestrol.
2.19-Nortestosteronederivatives-->Norethindrone,Lynesternol,
Allylesterone,Levonorgestrel,Desogestrel,Norgestimate,
Gestodene.
Syntheticestrogens
1. Steroidal---->Ethinylestradiol,mestranol,tibolone.
2. Nonsteroidal---->Diethylstilbestrol,hexestrol,dienestrol

559.Sideeffectofoxytocinisallexcept?
a)Placentalabruption
b)Fetaldistress
c)Peripheralvasculardisease
d)Waterintoxication
CorrectAnswer-C
Ans.is'c'i.e.,Peripheralvasculardisease
oSideeffectsofOxytocinaredueto:
i)ADHlikeaction-Waterintoxication
ii)Excessiveuterinecontractions,priortolabour-Fetaldistress,
Placentalabruption,Uterinerupture.

560.WhichisnotaS/EofCimetidine?
a)Impotence
b)Gynaecomastia
c)Atrophicgastritis
d)Galactorrhea
CorrectAnswer-C
Ans.is'c'i.e.,Atrophicgastritis

561.Antilipidemicdrugsthatprevent
hypercholesterolemiabyinhibiting
absorption-

a)Ezetimibe
b)Orlistat
c)Cholestyramine
d)Statins
CorrectAnswer-A
Ans.is'a'i.e.,Ezetimibe
Ezetimibeinhibitstheabsorptionofcholesterolbybidningto
transporter[NPC-1L1(NiemannPickCl,likel)SRBl,145KDa]
locatedinintestinalbrushborder.
Note:
Orlistatalsoreducescholesterolabsorption,butitisananti-obesity
drug(notanantilipidemicdrug).

562.Antifungalwhichcanbeusedorallybut
notivis?
a)Voriconazole
b)AmphoterecinB
c)Terbinafine
d)Noneoftheabove
CorrectAnswer-C
Ans.is'c'i.e.,Terbinafine

563.Mostpotentstatin-
a)Simvastatin
b)Pravastatin
c)Rosuvastatin
d)Simvastatin
CorrectAnswer-C
Ans.is'c'i.e.,Rosuvastatin
oTwomostpotentstatinsare
Pitavastatin(mostpotent)androsuvastatinmostpotent).

564.An86yearsoldladypresentedwith
severeconstipation.Shewasaknown
hypertensiveonmedicationsfor10
years.Inclinic,herBPwas157/98mm
Hgwithaheartrateof58/min.Ontaking
hereBPinthesupinepositionitwas
foundtobe90/60mmHg.Shehadthe
recenthistoryofdepression.Sheis
takingatenolol,thiazide,imipramine,
haloperidolanddocusate.Whatwillbe
thenextbeststepinthemanagement?

a)Changeatenololandthiazidetocalciumchannelblockerand
ACEinhibitorandaddbisacodylforconstipation
b)Changeimipramineandhaloperidoltofluoxetineand
risperidoneandaddbisacodylforconstipation
c)Onlyaddbisacodylforconstipationandcontinuerestofthe
medications
d)Discontinueallhermedicationsandstartheronsteroids
CorrectAnswer-B
Ans:B.Changeimipramineandhaloperidoltofluoxetineand
risperidoneandaddbisacodylforconstipation
(RefHarrison19/ep1623-1624,18/ep3531:GoodmanGilman12/e
p410.1333)
EffectsofImipramine:

Posturalhypotension-DuetoalphablockadebyImipramine&
thiazidesinteraction.
Anti-cholinergicside-effect.
Hence,Imipramine(TCA)mustbediscontinued.
ShouldbestartedonSSRI,fluoxetin&laxative(existing
constipation).
Effectofhaloperidol:
Anti-cholinergicsideeffects.
ShouldstartonatypicalantipsychoticRisperidone.

565.ProphylacticdoseofvitaminKgivento
newborninfantsatdeliveryis?
a)1mg
b)5mg
c)10mg
d)15mg
CorrectAnswer-A
Ans.is'a'i.e.,1mg
VitaminKDeficiencyinNewborns
ThesymptomsofvitaminKdeficiencyareduetohemorrhage
NewbornsareparticularlysusceptibletovitaminKdeficiency
becauseoflowfatstores,lowbreastmilklevelsofvitaminK,sterility
oftheinfantileintestinaltract,liverimmaturity,andpoorplacental
transport.
Intracranialbleeding,aswellasgastrointestinalandskinbleeding,
canoccurinvitaminK-deficientinfants17daysafterbirth.
Thus,vitaminK(1mgIM)isgivenprophylacticallyatthetimeof
delivery.

566.Lowmolecularweightheparinmainly
inhibitswhichfactor:
September2009

a)FactorIlia
b)FactorVIIIa
c)FactorXa
d)FactorXIIa
CorrectAnswer-C
Ans.C:FactorXa

567.Allofthefollowinghaveinteractionwith
warfarinexcept-
a)Barbiturate
b)Oralcontraceptive
c)Cephalosporins
d)Benzodiazepens
CorrectAnswer-D
Ans.is'd'i.e.,Benzodiazepenes

568.Clopidogrelmechanismofaction-
a)ThromboxaneA2inhibition
b)InhibitADPmediatedcAMPactivation
c)GPIIb/Inainhibitors
d)None
CorrectAnswer-B
Ans.is'b'i.e.,InhibitADPmediatedcAMPactivation

569.Romiplostimactsonwhichofthefollowingreceptors:
a)Thrombopoeitin
b)IL6
c)IL8
d)PGE1
CorrectAnswer-A
Romiplostim:GeneticallyengineeredproteininwhichtheFccomponentofahuman
antibodyisfusedtotwocopiesofapeptidethatstimulatesthethrombopoietinreceptors;
approvedfortreatmentofidiopathicthrombocytopenicpurpura
Ref:Katzung11theditionChapter33.

570.Mechanismofactionofquinolonesis?
a)DNAgyraseinhibitors
b)Bindto30sunit
c)Bindtobacterialcellmembrane
d)Bindtotetrahydofolatereductase
CorrectAnswer-D
DNAgyraseinhibitorsREF:GoodmanGillman's11thedp.722
ThequinoloneantibioticstargetbacterialDNAgyraseand
topoisomeraseIV.Formanygrampositivebacteria,topoisomerase
IVistheprimarytarget.Formanygram-negativebacteria,DNA
gyraseistheprimaryquinolonetarget.

571.Druginhibitingbacterialprotein
synthesisareallexcept-
a)Aminoglycosides
b)Chloramphenicol
c)Clindamycin
d)Sulfonamides
CorrectAnswer-D
Ans.is`d'i.e.,Sulfonamides
oSulfonamidesaffectintermediarymetabolismbyinhibitingfolate
synthase.

572.Longestactingsulphonamideis-
a)Sulfadiazine
b)Sulphadoxine
c)Sulfamethoxazole
d)Sulfamethiazole
CorrectAnswer-B
Ans.is'b'i.e.,Sulfadoxine
Sulfadoxineandsulfamethopyrazinearelongactingsulfonamides.

573.Widestspectrumaminoglycosideis-
a)Streptomycin
b)Amikacin
c)Framycetin
d)Netilmicin
CorrectAnswer-B
Ans.is'b'i.e.,Amikacin

574.Doseofoseltamivirinachildaged9monthsis:
a)2mg/kgtwicedailyfor5days
b)2.5mg/kgtwicedailyfor5days
c)3mg/kgtwicedailyfor5days
d)3.5mg/kgtwicedailyfor5days
CorrectAnswer-C
Agegroups
Recommendeddoseofoseltamivir
0to1month
2mg/kgtwicedailyfor5days
>1monthto3months
2.5mg/kgtwicedailyfor5days
>3monthsto12months
3mg/kgtwicedailyfor5days
Ref:Park21stedition,page148
Chapter:Epidemiologyincommunicablediseases.

575.IndicationofAcyclovirinpregnancy:
a)Disseminatedherpes
b)Chicken-poxinfirsttrimester
c)Prophylaxisinrecurrentherpes
d)Alloftheabove
CorrectAnswer-D
Alloftheabove

576.Whichofthefollowingantimalarialisa
slowactingschizonticide-
a)Artemether
b)Mefloquine
c)Pyrimethamine
d)Quinine
CorrectAnswer-C
Ans.is'c'i.e.,Pyrimethamine
oAntimalarialsthatactonerythrocyticphaseofschizogonyare
callederythrocyticschizontocides.Theavailabledrugscanbe
dividedinto?
1.Fastacting-Chloroquine,amodiaquine,quinine,mefloquine,
halofantrine,lumefantrine,atovaquone,artemisinin.
2.Slowacting-Pyrimethamine,Proguanil,sulfonamides,
tetracyclines.

577.Redmansyndromeisdueto-
a)Vancomycin
b)Polymyxin
c)Rifampicin
d)Teicoplanin
CorrectAnswer-A
Ans.is'a'i.e.,Vancomycin
Vancomycincancauseredmensyndrome.

578.Bleedingisseenwiththeuseof-
a)Cefaloridine
b)Cefazolin
c)Moxalactum
d)Ceftazidime
CorrectAnswer-C
Ans.is'c'i.e.,Moxalactam
oCeftriaxone,cefoperazone,moxalactam&cefamandolecancause
hypoprothrmbinemiaandbleeding.

579.Sulphonamideinjectioncausesdecrease
infolicacidby?
a)Competitiveinhibition
b)Noncompetitiveinhibition
c)Uncompetitiveinhibition
d)Allostericinhibition
CorrectAnswer-A
Ans.is'a'i.e.,Competitiveinhibition
Bacteriasynthesizetheirownfolicacidofwhichparaaminobenzoic
acid(PABA)isaconstituent-Sulfonamides,beingstructural
analoguesofPABA,
inhibitbacterialfolatesynthasecompetitively.

580.Allaretrueaboutciprofloxacinexcept?
a)C/Iinpregnancy
b)DNAinhibition
c)Mostpotent1stgenerationfluoroquinolone
d)MoreactiveatacidicpH
CorrectAnswer-D
Ans.is'd'i.e.,MoreactiveatacidicpH
CiprofloxacinisthemostpotentfirstgenerationFQ.
oCiprofloxacininhibitDNAgyraseandiscontraindicatedin
pregnancy.
ItislessactiveatacidicpH.

581.Whichofthefollowingisnotusedas
treatmentforlymphaticfilariasis-
a)Ivermectin
b)DEC
c)Praziquantel
d)Albendazole
CorrectAnswer-C
Ans.is'c'i.e.,Praziquantel
oDrugsusedinlymphaticfiliriasisareDEC,ivermectin,albendazale
anddoxycycline.

582.Aboutvincaalkaloidstrueis?
a)Inhibitsmitoticspindle
b)Enhancespolymerizationoftubulin
c)InhibitstopoisomeraseI
d)InhibitstopoisomeraseII
CorrectAnswer-A
Ans.is'a'i.e.,Inhibitsmitoticspindle
Vincaalkaloids(vincristine)inhibitmitoticspindlesbypreventing
polymerizationoftubulin

583.WhichofthefollowingAntineoplastic
drugsSHOULDNOTbegivenbyrapidIV
infusion?

a)Cyclophosphamide
b)Cisplatin
c)Bleomycin
d)Cytosinearabinoside
CorrectAnswer-B
Ans.is`b'i.e.,Cisplatin
oCisplatinisthemostcommonculpritcausingchemotherapy
inducednauseaandvomiting,thereforecisplatinisgivenasslowi.v.
infusion(neverbolus)topreventvomiting.

584.Modeofactionofazathioprine?
a)IL-2
b)T-cellblockade
c)Decreasedlymphophagocyticactivity
d)Wide-spreadantitumoractivity
CorrectAnswer-B
Ans.is'b'i.e.,T-cellblockade

585.ResistancetoMethotrexatedevelopsdue
to?
a)RapidCancercellmultiplication
b)Deficiencyofthymidylatekinase
c)Deficiencyofthymidylatesynthetase
d)Increasedproductionofdihydrofolatereductase
CorrectAnswer-D
Ans.is'd'i.e.,Increasedproductionofdihydrofolatereductase
Methotrexateresistance
oMethotrexateresistancemaybeduetoanyofthefollowing
mechanism:?
i)Defectivetransportintocells
ii)ProductionofalteredformofDHFRthathavedecreased
affinityformethotrexate
iii)IncreasedconcentrationsofintracellularDHFRthroughgene
amplificationoralteredgeneregulation
iv)Decreasedabilitytosynthesizemethotrexatepolyglutamates
v)Increasedexpressionofadrugeffluxtransporter,oftheMRP
(multidrugresistanceprotein)class


586.Whichofthefollowingpoisoning
presentswithabdominalpain,diarrhea,
Meeslineonnailsand
myelosuppression:
September2011

a)Lead
b)Arsenic
c)Alcohol
d)Mercury
CorrectAnswer-B
Ans.B:Arsenic
Inchronicarsenicpoisoning,aclassicalstateofillhealthresultsare
representedby4stages.Firststageofnutritionaland
gastrointestinaldisturbances(diarrhoeaiscommon)
Thirdstage(ofskinrashes)showswhitebandsknownasMeesline
crossingthenailoffingersandtoes
Inadditiontothis,theremaybeevidenceofliverdamage,kidney
damageandbonemarrowdepressionatsomestageArsenic
poisoning
Increasedlevelsoftheelementarsenicinthebody.
Arsenicinterfereswithcellularlongevitybyallostericinhibitionofan
essentialmetabolicenzyme.
Symptomsofarsenicpoisoningincludeheadache,confusion,
convulsion,diarrhea,vomiting,andinseverecasecomaanddeath
Routesofexposureincludecontaminatedwater,air,andfood.
Occupationalexposuretoarsenicmayoccurwithcopperorlead

smeltingandwoodtreatmentandamongworkersinvolvedinthe
productionorapplicationofpesticides
Symptomsofarsenicpoisoningbeginwithheadaches,confusion,
severediarrhea,anddrowsiness.
Asthepoisoningdevelops,convulsionsandchangesinfingernail
pigmentationcalledleukonychiamayoccur.
Whenthepoisoningbecomesacute,symptomsmayinclude
diarrhea,vomiting,bloodintheurine,crampingmuscles,hairloss,
stomachpain,andmoreconvulsions.
Theorgansofthebodythatareusuallyaffectedbyarsenic
poisoningarethelungs,skin,kidneys,andliver.
Chronicarsenicexposurecanremaininthebodysystemsfora
longerperiodoftimethanashortertermormoreisolatedexposure
andcanbedetectedinalongertimeframeaftertheintroductionof
thearsenic,importantintryingtodeterminethesourceofthe
exposure.
Hairisapotentialbioindicatorforarsenicexposureduetoitsability
tostoretraceelementsfromblood.
Incorporatedelementsmaintaintheirpositionduringgrowthofhair.
Thusforatemporalestimationofexposure,anassayofhair
compositionneedstobecarriedoutwithasinglehair
whichisnotpossiblewitholdertechniquesrequiringhomogenization
anddissolutionofseveralstrandsofhair.
Thistypeofbiomonitoringhasbeenachievedwithnewer
microanalyticaltechniqueslikeSynchrotonradiationbasedXray
fluorescence(SXRF)spectroscopyandMicroparticleinducedXray
emission(PIXE).
Dimercaprolanddimercaptosuccinicacidarechelatingagentswhich
sequesterthearsenicawayfrombloodproteinsandareusedin
treatingacutearsenicpoisoning.
Themostimportantsideeffectishypertension.
Dimercaprolisconsiderablymoretoxicthansuccimer

587.AllaretrueregardingSunitinibexcept-
a)Itinhibitstyrosinekinasereceptors
b)Itisusedforrenalcellcarcinoma
c)ItisusedforthetreatmentofGIST
d)Itisexcretedprimarilyinurine
CorrectAnswer-D
Ans.is'd'i.e.,Itisexcretedprimarilyinurine
SunitinibinhibitsmultipleTyrosinekinasereceptors.Itinhibits
PDGF,VEGFandc-kit.
oSunitinibandsorafinibareusedinrenalcellcarcinoma(inRCC
thereisoverexpressionofVEGF)andGIST(inGISTthereisover
expressionofC-Kit&PDGF).
oItiseliminatedprimarilybyhepaticroutewithexcretioninfaeces.

588.Whichofthefollowingdrugcauses
hirsutism?
a)Phenytoin
b)Valproate
c)Carbamazepine
d)Phenobarbitone
CorrectAnswer-A
Ans.is'a'i.e.,Phenytoin

589.Notaselectiveserotoninreuptakeinhibi-
tor
a)Fluoxitine
b)Fluoxamine
c)Buspirone
d)Citaloforamin
CorrectAnswer-C
Buspirone

590.Whichofthefollowingdrugcrosses
BBB?
a)Glycopyrrolate
b)Neostigmine
c)Physostigmine
d)Alloftheabove
CorrectAnswer-C
Ans.C.Physostigmine
[RefKDTp.07,117]
Physostigmine-RapidlyabsorbedfromGITandparenteralsites,
penetratescorneafreelyandcrossesBBB(bloodbrainbarrier).
Neostigmine-Itisaquarternaryammoniumcompoundwhichis
poorlyabsorbedorallywithpoorcornealpenetrationanddoesn't
crossBBB.
Glycopyrrolate-Itisapotentandrapidlyactinganti-muscarinic
lackingcentraleffectsandisusedasapre-anaestheticmedication.

591.Juvenilejusticeactdefinesajuvenile
whichis
a)Malebelow16years
b)Femalebelow16years
c)Malebelow18years
d)Noneoftheabove
CorrectAnswer-C
Ans.is'c'i.e.,Malebelow18years

592.CrPC26?
a)Divisionofoffence
b)Divisionofnegligence
c)Divisionofmalpractice
d)None
CorrectAnswer-A
Ans.is'a'i.e.,Divisionofoffence
AccordingtoSec26ofCrPC,1973,OffencesundertheCriminal
ProcedureCode(hereinafterthe"CrPC")aredividedinto:

1. OffencesunderIndianPenalCode(IPC)(triablebyHC,Sessions
Courtandothercourtshowninthe1stScheduletotheCrPC)
2. Offencesunderanyotherlaw(empowersHC,whennocourtis
mentionedforanyoffenceunderanylawotherthanIPC,totrysuch
offences)

593.

Whichsectionbelongstomaritalrape?
a)375A
b)376A
c)376B
d)375B
CorrectAnswer-B
Ans.is'b'i.e.,376A[RefReddy30"Vep.377]Maritalrape
Itisforcefulsexualintercoursewithwifewhoislivingseparately
fromhimunderadecreeofseparation,oranycustomorusage
withoutherconsent.
Itispunishablewithimprisonmentupto2yearsprovidedtheageof
wifeisnotbelow12years-Section376A,I.P.C.

594.Policeinquestisrequiredinallexcept:
b)Accidentaldeath
c)Dowrydeath
d)Murder
CorrectAnswer-C
Ans.C:Dowrydeath
Inpoliceinquest,sub-inspectororofficerinchargeofthepolice
stationconductstheinquestinsuicide,murder,accidentaldeathsor
deathundersuspiciouscircumstances.
Magistrateinquestisdonein:
Custodialdeath
Deathduetopolicefiring
Deathinprison
Deathinapsychiatrichospital
Dowrydeaths
Exhumation
Indowrydeaths,inquestshouldbecarriedoutbyamagistrateor
policeofficer,notbelowtherankofdeputysuperintendentofpolice.

595.Plantiffisapersonwho?
a)Filescaseincivilcourt
b)Actsasdefender
c)Givesjudgement
d)None
CorrectAnswer-A
Ans.is'a'i.e.,Filescaseincivilcourt[RefInternet]
Plaintiffisapersonwhofilesacaseincivilcourt.?

596.Incriminalcases,conductmoneyispaid
by:
NEET13

a)Court
b)Oppositeparty
c)Judge
d)Noconductmoneyisgiven
CorrectAnswer-D
Ans.Noconductmoneyisgiven

597.Oralevidenceismoreimportantthan
writtentestimonyas:
NEET13

a)Oralevidencecannotbecross-examined
b)Oralevidencecanbecross-examined
c)Documentaryevidencerequiresnoproof
d)None
CorrectAnswer-B
Ans.Oralevidencecanbecross-examined

598.Contributorynegligenceisnegligence
dueto:
NEET13

a)Doctoronly
b)Patientonly
c)Bothdoctorandpatient
d)Hospitaladministratoranddoctor
CorrectAnswer-C
Ans.Bothdoctorandpatient

599.Ina3monthfetus,characteristicfeature
seenis:
NEET13

a)Nailsarevisible
b)Limbswellformed
c)Anusisseenasdarkspot
d)Meconiumisfoundinduodenum
CorrectAnswer-A
Ans.Nailsarevisible

600.RuleofHasseisusedtodetermine:
a)Theageoffetus
b)Heightofanadult
c)Raceofaperson
d)Identification
CorrectAnswer-A
Ans.isai.e.Theageoffetus
Hasse'sruleisemployedincalculatingtheageoffetusbyits
length
Duringthefirstfivemonthsofpregnancy,thelengthincmsissquare
oftheageinmonthsi.e.Lengthincms=(Ageinmonths)?
Ageinmonths=Qlengthincms
Duringsecondfivemonthsofpregnancy,lengthincmsdividedby5
istheageinmonths.Lengthinems
Ageinmonths5

601.Cruralindexis:
a)Lengthoftibia/femurx100
b)Lengthofradius/humerusx100
c)Lengthoffibula/tibiax100
d)Lengthofradius/ulnax100
CorrectAnswer-A
Ans.Lengthoftibia/femurx100

602.Whichofthefollowingisnotseenin
fingerprints?
a)Loop
b)Circles
c)Whorl
d)Arch
CorrectAnswer-B
Ans.is'b'i.e.,Circles[RefReddy30`h/ep.75-76]
Classificationoffingerprints:
Loops(60-70%)-radial,ulnar
Whorls(25-35%)-concentric,spiral,doublespiral,almondshaped
Arches(6-7%)-plain,tented,exceptional
Composite(1-2%)-centralpocketloops,lateralpocketloops,
twinnedloops,accidentals
Mostcommontype-Loops.
Leastcommontype->Composite.

603.Methodofidentificationusinglips:
a)Dactylography
b)Poroscopy
c)Cheiloscopy
d)Tricology
CorrectAnswer-C
Ans.Cheiloscopy

604.Whatisfirstexternalsignof
decompositionofdeadbody-
a)Decompositionofliverandintestine
b)Greenishdiscolourationoverrightiliacfossa
c)Greenishdiscolourationoverdependentparts
d)Bloodstainedfrothfrommouth
CorrectAnswer-B
Ans.is'b'i.e.,Greenishdiscolourationoverrightiliacfossa
[RefReddy30th/ep.150-152]
Firstexternalsignofputrefaction-Greendiscolorationinrightiliac
fossa.
FirstinternalsignofputrefactionGreendiscolorationunderliver.
Theseareduetoinvolvementofcaecumwhichcontainsmoregas
andisfullofbacteria.

605.Invitrouswhatismeasuredfortime
sincedeath-
a)Sodium
b)Potassium
c)Proteins
d)Chloride
CorrectAnswer-B
Ans.is'b'i.e.,Potassium[Ref:Reddy30th/ep.139]
Postmortemchangesineyeare:
Lossofcornealreflex
Opacityofcornea-corneabecomesopaquein2to4hours.
Flaccidityofeyeball-withinminutesafterdeathduetodecreased
intraoculartension.
Pupilsaredilatedwithinoneminuteofdeath,pupilsreacttoatropine
foraboutonehour.
Retina-fragmentationofbloodcolumninretinalvesselsoccurwithin
minutestoonehour.
Chemicalchange-asteadyriseinthepotassiumvaluesoccurinthe
virtoushumorafterdeathupto100times.

606.Postmortemcaloricityisseeninall,
except:
AP06;Bihar12;BHU12

a)Burns
b)Sunstroke
c)Tetanus
d)Septicemia
CorrectAnswer-A
Ans.Burns

607.Colorofpostmortemlividityin
hypothermicdeaths:
NEET13

a)Purple
b)Deepred
c)Cherryred
d)Brightpink
CorrectAnswer-D
Ans.Brightpink

608.Inhowmanyhoursdoesadeadbody
floatinIndiainsummer?
a)6hours
b)12hours
c)24hours
d)48hours
CorrectAnswer-C
Ans.is'c'i.e.,24hours
Timeoffloatationofdeadbodyindrowiningisordinarily24
hoursafterdeathinsummerand2to3daysincoldseason.

1. Thebodyfloatsquickerinsummerduetoearlyputrefaction)
2. Itfloatsupsoonerinshalloworsalinewaterbecauseofitshigher
specificgravity)
3. Floatationisquickerinpollutedwaterduetoquickerdecomposition)
4. Bodiesofwomenandfattypersonsfloatearlierastheyarehighter.
Bodiesofthechildrenfloatupearlierasbonesarelighter.

609.Foamyliverisseenin:
TN08;UP08;NIMS11;NEET13
a)Arsenicpoisoning
b)Electrocution
c)Hanging
d)Putrefaction
CorrectAnswer-D
Ans.Putrefaction

610.Inmedicolegalautopsy,cavitytobe
openedfirstis?
a)Thoracic
b)Abdomen
c)Cervical
d)Anyoftheabove
CorrectAnswer-D
Ans.is'd'i.e.,Anyoftheabove[RefReddy30thlep.97]
Dependingontypeofcase,anyofthebodycavitycanbeopened
first.Spinalcordisroutinelynotopened.
Itisconvenienttostarttheexaminationwiththecavitychiefly
affected

611.Incisedlookinglacerationisseenat?
a)Forehead
b)Hand
c)Thorax
d)Abdomen
CorrectAnswer-A
Ans.is'a'i.e.,Forehead
LACERATIONS(TearorRupture)
Lacerationsaretearsorsplitsofskin,mucousmembraneand
underlyingtissue(e.g.,muscleorinternalorgans).Lacerationsare
producedbyapplicationofbluntforcetobroadareaofthebody,
whichcrushorstretchtissuesbeyondthelimitsoftheirelasticity.
Localizedportionsoftissuearedisplacedbytheimpactoftheblunt
force,whichsetsuptractionforcesandcausestearingoftissues.
Featuresoflacerationsare:?
i)Hairandhairbulb,nervesandbloodvesselsarecrushedThere
maybeparalysis(nervecrushed)andhemorrhageisnot
pronounced(bloodvesselscrushed).
ii)Siteofinjuryisthesiteofimpact.
iii)Shapeofinjuryisirregular,marginsareirregularand
contused/abradedandshowtagsoftissue.
iv)Sizeofinjurydoesnotcorrespondstoimpactingsurface.
Therearefollowingtypesoflaceration:?
1)Splitlaceration:
Splittingoccursbycrushingofskinbetween
twohardobjects.Bluntforceonareaswheretheskinisclosetorigid
structureslikebonewithscantysubcutaneoustissue,mayproduce
awoundthatbylinearsplittingoftissuemaylooklikeincisedwound,
i.e.,incisedlikeorincisedlookingwound.Examplesofsucharea

arescalp,eyebrows,cheekbones(zygomatic),lowerjaw,iliac
crest,perineumandskin.Awoundproducedbyafallonkneeor
elbow
withlimbflexedandbyasharpstonealsosimulatesincised
wound.
2)Strechlacerations:Overstretchingoftheskin,ifitisfixed,will
causelaceration,forexample,bykicking,suddendeformityofbone
occursafterfracture,makingitcompound.
3)Avulsion(shearinglaceration):Anavulsionisalaceration
producedbysufficientforce(shearingforce)deliveredatanacute
angletodetach(tearoff)aportionofatraumatizedsurfaceorviscus
fromitsattachment,theshearingandgrindingforcebyaweight.
Flayingistypeofavulsioninwhichshearingandgrindingforceby
weight(suchasoflorrywheelpassingoveralimb)mayproduce
avulsion(separationofskinfromunderlyingtissue/deglovingofa
largearea).
4)ears:Tearsoftheskinandtissuescanoccurfromimactbya
againstirregularorsemi-sharpobjects,suchasdoorhandleofa
car.Thisisanotherformofoverstretching.
5)Cutlaceration:Cutlacerationsmaybeproducedbyaheavy
sharpedgedinstrument.

612.Incisedlookinglacerationisseeninall,
except
AFMC11;NEET13

a)Iliaccrest
b)Zygomaticbone
c)Shin
d)Chest
CorrectAnswer-D
Ans.Chest

613.Blackeningofeyemostcommon
becauseof:
NEET13

a)Frictionabrasion
b)Patternedabrasion
c)Imprintabrasion
d)Contusion
CorrectAnswer-D
Ans.Contusion

614.Ectopicbruiseismostcommonlyseen
in:
NEET13

a)Leg
b)Eye
c)Pinna
d)Scalp
CorrectAnswer-B
Ans.Eye

615.Inacaseofhangingneckligaturemarks
areexampleof
a)Contussion
b)Printedabrasion
c)Laceration
d)Bruise
CorrectAnswer-B
Bi.e.Printedabrasion

616.TrueaboutStabWounds?
a)DepthisgreaterthanBreadth
b)Breadthisgreaterthandepth
c)Lengthisgreaterthanbreadth
d)Ithaswoundofentryandexit
CorrectAnswer-A
Ans.is'a'i.e.,Depthisgreaterthanbreadth[RefReddy30thle
p.179]
Stab/puncturewoundisaninjurycausedbypointedweaponssuch
asdagger,knife,needle,arrow,scissoranditsdepthisthegreatest
dimension.

617.Marshallstriadisseenin?
a)Explosiveinjury
b)Gunshotinjury
c)Drowninginjury
d)None
CorrectAnswer-A
Ans.is'a'i.e.,Explosiveinjury[Ref
epository.up.ac.za/bitstream/handle/2263/19400/Blumenthal_Does(2012).pdj]
Marshall'striadincludespunctate-bruises,abrasionsandsmall
punctatelacerationsallofwhicharetypicallyfoundinanexplosive
bombblast.

618.Kennedyphenomenonisseenin:
NEET13
a)Roadtrafficaccident
b)Gunshotinjury
c)Burns
d)Contusion
CorrectAnswer-B
Ans.Gunshotinjury

619.Gunshotresidueonhandscanbe
detectedby:
DNB10;NEET13

a)Phenolphthaleintest
b)Dermalnitratetest
c)Benzidinetest
d)H2activationtest
CorrectAnswer-B
Ans.Dermalnitratetest

620.Shotgundoesnotcontainuse:
NEET13
a)Barrel
b)Chokebore
c)Bullets
d)Muzzle
CorrectAnswer-C
Ans.Bullets

621.Dirtcollarorgreasecollarisseen
in:
NEET13

a)Puncturedwoundbysharpweapon
b)Laceratedwound
c)Firearmentrywound
d)Stabwound
e)None
CorrectAnswer-C
Ans.Firearmentrywound
Smudgering/Leadring/Greasecollar/Dirtcollar
Thisisduetothewipeofthesoftmetalofthebullet,ordirtpresent
onit,orgreasecarriedfromthebarrel&isdepositedroundthe
entrancewoundinternaltotheabradedcollar
Thesmudgeringmaythereforebeabsentwhenthejacketedbullet
haspassedthroughclothing
Thesmudgingincaseofleadshotorunjacketedbulletscan
bedetectedmicrochemicallyonthetarget(skin/cloth)
Theforensicvalueofbulletwipeistoestablishaholeasabullet
hole,todeterminetheentrysite,&onoccasionthesequenceof
shotsorbullet'spassagethroughmultipleobjects.

622.Lafaciessymapthiqueisseenin?
a)Hanging
b)Strangulation
c)Myocardialinsufficiency
d)Railwayaccident
CorrectAnswer-A
Ans.is'a'i.e.,Hanging[RefReddy30th/ep.526]
Lafaciessymapthique:eyeonthesideoftheknotinhanging
remainsopen,duetooverstretchingofcervicalsympatheticchainof
theneckonthisside.
Itoccursinhangingduetopressureofligatureknotonthecervical
sympatheticchain.

623.Burkingincludes:
NEET13
a)Choking
b)Ligature
c)Overlaying
d)Traumaticasphyxia
CorrectAnswer-D
Ans.Traumaticasphyxia

624.Hyoidbonefracturemostcommon
occursin?
a)Manualstrangulation
b)Hanging
c)Smothering
d)Traumaticasphyxia
CorrectAnswer-A
Ans.is'a'i.e.,Manualstrangulation
Asmanualstrangulation(throttling)isamongthemostviolentform
ofasphyxia,hyoidfractureandotherinjurytoneckstructuresis
morecommon.

625.Victimwaschokedwithmouth&nose
coveredwithelbowaroundneck.Itis
calledas?

a)Mugging
b)Garrotting
c)Bansadola
d)Noneoftheabove
CorrectAnswer-A
Ans.is'a'i.e.,Mugging[RefReddy30th/ep.321]
Mugging:Strangulationiscausedbyholdingtheneckofthevictimin
thebendoftheelbow.
Garrotting:Thevictimisattackedfrombehind.Thethroatmaybe
graspedoraligatureisthrownovertheneck&quicklytightened,by
twistingitwithalever,whichresultsinsuddenlossinconsciousness
andcollapse.
Bansdola:onestrongbambooorstickisplacedacrossthebackof
theneck&anotheracrossthefront.Boththeendsaretiedwitha
ropeduetowhichthevictimissqueezedtodeath

626.Causeofdeathfordrowningincold
water:
NEET13

a)Vagalinhibition
b)Asphyxia
c)Lossofconsciousness
d)Ventricularfibrillation
CorrectAnswer-A
Ans.Vagalinhibition

627.Insexualassaultofachild,thehymenis
usuallynotruptureddueto:
NEET13

a)Deepseated
b)Underdeveloped
c)Tootoughtorupture
d)Distensible
CorrectAnswer-A
Ans.Deepseated

628.Mostcommonhymenruptureinavirgin
is?
a)Anterior
b)Anterolateral
c)Posterolateral
d)Posterior
CorrectAnswer-C
Ans.is'c'i.e.,Posterolateral
Hymenrupture:
Congenital:anterior
Duetointercourseorforeignbody:posterolateral(4/8or5/70'
clock)>posterior(60'clock).
Invirginrupture(tears)ofhymenduetosuddentretchingoccursin
posteriorhalfofmembraneusuallyatthesides(i.e.posterolaterally)
in4or8'Oclockor5or7'Oclockposition,orinthemidlineofhymen
(6'Oclockposition).
Withfirstintercoursetearsusuallyoccurinposteriormidline
becausethehymenliessuspendedacrossapotentialspacehere,
whereasanteriorlyperiurethraltissuesbuttressthehymen.
Morethan2tearsareunusual,Semilunarhymenoftenruptureson
bothsides.Annularhymenwhichnearlyclosesupthevaginalorifice
maysufferseveralhymenallacerationsindicatefirstsexual
intercourse.
Onedeep'Vshapedcleft/tearat6'0clockoranumberofclefts
usuallyinposteriorhafthymenmembraneindicatepassageofany
objectthroughhymeorificewhichislargeritsoriginalopening.'
Inprepubertalchildrenposteriortearmayinvolvefourchette
producingadeepUshapeddefect.Fourchetteistorn,fossa

navicularisdisappearandposteriorcommissuremayberuptured.
Thelatterinjuryusuallydoesnotoccurinconsentingsexual
intercourseunlessthereismuchdisproportionbetweenthemale
andfemaleparts.

629.Frotteruismis?
a)Sexualpleasureisobtainedbywitnessingtheactofurination
b)Sexualgratificationbyrubbingprivateparts
c)Sexualpractiseinvolvingthreepeople
d)None
CorrectAnswer-B
Ans.is'b'i.e.,Sexualgratificationbyrubbingprivateparts[Ref
Reddy30thiep.395]
Frotteurism:iscontactwithanotherpersoninordertoobtainsexual
gratification.Sexualgratificationbyrubbingprivatepartsagainsta
femalebodyincrowd.ItispunishableunderSection290I.P.C.,
withfineuptoRs200.

630.Disputedmaternitycanbesolvedby
usingthefollowingtests,EXCEPT:
a)Bloodgrouping
b)HLAtyping
c)Preciptintest
d)DNAfingerprinting
CorrectAnswer-C
Precipitintestisanantigen-antibodyreactiontest,usedto
distinguishbetweenspecies.
Itusesspeciesspecificantiserum.
Itwillnotbeusedfordisputedmaternity.
Ref:TextbookofMedicalJurisprudence,ForensicMedicineand
ToxicologyByParikh,6thEdition,Pages7-21;TheEssentialsof
ForensicMedicineandToxicologyByKSNarayanReddy,27th
Edition,Pages402

631.Gastriclavageiscontraindicatedin
whichofthefollowing:
September2010

a)Arsenicpoisoning
b)Kerosenepoisoning
c)OrganophosphorusPoisoning
d)Dhaturapoisoning
CorrectAnswer-B
Ans.B:KerosenePoisoning
Lavageiscontraindicatedwhenpatientshaveacompromised,
unprotectedairwayandinpatientsatriskofgastrointestinal
hemorrhageorperforation.
Relativecontraindicationsincludewhenthepoisoningisduetoa
corrosivesubstance,hydrocarbons,orforpoisonsthathavean
effectiveantidote.

632.Whatactsasbothpoison&antidote?
a)CuSO4
b)HgC12
c)Ar203
d)Thalliumarsenate
CorrectAnswer-A
Ans.is'a'i.e.,CuSO4[Ref:Reddy30th/ep.500,503]
Coppersulphateactsasapoison&anantidoteforphosphorus
poisoning.
Coppersulphatewheningestedcausesburningpaininstomach
withcolickyabdominalpain,vomiting.Inseverecases,hemolysis,
hemoglobinuria,methemoglobinemia,jaundice,pancreatitis&
cramps&convulsions.Deathcanoccurduetohepatic&renal
failure.
Itisgiveningastriclavageforphosphorouspoisoningasitcoatsthe
particlesofphosphorouswithafilmofcoppersulphidewhichis
harmless.

633.Firstaidshouldbegiveninacidcontact
?
a)Washwithnormalwatersoap
b)Washwithmildalkaliagent
c)Washwithstrongalkali
d)Refertohighercentre
CorrectAnswer-B
Ans.is'b'i.e.,Washwithmildalkaliagent[RefReddy30Thlep.
485]
Themostcriticalaspectofacidburncareistheconstantand
prolongedwashingoftheareawithwaterataroomtemperatureto
gettheacidoutimmediately.
Totryandminimizethedamage,theeffectsofacidshouldbe
neutralizedbyapplyingamildalkalisolutiontotheaffectedarea
(solutionofsodiumbicarbonateandwater.

634.Amotivationalsyndromeisseenwith:
Maharashtra10;NEET13
a)Heroin
b)Cannabis
c)Cocaine
d)Clonidine
CorrectAnswer-B
Ans.Cannabis

635.Cyanideodourisof?
a)Rottenegg
b)Fish
c)Fruity
d)Bitteralmond
CorrectAnswer-D
Ans.is'd'i.e.,Bitteralmond[RefReddy30th/ep.578]
Odorsassociatedwithpoisoning
Garliklike:Phosphorus,arsenic,zincphosphide,aluminium
phosphide(celphos),arsinegas,tellurium,parathion,malathion,
arsenic.
Rotteneggs:Hydrogensulphide,mercaptans,disulphiram.
Fishyormusty:Zincphosphide.
Bitteralmonds:Cyanide,HCN.
Acrid:Paraldehyde,chloralhydrate.
Burntrope:Cannabis.
Kerosenelike:Keroseneandorganophosphates.
Phenolicsmell:Carbolicacid.

636.Instrychnosnuxvomicapoisoning,
patient:
NEET13

a)Becomesunconsciousimmediately
b)Becomesunconsciousin1horso
c)Becomesunconsciousatendstage
d)Remainsconsciousthroughout
CorrectAnswer-D
Ans.Remainsconsciousthroughout

637.Innuxvomicapoisoning,posture
commonlyassumedbythespineis:
NEET13

a)Opisthotonus
b)Emprosthotonus
c)Pleurosthotonus
d)BandC
CorrectAnswer-A
Ans.Opisthotonus

638.Phossyjawiscausedby?
a)Whitephosphorus
b)RedPhosphorus
c)Arsenic
d)Antimony
CorrectAnswer-A
Ans.is'a'i.e.,WhitePhosphorus
Phossyjawiscausedbyphosphoruspoisoning.Allphosphorus
poisoningarecausedbywhite(yellow)phosphorus(Red
phosphorusisnontoxic).
Phosphoruspoisoning
Phosphorusisaprotoplasmicpoisonaffectingcellularoxidationand
causinganoxicnecorbiosis,classicallyaffectingliver.Itincreasesfat
depositionandinhibitsglycogendepositioninliver.Itisusedinfire
works(Diwalipoisoning)andasratpoison.Lethaldoseis60-120
mg.
Phosphorusoccursintwoforms:?
1)White/yellowphosphorus:Itiswhite,andbecomesyellowon
exposuretoair.Itistranslucent,waxy,luminousandcrystalline
cylinders.Ithasgarliclikeodor.Itisinsolubleinwaterandluminous
indark.Itsfumesshowphosphorescence.
2)Redphosphorus:Itisreddishbrown,inert,odourlessand
tasteless.Itisnontoxic(thuspoisoningoccursonlyduetowhite
phosphorus).Itisputonthesides(strikingsurface)ofmatchbox
(alongwithpowderedgalss).
Acutepoisoning
Ithasfollowingstages:
i)1stStage(GIirritation):Thereisnausea,vomiting,diarrheaand
garlicodor.Thisstagelastsfor8hoursto3days.

garlicodor.Thisstagelastsfor8hoursto3days.
ii)2ndStage(Asymptomatic):Thisstagelastsfor3days.
iii)3rdStage:Thereisliverandkidneydamageduetoabsorbed
phosphorus.Initiallyliverisenlargedduetoacutefattyinfiltration.
Laterlivershruksduetonecrosis,i.e.acuteyellowatrophy.
Chronicpoisoning
Toothacheisthefirstsymptomwhichisassociatedwithlooseningof
teeth,necrosisofgumsandosteomyelitisofjaw.Thereforechronic
phosphoruspoisoningisalsoknowasphossyjaw(orglassjaw).
Postmortemappearance
Thereisgarlicodor.Visceraandstoolglowindark(dueto
luminosity).
Topreserveluminosity,visceraarepreservedinsaturatedsaline
solution.
Rectifiedspiritisnotusedasitcauseslossofluminosity.

639.Mutteringdeliriumisseenwith:
NEET13
a)Ricinus
b)Dhatura
c)Cocaine
d)Aconite
CorrectAnswer-B
Ans.Dhatura

640.Ethyleneglycolwheningestedaffects
kidneybyforming:
NEET13

a)Formaldehyde
b)Oxalates
c)Phytates
d)Phosphates
CorrectAnswer-B
Ans.Oxalates

641.Chronicarsenicpoisoningdoesnot
cause:
NEET13,15

a)Mixedsensoryandmotorneuropathy
b)Mesothelioma
c)Hyperkeratosisofskin
d)Anemia
CorrectAnswer-B
Ans.Mesothelioma

642.Raindroppigmentationiscausedby?
a)Clofazimine
b)Dapsone
c)Minocycline
d)Arsenic
CorrectAnswer-D
Ans.is'd'i.e.,Arsenic

643.Brutonianlinesongumsisseenin
poisoningwith?
a)Lead
b)Mercury
c)Mercury
d)Zinc
CorrectAnswer-A:B
Ans.is'a>b'i.e.,Lead>Mercury[RefReddy306/ep.497-498]
Burtonianlineisblueline,whichisseenonuppergumsinlead
poisoning.
Similarblueline(butnotcalledburtonianlineinthesecases)isalso
seeninpoisoningwithMercury(Hg),Copper(Cu),Silver(Ag),
bismuth(Bi),andIron(Fe).

644.Oochronosisisseeninwhichpoisoning
?
a)Hydrochloricacid
b)Carbolicacid
c)Oxalicacid
d)Formicacid
CorrectAnswer-B
Ans.is'b'i.e.,Carbolicacid[RefReddy30th/ep.489]
Chroniccarbolicacidpoisoning/phenolmarasmus
Thepoisoningischaracterisedbyanorexia,weightloss,headache,
vertigo,darkurineandpigmentationofskin&sclera.
Carbolicacid/Phenolisconvertedintohydroquinone&pyrocatechol
inthebodybeforebeingexcretedintheurine.
Hydroquinoneerpyrocatecholmaycausepigmentationinthe
corneaandvariouscartilages-Oochronosis.
Oochronosisiscommonlyassociatedwithalkaptonuria,inwhich
homegentisicacidgetsdepositedincartilages,ligaments&fibrous
tissues.

645.Opiumisderivedfrom:
NEET13
a)Leaf
b)Root
c)Poppyseed
d)Unripecapsule
CorrectAnswer-D
Ans.Unripecapsule

646.Mechanismofcyanidepoisoningisby
inhibiting:
NEET13

a)DNAsynthesis
b)Cytochromeoxidase
c)Proteinbreakdown
d)Proteinsynthesis
CorrectAnswer-B
Ans.Cytochromeoxidase

647.Meaningoftermvitriolageis:
a)Usingvitriolforsuicide
b)Usingvitriolformurder
c)Vitriolthrowing
d)PerforationofstomachcausedbyH2SO4
CorrectAnswer-C
Ci.e.Vitriolthrowing
Vitriolage(Vitriolthrowing)isthrowingofanycorrosive,not
necessarilysulphuricacid(H2SO4)onanotherpersonQ.
Eyesare
themostaffectedorgans.

648.Antidoteforstrychninepoisoning
is:
NEET13

a)Fomepizole
b)Physotigmine
c)Barbiturates
d)Naloxone
CorrectAnswer-C
Ans.Barbiturates

649.Bariumcarbonatepoisoning
causes:
NEET13

a)Respiratorydistress
b)Gastrointestinalirritation
c)Muscularweakness
d)Cyanosis
CorrectAnswer-C
Ans.Muscularweakness

650.Oximesarecontraindicatedinwhich
poisoning:
NEET13

a)Malathion
b)Diazinon
c)Phorate
d)Carbamate
CorrectAnswer-D
Ans.Carbamate

651.EthyleneGlycolantidoteis?
a)Barbiturates
b)Acetycysteine
c)Ferricchloride
d)Fomipizole
CorrectAnswer-D
Ans.is'd'i.e.,Fomipizole[RefReddy30th/ep.533]
Antidoteforethyleneglycolpoisoningisfomipizole.

652.Tetanyiscausedbypoisoningwith:
NEET13
a)Oxalicacid
b)Carbolicacid
c)Sulphuricacid
d)Nitricacid
CorrectAnswer-A
Ans.Oxalicacid

653.Widmark'sformulahelpsinthe
measurementsofbloodlevelof:
a)Barbiturates
b)Cocaine
c)Alcohol
d)Benzodiazepines
CorrectAnswer-C
Ci.e.Alcohol

654.Fatheroftoxicology
is:
NEET13

a)Paracelsus
b)Galen
c)Galton
d)Orfila
CorrectAnswer-A
Paracelsusiscalledfatheroftoxicology.Mathieuorfilais
thoughtofasfatherofmoderntoxicology

655.MostcommondrugabuseinIndia?
a)Cannabis
b)Amphetamine
c)Cocaine
d)Heroine
CorrectAnswer-D
Ans.is'd'i.e.,Heroin
Amongstthegivenoptions,heroinisthemostcommonly
abusedsubstance:
Alcohol43.9%
Opioids(includingheroin)26%
Cannabis11.6%

656.Blackpapperadultrantis?
a)Khesaridal
b)Driedpapayaseed
c)Finesand
d)None
CorrectAnswer-B
Ans.is`b'i.e.,Driedpapayaseed

657.Hydrogenperoxideisusedinallofthe
followingchemicaltestsforblood
except:
NEET13

a)Benzidinetest
b)KastleMayertest
c)Orthotoludinetest
d)Teichmanntest
CorrectAnswer-D
Ans.Teichmanntest

658.Whichsnakebitecauseshematologic
abnormalities?
a)Cobra
b)Crait
c)Viper
d)Seasnake
CorrectAnswer-C
Ans.is'c'i.e.,Viper[RefParikh&Yep.9.44]
Hematotoxic Vipers
Myotoxic
Seasnakes
Neurotoxic
Elapids-cobra,Krait,coral

659.Whichofthefollowingshouldnotbe
donewhiledealingwithapatientof
snakebite?

a)Tightbandapplied
b)Reassuarance
c)Localincision
d)Cleanwithsoapandwater
CorrectAnswer-C
Ans.is'c'i.e.,Localincision
Treatmentofsnakebite
Reassuranceshouldbegiventopatient.
Applicationofpressureoverbittenareawhichdelaysabsorptionof
venom.
Tieabroadfirmbandage(tourniquet)proximaltothebittenareaand
aroundthelimbwhichwouldoccludevenousandlymphatic
drainagebutnotarterialordeepvenousflow.
Limbimmobilization.
Localincisionandsuctionshouldnotbedoneasitcancauselocal
injuryandnervedamage.
Cleanthewoundwithsoapandwateroriodineandcoverwith
steriledressing.
Polyvalentsnakeantivenomisgiven.Howeveritisgivenonlyin
followingsituations:
i. Rapidlyprogressiveandseverelocalfindings.
ii. Therearemanifestationsofsystemictoxicity.

660.DNAfingerprintingcannotbetakenfrom
?
a)Saliva
b)Tooth
c)Buccalmucosa
d)Blood
CorrectAnswer-B
Ans.is'b'i.e.,Tooth[RefParikh6thlep.7.2-7.15]
DNAfinger-printing/DNAprofiling
Isatechniqueemployedbyforensicscientiststoassistinthe
identificationofindividualsbytheirrespectiveDNAprofile
itisafoolproofmethodtoconclusivelyfixthepaternity/maternity.
Themostdesirablemethodofcollectingareferencesampleisthe
useofabuccalswab.Whenthisisnotavailable(e.g.
becauseacourtordermaybeneededandnotobtainable)other
methodsmayneedtobeusedtocollectasampleofblood,
saliva,semen,orotherappropriatefluidortissuefrompersonal
items(e.g.toothbrush,razor,etc.)orfromstored
samples(e.g.bankedspermorbiopsytissue)

661.Notagrievousinjury:
September2005
a)Multiplescarsofface
b)Fractureoffemur
c)Emasculation
d)Contusionofbreast
CorrectAnswer-D
Ans.D:Contusionofbreast
Sec320IPC:anyofthefollowinginjuresaregrievous?
Emasculation
Permanentprivationofsightoreithereye
PermanentprivationofHearingoreitherear
Permanentprivationofanymemberorjoint
Permanentdisfigurationoftheheadorface
Destructionorpermanentimpairingofthepowerofanymemberor
joint
Fractureordislocationofaboneortooth
Anyhurtswhichendangerslife,orwhichcausesthevictimtobein
severebodilypainorunabletofollowhisordinarypursuitsfora
periodoftwentydays

662.Lendrumstainisdonefor?
a)Airembolism
b)Fatembolism
c)Amnioticfluidembolism
d)None
CorrectAnswer-C
Ans.is'c'i.e.,Amnioticfluidembolism[RefHandbookof
autopsypractise3rdlep.283]
Amnioticfluidembolismdiagnosis
SectionsoflungsandotherorgansarestainedusingPhloxine-
Tartrazine(Lendrumstaining)todetectsquames,
AlcianBluetodetectmucin.
SudanBlackorOilRedtodetectvernixcaseosa.

663.Malarialparasitewasdiscoveredby-
a)Ronaldross
b)Paulmuller
c)Laveran
d)Pampania
CorrectAnswer-C
Ans.is'c'i.e.,Laveran
.Thespecificcausativeagentofmalariawasdiscoveredinthered
bloodcellsofapatientin1880byAlophonseLaveran,aFrench
armysurgeoninAlgeria.
Remember
.Romanowskydevelopedamethodofstainingmalaria
parasitesinbloodfilms.

664.Whichofthefollowingisaprotista-
a)Algae
b)Fungi
c)Protozoa
d)Bacteria
CorrectAnswer-C
Ans.is'c'i.e.,Protozoa

665.Katayamafeveriscausedby-
a)F.hepatica
b)C.sinensis
c)S.haematobium
d)A.lumbricoides
CorrectAnswer-C
Ans.is'c'i.e.,S.haematobium
Katayamafever(AcuteSchistosomiasis)occursinaboutamonth
afterinfecionwithS.japonicumandS.mansoniandrarelywithS.
haematobium.

666.

Whichpartofbacteriaismostantigenic?
a)Proteincoat
b)Lipopolysaccharide
c)Nucleicacid
d)Lipids
CorrectAnswer-A
Ans.isdi.e.,Lipids
Proteinsaremostimmunogenic,whilelipids&nucleicacidsare
leastimmunogenic.
Polysaccharides(carbohydrates)arelessimmunogenicthanprotein
antigens,butaremoreantigenicthanlipids&nucleicacids.

667.Thecorrectorderofgramstainingis
a)GentianvioletIodineCarbolfuchsin
b)IodineGentianvioletCarbolfuchsin
c)CarbolfuchsinIodineGentianviolet
d)CarbolfuchsinGentianvioletIodine
CorrectAnswer-A
Answer-A-GentianvioletIodineCarbolfuchsin
1. Applicationoftheprimarystain(crystalviolet).Gentianvioletalso
knownascrystalvioletstainsallcellsblue/purple
2. Applicationofmordant:Theiodinesolution(mordant)isaddedto
formacrystalviolet-iodine(CV-I)complex;allcellscontinueto
appearblue.
3. Decolourizationstep:Thedecolourizationstepdistinguishesgram-
positivefromgram-negativecells.Theorganicsolventsuchas
acetoneorethanolextractsthebluedyecomplexfromthelipid-rich,
thin-walledgram-negativebacteriatoagreaterdegreethanfromthe
lipid-poor,thick-walled,gram-positivebacteria.Thegram-negative
bacteriaappearcolourlessandgram-positivebacteriaremainblue.
4. Applicationofcounterstain(safranin):Thereddyesafraninstains
thedecolourizedgram-negativecellsred/pink;thegram-positive
bacteriaremainblue.
NOTEIfyouarestrugglingtorememberthestainingreagents
usedinthisprocedureandtheirorderyoucanrememberthis
sentence"ComeInAndStain"i.e.theorderisCrystalviolet,Iodine,
Alcohol/AcetoneandthefinaloneisSafranin.

668.Flagellanottrue-
a)Locomotion
b)Attachment
c)Proteininnature
d)Antigenic
CorrectAnswer-B
Ans.is'b'i.e.,Attachment

669.Innutrientagarconcentrationofagaris-
a)1%
b)1.5%
c)3%
d)4%
CorrectAnswer-B
Ans.is'b'i.e.,1.5%
NutrientAgarisageneral-purpose,nutrientmediumusedforthe
cultivationofmicrobessupportingthegrowthofawiderangeofnon-
fastidiousorganisms.Nutrientagarispopularbecauseitcangrowa
varietyoftypesofbacteriaandfungi,andcontainsmanynutrients
neededforthebacterialgrowth.
Nutrientagarismadebyadding1.5%agartothenutrientbroth.
0.5%Peptone
0.3%beefextract/yeastextract.
0.5%NaCl
Distilledwater
pHisadjustedtoneutral(7.4)at25?C.

670.Enrichmentmediaforcholera?
a)VRmedium
b)TCBSmedium
c)Cary-Blairmedium
d)Alkalinepeptonewater
CorrectAnswer-D
Ans.is'd'i.e.,Alkalinepeptonewater

671.Bloodagarisanexampleof?
a)Enrichedmedia
b)Indicatormedia
c)Enrichmentmedia
d)Selectivemedia
CorrectAnswer-A
Ans.is'a'i.e.,Enrichedmedia

672.Indonovanosis-
a)Pseudolymphadenopathy
b)Penicillinisusedfortreatment
c)Painfululcer
d)Suppurativelymphadenopathy
CorrectAnswer-A
Ans.is`a'i.e.,Pseudolymphadenopathy
Donovanosis
CausedbyCalymmatobacteriumgranulomatis.
C.granulomatisis?
Gramnegative
Encapsulated
Nonmotile
Intracellular
Itsharesmanymorphologicandserologiccharacteristic(antigenic)
and>99%homologyatthenucleotidelevelwithKlebsiella.
Clinicalmanifestations
IP1-4weeks
Beginsasoneormoresubcutaneousnodulesthaterodethrough
skintoproduceclean,granulomatous,sharplydefined,usually
painlesslesions.
Thegenitaliaareinvolvedin90%ofcases.
Genitalswelling,particularlyoflabia,iscommon.
Indonovanosis,heaped-upgranulomatoustissuemayfollowandvia
subcutaneousextensiontoinguinalareamayform"pseudo-
buboes";however,theabsenceoftruelymphadenopathyisthe
hallmarkofthisinfection.
ComplicationsPseudoelephantiasis,phimosisandparaphimosis.

Diagnosis:
Thepreferreddiagnosticmethodinvolvesdemonstrationoftypical
intracellularDonovanbodieswithinlargemononuclearcells
visualizedinsmearspreparedfromlesionsorbiopsyspecimens.
Stainusediswright-Giemsa
Treatment:
Azithromycin(DOC)
Doxycycline(2'dchoice)
Chloramphenicol

673.Necrotizingfascitisiscausedby-
a)Staphylococcusaureus
b)Betahemolyticstreptococci
c)Clostridiumperfringens
d)Pneumococcus
CorrectAnswer-B
Ans.is'b'i.e.,Betahemolyticstreptococci
Thespectrumofinfectionsofthedeepsofttissuesrangesfrom
localizedbacterial,viralandparasiticlesionstorapidlyspreading,
tissuedestructiveinfectionssuchasnecrotizingfascitisand
myonecrosis.
A)Pyomyositis:-Itiscommonintropics,thereforealsocalled
tropicalpyomyositis.Itisalocalizedinfectionofskeletalmuscles.It
iscausedmostcommonlybystaphylococcusaureus.
B)NecrotizingfascitisNecrotizingfascitisisaninfectionofthe
deeperlayersofskinandsubcutaneoustissues,easilyspreading
acrossthefascialplanewithinthesubcutaneoustissues.Thereare
twotypesofnecrotizingfascitis:?
i)Type1:-
Itisapolymicrobialinfection,i.e.mixedaerobicand
anaerobicinfection.Itoccursmostcommonlyaftersurgical
proceduresindiabeticpatientsorinthosewhohaveperipheral
vasculardisease.
ii)Type2:-Itiscausedmostcommonlybystreptococcus
pyogenes(GroupAbetahemolyticstreptococci).
C)Clostridialmyonecrosis(Gasgangrene):-Itischaracterizedby
rapidandextensivenecrosisofmuscleaccompaniedbygas
formationandsystemictoxicity.ItiscausedbyC.perfringens(most
common),C.novyi,C.septicumandC.histolyticum.
Itisalsocalled

typeIIInecrotizingfascitis.

674.Staphaureuscauses-
a)Erythrasma
b)Chancroid
c)Acnevulgaris
d)Bullousimpetigo
CorrectAnswer-D
Ans.is'd'i.e.,Bullousimpetigo
Impetigoisdividedintotwotypes:?
i)Non-bullousimpetigo(Impetigocontagiosum):-
Causedby
staphylococcusaureusandstreptococcuspyogenes.
ii)Bullousimpetigo:-Causedbystaphylococcusaureus.

675.TRUEaboutmycoplasmais-
a)Causeslunginfection
b)Penicillinisdrugofchoice
c)Thickcellwall
d)Thalliumacetateinhibitsthegrowth
CorrectAnswer-A
Ans.is'a'i.e.,Causeslunginfection
Mycoplasmainfections
.Mycoplasmapneumoniaecausesinfectionofupperandlower
respiratorytract.
.Highestattackratesin5-20yearsold.
.Children<5yrsUpperrespiratorysymptoms
.Children>5yrs--0.Bronchitisandpneumonia
.disease.Aboutotheroptions
.Penicillinisnotactiveagainstmycoplasmas.
.Mycoplasmaslackcellwall.
Mycoplasmasareresistanttothalliumacetateinaconcentrationof
1:10000

676.Achildcomewithfever,cold,cough,
membraneovertonsils;nasalswabis
taken,cultureshouldbedoneonwhich
mediumforearliestdiagnosis?

a)Loffelersserumslop
b)L.J.media
c)MCConkey'sAgar
d)Citratemedia
CorrectAnswer-A
Ans.is'a'i.e.,Loffelersserumslop[Ref:Ananthanarayanelep.
233]
Fever,cold,coughwithmembraneontonsilssuggestthediagnosis
ofdiphthesia.
ForrapidgrowththespecimenisinoculatedonLoeffer'sserumslop.
DiphtheriabacilligrowonLoeffler'sserumslopeveryrapidlyand
coloniescanbeseenin6-8hours,longbeforeotherbacteriagrow

677.Botulismismostcommonlydueto-
a)Egg
b)Milk
c)Meat
d)Pulses
CorrectAnswer-C
Ans.is'c'i.e.,Meat
Followingnewtypesofbotulismhavebeenaddedin18th/eof
Harrison
.Adultintestinaltoxemiabotulism:-resultsfromabsorptionof
toxinproducedinsituafterrarelyoccuringintestinalcolonizationwith
toxigenicclostridia.
.Iatrogenicbotulism:-resultsfrominjectionofbotulismtoxin.

678.Trueaboutchlamydiaareallexcept:
a)Obligateintracellularorganism
b)Grampositive
c)Reticulatebodyismetabolicallyactive
d)Replicatebybinaryfission
CorrectAnswer-B
Ans.is.'b'i.e.,Grampositive

679.Whichgroupofstreptococcusgrowat>
60?C
a)A
b)B
c)C
d)D
CorrectAnswer-D
Ans.is'd'i.e.,D
Amongstreptococci,enterococcus(groupDstreptococcus)isheat
resistant.

680.Pigmentproducingatypical
mycobacteria?
a)M.fortutionandM.chelonae
b)M.xenopiandMAC
c)M.gordonaeandM.szulgai
d)M.ulcerans
CorrectAnswer-C
Ans.is'c'i.e.,M.gordonaeandM.szulgai
Non-tubercularmycobacteria(alsocalledatypicalmycobacteria)
havebeenclassifiedintofourgroupsbyRunyonbasedonpigment
productionandrateofgrowth.
1)GroupI(photochromogens):-Theseproducepigmented
colonies(yellow-orange)onlywhenexposedtolight,butnotindark.
ExamplesofphotochromogensareM.asiaticum,M.kansasii,M.
marinum,
andM.simiae.
2)
GroupII(scotochromogens):-Thesealwaysproduce
pigmentedcolonies(yellow-orange-red),i.e.indarkaswellasin
light.ScotochromogensareM.flovescens,M.gordonae,M.
scrofulaceumandM.szulgi.
3)
GroupHI(Nonchromogens):-Thesedonotproducepigment.
NonchromogensareM.aviumcomplex(MAC),M.haemophilum,M.
gastri,M.ulcerans,M.xenopiandM.nonchromogenicum.
4)
GroupIV(rapidgrowers):-Thisisheterogeneousgroupof
mycobacteriacapableofrapidgrowth,coloniesappearingwithin7
daysofincubation.Withinthegroup,photochromogenic,
scotochromogenicandnonchromogenicspeciesoccur.
Chromogenic(pigmentproducing)rapidgrowersaremostly
saprophytic,e.g.M.phleiandM.smegmetis.M.fortuitumandM.

cheloneidonotproduceanypigment.OtherrapidgrowersareM.
abscessus,M.vaccae,M.genevense,M.confluentis,
andM.
intermedium.


681.Sevensheathedflagellaisseenin-
a)Vcholera
b)Hpylori
c)Psaeroginosa
d)Spirochetes
CorrectAnswer-B
Ans.is'b'i.e.,H.pyroli
H.pylorihasfivetosevensheathedpolarflagella.
VibriocholeraeandPseudomonasaeruginosahavesinglepolar
flagellum.Occasionalstrainsofpseudomonasmaycontain2or3
flagella.
Spirochetes(Treponema)ismotilebyendoflagella.

682.TrueaboutHinfluenza-
a)Grownonsheepbloodagar&CO2
b)itisnotcapsulated
c)Invasivestrainismostcommon
d)Grampositive
CorrectAnswer-C
Ans.is'c'i.e.,Invasivestrainismostcommon
Invasivediseaseismorecommonthannon-invasivedisease.
H.influenzaedoesnotgrowonbloodagar.
Itisgramnegativeandcapsulated.

683.Meningococcidifferfromgonococciin
thatthey?
a)Areintra-cellular
b)Possessacapsule
c)Causefermentationofglucose
d)Areoxidasepositive
CorrectAnswer-B
Ans.is'b'i.e.,Possessacapsule

684.Choleratoxinisdueto-
a)Chromosome
b)Plasmid
c)Phage
d)Transposons
CorrectAnswer-C
Ans.is'c'i.e.,Phage
.Choleratoxinproductionisdeterminedbyafilamentousphage
integratedwithbacterialchromosome.

685.Whichofthefollowingisthemechanism
ofactionoftetanospasmin?
a)InhibitionofreleaseofGABAandglycine
b)InhibitionofAchreleasefromsynapse
c)Inhibitionofproteinsynthesis
d)Activationofadenylylcyclase
CorrectAnswer-A
Ans.is'a'i.e.,InhibitionofreleaseofGABAandglycine
Pathogenicity
CLtetanihaslittleinvasivepropertyandisconfinedtotheprimary
siteoflodgment.
Tetanusresultsfromtheactionofthepotent
exotoxinitproduces.

686.Virulenceofgonococciisdueto-
a)Pili
b)Endotoxin
c)Exotoxin
d)None
CorrectAnswer-A
Ans.is'a'i.e.,Pili
.Majorvirulencefactorforgonococciispill
(fimbriae).

687.Tabesdorsalisisseenin-
a)Primarysyphilis
b)Secondarysyphilis
c)Tertiarysyphilis
d)Latentsyphilis
CorrectAnswer-C
Ans.is'c'i.e.,Tertiarysyphilis

688.Meningitiswithrashisseenin-
a)Neisseriameningitidis
b)H.influenzae
c)Strepto.agalactae
d)Pneumococcus
CorrectAnswer-A
Ans.is'a'i.e.,Neisseriameningitidis

689.Ehrlichiachaffeensisiscausativeagents
of
a)HME
b)HGE
c)Glandularfever
d)None
CorrectAnswer-A
Ans.a.HME

690.Invasiveinfectionscausedbyallexcept
?
a)Shigella
b)Salmonella
c)V.cholerae
d)Yersinia
CorrectAnswer-C
Ans.is'c'i.e.,V.cholerae[RefHarrisontelep.1084]

691.AchildwithfeverwithABCs&pusin
stools,causativeorganismis?
a)ETEC
b)EHEC
c)EPEC
d)EAEC
CorrectAnswer-B
Ans.is'b'i.e.,EHEC[RefHarrison18thlep.1084]
FeverwithRBCandpusinstoolssuggestinflammatorydiarrhea.
AmongstthegivenoptionsEHECcausesinvasion.

692.WhatisNOTtrueaboutyersiniosis-
a)Gram-negativebacillus
b)CausedbyYpestis
c)Byyersiniaenterocolitica
d)Byyersiniapseudotuberculosis
CorrectAnswer-B
Ans.is'b'i.e.,Causedbyyersiniapestis
Yersiniosisisaninfectiousdiseasecausedbyabacteriumofthe
genusYersinia.
YersiniaenterocoliticaisaGram-negativebacillus-
shapedbacterium,belongingtothefamilyYersiniaceae.It
ismotileattemperaturesof22?29?C(72-84?F),butbecomes
nonmotileatnormalhumanbodytemperature.[1][2]Y.
enterocoliticainfectioncausesthediseaseyersiniosis,whichis
ananimal-bornediseaseoccurringinhumans,aswellasinawide
arrayofanimalssuchascattle,deer,pigs,andbirds.
Infectioncausedbyyersiniagenusaredividedinto:-
i)Plague:-Itisadeadlyinfectiousdiseasecausedbyyersinia
pestis.
ii)Yersinosis:-
Itischaracterizedbyinfectiousdiarrhea,
enteritis,ileitisandoccasionallysepticemia.Itiscausedbyyersinia
enterocolitica
(mostcommon)andyersiniapseudotuberculosis.

693.AllaretrueaboutB.Quintanaexcept-
a)Causestrenchfever
b)Notdetectedbyweilfelixreaction
c)Recurrenceiscommon
d)Tickisthevector
CorrectAnswer-D
Ans.is'd'i.e.,Tickisthevector
Trenchfever
Trenchfever,alsocalled5-dayfeverorquintanfever,iscausedby
Bartonellaquintana(Rochalimaeaquintana).
Thehumanbodylouce(Pediculushumanscorporis)isthevector
andhumansistheonlyknownreservoir.Clinicalmanifestations
Theincubationperiodis15-25days(range,3-38days).
'Classical'trenchfeverpresentsasfebrileillness.Feveris
exceedinglyvariable,butcommonlylastsforabout5days.Thefever
isfollowedbyaremissionandarecurrenceafter5days.These
recurrencesmaybesingleormultipleandupto12recurrences
every5-6daysarenotuncommon.
Othersymptomsandsignsincludeheadache,backandlimbpain,
profusesweating,shivering,myalgia,arthralgia,splenomegaly,a
maculopapularrashinoccasionalcases,andnuchalrigidityinsome
cases.
Diagnosis
DefinitivediagnosisrequiresisolationofB.quintanabyblood
culture.
Weil-Felixtestusedfordiagnosisofricketssialinfectionisnegative
intrenchfever.
Treatment

Itistreatedwithgentamycinalongwithdoxycycline.

694.Painlessgenitalulcerinmalewith
evertedmarginisseenin?
a)Syphilis
b)Chancroid
c)Herpes
d)LGV
CorrectAnswer-A
Ans.is'a'i.e.,Syphilis[RefHarrison18th/ep.1382,Jawetz
22"/ep.642]
Painlessinduratedulcerwithevertedmargins,h/oofsexual
exposureandlackofsystemicsymptomsfavoursthediagnosisof
syphilis.

695.Leptospirosisistransmittedby:
a)Rat
b)Cat
c)Dog
d)Fish
CorrectAnswer-A
Ans.is.'a'i.e.,Rat

696.Fishtankgranulomaisseenin-
a)Mfortuitum
b)Mkansasi
c)Mmarinum
d)Mleprosy
CorrectAnswer-C
Ans.is'c'i.e.,M.Marinum
'Fishtankgranuloma',alsocalledswimmingpoolgranuloma',is
causedbyM.marinum.

697.Actinomycosisissensitiveto?
a)Streptomycin
b)Nystatin
c)PenciIlin
d)Iodox-uridine
CorrectAnswer-C
Ans.is'c'i.e.,Pencillin

698.Allcausefourniergangreneexcept-
a)Staphylococcus
b)Streptococcus
c)Clostridium
d)Bacteroides
CorrectAnswer-C
Ans.is'c'i.e.,Clostridium
Fournier'sgangreneisanecrotisingfascitisofgenitalia,usually
affectingthescrotumandpenis.
Therehavebeenmanytypesofbacteriologicalcultureencountered
inFournier'sgangrene,bothsinglestrainandpolymicrobialculture.
Majorityofcasesareduetomixedinfectioncausedbybothaerobic
andanaerobicbacteria.
Followingarecommoncausativeorganisms:Staphylococcus
aureus,streptococcuspyogenes
((3-hemolyticstreptococci),
enterobacteriaceae(E.coli,klebsiella,proteus),enterococci,
pseudomonas,andanaerobeslikebacteroidesand
peptostreptococcus.

699.Reactivetuberculararthritis:
a)Spinaventusa
b)Pott'sdisease
c)Poncet'sdisease
d)None
CorrectAnswer-C
Ans.is.'c'i.e.Poncet'sdisease

700.Schistosomiasisistransmittedby?
a)Cyclops
b)Fish
c)Snaile
d)Cattle
CorrectAnswer-C
Ans.is'c'i.e.,Snails
Intermediatehostforschistosomasp.issnail.

701.Trueabouttrematodes
a)Twohostrequired
b)Segmented
c)Anuspresent
d)Bodycavitypresent
CorrectAnswer-A
Ans.is'a'i.e.,Twohostrequired
Medicallyimportantmemberoftheclasstrematodabelongto
subclassDigenea,astheyaredigenetic,i.e.requiretwohosts.The
definitivehostinwhichtheypassthesexualoradultstageare
mammals,humansoranimals,andtheintermediatehostsinwhich
theypasstheirasexualorlarvalstagesarefreshwatermolluscsor
snails.
Trematodesareunsegmented,andhavenoanusandbodycavity.

702.
Whichorganismcanbeisolatedfrom
stool&sputum-
a)Paragnomus
b)Fasciola
c)Chlornchis
d)P.carini
CorrectAnswer-A
Ans.is'a'i.e.,Paragnomus[RefRajeshkaryakartep.212]
Twoorganismcanbeisolatedfrombothsputumandstool:-

1. TrophozoiteofE.histolytica
2. EggsofParagonimus

703.Trueaboutdiphyllobothrium:
a)Manissinglehost
b)Irondeficiencyanemiaisseen
c)Operculatedeggisdiagnostic
d)Fishisthedefinitivehost
CorrectAnswer-C
Ans.c.Operculatedeggisdiagnostic

704.Cercariaeareinfectiveformof-
a)S.hematobium
b)P.westermanii
c)F.hepatica
d)T.solium
CorrectAnswer-A
Ans.is'a'i.e.,S.hematobium
CercariaofSchistosomahematobiumistheinfectiveform.

705.Unsegmentedeggsareinwhich
parasite?
a)Trichuristrichura
b)Ancylostoma
c)Necatoramericanus
d)Dracunculus
CorrectAnswer-A
Ans.a.Trichuristrichura

706.Inmalaria,sexualcycleis-
a)Sporozoitetogametocytes
b)GametocytestoSporozoite
c)Occursinhuman
d)Responsibleforrelapse
CorrectAnswer-B
Ans.is'b'i.e.,Gametocytestosporozoite
LIFECYCLEOFPLASMODIUM
Plasmodiumpassesitslifecycleintwodifferenthosts?
1. Human(intermediatehost)
2. Femaleanaphelinemosquito(Definitivehost)
Humancycle
.Humancyclestartswiththeintroductionofsporozoitesbythe
biteofaninfectedanaphelinemosquito.
.Itcomprisesthefollowingstages.
1)Pre-erythrocyticschizogony
-Occursinsidetheparenchymacellsoftheliver.
-Duringthisphasetheparasitesarenotfoundinthe
peripheralblood(bloodissterile).
-Thelibratedmerozoitesarecalledcryptozoites.
-
MicromerozoitesenterthecirculationandstartErythrocytic
schizogony,whilemacromerozoitesre-enterthelivercellsandstart
exoerythrocyticycle(exoerythrocyticcycledoesnotoccurinP
falciparum).
-Durationofpre-erythrocyticschizogony
P.vivax-->8daysP.malariae--f
15days
P.falciparum-->6daysP.ovale--

>9days
2)Erythrocyticschizogony
-ParasiteresidesinsidetheRBCs.
-Passesthroughthestagesoftrophozoite,Schizontandmerozoite.
-Theparasiticmultiplicationduringtheerythrocyticphaseis
responsibleforbringingonaclinicalattackofmalaria.
-Duration
P.vivax,P.ovale,P.falciparum-->48hrs
P.malariae-->72hours
3)Gametogony
Aftererythrocyticschizogony,someofthemerozoitesdevelopinto
gametocytes.
Theindividualwhoharboursthegametocytesisknownasacarrier.
4)Exoerythrocyticschizogony
Someofmerozoites,afterpre-erythrocyticschizogony,reinfectliver
parenchymacellstostartexoerythrocyticschizogony.
Merozoitesliberatedfromexo-erythrocyticschizogonyarecalled
phanerozoites.
ItisabsentinP.falciparum.
Itisresponsiblefortherelapse.
Mosquitocycle
SexualcycleoccursinfemaleAnopheles.
Gametocytesaretransferredtotheinsect.
Gametocytesdevelopintosporozoitesaftercompletesexualcycle.
Sporozoitesareinfectivetoman.

707.Whichtypeofmalariaisassociatedwith
renalfailure-
a)Falciparum
b)Vivax
c)Malariae
d)Ovale
CorrectAnswer-A
Ans.is'a'i.e.,Falciparum
Nephroticsyndrome-->quartamalaria(P.malariae)
Acutetubularnecrosis(renalfailure)malignanttertianmalariaor
perniciousmalaria(causedbyPfalciparum).

708.Malariacausingnephroticsyndrome-
a)P.vivax
b)P.falciparum
c)P.malariae
d)P.ovale
CorrectAnswer-C
Ans.is'c'i.e.,P-malariae
NephroticsyndromeisseeninQuartanmalarialnephropathy,
causedbyrepeatedorchronicinfectionwithP.malariae.

709.Hangingdropmethodisusedfor-
a)T.trichomonas
b)Plasmodium
c)Toxoplasma
d)Cryptosporidium
CorrectAnswer-A
Ans.is'a'i.e.,T.trichomonas
Hangingdropmethodisusedforexaminingmotilityofmicro-
organisms.
Motilityoftrichomonasvaginalismayalsobeobservedbythis
method.

710.Rhaditiformlarvaeisseenin?
a)Teniasolium
b)Strongyloides
c)D.latum
d)Trichenella
CorrectAnswer-B
Ans.is`b'i.e.,Strongyloides[RefPanikar6th/ep.171]
Rhabditiformlarvaisthefirststagefeedinglarvafoundinsome
nematodes.Itisnon-infective.
Filiformlarvaisthenonfeedinginfectivelarvainsomenematodes.
Rhabditiformlarvaandfiliformlarvaarefoundinfollowingimportant
nematodes

711.Wucheriabancrofti,trueis-
a)Unsheathed
b)Tailtipfreefromnuclei
c)Non-periodic
d)All
CorrectAnswer-B
Ans.is'b'i.e.,Tailtipfreefromnuclei
W.bancroftiissheathedandperiodicwithtailtipfreefromnuclei.

712.Flamecellsareseenin:
a)Protozoa
b)Cestode
c)Nematodes
d)None
CorrectAnswer-B
Ans.b.Cestode
Flamecell(alsocalledsolenocyte)istheexcretorycellin
cestodesandtrematodes,thenumberandarrangementofwhichis
usedasabasisforidentification.
Thecellhasatuftofcilia,whosebeatingresemblestheflickeringof
aflame.
Theflamecellsopenintoacollectingtubule.

713.Whichofthefollowingistoxicto
parasite-
a)Peroxidase
b)Interferon
c)IL-2
d)IL-6
CorrectAnswer-A
Ans.is'a'i.e.,Peroxidase
Neutrophilsandmonocytescontainaperoxidase(myeloperoxidase),
thathasbeenimplicatedinantiparasiticactivity.However,
monocyteslosethisenzymewhentheymatureintomacrophages.
Eosinophilsalsocontainperoxidasethatdiffersfrom
myeloperoxidaseofneutrophils.However,likemyeloperoxidase,
eosinophilperoxidasecombineswithH20,andahalidetoforman
antiparasitesystem.
ParasiteEggsize
1)Fasciolagigantica190X100m
2)Echinostomailiocanum100X70m
3)Gastrodiscoideshominis150X70-100m
4)Opisthorcisviverni<30X15m

714.Toxoplasmainchildrencauses:
a)Chorioretinitis
b)Conjunctivities
c)Keratitis
d)Papillitis
CorrectAnswer-A
Ans.a.Chorioretinitis

715.Whichofthefollowingisonlyyeast?
a)Candida
b)Mucor
c)Rhizopus
d)Cryptococcus
CorrectAnswer-D
Ans.isdi.e.,Cryptococcus

716.Regardingfungalcellwallallaretrue
except:
a)Containschitin
b)Preventosmoticdamage
c)Azolesactonthem
d)Doesnotcontainpeptidoglycan
CorrectAnswer-C
Ans.c.Azolesactonthem

717.RenauldBraudphenomenonisseenis:
a)Candidaalbicans
b)Candidapscitasi
c)Histoplasma
d)Cryptococcus
CorrectAnswer-A
Ans.a.Candidaalbicans

718.Largestintestinalprotozoais?
a)E.coli
b)Balantidiumcoli
c)Giardia
d)T.gondii
CorrectAnswer-B
Ans.is'b'i.e.,Balantidiumcoli[RefPanikerp.111]
Largestprotozoa
Balantidumcoli
Smallestintestinalamoeba Dientamoebafragilis
Smallesttapewormfoundin
H.nana
humanintestine
Largesthelminth(largest
T.saginata(beeftapeworm)
worm)
Largestliverfluke F.hepatica
Largesttrematodeinfectingman Fasciolopsisbuski
LargestNematode
Ascaris
SmallestNematode-
Trichinella
Onlyprotozoanparasitefoundin
Giardialamblia
smallintestineofman
Onlyciliateprotozoanparasiteof
Balantidumcoli
man-
Parthenogenicworm(femaleis
Strongyloidesstercoralis.
abletoproducefertile
eggsorlarvaewithoutfertilization)


719.Trichophytonspecieswhichiszoophilic
?
a)T.tonsurans
b)T.violaceum
c)T.schoenleinii
d)T.mentagrophytes
CorrectAnswer-D
Ans.is'd'i.e.T.mentagrophytes
Zoophilicdermatophytesarethespecieswhichprimarilyinfect
animalsandoccasionallytransmittedtohumans.
ZoophilicspeciesoftrichophytonareT.montegrophytesandT
verrucosum.
OtherzoophilicspeciesofdermatophytesisM.canis.

720.Aseptatehyphaeisnotseenin-
a)Rhizopus
b)Mucor
c)Aspergillus
d)None
CorrectAnswer-C
Ans.is'c'i.e.,Aspergillus
Non-septate(aseptate)hyphae-->Rhizopus,mucor.
Septatehyphae-->Aspergillus.

721.Whichofthefollowingisprimarycellline
?
a)Chickembryofibroblast
b)Helacells
c)Verocells
d)WI-38
CorrectAnswer-A
Ans.is'a'i.e.,Chickembryofibroblast
Chickembryofibroblastisprimarycellculture.

722.Virusquantificationisdoneby-
a)Egginoculation
b)Hemadsorption
c)Plaqueassay
d)Electronmicroscopy
CorrectAnswer-C
Ans.is'c'i.e.,Plaqueassay
PlaqueassayandPockassayarequantitativeinfectiveassays.

723.Whichisenvelopedvirus-
a)Denguevirus
b)Norwalkvirus
c)HepAvirus
d)Adenovirus
CorrectAnswer-A
Ans.is'a'i.e.,Denguevirus
Denguevirus(amemberofflavivirodae)isanenvelopedvirus.
Adenovirus,norwalkvirus(calciviridae)andhepatitisAvirus
(Picornaviridae)arenon-envelopedviruses.

724.Coxsackievirusis-
a)Harpesvirus
b)Poxvirus
c)Enterovirus
d)Myxovirus
CorrectAnswer-C
Ans.is'c'i.e.,Enterovirus

725.Serologicaltestingofpatientshows
HBsAg,IgManti-HBcandHBeAg
postive.Thepatienthase-

a)ChronichepatitisBwithlowinfectivity
b)AcutehepatitisBwithhighinfectivity
c)Chronichepatitiswithhighinfectivity
d)Acuteonchronichepatitis
CorrectAnswer-B
Ans.is`b'i.e.,AcutehepatitisBwithhighinfectivity

726.Congenitalvaricellainfectioncausesall
except:
a)Macrocephaly
b)Limbhypoplasia
c)Corticalatrophy
d)Cicatrix
CorrectAnswer-A
Ans.a.Macrocephaly

727.Diagnosisofrotavirusisby:
a)Stoolantigen
b)Stoolantibody
c)Stoolculture
d)Bloodantibody
CorrectAnswer-A
Ans.is'a'i.e.,Stoolantigen[Ref:Hanison18n/ep.1591,
1592;Greenwood1&/ep.5251
InRotavirusdiarrhoea,largeno.ofvirusesareshedinfaeces(atthe
peakofthedisease,asmanyasl0rrvirusparticlescanbedetected
permloffeces).Thesevirusescanbeeasilydetectedby
thefollowingmethods:
i)Enzymeimmunoassay(ELISA)
Itoffersapproximately90%specificity&sensitivityfordetection
ofvirusinstools.
ii)Latexagglutination
iii)Immuneelectronmicroscopy
-->Viralsheddingdetectablebythesemethodsusually
subsideswithinaweek
-->Virusinstoolscanbedetectedforlongerperiodsbyusing
techniquesfordetectingviralRNAs,suchasPCR,Gel
electrophoresis,
probehybridization.


728.Bollingerbodiesareseenin?
a)Chickenpox
b)Cowpox
c)Fowlpox
d)Smallpox
CorrectAnswer-C
Ans.is'c'i.e.,Fowlpox

729.Followingvirusisofpoxvirus-
a)Variola
b)Coxsachie
c)ECHO
d)HSV
CorrectAnswer-A
Ans.is'a'i.e.,Variola

730.Amplifierhostis-
a)PiginJE
b)Doginrabies
c)ManinJE
d)CattleinJE
CorrectAnswer-A
Ans.is'a'i.e.,PigJE

731.Falseaboutp24is
a)Seenafter3weeksofinfection
b)Cantbeseeninfirstweek
c)Cantbedetectedafterseroconversion
d)aandc
CorrectAnswer-D
Ans.is'a>c'i.e.,Seenafter3weeksofinfection>Cantbe
detectedafterseroconversion
AntigendetectioninHIV
Followingasinglemassiveinfection,asbybloodtransfusion,the
virusantigensmaybedetectableinbloodafterabouttwoweeks.
Themajorcoreantigen,p24istheearliestvirusmarkertoappearin
thebloodandistheonetestedfor.IgMantibodiesappearinabout
4-6weeks,tobefollowingbyIgGantibodies.
Iftheinfectingdoseissmall,asfollowinganeedle-stickinjury,the
processmaybeconsiderablydelayed.
Theappearanceofp24antigenemiaandviremia,followedbyIgM
antibodyresponse,coincideswithacuteorseroconversionillness.
Afterwards,freep24antigendisappearsfromcirculationand
remainsabsentduringthelongasymptomaticphase,
toreappear
onlywhensevereclinicaldiseasesetsin.
Howeverantibody-boundp24antigencontinuestobedemonstrable,
afterdissociation.Thep24antigencaptureassay(ELISA)which
usesanti-p24antibodyasthesolidphasecanbeusedforthis.The
testispositiveinabout30%ofHIVinfectedpersons.Withprior
dissociationofantigen-antibodycomplex,thepositivityrate
increasestoabout50%.
Thetestismostusefulinpersonsrecentlyexposedtoriskof

infection,inwhomtheantibodytestisnegative.
Nowcomingtothequestion
Optionb&darestraight-forward.p24antigenisdetectedbyELISA
(enzymeimmuno-assayorenzymelinkedimmuno-assay)andisnot
detectableinfirstweekaftertheinfection.
Option'a'incorrectbecausethoughthep24antigencanbedetected
after2weeksofinfection:"Thep24antigentestcandetectthep24
antigenonaverage10-14daysafterinfectionwithHIV".
"Followingasinglemassiveinfection,thevirusantigensmaybe
detectableinbloodafterabouttwoweeks".
Option'c'ispartlycorrectandpartlyincorrectbecause:-
i)Freep24antigenisnotdetectableafterseroconversion.
However,in30-50%ofinfectedpersons,antibody-boundp24
antigencanbedetectedafteritsdissociationfromantibody.

732.Whichpoxwontgrowinegg,animal
cells:
a)Cowpox
b)Vaccinia
c)Variola
d)Molluscum
CorrectAnswer-D
Ans.d.Molluscum

733.AppearanceofcowdrytypeAinclusion
bodies?
a)Granular
b)Circumscribed
c)Inpolio
d)None
CorrectAnswer-A
Ans.is'a'i.e.,Granular
Intranuclearinclusionbodieswereclassifiedintotwotypesby
cowdry:
a)CowdrytypeA:-Theseareofvariablesizeandgranularin
appearance,
e.g.inherpesvirusandyellowfevervirus.
b)
CowdrytypeB:-Thesearemorecircumscribedandoften
multiple,aswithadenovirusandpoliovirus.

734.Smallpoxbelongstowhichclassof
poxviruses?
a)Parapoxvirus
b)Capripoxvirus
c)Leporipoxvirus
d)Orthopoxvirus
CorrectAnswer-D
Ans.is'd'i.e.,Orthopoxvirus
i)Entomopoxvirinae:Poxvirusesofinsectswhichdonotinfect
vertebrates.
Chordopoxvirinaeareclassifiedintosixgeneraorsubgroups-
i)Orthopoxvirus:Thesearemammalianpoxvirusesthattendto
causegeneralizedinfectionwithrash.Exmplesarevariola(smallpox
virus),
vaccinia,cowpox,monnkeypox,rabbitpox,buffalopox,
camelpox,mousepox.
ii)Parapoxvirus:Virusesofungulatesthatmayoccasionally
infacthuman,eg.Orf(contagiouspustulardermatitis)and
paravaccinia(milker'snode,bovinepupularstomatitis).
iii)Copripoxvirus:Virusesofgoatandsheeps,eg.sheep-pox,
goatpox,lumpyskindisease.
iv)Leporipoxvirus:Virusesofofleporids(rabbits,hares,
squirrels),e.g.myxomaandfibromas.
v)Avipoxvirus:Virusofbirds,eg.fowlpox,turkeypox,
pigeonpox,canarypox.
vi)Suipoxvirus:Virusofswine,eg.swinepox.

735.Brick-shapedvirus-
a)Chickenpox
b)Smallpox
c)CMV
d)EBV
CorrectAnswer-B
Ans.is'b'i.e.,Smallpox
Variolavirus
BelongstoPoxvirusesenvelopedDNA(dsDNA)virus.
Brickshaped
Instainedpreparationelementarybodiesareseen--)Paschen
bodies.
Thevariolavirusisthecausativeagentofsmallpox.
On8thmay1980whoanouncedglobaleradicationofsmallpox.
Vacciniavirus
Itissimilartovariolavirusinproperties.
Itisanartificialvirusanddoesnotoccurinnatureassuch.
Vacciniavirusisbeingemployedasavectorforthedevelopmentof
recombinantvaccines.
Vacciniagenomecanaccomodateabout25000foreignbasepairs,
sufficientforintroducingseveralgenes.
Manygeneshavebeeninsertedeg-HBV,HIV,rabiesandfor
pharmacologicallyimportantproductssuchasneuropeptides.
Howeveritisnotsuitableasavectorforhumanuseduetoits
pathogeniceffects.

736.Bindingofgp120causes:
a)Infectionoftargetcell
b)Facilitationofco-receptor
c)Fusingofvirusandtargetcell
d)None
CorrectAnswer-B
Ans.b.Facilitationofco-receptor

737.NefgeneinHIVisforuse-
a)Enhancingtheexpressionofgenes
b)Enhancingviralreplication
c)Decreasingviralreplication
d)Maturation
CorrectAnswer-C
Ans.is'c'i.e.,Decreasingviralreplication
NonstructuralHIVgenes
o
Theseare?
1)Tat
(Trans-activatinggene):Enhancingtheexpressionofall
viralgenes.
2)Nef(Negativefactorgene):Down-regulatingviralreplication.
3)Rev(Regulatorofviralgene):Enhancingexpressionof
structuralproteins.
4)Vif(Viralinfectivityfactorgene):Influencinginfectivityofviral
particles.
5)Vpu(InHIV-1)andvpx(InHIV-2):Enhancematurationand
releaseofprogenyofvirusfromcells.
6)Vpr:Stimulatespromotorregion.
7)LTR(Longterminalrepeat)sequence:Contains
sequenceswhichgivepromotor,enhancerandintegrationsignals.

738.Whatisp24?
a)EnvelopantigeninHIV
b)CoreantigeninHIV
c)GenomeofHIV
d)Shellantigen
CorrectAnswer-B
Ans.is'b'i.e.,CoreantigeninHIV[RefAnanthanarayan
8thiep.571]
A.Envelopantigens
Spikeantigen-gp120(Principalenvelopeantigen)
Transmembranepedicleprotein-gp41
B.Shellantigen
Nucleocapsidprotein-p18
C.Coreantigens
Principalcoreantigen-p24
Othercoreantigens-p15,p55
Polymeraseantigens-p31,p51,p66

739.Hemorrhagicfeveriscausedby-
a)West-Milefever
b)Sandflyfever
c)Ebolavirus
d)Alloftheabove
CorrectAnswer-C
Ans.is'c'i.e.,Ebolavirus
Ebolavirusbelongstohemorrhagicfever(seeaboveexplanation).

740.Influenzaviruscultureisdoneon?
a)Chorioallantoicmembrane
b)Allantoiccavity
c)Yolksac
d)All
CorrectAnswer-B
Ans.isbi.e.,Allantoiccavity
Inembryonatedeggcultivationforinfluenzavirussiteofinoculation
are:-Allontoiccavityoramnioticcavity
Inoculationsitesinembryonatedeggs
Chorioallantoicmembrane-4HSV,Poxvirus,Rous-sarcoma
virus
Amnioticcavity-4Influenzavirus,mumpsvirus
Allantoiccavity-Influenzavirus,mumpsvirus,
avianadenovirus,newcastlediseasevirus
Yolksac-->HSV,chlamydia,rickettisia

741.ColoradoTicfeveriscausedby:
a)Filoviridae
b)Reoviridae
c)Coronaviridae
d)Calciviridae
CorrectAnswer-B
Ans.b.Reoviridae

742.Idiotypicclassofantibodyisdetermined
by-
a)Fcregion
b)Hingeregion
c)Carboxyend
d)Aminoend
CorrectAnswer-D
Ans.is'd'i.e.,Aminoend
TheidiotypeisdefinedasthespecificregionoftheFabportionof
theIgmoleculetowhichantigenbinds.
Itisonaminoterminalwhereantigenbindingsiteispresent.

743.Prozonephenomenonisseenwith?
a)Sameconcentrationofantibodyandantigen
b)Inantigenexcesstoantibody
c)Antibodyexcesstoantigen
d)Hyperimmunereaction
CorrectAnswer-C
.Ans.is'c'i.e.,Antibodyexcesstoantigen

744.Apatientwithsorethroathasapositive
PaulBunnelltest.Thecausative
organismis-

a)EBV
b)Adenovirus
c)CMV
d)Herpesvirus
CorrectAnswer-A
Ans.is'a'i.e.,EBV
.EBVassociatedmalignancies
-Burkitt'slymphoma0
-Nasopharynagealcarcinoma
-Hodgkin'sdisease(mixedcellularity
type)-TonsillarCarcinoma
-TCell
lymphoma-
Thymoma
-Gastriccarcinoma
-CNSlymphomainAIDSand
-AngiocentricnasalNK/Tcellimmunoproliferativelesions
transplantrecipient
-Angioimmuno-blasticlymphadenopathy
-LeiomyosarcomaOtherassociatednonmalignantconditions
-OralhairyleukoplakiainAIDSpatients
-Chronicfatiguesyndrome
-X-linkedlymphoproliferativesyndrome(Duncan'sdisease)
Laboratorydiagnosis

1.Heterophileantibodiestest
.Thestandarddiagnosticprocedureinchildrenandyoung
adultsisheterophileantibodiestest-Paul-BunnellTest
.Atitreof40foldorgreaterisdiagnosticofacuteEBVinfection
inapatientwhohassymptomscompatiblewithinfectious
mononucleosis.
Testusuallyremainspositivefor3months.
Testisusuallynegativeinchildren<5years,inelderlyorinpatients
withsymptomsnottypicalofinfectiousmononucleosis.
.Monospottestforheterophileantibodiesismoresensitivethan
theclassicalheterphiletest.
2.EBVspecificantibodytest
.Usedinpatientswholackheterophileantibodies(children
anti-viralcapsidantigen(anti-VCA)-->mostcommon
anti-EBVnuclearantigen(anti-EBNA)
anti-earlyantigen(anti-EA)

745.Paulbunnelreactionisatypeof
a)Agglutination
b)CF
c)Precipitation
d)Flocculationtest
CorrectAnswer-A
Ans.is'a'i.e.,Agglutinationtest
PaulBunnelltestistubeagglutinationtest.

746.Whichcellscauserosetteformationwith
sheepRBCs-
a)Tcells
b)NKcells
c)Monocytes
d)All
CorrectAnswer-A
Ans.is'a.Tcells

747.Allaretureaboutinnateimmunityexcept
?
a)Non-specific
b)Firstlineofdefence
c)Notaffectedbygeneticaffected
d)Includescomplement
CorrectAnswer-C
Ans.is'c'i.e.,Notaffectedbygeneticaffected

748.Pentavalentimmunoglobinis
a)IgA
b)IgG
c)IgM
d)IgE
CorrectAnswer-C
Ans.is'c'i.e.,IgM

749.Centerofcomplementpathway-
a)C3
b)CI
c)C5
d)C2
CorrectAnswer-A
Ans.is'a'i.e.,C3

750.Acontinuousquantitativedatacanbe
depictedwiththehelpof:
September2009

a)Bardiagram
b)Piechart
c)Histogram
d)Pictogram
CorrectAnswer-C
Ans.C:Histogram
Numericaldata/quantitativedataisdatameasuredoridentifiedona
numericalscale.
Numericaldatacanbeanalyzedusingstatisticalmethods,and
resultscanbedisplayedusingtables,charts,histogramsand
graphs.
Forexample,aresearcherwillaskaquestionstoaparticipantthat
includewordshowoften,howmanyorpercentage.Theanswers
fromthequestionswillbenumerical.
Examplesofquantitativedatawouldbe:'thereare643dotsonthe
ceiling'or'thereare735piecesofbubblegum'.or'thereare8
planetsinthesolarsystem'

751.WhichofthefollowingTcell
independentAntigenactsthrough-
a)T-cell
b)B-cell
c)Macrophages
d)CD8+Tcells
CorrectAnswer-B
Ans.is'b'i.e.,B-cells
T-cellindependentantigensdirectlystimulatesB-cellswithout
processingbyantigenpresentingcells.

752.Maximumhalflife-
a)IgG
b)IgA
c)IgM
d)IgE
CorrectAnswer-A
Ans.is'a'i.e.,IgG

753.Whichofthefollowingimmunoglobulin
isresponsibleforopsonisation-
a)IgA
b)IgG
c)IgM
d)none
CorrectAnswer-C
Ans.is'c'i.e.,IgM>'b'i.e.,IgG

754.Opsonizationtakesplacethrough-
a)C3a
b)C3b
c)C5a
d)C5b
CorrectAnswer-B
Ans.is'b'i.e.,C3b

755.Antigenidiotypeisrelatedto-
a)Fcfragment
b)Hingeregion
c)C-terminal
d)N-terminal
CorrectAnswer-D
Ans.is'd'i.e.,N-terminal
Theidiotypeisthespecificregionofthe
fabportion(notFcfragment)ofIgmoleculetowhichantigenbinds.
Idiotype(antigenbindingsite)isonvariableregion,whichisat
aminoterminal(N-terminal).
Theaminoacidsequencesofthevariable
regionsarenotuniformlyvariablealongtheirlength,butconsistof
relativelyinvariable
andsomehighlyvariablezones.
ThehigltlyvariableZonesareinvolvedwlththeformationofantigen
bindingsites.
Thesitesonthehyervariableregionsthatmakeactual
contactwiththeepitopesarecalled"complementaritydetermining
Regions.

756.Activationofclassicalcomplement
pathway?
a)IgA
b)IgG
c)IgM
d)IgD
CorrectAnswer-C
Ans.is'c'i.e.,IgM

757.Superantigencauses-
a)PolyclonalactivationofT-cells
b)StimulationofBcells
c)Enhancementofphagocytosis
d)Activationofcomplement
CorrectAnswer-A
Ans.is'a'i.e.,PolyclonalactivationofT-cells
Superantigensarecapableofactivatingupto20%oftheperipheral
T-cellpool,whereasconventionalantigensactivate<1in10,000.


758.Complementformedinliver-
a)C2,C4
b)C3,C6,C9
c)C5,C8
d)C1
CorrectAnswer-B
Ans.isb'i.e.,C3,C6,C9
Biosynthesisofcomplementcomponents
.Complementcomponentsaresynthesizedinvarioussitesof
body:-
i)Intestine-->Cl
ii)Macrophages-->C2,C4
iii)Spleen-->C5,C8
iv)Liver-->C3,C6,C9

759.GenecomponentsofHLAclassI
includes-
a)A,B,C
b)DR
c)DQ
d)DP
CorrectAnswer-A
Ans.is'a'i.e.,A,B,C
.ClassIHLAcomprisesA,BandCloci.

760.Antibodyelevatedinparasiticinfection?
a)IgA
b)IgE
c)IgG
d)IgM
CorrectAnswer-B
Ans.is'b'i.e.,IgE

761.Testforcoliformcount?
a)Eijkmantest
b)Casoni'stest
c)Nitratetest
d)Ureasetest
CorrectAnswer-A
Ans.is'a'i.e.,Eijkmantest
Adleranalysingpresumptivecoliformcount,Ecolicountiscofirmed
byothertestslikeEijkmanttestandindoleproduction.

762.Trueaboutinterferonis:
a)Itisasyntheticantiviralagent
b)Inhibitsviralreplicationincells
c)Isspecificforparticularvirus
d)None
CorrectAnswer-B
Ans.(b)InhibitsviralreplicationincellsRt,11itlii
ilallarayl111CO,
ppIiiiitC,i:;r,p
Interferon(hostcodedprotein)hasnodirectactiononvirusesbut
inhibitviralreplicationbyselectivelyinhibitingtranslationofviralm-
RNAwithoutaffectingcellularm-RNA.
IFNarenotvirusspecificbutspeciesspecific.
Itisof3types:
TypeCellsource.Cell
targetBiological
ctivity
IFNa(protein)orleukocyte
Antiviralactivity;stimulates
Allcells
IFNAllcells
TL.ell,macrophagesand
NKcellactivity
IFN13(glycoprotein) Allcells AllcellsDirectantitumoreffects
orFibroblastIFN
UpregulatesMHCclassI
antigenexpression.Used
therepeuticallyinviraland
autoimmunedisease
?IFNy
Regulatesmacrophage
(glycoprotein)or Tcells
Allcells andNKcellsactivation
immuneIFN
NK
StimulatesIgsecretionbyB

NK
StimulatesIgsecretionbyB
cells
cellsInductionofclassII
histocompatibilityantigens
TH1Tcelldifferentiation

763.Schistosomiasisisanexmpleof:
a)Meta-zoonoses
b)Cyclo-zoonoses
c)Direct-zoonoses
d)Sporo-zoonoses
CorrectAnswer-A
Ans.a.Meta-zoonoses

764.Whichofthefollowingorganismsdoes
notentersthroughabrasionsintheskin
-

a)Erhusiopathiae
b)Ecorrodens
c)Chominis
d)Cviolaceum
CorrectAnswer-C
Ans.is'c'i.e.,C.hominis
Bacteriacirculateandmultiplyinblood--->Septicemia
.Bacteriacirculateinblood-->Bacteremia
.Toxinscirculateinblood-->Toxemia
.Pusinblood-->Pyemia
Eikenellacorrodens(E.corrodens),chromobacteriumviolaceum(C.
violaceum)andEikenellacorrodens(E.corrodens)entersthebody
throughskinabrasions.
Awidevarietyofmicroorganismsthatresideontheskinand
mucousmembranesofthebody,aswellasthosefoundinthe
environment,cancauseskinandsofttissueinfections.
Theseorganismsenterthebodythroughbreaksintheskinor
mucousmembranes,throughwoundsmadebytraumaorbitesoras
acomplicationofsurgeryorforeign-bodyimplantation.
Theseorganismshavebeenmentionedinfollowingtable:
Aerobicand
Anaerobicbacteria Aerobic
Yeast
facultative
microorganismsfrom
microorganisms
unusual,Vspecialized
andzoonoticinfections

?Coagulase
?
Actinobacillus
?Candida
negative
Peptostreptococcus ?
albicans
?Candida
staphylococci spp.
actinomycetemcomitans krusei
?
?Candida
Staphylococcus ?Clostridiumspp. ? Aeromonasspp.
parapsilosis
aureus
?Enterococcus ?Eubacterium
? Bacillusanthracis
spp.
limosum
?
?Bacteroides
Streptococcus
Bergeyellazoohelcum
fragilis
viridans
?
?
Chromobacterium
Corynebacterium ?Prevotellaspp.
violaceum
spp.
?
?Bacillus
?Porphyromonas ? Eikenellacorrodens
cereus
?E.coli
?Fusobacterium
? Erysipalothrix
?Serratia
?Veillonellaspp.
rhusiopathiae
?Enterobactor
? Francisellatularensis
?Proteus
? Haemophillusspp.
?Morganella
? Kingellakingae
?
? Pasteurellamultocida
Pseudomonas
Streptobacillus
?Acinetobactor
?
moniliformis
? Vibriovulnificus

765.18yearsoldgirlpresentswithwatery
diarrhea.Mostlikelycausativeagent-
a)Rotavirus
b)V.cholerae
c)Salmonella
d)Shigella
CorrectAnswer-B
Ans.is'b'i.e.,V.cholerae
AmongstthegivenoptionsRotavirusandV.choleraecausewatery
diarrhea.
Acutewaterydiarrhainchildrenisusuallybacterialinorigin,most
commonlyduetoenterotoxigenicE.coli(ETEC).Vcholeraeisalso
acommoncause.
Rotavirusisthemostcommoncauseofdiarrheaininfantand
children(thepatientinquestionisadult).

766.'Secreteofnationalhealthliesinthe
homesofpeople'statementby?
a)IndiraGandhi
b)Abhrahamlincon
c)Bhore
d)FlorenceNightingale
CorrectAnswer-D
Ans.is'd'i.e.,FlorenceNightingale[RefHousingandplanning
reviewp.35]
FlorenceNighingalepointedoutnearlyahundradeyearsago,"the
secretofnationalhealthliesinthehomesofpeople".

767.Fatherofpublichealth-
a)Cholera
b)Plague
c)Leptospirosis
d)Anthrox
CorrectAnswer-A
Ans.is'a'i.e.,Cholera
Historyofcholera
oFatherofpublichealthisadisease,notaperson.
oFatherofpublichealthischolera.
o
JohnSnow(1813-1858)foundthelinkbetweencholeraand
contaminateddrinkingwaterin1854,usingspotmap.
oRobertKochindifiedV.cholerae(1885).
Choleramorbus-Usedin19th&early20thcenturiesforbothnon-
epidemiccholeraandothergastrointestinaldiseasesthatresembled
cholera.

768.Whichagencymonitorsairqualityin
India?
a)Centralpollutioncontrolboard
b)Centralairqualityboard
c)Centralpublicworksdept
d)None
CorrectAnswer-A
Ans.is'a'i.e.,Centralpollutioncontrolboard
TheNationalAirQualityMonitoringProgramme,sponsoredbythe
CentralPollutionControlBoard(CPCB)since1990,hasgenerated
databaseoverlast14yearsin10majorIndiancities.

769.Worksampling?
a)Samplingdoneforindividualwork
b)Assessmentoftimespentbyworkersinwork
c)Doneinveryshortperiod
d)None
CorrectAnswer-B
Ans.is'b'i.e.,Assessmentoftimespentbyworkersinwork
Worksamplingisatechniqueusedtoinvestigatetheproportionof
totaltimedevotedtothevariousactivitiesthatconstituteajobor
worksituation.

770.Childprotectionschemeisunderwhich
ministry-
a)Ministryofhealthandfamilywelfare
b)MinistryofSocialwelfare
c)Ministryofwomenandchilddevelopment
d)Ministryofeducation
CorrectAnswer-C
Ans.is'c'i.e.,Ministryofwomenandchilddevelopment
oIn2006theMinistryofWomenandChildDevelopment(MWCD)
proposedadoptionofIntegratedChildProtectionScheme(ICPS).
oIn2009thecentralgovernmenttaketheschemeitsapprovaland
hasbeguntheextensivetaskofprovidingchildrenwithaprotective
andsafeenvironmenttodevelopandflorish.
oThepurposeoftheschemeistoprovideforchildrenindifficult
circumstances,aswellastoreducetherisksandvulnerabilities
childrenhaveinvarioussituationsandactionsthatleadtoabuse,
neglect,exploitation,abondonmentandseparationofchildren.

771.Schoolhealthcheckupcomesunder-
a)Disricthospital
b)PHC
c)CHC
d)Schoolhealthcommittee
CorrectAnswer-D
Ans.is'd'i.e.,Schoolhealthcommittee[RefPark22"d/ep.534,
535]
Theschoolhealthcommittee(1961)inIndiarecommendedmedical
examinationofchildrenatthetimeofentryandthereafterevery4
years.

772.
WebCausationofdiseaseismost
appropriate?
a)Mostlyapplicableforcommondisease
b)Betterforalltherelatedfactorsassociatedwithcausationof
disease
c)Epidemiologicalratio
d)Helpstointerrupttheriskoftransmission
CorrectAnswer-B
Ans.is'b'i.e.,Betterforalltherelatedfactorsassociatedwith
causationofdisease
Webofcausation
oThismodelisideallysuitedinthestudyofchronicdisease,where
thediseaseagentisoftennotknownbutistheoutcomeof
interactionofmultiplefactors.
oThe"webofcausation"considersallthepredisposingfactorsof
anytypeandtheircomplexrelationshipwitheachother.
o
Thecausalwebprovidesamodelwhichshowsavarietyof
possibleinterventionsthatcouldbetakenwhichmightreducethe
occurrenceofdisease(e.g.MI)
oThewebofcausationdoesnotimplythatthediseasecannotbe
controlledunlessallthemultiplecausesorchainsofcausationorat
leastanumberofthemareappropriatelycontrolled.Thisisnotthe
case.Sometimesremovaloreliminationofjustonlyonelinkorchain
maybesufficienttocontroldisease,providedthatlinkissufficiently
importantinthepathogenicprocess.
oTherefore,inamultifactorialevent,individualfactorsarebyno
meansalloftheequalweight.

773.Iodizedsaltiniodinedeficiencycontrol
programmeis?
a)Primaryprevention
b)Secondaryprevention
c)Teriaryprevention
d)None
CorrectAnswer-A
Ans.is'a'i.e.,Primaryprevention
Fortificationoffoode.g.iodizedsaltisprimaryprevention.
Primarypreventionhastwomaincomponents:-
i)Healthpromotion
ii)Specificprotection
ModeoftheinterventionofPrimarypreventionHealthpromotion
Specificprotection
1. Healtheducation1.Immunization
2. Environmentalmodifications2.Useofspecificnutrientsiron
andfolicacidtablet
3. Nutritionalintervention3.Chemoprophylaxis
4. Lifestyleandbehavioralchanges4.Protectionagainst
occupationalHazards
5. Protectionagainstaccident
6. Protectionfromcarcinogens
7. Avoidanceofallergens
8. ControlofspecificHazardsingeneralenvironmente.g.airpollution,
noisecontrol
9. Controlofconsumerproductqualityandsafetyoffoodanddrug
10. Usingamosquitonet
11. Contraception


774.Secondarypreventionisapplicableto
a)Causalfactors
b)Earlystageofdisease
c)Latestageofdisease
d)Noneoftheabove
CorrectAnswer-B
Ans.is'b'i.e.,Earlystageofdisease
PrimordialpreventionBeforeriskfactorpresent.
PrimarypreventionBeforeonsetofdisease(riskfactorpresent).
SecondarypreventionInearlystageofdisease.
TertiarypreventionLatestageofdisease.

775.DALEisreplacedby?
a)DALY
b)HALE
c)OALY
d)None
CorrectAnswer-B
Ans.is'b'i.e.,HALE[RefPark20th/ep.24]
HALE(Health-Adjustedlifeexpectancy):-Thenameofthe
indicatorusedtomeasurehealthylifeexpectancyhasbeenchanged
fromdisabilityadjustedlifeexpectancy(DALE)tohealthadjustedlife
expectancy(HALE).
HALEisbasedonthelifeexpectancyatbirthbutincludesand
adjustmentfortimespentinpoorhealth.Itismosteasilyunderstood
astheequivalentnumberofyearsinfullhealththatanewborncan
expecttolivebasedoncurrentratesofillhealthandmortality.

776.Naturalhistoryofdiseaseisstudiedwith
?
a)Longitudinalstudies
b)Cross-sectionalstudies
c)Both
d)None
CorrectAnswer-A
Ans.is'a'i.e.,Longitudinalstudies
Longitudinalstudies
Inthistypeofstudy,observationsarerepeatedinthesame
populationoveraprolongedperiodusingfollowupexaminations.
Theyareusefulto

1. Studythenaturalhistoryofthedisease
2. Foridentifyingriskfactorsofdisease
3. Forfindingouttheincidencerateorrateofrecurrenceofnewcases
ofthedisease.
Note:Longitudinalstudiesaredifficulttoorganizeandmoretime
consumingthancross-sectionalstudies.

777.Problemofbiasismaximumwith-
a)Cohortstudy
b)Casestudy
c)Casecontrolstudy
d)Experimentalstudy
CorrectAnswer-C
Ans.is'c'i.e.,Casecontrolstudy

778.Berkesonianbiasisatypeof?
a)Selectionbias
b)Intervieverbias
c)Informationbias
d)Recellbias
CorrectAnswer-A
Ans.is'a'i.e.,Selectionbias
Berksonianbiasisatypeofselectionbias.
Bias
oItisanysystemicerrorintheanalysisofstudythatresultsina
mistakenestimateofanexposure'seffectontheriskofdisease.
oMainlybiasesareoffollowingtypes.
1.Selectionbias
Surveillance/detectionbiasReferrelbiasor
volunteerbiasBerksonianbias
NeymansurvivalbiasResponsebias
2.Information(Misclassification)bias
ReportingbiasInterviewerbias
RecallbiasHawthornebias(attentionbias)
3.Confoundingbias
Confoundingissometimesisreferredtoasathirdmajorclassof
bias.

779.Allofthefollowingarecharacteristicsof
casecontrolstudyexcept-
a)Quickresultsareobtained
b)Measuresincidencerate
c)Proceedsfromeffecttocause
d)Inexpensivestudy
CorrectAnswer-B
Ans.is'b'i.e.,Measuresincidencerate
oIncidenceratecannotbemeasuredbycase-controlstudyas
denominator(populationatriskisnotavailable).

780.Beststudyfordefinitivecauseofdisease
?
a)Case-control
b)Cohort
c)Ecological
d)Cross-sectional
CorrectAnswer-B
Ans.is'b'i.e.,Cohort
AmongstthegivenoptionsCohortstudyisbesttotestthe
associationbetweenriskfactoranddisease.
Herearethedifferentepidemiologicalstudieswithdecreasing
orderofaccuracytotesttheassociationbetweenriskfactor
anddisease:-
Systematicreviewandmeta-analysis-->Overallmostreliable
Randomizedcontrolledtrials(controlledclinicaltrails)-->Most
reliableindividualstudy.
Retrospective(Non-concurrent/historic)Cohortstudy.
Prospective(concurrent)Cohortstudy.
Casecontrolstudy
Cross-sectionalstudy
Ecologicalstudy

781.Suspectedcauseprecedingthe
observedeffectisanexamplefor-
a)Temporalassociation
b)Consistencyofassociation
c)Strengthofassociation
d)Coherenceofassociation
CorrectAnswer-A
Ans.is'a'i.e.,Temporalassociation
Temporalassociation
oItimpliescauseprecedseffectoreffectfollowscause,i.e.,
suspectedcauseprecedingtheobservedeffect.
oItisthemostimportantcriteriaforcausalassociation
o
Itisbestestablishedbyconcurrentcohortstudy.
Strengthofassociation
oItimplieshowstronglyexposureisassociatedwithdisease.
oItisdeterminedby
i)Relativerisk(InCohortstudy)IHavebeenexplained
ii)Oddsratio(Incase-controlstudy)
iii)Dose-responserelationship
Withincreasinglevelofexposureto
theriskfactor,onincreasingrisinincidenceisfound.
iv)Cessationofexposure(Reversibility)-->Removalofpossible
causereducesriskofdisease.
Consistencyofassociation
oTheassociationisconsistentiftheresultsarereplicatedwhen
studiesindifferentsettingsandbydifferentmethods.
oForexample,differentstudiesindifferentsettingshaveproved
smokingasoneofthecauseforlungcancer-->Smokinghas
consistentassociationwithlungcancer.

Coherenceofassociation
oThecausalassociationmustbecoherent(supportedby)with
relevantfacts.
oForexample:-
Thedeathratesduetolungcancerincreasingmorerapidlyin
femalescomparisontomales.
Therelevantfact--->Thisincreasingrateisduetomorerecent
adoptionofcigarretesmokingbywomen.

782.Resultsofanystudyarebetterdefinedin
?
a)Costeffectiveness
b)Costbenefit
c)Botharesame
d)None
CorrectAnswer-A
Ans.is'a'i.e.,Costeffectiveness[Ref.Park22"dlep.814]
Thestudyresultscanbeevaluatedbycost-benefitanalysisand
cost-effectivenessanalysis.
Incost-benefitanalysisallcostsandbenefitsareevaluatedinterms
ofmoney,i.e.economicbenefitsoftheprogramme/studyare
comparedwiththecostofprogramme.Themaindrawbackwiththis
techniqueisthatthebenefitsinthehealthfieldcanotalwaysbe
expressedinmonetaryterm.Forexamplebirthordeathprevented,
orillnessavoidedetc.Hencethescopeofapplyingthismethodis
rathervague.
Cost-effectiveanalysisismorepromisingtoolforapplicationinthe
healthfield.Itevaluateshowbesttospendagivenamountofmoney
toachievespecificgoals,i.e.benefitsareexpressedintermsof
resultsachieved,e.g.numberoflivessaved,orthenumberofdays
freefromdisease.

783.Forcalculationofincidencedenominator
istakenas?
a)Midyearpopulation
b)Populationatrisk
c)Totalnumberofcases
d)Totalnumberofdeaths
CorrectAnswer-B
Ans.is'b'i.e.,Populationatrisk
Denominatorforcalculatingincidenceispopulationatrisk

784.Whichoneofthefollowingisnota
specialincidencerate-
a)Attackrate
b)Secondaryattackrate
c)Hospitaladmissionrate
d)Standardizedmortalityrate
CorrectAnswer-D
Ans.is'd'i.e.,Standardizedmortalityrate
Specialincidencerates
i)Attackrate(caserate)
ii)Secondaryattackrate
iii)Hospitaladmissionrate


785.Attributableriskmeans
a)Fatalityofadisease
b)Diseaseriskratiobetweenexposedandnon-exposed
c)Riskdifferencebetweenexposedandnon-exposed
d)Communicabilityofadisease
CorrectAnswer-C
Ans.is'c'i.e.,Riskdifferencebetweenexposedandnon-exposed
-Attributablerisk(AR)isthedifferenceinincidenceratesofdisease
ordeathbetweenanexposedandnon-exposedgroup.
-Itisexpressedinpercentageandgivestheextenttowhichthe
diseasecanbeattributedtotheexposureinacohortstudy.

786.Proportionalmortalityrateis?
a)Numberofdeathduetoaparticularcause
b)Numberofdeathduringthatyear
c)Numberofdeathinonemonth
d)None
CorrectAnswer-A
Ans.is'a'i.e.,Numberofdeathduetoaparticularcause
Proportionalmortalityrate(ratio)
Proportionalmortalityratemeasurestheproportionoftotaldeath
duetospecificcauseorproportionofdeathsinaparticularage
group.
Itisdefinedas"numberofdeathsduetoaparticularcause(orin
specificagegroup)per100totaldeaths".
Itisthe'simplestmeasureofestimatingtheburdenofdisears'inthe
community.
Itisauseful'healthStatusindicator';indicatesmagnitudeof
preventablemortality.
Itisusedwhenpopulationdataisnotavailable.
Itdoesnotindicatetheriskofmembersofpopulationcontractingor
dyingfromthedisease.

787.Nameofmumpsvaccineis-
a)JerylLynn
b)Edmonshonzagreb
c)Schwatz
d)Moraten
CorrectAnswer-A
Ans.is'a'i.e.,Jeryllynn
oMumpsvaccinestrainisJeryllLynnstrain.
oOtherthreestrainsareofmeaslesvaccine.

788.Timebetweeninfectionandmaximum
infectivityisknownas?
a)Incubationperiod
b)Serialinterval
c)Generationtime
d)Communicableperiod
CorrectAnswer-C
Ans.is'c'i.e.,Generationtime
Generationtime
oIntervalbetweenreceiptofinfectionbyhostandmaximalinfectivity
ofthehost.
oGenerationtimeisroughlyequaltotheincubationperiod.
Ref:PARK22ndeditionpg96

789.
TransovariantransmissionIsseenin
whichinfection-
a)Plague
b)Guinea
c)Yellowfever
d)All
CorrectAnswer-C
Ans.is'c'i.e..Yellowfever

790.Transovariantransmissionofinfection
occursin-
a)Fleas
b)Ticks
c)Mosquitoes
d)bandc
CorrectAnswer-D
Ans.is'b'i.e.,Ticks;'c'i.e.,Mosquitoes

791.Chroniccarrierstateisnotseeninall
except?
a)Poliomyelitis
b)Measles
c)Malaria
d)Tetanus
CorrectAnswer-C
Ans.is'c'i.e.,Malaria

792.Followingareexamplesofhuman"dead
end"diseaseexcept-
a)Bubonicplague
b)Japaniesecephalitis
c)Hydatiddisease
d)Leishmaniasis
CorrectAnswer-D
Ans.is'd'i.e.,Leishmaniasis
Deadandhost
oAdeadendhostisaninfectedpersonfromwhichinfectious
agentsarenottransmittedtoothersusceptiblehostorfromwhicha
parasitecannotescapetocontinueitslifecycle.
oThediseasesinwhichhumanactsasdeadendhost,i.e.,dead
anddisease:?
1. Japaniesencephalitis3.
Trichinosis5.Bubonicplaque
2. Echynococcosis(hydatiddisease)4.Tetanus

793.Propagativecycleisseenin?
a)Plague
b)Filaria
c)Malaria
d)All
CorrectAnswer-A
Ans.is'a'i.e.,Plague[RefPark22"/ep.94]
PropagativePlaguebacilliinratfleas
Cyclo-developmentalMicorfilariainmosquito.
Cyclo-propagativeMalarialparasiteinmosquito.

794.Internationaldiseasesurveillanceisfor?
a)Hepatitis
b)Polio
c)TB
d)Leprosy
CorrectAnswer-B
Ans.is'b'i.e.,Polio

795.Massvaccinationisineffectivein-
a)Measles
b)Polio
c)Tetanus
d)None
CorrectAnswer-D
Ans.is'None'
oMassvaccinationisusedinallthethreegivenoptions.

796.Ifprevalenceofadiseaseincreases,
whatistrue?
a)PPVincreases
b)PPVdecreases
c)NoeffectonPPV
d)PPVmayincreaseordecrease
CorrectAnswer-A
Ans.is'a'i.e.,PPVincrease

797.ScreeningprocedureisbestforCaof-
a)Prostate
b)Colon
c)Gastric
d)None
CorrectAnswer-B
Ans.is'b'i.e.,Colon
oTwomostimportantcancers,whichcanbepreventedby
screeningarecarcinomaofcolonandcervix.

798.Highsensitive-
a)Lowfalsepositive
b)Lowfalsenegative
c)Lowtruenegative
d)Lowtruepositive
CorrectAnswer-B
Ans.is'b'i.e.,LowFalsenegative
TruepositiveisdirectlyrelatedtosensitivityoFalsenegativeis
inverselyrelatedtosensitivity
TruenegativeisdirectlyrelatedtospecificityoFalsepositiveis
inverselyrelatedtospecificity
oIfatesthashighsensitivity-->moretruepositive,lessfalse
negative,andalsomorefalsepositive.oIfatesthashighspecificity
-->
moretruenegative,lessfalsepositiveandalsomorefalse
negative.

799.Mostreliabletestforscreeningof
diabetesmellitus?
a)Randomsugar
b)Fastingsugar
c)Glucosetolerancetest
d)Urinesugar
CorrectAnswer-B
Ans.is'b'i.e.,Fastingsugar
"Thebestscreeningtestfordiabetes,thefastingplasmaglucose
(FPG),isalsoacomponentofgnostictesting"diabetesjournals.org
oThefastingplasmaglucosetestandthe75goralglucose
tolerancetest(GTT)arebothsuitabletestsforscreeningof
diabetes.
oHowever,theFPGtestispreferredinclinicalsettingsbecauseitis
easierandfastertoperform,moreconvenientandacceptableto
patients,andlessexpensive.
"Fastingplasmaglucoselevelisthemostreliableandconvenient
test"
Problemorientedpatientmanagement

800.Bloodscreeningisnotdonefor?
a)HIV
b)HBV
c)EBV
d)HCV
CorrectAnswer-C
Ans.is'c'i.e.,EBV
Screeningrecommendedinall
Selectivescreeninginsome
countries
countries
HIV
Malaria
HBV
HTLV
HCV
CMV
Syphilis
Chagasdisease

801.Toeradicatemeaslesthepercentageof
populationtobevaccinatedisat
least............%

a)70
b)80
c)85
d)95
CorrectAnswer-D
95

802.Inmeaselesvaccinecanbegiven
within-
a)3months
b)5months
c)7months
d)6months
CorrectAnswer-D
Ans.is'd'i.e.,6thmonths
oThebestageformeaslesvaccinationis9months.
oTheagecanbeloweredto6monthsifthereismeaslesoutbreak
inthecommunity.Forinfantsimmunizedbetween6monthsand9
monthsofage,aseconddoseshouldbeadministeredassoonas
possibleafterthechildreachestheageof9monthsprovidedthatat
least4weekshaveelapsedsincethelastdose.

803.Measlesvaccinationisgivenat-
a)9months
b)Atbirth
c)4weeks
d)8weeks
CorrectAnswer-A
Ans.is'a'i.e.,9months
oTheWHOexpandedprogrammeonimmunizationrecommends
immunizationat9monthsofage.
oNow,seconddoseofmeaslesisalsogivenat16-24monthsin
NationalImmunizationSchedule.

804.Incubationperiodofswineflu-
a)1-3days
b)2-3weeks
c)10-15days
d)5weeks
CorrectAnswer-A
Ans.is'a'i.e.,1-3days
Swineflu
oSwineflu,alsocalledswineinfluenza,orpiginfluenza,iscaused
byinfluenzavirus.
oItismainlyoccursinswines(pigs)andoccasionallytransmittedto
human.
oItisusuallycausedbyinfluenzavirustypeA(H1-N1).
oIncubationperiodisabout1-3daysandthesymptomsofswineflu
inhumansaresimilartomostinfluenzainfections,e.g.fever,cough,
rhinorrhea,fatigueandheadache.

805.AsperRNTCPguidelines,Multidrugresistance(MDR)TBisdefinedas
resistanceto:
a)Rifampicin
b)Rifampicinandisoniazide
c)Rifampicin,isoniazideandethambutol
d)Noneoftheabove
CorrectAnswer-B
ConfirmedMDR-TBcase:MDR-TBsuspectisonewhoissputum
culturepositiveandwhoseTBisduetoMycobacteriumtuberculosis
thatareresistantin-vitrotoatleastisoniazidandrifampicin(the
cultureandDSTresultbeingfromanRNTCPaccreditedlaboratory).
Ref:Park21stedition,page178.
http://health.bih.nic.in/Docs/Guidelines-DOTS-Plus.pdf.

806.WhyaTBpatientisrecommenda
regimenof4drugson1stvisit-
a)Toavoidemergenceofpersistors
b)Toavoidsideeffects
c)Tocureearly
d)None
CorrectAnswer-A
Ans.is'a'i.e.,Toavoidemergenceofpersistors
Twophasechemotheraphv
oTherearetwophaseoftreatmentoftuberculosis
i)Intensivephase
Thisisshortphaseintheearlycourseoftreatmentandlastsfor1-
3months.
Threeormoredrugsaregiventokillasmanybacilliaspossible,
whichpreventsemergenceofpersisters.
Theriskofrelapseisalsolessened.Continuationphase
Itisaimedatsterilizingthesmallernumberofdormantorpersisting
bacilli.

807.TBmultidrugregimenisgivento-
a)Preventresistance
b)Broadspectrum
c)Preventsideeffects
d)None
CorrectAnswer-A
Ans.is'a'i.e.,Preventresistance
oMultidrugtreatmentinTBisgivento?
i)Preventemergenceofpersisters
ii)Preventrelapse
iii)Preventemergenceofresistance
iv)shortenthedurationoftreatment

808.4drugsinAKTusedbecause-
a)Decreaseinresistancebymutation
b)Decreaseinresistancebyconjugation
c)Tocurediseaseearly
d)None
CorrectAnswer-A
Ans.is'a'i.e.,Decreaseinresistancebymutation
oMostcommonmethodforproductionofresistanceagainstATTis
mutation.
oMultidrugtreatmentpreventsemergenceofresistance.Thus,
multidrugtreatmentpreventemergenceofresistancedueto
mutation.

809.Chickenpoxvaccineis-
a)Livevaccine
b)Killedvaccine
c)Conjugatedvaccine
d)None
CorrectAnswer-A
Ans.is'a'i.e.,Livevaccine
Preventionofchickenpox
oForpreventionofchickenpoxfollowingareused?
i)Varicellazosterimmunoglobulin(VZIG)
VZIGisgivenwithin72hoursofexposureinexposed
susceptibleindividuals.
Itisgivenintramuscularwitharepeatdosein3weeks.
BecauseVZIGbindtovaricellavaccine,thetwoshouldnot
begivenconcomitantly.
ii)Vaccine
Varicellavaccineisliveattenuatedvaccineandis
recommendedforchildrenbetween12-18monthsofage.
Efficacyofvaccineis90-95%anddurationofimmunityis
probably10years.
Twodosesarerecommendedinpersonolderthan12yearsofage.

810.AmountofdiphtheriatoxoidinDTis?
a)5Lf
b)10Lf
c)15Lf
d)25Lf
CorrectAnswer-D
Ans.is'd'i.e.,25Lf[Ref.Park22"dlep.153]
Oridinary(Pediatric)DPT/DTvaccinescontain25Lfofdiphtheria
toxin.
Forimmunizingadultsandolderchildren(>12years),dTisused
whichcontains2Lfofdiphtheriatoxin.
DPTvaccine
Itcontainscomponentsforimmunizationagainstthreediseases,i.e.,
toxoidofdiphtheriaandtetanus,andkilledB.pertussis.Pertussis
componentenhancesthepotencyofdiphtheriatoxoid.
Aluminiumsalts(hydroxideorphosphate)areusedasadjuvantto
increaseimmunogenicity.Thiomersalisusedaspreservative.
UsualstoragetemperatureforDPTvaccineincoldchainis+2to
+8?C,storedinrefrigerator.Itshouldneverbestoredindeepfreezer
(shouldnotbefrozen)andifitgetsfrozenaccidently,vaccineshould
bediscarded.
Exposuretosunlightshouldbeavoided.Openvialswhichhavenot
beenfullyusedshouldbediscardedattheendofsession.
Vaccineisgivenbyintramuscularrouteinthemiddlethirdof
anterolateralaspectofthigh.
OptimumagetostartDPTvaccinationis6weeksafterbirth.
Totalthreedosesaregiveninprimarlyimmunizationwithaninterval
of4weeksbetweenthreedoses.Firstboosterisgivenat16-24

monthswithsecondboosterat5-6years.

811.Bivalentmeningococcalvaccineis?
a)AY
b)AC
c)Cy
d)AW-I35
CorrectAnswer-B
Ans.is'b'i.e.,AC
Twotypeofmeningococcalvaccinedevelop
Unconjugatedpolysaccharidevaccine.
ConjugatedgroupCvaccine.
Polysaccharidevaccines
Internationallymarketedmeningococcalpolysaccharidevaccines
areoBivalent(AandC),
Trivalent(A,CandW-135)
Tetravalent(A,C,YandW-135).
Thevaccinesarepurified,heat-stable,lyophilizedcapsular
polysaccharidesfrommeningococcioftherespectiveserogroups.
Aprotectiveantibodyresponseoccurswithin10daysofvaccination.
Inschoolchildrenandadults,onedoseofthesepolysaccharide
vaccinesappearstoprovideprotectionforatleast3years,butin
childrenunder4yearsofagethelevelsofspecificantibodies
declinerapidlyafter2-3years.

812.AccordingtoEVINCEfastbreathingin5
mthchildisdefinedas-
a)>30/min
b)40
c)50
d)60
CorrectAnswer-C
Ans.is'c'i.e.,50

813.1955HepatitisoutbreakinDelhi?
a)A
b)B
c)C
d)E
CorrectAnswer-D
Ans.is'd'i.e.,E[RefInternet]
HepatitisEwasfirstdocumentedinNewDelhiin1955when29000
casesoficterichepatitisoccured.

814.Whichischoleravaccine-
a)Ty21A
b)HGD-103
c)WC-rBS
d)None
CorrectAnswer-C
Ans.is'c'i.e.,WC-rBS
CholeraVaccine
Killedvaccines
Dukoral(WC-rBS)
SancholandmORCVAX

815.Trivalentoralpoliovaccinecontains,
type3virus-
a)100,000TCID50
b)200,000TCID50
c)300,000TCID50
d)400,000TCID50
CorrectAnswer-C
Ans.is'c'i.e.,300,000TCID50
Oral(sabin)poliovaccine
Itcontainsliveattenuatedviruss(type1,2and3)growninprimary
monkeykidneyorhumandiploidcellculture.
oThevaccinecontains:-
i)Over300,000TCID50oftype1poliovirus
ii)Over100,000TCID50oftype2poliovirus
iii)Over300,000TCID50oftype3poliovirus
oDose2drop(0.1
ml)
ScheduleinNationalImmunizationProgrammeofIndia.
DoseAge
OPV-0(Zerodose)Atbirth
OPV-16weeks
OPV-210weeks
OPV-314weeks
OPV-B(Boosterdose)16-24months
oDevelopmentofimmunity-->OPVinduceslocalintestinal
immunitybyproductionofsecretoryIgAaswellashumoralimmunity
byinducingproductionofserumantibodies(IgG).So,itgives
protectionfromparalysisandalsopreventsinfectionofthegutby
wildviruses.


816.ORScontains75mEq/l-
a)Sodium
b)Potassium
c)Glucose
d)Chloride
CorrectAnswer-A
Ans.is'a'i.e.,Sodium
Glucose
shouldatleastequalthatofsodiumbutshouldnotexceed111mmol/l
Sodium
shouldbewithintherangeof60-90mEq/l
Potassium shouldbewithintherangeof15-25mEq/l
Citrate

shouldbewithintherangeof8-12mmol/l
Chloride
shouldbewithintherangeof50-80mEq/l

817.ORSnewosmolarityis-
a)270
b)245
c)290
d)310
CorrectAnswer-B
Ans.is'b'i.e.,245
oOsmolarityofnewORS(reducedosmolarityORS)is245
mmol/litre.

818.Prophaylaxisformalarianotused-
a)Doxycycline
b)Artesunate
c)Chloroquine
d)Mefloquine
CorrectAnswer-B
Ans.is'b'i.e.,Artesunate
oDrugsusedforprophylaxisofmalariaarechloroquine,proguanil,
doxycycline,mefloquineandhydroxychloroquine.

819.Yellowfeveraedesagyptiindexshould
be?
a)<1%
b)<5%
c)<10%
d)<20%
CorrectAnswer-A
Ans.is'a'i.e.,<1%

820.WhichisthemainvectorofDengue?
a)A.aegypti
b)Aschleri
c)Culex
d)Anopheles
CorrectAnswer-A
Ans.is'a'i.e.,A.aegypti[RefPark22"/ep.225]
Denguefeveriscausedbyarboviruses(atleast4serotypeshave
beenrecognized)
ItistransmittedbyAedes(Aedesaegyptiisthemainvector).
Thereservoirofinfectionisbothmanandmosquito.
ThetransmissioncycleisMan-mosquito-man
Denguefeveroccursbothepidemicallyandendemically.Epidemics
startsinrainyseasonandareusuallyexplosive.
Aedesmosquitobecomesinfectivebyfeedingonapatientfromthe
daybeforeonsettothe5thdayofillness

821.Chemoprohylaxisofchloroquine
includes-
a)300mgBD/week
b)600mgOD/week
c)600mg/week
d)300mgOD/week
CorrectAnswer-D
Ans.is`d'i.e.,300mgOD/week
DrugDoseforchemoprophylaxis
Chloroquine300mg(3tabletsof100mgor2tabletsof
150mg)onceaweekor100mg/dayfor6dayeveryweek
Proguanil400mgperday(2tabletsof200mg)
Mefloquine250mgonceaweek
Doxycycline100mgperday.

822.Epidemictyphuscause&vector-
a)Rickettessiaeprowazki&Louse
b)R.typhi&mite
c)R.conorii&tick
d)R.akari&mite
CorrectAnswer-A
Ans.is'a'i.e.,Rickettessiaeprowazki&Louse

823.Vectorofscrubtyphusinman?
a)O.tsutsugamushi
b)Leptotrombidiumdeliense
c)Lice
d)Pediculoushumanus
CorrectAnswer-B
Ans.is'b'i.e.,Leptotrombidiumdeliense
oForscrubtyphus:-
CausativeagentR.tsutsugamushi
Vectortrombiculidmite(LeptotrombidiumdelineseandL.akamushi)


824."Multibacillary"isaspectrumofdisease,
seenin-
a)Leprosy
b)TB
c)Tetanus
d)Trachoma
CorrectAnswer-A
Ans.is'a'i.e.,Leprosy

825. 2yrsdurationintermsofleprosyiswith
regardto-
a)Rxofpaucibacillaryleprosy
b)Rxofmultibacillaryleprosy
c)PostRxsurveillanceofpaucibacillaryleprosy
d)PostRxsurveillanceofmultibacillaryleprosy
CorrectAnswer-C
Ans.is'c'i.e.,PostRxsurveillanceofpaucibacillaryleprosy

826.Whichvirusisusedtoproducerabies
vaccine?
a)Wild
b)Street
c)Fixed
d)LiveAttenuated
CorrectAnswer-C
Ans.is'c'i.e.,Fixed
oTherearetwostrainsofrabiesvirus:?
i)Streetvirus-
Thisthevirus,responsiblefornaturalrabiesand
isisolatedfromnaturalhumanoranimalinfection.
ii)Fixedvirus-Itisisolatedafterseveralserialintracerebral
passageinrabbit.Itisusedtopreparerabiesvaccine.

827.SecondmostcommonSTDafter
gonococcus?
a)Chylamydia
b)HSV
c)HIV
d)Syphilis
CorrectAnswer-A
Ans.is'a'i.e.,Chlamydia[RefPark2151/ep.304]
FiveclassicalSTDsaresyphilis(T.pallidum),gonorrhoea(N.
gonorrhoeae),chanchroid(H.ducreyi),lymphogranulomavenerum
(chlamydiatrachomatis),anddonovanosis(calymmatobacterium
granulomatosis).
MostcommonSTDinIndiaisherpesgenitalis(20%)followedby
chancroid(11%),viralwarts/HPV(11%),syphilis(11%)and
gonorrhoea(9%).
Overall(inworld)mostcommonSTDischlamydiafollowedby
gonorrhoea.

828.ChandlersindexforHookworm,whenit
ishealthproblem?
a)>300
b)>200
c)>100
d)>50
CorrectAnswer-A
Ans.is'a'i.e.,>300[RefPark21"/ep.221]
Below200Hookworminfectionisnotmuchofsignificance
200-250Potentialdanger
250-300--Minorpublichealthproblem
Above300Importantpublichealthproblem

829.Whichofthefollowingisazoonotic
disease?
a)Hydatidcyst
b)Malaria
c)Filariasis
d)denguefever
CorrectAnswer-A
Ans.is'a'i.e.,Hydatidcyst
Importantzoonoses
I.Bacterial?4Anthrax,Brucellosis,Ornithoses,Q-fever,
Leptospirosis,TB,Plague,Tularemia,Salmonellosis.
2.Viral-->Cowpox,Monkeypox,Easternequine,encephalitis,
Rossriverfever,Yellowfever,Japaneseencephalitis,Lassafever,
Rabies.
3.Protozoal--->Leishmaniasis,Toxoplasmosis,Trypanosomiasis,
Babesiosis.
4.Helminthic---->Clonorchiasis,Fasciolopsis,Schistosomiasis,
Echinococus(hydatiddisease),Taeniasis,Trichinellosis.

830.HIVpostexposureprophylaxisshouldbe
startedwithin?
a)1-2hrs
b)14hrs
c)18hrs
d)72hrs
CorrectAnswer-A
Ans.is'a'i.e.,1-2hrs
Anti-retroviraldrugforpost-exposureprophylaxisshouldbeinitiated
assoonaspossibleaftertheexposurewithinthefirstfewhoursand
nolaterthan72hours.
So,thebestanswerhereis1-2hours(firstfewhours).

831.HIVprevalencecanbeassessedby-
a)Sentinelsurveillance
b)Active
c)Passive
d)Register
CorrectAnswer-A
Ans.is'a'i.e.,Sentinelsurveillance
oSentinelsurveillanceinIndiaisdoneinnationalAIDScontrol
programme.


832.Mostcommoncancerinmenis
a)Bladdercancer
b)Colorectalcancer
c)Prostatecancer
d)Oralcancer
CorrectAnswer-C
Answer-C.Prostatecancer
CancersinmalesinIndia:Lip/oralcavity>Prostate>Colorectum>
Pharynx(otherthannssopharynx)>Larynx
Lungcancerwasthemostcommoncancerinmenworldwide,
contributing15.5%ofthetotalnumberofnewcasesdiagnosedin
2018.
Thetopthree?lung,prostateandcolorectalcancers?contributed
44.4%ofallcancers(excludingnon-melanomaskincancer).

833.Whichindexofobesitydoesnotinclude
height?
a)BMI
b)Ponderal'sindex
c)Broca'sindex
d)Corpulenceindex
CorrectAnswer-D
Ans.isi.e.,D.Corpulenceindex[RefPark22ndlep.369,370]
Differentindicesusetodetermineobesityare
i)Bodymassindex(Qetelet'sindex)
Itisusedinternationallyasreferencestandardforassessingthe
prevalenceofobesity.
Itisdependentbothonheightandweight(hasbeenexplained
earlier).
ii)Ponderalindex
Itisdependentbothonheightandweight.
Itisdefinedasheight(cm)dividedbycuberootofweight(kg).
iii)Brocaindex?dealweight=Height(cms)-100
iv)Corpulenceindex?Itisdependentonlyonweight(height
independent.Itshouldbe1.2.


834.Whichofthefollowingstudieshave
givencoronaryriskfactor?
a)Framingham
b)Stanfordstudy
c)NorthKerala
d)MONICA
CorrectAnswer-A
Ans.is'a'i.e.,Framingham[RefPark22"diep.342]
Optiona,b&callthreeareriskfactorinterventiontrials.
However,optionaisthebestanswer:?
"Since1951,oneofthebestknownlargeprospectivestudies,the
Framinghamstudy,hasplayedamajorroleinestablishingthe
natureofCHDriskfactorsandtheirrelativeimportance.
Standford-threecommunitystudy.Todeterminewhether
communityhealtheducationcanreducetheriskofcardiovascular
disease,thisstudywasundertakenin1972.
TheNorthKereliaProject:Thisisamultipleriskfactor
intervention
trialestablishthenatureofCHDriskfactorsandtheir
relativeimportance.Largestprospectivestudywhichstartedsince
1951.

835.Nicotineresponsiblefororalcanceris?
a)10%
b)40%
c)90%
d)60%
CorrectAnswer-C
Ans.is'c'i.e.,90%[RefPark22"/ep.358]
Approximately90%oforalcancersinSouthEastAsiaarelinkedto
tobaccochewingandtobaccosmoking"--Park

836.Allofthefollowingaregivenglobal
prominenceintheVISION2020goal,
expect:

a)Refractiveerrors
b)Cataract
c)Trachoma
d)Glaucoma
CorrectAnswer-D
Ans.Glaucoma

837.ForAsianpopulationB.M.I.trueis?
a)Differentfrominternationalvaluestodefineobesity
b)Increasedmorbidityatlowervalues
c)Increasedmorbidityathighervalues
d)Obesityis>25kg/m2
CorrectAnswer-B
Ans.is'b'i.e.,Increasedmorbidityatlower
values
[RefWHOexpertconsultation]
AWHOexpertconsulationaddressedthedebateabout
interpretationofrecommendedbody-massindex(BMI)cut-offpoints
fordeterminingoverweightandobesityinAsianpopulations,and
consideredwhetherpopulationspecificcut-offpointsforBMIare
necessary.
TheyreviewedscientificevidencethatsuggeststhatAsian
populationshavedifferentassociationsbetweenBMI,percentageof
bodyfatandhealthrisksthandoEuropeanpopulation.
TheconsultationconcludedthattheproportionofAsianpopulation
withahighriskoftype2diabetesandcardiovasculardiseaseis
substantialatBMI,lowerthanexistingWHOcut-offpointsfor
overweight(25mg/kg2).AndAsiansgenerallyhaveahigher
percentageofbodyfatincomparisontowhitepeopleofthatsame
age,sexandBMI.
However,availabledatadonotnecessarilyindicateaclearBMIcut-
offpartforallAsiansforoverweightorobesity.
Thecut-offpointforobservedriskvariesfrom22kg/m2to25
kg/m2indifferentAsianpopulation,forhighriskitvariesfrom26
kg/m2to31kg/m2.
FormanyAsianpopulations,additiondtriggerpointsforpublic

healthactionwereidentifiedas-
1. 18.5kg/m24Underweight
2. 18.5-23kg/m2Increasedbutacceptablerisk
3. 23-27.5kg/m2Increasedrisk
4. 27.5kg/m2Higherhighrisk

838.Whichofthefollowingstatementsabout
'LateExpandingPhaseofDemographic
Cycle'isTRUE?

a)BirthRateislowerthanDeathRate
b)HighDeathRateandHighBirthRate
c)DeathRatedeclinesmorethanBirthRate
d)DeathRatebeginstodeclinewhileBirthRateremains
unchanged
CorrectAnswer-C
Inlateexpandingphaseofdemographiccycledeathratesdeclines
fasterthanthebirthrateandthereisasteadydecreaseinthe
demographicgap.
Inthisstatepopulationgrowatasteadilydecreasingrate.
Mostofthedevelopingcountriesarenowatearlyexpandingand
lateexpandingstagesofdemographiccycle.
Ref:Park'sTextbookofPreventiveandSocialMedicineByK.Park,
19thEdition,Page379;FoundationsofCommunityMedicineBy
Dhaar,2ndEdition,Page667

839.Nottrueaboutpopulationpyramid?
a)Indiahasbroadbase
b)Indiahasnarrowbase
c)Indiahasnarrowapex
d)Developingcountrieshavebulgeinthecenter
CorrectAnswer-B
Ans.is'b'i.e.,Indiahasnarrowbase
Incountrieswithhighbirthratesasours,populationpyramidis
broadbasedconical(narrowapex)becauseofhighbirthrateand
taperingofpopulationwithincreaseinage.
Thepyramidofdevelopingcountry(e.g.India)hasabroadbaseand
ataperingtop.
Indevelopedcountries,thepyramidgenerallyshowsabulgeinthe
middleandhasanarrowerbase(asinfiguregiveninthequestion).

840.Netreproductionrateis?
a)Numberoflivebirthsper1000mid-yearpopulation
b)Numberoflivebirthper1000womenofchildbearingage
c)Numberofdaughtersanewborngirlwillhaveduringlifetime
d)Noneoftheabove
CorrectAnswer-C
Ans.is'c'i.e.,Numberofdaughtersanewborngirlwillhave
duringlifetime
Netreproductionrate
Netreproductiverateisdefinedas
"Theno.ofdaughtersanewborngirlwillbearduringherlifetime
assumingfixedagespecificfertilityandmortalityrates"
Itistheonlyfertilityrelatedstatisticswhichalsotakesmortalityrates
intoconsideration.
NRRofoneisequivalenttoattainingapproximatelythetwochild
norms.
GoalofNRR=1canbeachievedonlyifatleast60%*oftheeligible
couples
areeffectivelypracticingfamilyplanning.

841.Homeswherechildrenareplacedunder
thecareofdoctorsandpsychiatristsare
called-

a)Fosterhomes
b)Borstals
c)Remandhomes
d)Childguidanceclinics
CorrectAnswer-C
Ans.is'c'i.e.,Remandhomes
Childplacement:
oOrphanages:Forchildrenwhohavenohomeorcannotbetaken
careofbytheirparents.
oFosterHomes:Severaltypesoffacilitiesforrearingchildrenother
thaninnaturalfamilies.
oAdoption:Legaladoptionconfersuponchildandtheadoptive
parents,rightsandresponsibilitiessimilartothatofnaturalparents.
oRemandHomes:Childisplacedunderthecareofdoctors,
psychiatristsandothertrainedpersonneltoimprovethementaland
physicalwellbeingofthechild.

842.MeanbirthweightinIndia-
a)2.0-2.4kg
b)2.4-2.5kg
c)2.5-2.9kg
d)>3.0kg
CorrectAnswer-C
Ans.is'c'i.e.,2.5-2.9kg
MeanbirthweightinDifferentpartsoftheworldRegion
Meanbirthweight
NorthAmerica,WesternEurope,Australia
3.5-3.6kg
EasternEurope3.1-
3.3kg
AfricaandEastAsia2.9-
3.1kg
SouthAsiancountries2.7kg
oInIndia,meanbirthweightrangesfrom24.9
kgto28.8kg.

843.Highestfundingforreproductivehealth
isby-
a)UNFPA
b)UNICEF
c)ILO
d)None
CorrectAnswer-A
Ans.is'a'i.e.,UNFPA[RefMaternalhealthfifthreportsession
2007-2008]
UNFPAworkswitharangeofpartnerstopromotereproductive
healthinIndia.
Itpoolsasignificantproportionofitscountryprogrammeresources
inthereproductiveandchildhealthII(RCHII)programme,aimedat
reducingmaternalmortalityandchildmortality,aswellasprovision
ofrangeofcontraceptiveservices.
UNFPAalsodeliverstechnicalassistanceforeffective
implementationofRCH-IIprogrammeatthenationalaswellasstate
levelparticularlyinthestateofRajasthan,M.P.Maharashtra,Orrisa
andBihar.

844.Mostcommonmethodofsterilisation
practisedinIndia?
a)Femalesterilization
b)Malesterilization
c)Both
d)None
CorrectAnswer-A
Ans.is'a'i.e.,Femalesterilization[RefPark22"/ep.454]
During2010-2011
Totalsterilization-5.0million
Femalesterilization(tubectomy)4.78million
Malesterilization(vasectomy)0.219million

845.Lifespanofcut380a-
a)10yrs
b)20yrs
c)1yrs
d)None
CorrectAnswer-A
Ans.is'a'i.e.,10years

846.WhichofthefollowinghasLEAST
pregnancyfailurerate:
a)OCP
b)IUCD
c)Diaphragm
d)Condom
CorrectAnswer-A
OCP

847.Whichistheleastcommoncauseamong
theseofinfantmortalityinIndia-
a)Infections
b)Prematurity
c)Birthinjuries
d)Congenitalmalformations
CorrectAnswer-C
Ans.is'c'i.e.,Birthinjuries

848.MCHcareisassessedby-
a)Deathrate
b)Birthrate
c)Maternalmortalityrate
d)Anemiainmother
CorrectAnswer-C
Ans.is'c'i.e.,Maternalmortalityrate
ImportantMCHindicators
1. Maternalmortalityrate
2. Mortalityininfancyandchildhood
a)Perinatalmortalityrate
b)Neonatalmortalityrate
c)Post-neonatalmortalityrate
d)Infantmortalityrate
e)1-4yearmortilityrate
f)Under5mortalityrate
g)Childsurvivalrate

849.Innormaldelivery,breastfeedingshould
bestarted?
a)6hourafterdelivary
b)2hourafterdelivary
c)4hourafterdelivary
d)None
CorrectAnswer-D
AnsisdNone
oBreastfeedingshouldbeinitiatedwithin30min.ofanormal
vaginaldelivery.
oBreastfeedingshouldbeinitiatedwithin4hrsofdeliveryby
caesariansection.


850.Bestindicatorofavailability,utilisation&
effectivenessofhealthservices-
a)IMR
b)MMR
c)HospitalbedOCR
d)DALY
CorrectAnswer-A
Ans.is'a'i.e.,IMR

851.Perinatalmortalityincludesdeaths-
a)After28weeksofgestation
b)First7daysafterbirth
c)Both
d)Fromperiodofviability
CorrectAnswer-C
Ans.is'c'i.e.,Both

852.Colostrumshasincomparedtonormal
milk?
a)DecreasedK
b)DecreasedNa
c)Increasedproteins
d)Increasedcalories
CorrectAnswer-C
Ans.is'c'i.e.,Increasedproteins

853.Maternalmortalityismaximuminwhich
period
a)Antepartum
b)Peripartum
c)Postpartum
d)None
CorrectAnswer-B
Ans.is'b'i.e.,Peripartum
oMaternaldeathmostlyoccurfromthethirdtrimestertothefirst
weekafterbirth.
Studiesshowthatmortalityrisksformothersare
particularlyelevatedinthefirsttwodaysafterbirth.

854.Energyrequirementinlatepregnancy-
a)2500cal
b)1400cal
c)1000cal
d)1000cal
CorrectAnswer-A
Ans.is'a'i.e.,2500Cal

855.'VitaminArequirementininfantis-
a)350g
b)600g
c)800g
d)1000g
CorrectAnswer-A
Ans.is'a'i.e.,350g

856.Adultnon-pregnantfemalerequires,
Calciumperday-
a)400mg
b)600mg
c)800mg
d)1000mg
CorrectAnswer-B
Ans.is'b'i.e.,600mg
oAdultnon-pregnantfemalerequires600mgcalciumperday.

857.Calciumrequirementabovethenormal
duringthefirstsixmonthoflactationis-
a)400Mgiday
b)550mg/day
c)600mg/day
d)750mg/day
CorrectAnswer-C
Ans.is'c'i.e.,600mg

858.IodineRDAis-
a)300microgram
b)500microgram
c)150microgram
d)50microgram
CorrectAnswer-C
Ans.is`c"i.e.,150microgratn
oTheRDAofiodineforadultsis150microgram.

859.Iodinecomesiniodinesalt.Requirmen
tatproductionandconsumerlevel
respectively-

a)20&10PPNI
b)30&10PPM
c)30&15PPM
d)30&20PPM
CorrectAnswer-C
Ans.is"c'i.e.,30&15PPM

860.Bitot'sspotprevalenceaspublichealth
problem-
a)>1%
b)>2%
c)>0.5%
d)None
CorrectAnswer-C
Ans.isi.e.,c.>0.5%[RefPark22"d/ep.571]
Nightblindness
Bitot'sspots
>0.5%
Cornealxerosis/cornealulceration/keratomalacia >0.01%
Cornealulcer
>0.05%
Serumretinol(lessthan10mcg/dl)
>5%

861.JowarisPellogerogenicduetoexcess
of-
a)Leucine
b)Lysine
c)Tryptophan
d)Methioninc
CorrectAnswer-A
Ans.is'a'i.e.,Leucine
oSimilartomaize,JowaralsoContainsexcessofleucine.
oLeucineinterfereswithconversionoftryptophantoniacin.
oPellagrahasbeenreportedinIndiainTelanganaareaofAndhra
Pradesh
becauseofSower(Sorghumvulgare)consumption.

862.Toxinresponsibleforepidemicdropsy:
AIIMS07;UP09;PGI11
a)BOAA
b)Aflatoxin
c)Sanguinarine
d)Pyrrozolidine
CorrectAnswer-C
Ans.Sanguinarine

863.Maximumlinolenicacidispresentin?
a)Coconutoil
b)Soyabeanoil
c)Groundnutoil
d)Saffloweroil
CorrectAnswer-D
Ans.is'd'i.e.,Saffloweroil[RefPark22"d/ep.566]
Therichestsourceoflinoleicacidissaffloweroil.Sourcesoflinoleic
acidindecreasingorderaresaffloweroil>cornoil>Sunfloweroil>
Soyabeanoil>Sesameoil>groundnutoil.

864.Thehearttotheactivatedsludge
processis?
a)Primarysedimentationtank
b)Sludgedigester
c)Aerationtank
d)Finalsettlingtank
CorrectAnswer-C
Ans.is'c'i.e.,Aerationtank
oTheheartoftheactivatedsludgeprocessisaerationtank.


865.Thicknessofleadaprontoprevent
radiation:
a)1mm
b)3mm
c)0.5mm
d)7mm
CorrectAnswer-C
Ans.C.0.5mm
"Itisrecommendedthatforgeneralpurposeradiographytheminimal
thicknessofleadequivalentintheprotectiveapparelshouldbe
0.5mm."
-TextbookofRadiologyPhysicsp.39
Leadapronof0.5mmthicknessreduceintensityofscatteredX-rays
byover90%.

866.DalyrequirmentofvitaminK?
a)3mg/kg
b)0.3mg/kg
c)0.03mg/kg
d)1mg/kg
CorrectAnswer-C
Ans.is'c'i.e.,0.03mg/kg[Ref.Park22"d/ep.572]
VitaminA600mcgretinol
VitaminB7(Thiamine)-->0.5mgper1000Kcalofenergyintake
VitaminB,(Riboflavin)0.6mgper1000Kcalofenergyintake
VitaminB,(Niacin)6.0mgper1000Kcalofenergyintake
Vitamin135(PantothenicAcid)10mg
VitaminB6(Pyridoxine)2mg
VitaminB,(FolicAcid)200mcg
VitaminB12(Cobalamin)1mcg
VitaminD100IU(2.5mcgcalciferol)
VitaminE(Tocopherol)0.8mgpergmofessentialfattyacids
VitaminK0.03mgperkg

867.Avidinhasaffinityfor?
a)Folicacid
b)Thiamine
c)Biotin
d)Riboflavin
CorrectAnswer-C
Ans.is'c'i.e.,Biotin[Ref.Harper29iVep.539]
Peoplewhoeatabnormallylargeamountofuncookedeggwhite
mayhavebiotindeficiencybecauseitcontainsavidin,aproteinthat
bindsbiotinandpreventsitsabsorption

868.Argemainoilcontaminationofmustard
oilcanbedetectedby?
a)Phosphatasetest
b)Nitricacidtest
c)Coliformcunel
d)Methylenebluetest
CorrectAnswer-B
Ans.is'b'i.e.,Nitricacidtest[Ref.Park22"d/ep.610]
DetectionofArgemoneoil:
1. Nitricacidtest:brownorangeredcolour/ringshowsitispresent
minimumconcentrationofArgemoneoilrequiredisabout0.2%.
2. Paperchromatographytest-Themostsensitivetest

869.Iodinedeficiencycontrolprogramme?
a)Healtheducation
b)Watertesting
c)Fortificationofsalt
d)None
CorrectAnswer-C
Ans.is'c'i.e.,Fortificationofsalt
Iodizedsalt(saltfortifiedwithiodine)ismosteconomical,
convenientandeffectivemeansofmassprophylaxisin
endemicarea.


870.Amountofproteinsinhumanmilk(in
gms):
September2007

a)1.1
b)2.2
c)3.3
d)4.4
CorrectAnswer-A
Ans.A:1.1
Each100gramsofbreastmilkyieldsapproximately:
65Kilocalories
88gwater
7.4gcarbohydrates(primarilylactose)
3.4gfat
1.1gprotein

871.Itwastecontaintoxicsubstances,
organicloadismeasuredby?
a)Biologicaloxygendemand
b)Chemicaloxygendemand
c)Suspendedsolid
d)None
CorrectAnswer-B
Ans.is'b'i.e.,Chemicaloxygendemand[RefPark22"d/ep.708]
"Thestrengthofsewageisexpressedintermsof:
1. Biochemicaloxygendemand
2. Chemicaloxygendemands
3. Suspendedsolidsdemand
BiochemicalOxygendemand
Itisthemostimportanttestdoneonsewage.Itisdefinedasthe
amountofoxygenabsorbedbyasampleofsewageduringa
specifiedperiod,generally5days,ataspecifiedtemperature,
generally20deg.C,fortheaerobicdestructionoruseoforganic
matterbylivingorganisms.BODvaluerangesfromaboutImgper
litrefornaturalwaterstoabout300mgperlitreforuntreated
domesticsewage.IftheBODis300mg/1andabove,sewageis
saidtobestrong;ifitis100mg/I,itissaidtobeweak."
Chemicaloxygendemand(COD)
TheCODmeasurestheoxygenequivalentofthatportionofthe
organicmatterinasamplewhichissusceptibletooxidationbya
strongchemicaloxidiser.Ifwastescontaintoxicsubsances,COD
maybetheonlymethodfordeterminingtheorganicload.
Suspendedsolids
Iftheamountofsuspendedsolidsis100mg/1,thesewageissaidto

beweak,iftheamountis500mg/dlthesewageissaidtobestrong

872.Fenthionis?
a)Spacespray
b)Residualspray
c)Stomachpoison
d)Fumgant
CorrectAnswer-A
Ans.is'a'i.e.,Spacespray[RefPark22"dlep.727]
Residualsprapy
Inresidualspray,sprayingofhouseswithresidualinsectisidesis
done.
Residualinsectisidesremainsactiveoverextendedperiodsi.e.,they
haveresidualactionevenafterthetimeofspray.
CommonlyusedresidualinsectisidesareMalathion,DDT,
Lindane,propoxure(OMS-33)
Spacespray
Spacespraysarethosewheretheinsecticidalformationissprayed
intotheatmosphereintheformofamistorfogtokillinsect.
Actionisshortlivedandtemporarysincethereisno.residualaction.
ThemostcommonlyusedspacesprayinsecticideisPyrethrum.
Nowresidualinsectisidesarealsobeingusedasresidualsprayby
newequipmentforultra-lowvolume(ULV)spacespraying.
Melathionandfenthionareusedforthispurpose.

873.Notsafedisposalbutgelidforsoil
building-
a)Incineration
b)Controlledtipping
c)Composting
d)None
CorrectAnswer-C
Ans.is'c'i.e.,Composting
Composting
C'ompostigisamethodofcombineddisposalofrefuseandnightsoil
orsluge.
oItisprocessofnaturewherebyorgaicmatterbreaksdownunder
bacterialactionresultingintheformationof
relativelystablehumus-likematerial,calledthecompostwhichhas
considerablemanurialvalueforthesoil.
oCompostcontainsnoorfewdiseaseproducingorganismsandisa
goodsoilbuildercontainingsmallamountsof
themajorplantnutrientssuchasnitratesandphosphates.
oTherearefollowingtwomethodsofcomposting.
i)Bangaloremethod(Anaerobicmethod)
ii)Mechanicalcomposting(Aaerobicmethod).


874.Inwinter,watervapoursandpollutants
comestolieinthelowermostlayerof
atmosphereby-

a)Acidrain
b)Greenhouseeffect
c)Temperatureinversion
d)None
CorrectAnswer-C
Ans.is`c'i.e.,Temperatureinversion
Atemperatureinversionisathinlayeroftheatmospherewherethe
normaldecreaseintemperaturewithheightswitheestothe
temperatureincreasingwithheight.
Aninversioncanleadtopollutionsuchassmogbeingtrappedclose
tothegrand
(lowerlayersofatmosphere).
Temperatureinversionmayoccurduringthepassageofacoldfront
orresultfromtheinvasionofseaairbyacooleronshorebreeze.
oOvernightradiativecoolingofsurfaceairoftenresultsina
nocturnaltemperatureinversionthatisdissipatedaftersunriseby
thewarmingofairneartheground.

875.Vagabonddiseaseistransmittedby?
a)Louse
b)Mite
c)Tick
d)BlackFly
CorrectAnswer-A
Ans.is'a'i.e.,Louse
Vagabond'sdiseaseispediculosiscorporis,causedbybodylouse.

876.Sourceofenvironmentalradiationareall
except?
a)CO
b)Radium
c)Uranium
d)Radon
CorrectAnswer-A
Ans.is'a'i.e.,CO[RefPark22'/ep.690]
Environmentalratiationareoftwotypes?
1. Terrestrial
2. Atmospheric
Terrestrialradiation
Radioactiveelementssuchasthorium,Uranium,radiumandan
isotopofpotassium(K40)arepresentinman'sinvironment,e.g.,
soil,rocks,boiling.
Itisestimatedthatmanderivesabout50mradperyearfrom
terrestrialradiation.
Areaexists(KeralainIndia)wheretherockformationcontaining
uranium,itcanbeashighas2000,mradayear.
Atmosphericradiation
Theseareradioactivegasesradonandthoron

877.Theamountofbleachingpowder
necessarytodisinfectcholericstools,is
-

a)50gm/lit
b)75gm/lit
c)90gm/lit
d)100gm/lit
CorrectAnswer-A
Ans.is'a'i.e.,50gm/lit

878.Socialpsychologyis?
a)Humanrelationships&behaviour
b)Psychologyofindividualsinsociety
c)Culturalhistoryofman
d)None
CorrectAnswer-B
Ans.is'b'i.e.,Psychologyofindividualsinsociety[Ref:Park
22"/e
p.622]
StudyofhumanrelationshipsandhumanbehaviourSociology
PsychologyofindividualslivinginhumansocietySocial
psychology
Studyofphysical,socialandculturalhistoryofmanAnthropology

879.Sociology-
a)Studyofhumanrelationship
b)Studyofbehavior
c)Both
d)None
CorrectAnswer-C
Ans.is'c'i.e.,Both
oSociologydealswiththestudyofhumanrelationshipsandof
humanbehaviour.

880.Studyofphysical,socialandcultural
historyofmanisknownas?
a)Socialscience
b)Anthropology
c)Acculturation
d)Sociology
CorrectAnswer-B
Ans.is'bi.e.Anthropology
oAnthropologyisstudyofphysical,socialandculturalhistoryof
man.
oAnthropologymaybe:
(i)Physicalanthropology:Studyofhumanevolution,racial
differences,inheritanceofbodilytraits,growthanddecayofhuman
organisms
(ii)Socialanthropology:Studyofthedevelopmentandvarious
typesofsociallife.
(iii)Culturalanthropology:Studyoftotalwayoflifeofcontemporary
primitiveman.hiswayofthinking,feelingandaction.
(iv)Medicalanthropology:Dealswiththeculturalcomponentinthe
ecologyofhealthanddisease.

881.Organizedgroupofpeoplewithsocial
relationship?
a)Community
b)Association
c)Society
d)None
CorrectAnswer-C
Ans.is'c'i.e.Society[RefPark22'/ep.622]
Asocietyisabodyofindividualsofspecies,generallyseenasa
communityorgroup,thatisoutlinedbytheboundsoffunctional
interdependence,comprisingalsopossiblecharactersorconditions
suchasculturalidentity,socialsolidarityoreusociality.
Humansocietiesarecharacterizedbypatternsofrelationships
betweenindividualsthatshareadistinctivecultureorinstitution.
Theimportanceofsocietyliesinthefactthatitcontrolsand
regulatesthebehaviouroftheindividualbothbylawandcustoms.

882.Acculturationis?
a)Traige
b)Culturalchangesduetosocialisation
c)Attitude
d)Belief
CorrectAnswer-B
Ans.is'b'i.e.,Culturalchangesduetosocialisation
Acculturationisaprocessofsocial,psychological,andcultural
changethatstemsfromthebalancingoftwocultureswhile
adaptingtotheprevailingcultureofthesociety.Acculturation
isaprocessinwhichanindividualadopts,acquiresand
adjuststoanewculturalenvironment.


883.Chronologicalage10yrs,mentalage
4yrs.Whatthatpersoncalledas?
a)Idiot
b)Imbecile
c)Normal
d)Genius
CorrectAnswer-B
Ans.is'b'i.e.,Imbecile

884.M/CHeavyMetalpoisoninginThe
World?
a)Lead
b)Arsenic
c)Mercury
d)Cadmium
CorrectAnswer-A
Ans.is'a'i.e.,Lead[RefPark22"'Ilep.752]
Moreindustrialworkersareexposedtoleadthananyothertoxic
metal.

885.Allareoccupationalcancersexcept?
a)Lung
b)Bladder
c)Breast
d)Liver
CorrectAnswer-C
Ans.is'c'i.e.,Breast
AsbestosMesothelioma
ArsenicSkin,Lung,Liver
BenzeneLeukemia
BenzidineUrinarybladder
BeryliumLung
CadmiumLung
ChromiumNasalSinus,Lung

886.Whichoccupationalexposuremaycause
sterilityinfemales?
a)Lead
b)Carbonmonoxide
c)Mercury
d)Agriculturalinsecticides
CorrectAnswer-D
Ans.is'd'i.e.,Agriculturalinsecticides[RefHandbookofpesticide
toxicologyp.
787
Pesticidesexposurecancause-
1. Cancers:-Inmultipleorgansystems
2. Endocrineabnormalities
3. Infertilityandsterility
4. Braindamage
5. Birthdefects:-Oralclefts,neuraltubedefects,heartdefects,limb
defects
6. Respiratorydisorders:-Wheezing,bronchitis,asthma
7. Organfailure:-Chronickidneydiseaseorinterstitialnephritis
8. Skinirritation

887.Effectofenvironmentongenesis
called?
a)PositiveEugenics
b)NegativeEugenics
c)Euthenics
d)Enthenics
CorrectAnswer-C
Ans.is'c'i.e.,Euthenics
Eugenics
Sciencewhichaimstoimprovethegeneticendomentofhuman
population.
oThatisimprovingthequalityofthehumanspeciesorahuman
populationbygeneticmanipulation.
oEugenismaybe:
1.NegativeDiscouragingreproductionbypersonshaving
geneticdefectorpresumedtohaveinheritableundesirabletraitsthis
includesabortions,sterilization&othermethodsfamilyplaning.
2.PostiveEugenics->Encouragingreproductionpresumedto
haveinheritabledesirabletrait.Forexample,invitro-fertilization,
cloning,eggtransplantationetc.
Euthenick
Euthenicsdealwithhumanimprovementthroughalteringthe
externalenvironment(environmentalmanipulation).
Itincludeseducation,preventionandremovalofcontagiousdisease
andparasites,educationregardinghomeeconomics,sanitationand
housing.

888.Hardly-weinberglawisrelatedto-
a)Populationgenetics
b)Healtheconomics
c)Socialmedicine
d)Noneoftheabove
CorrectAnswer-A
Ans.is'a'i.e.,Populationgenetics
Hardly-Weinberglaw
Thehardly-weinherglawstatesthat"Therelativefrequencies
ofeachgenealleletendstoremainconstantfromgenerationto
generation".
oThus,thestudyofgenefrequencies,andtheinfluences
whichoperatetoalterthe"genepool"andtheirlongterm
consequencesisthecentralthemeinpopulationgenetics.


889.Blackcolorintriage-
a)Death
b)Transfer
c)Highpriority
d)Lowpriority
CorrectAnswer-A
Ans.is'a'i.e.,Death

890.Trueabouttriage-
a)Yellow-leastpriority
b)Red-morbidity
c)Green-ambulatory
d)Blue-ambulatory
CorrectAnswer-C
Ans.is'c'i.e.,Greenambulatory

891.Amountofwasteinfectiousproducedin
hospitals?
a)45%
b)65%
c)80%
d)100%
CorrectAnswer-A
Ans.is'a'i.e.,45%

892.BestforIncinerationofinfectiouswaste?
a)Double-chamber
b)Single-chamber
c)Triple-chamber
d)None
CorrectAnswer-A
Ans.is'a'i.e.,Double-chamber
oThreebasickindsofincinerationtechnologyareofinterestfor
treatmentofhealhcarewastes:
i)Double-chamberpyrolyticincineratorswhichmaybeespecially
designedtoburninfectioushealthcarewaste.
ii)Single-chamberfurnaceswithstaticgrate,whichshouldbe
usedonlyifpyrolyticincineratorsarenotaffordable.
iii)Rotarykilnsoperatingathightemperatures,capableofcausing
decompositionofgenotoxicsubstancesandheat-resistant
chemicals.

893.Allaretrueaboutpaneldiscussion
except?
a)Twowaydiscussion
b)6to20membersparticipates
c)Chiefmembersinitiates
d)Eachonepreparesthetopicofdiscussion
CorrectAnswer-B
Ans.is'b'i.e.,6-20membersparticipates
Paneldiscussion
oItisatwowaycommunication.
4-8persons
whoarequalifiedtotalkaboutatopicanddiscussa
givenprobleminfrontofalargegrouporaudience.
oThepanelcomprisesachairman.
oThechairmanopensthemeeting,welcomesthegroupand
introducesthepanelspeakers.
oHeindroducestopicbrieflyandinvitethepanelspeakersto
presenttheirpointofview.
oAfterthemainaspectsofsubjectareexploredbypanelspeakers,
theaudienceisinvitedtotakepart.
oIfmembersofthepanelareunacquaintedwiththismethod,they
mayhaveapreliminarymeeting,preparethematerialonthesubject
anddecideuponthemethodandplanofpresentation.

894.ICDSwaslaunchedat-
a)Communitydevelopmentblock
b)Townlevel
c)Citylevel
d)Districtlevel
CorrectAnswer-A
Ans.is'a'i.e.,Communitydevelopmentblock
oICDSwaslaunchedon2"dOctober1975in33Community
developmentblock.
oICDSprogrammeisagloballyrecognizedcommunitybasedearly
childcareprogramme,whichaddressesthebasicinterrelatedneeds
ofyoundchildren,expectantandnursingmothersandadolescent
girlsacrossthelifecycle,inaholisticmanner.

895.KitBisgivenat-
a)PHC
b)Subccnter
c)CHC
d)FRUlevel
CorrectAnswer-B
Ans.is'b'i.e.,Subcenter

896.Atraineddalcatersforapopulationof-
a)1000
b)2000
c)3000
d)4000
CorrectAnswer-A
Ans.is'a'i.e.,1000
Localdais(Traditionalbirthattendants)
UnderRuralHealthScheme,allcategoriesoflocaldiesaretrained
toimprovetheirknowledgeintheelementryconceptsof:-
i)MCH
ii)Sterilization(Smallfamilynorm)
Training
Trainingisfor30workingdays.
oTheyarepaidastipendo/Rs.300duringtrainingperiod.
oTrainingisgivenfor2daysinaweekatPHC,subcentreorMHC
centre.
oOntheremainingfourdaysoftheweektheyaccompanythe
healthworkerfemale.
oDuringhertrainingperiodeachdaiisrequiredtoconductatleast2
deliveries
underthesupervisionofhealthworker.
Thenationaltargetistotrainonelocaldaiineachvillage.
Note:Onevillageisequivalentto1000ruralpopulation.

897.Heighttoweightisa/an-
a)Association
b)Correlation
c)Proportion
d)Index
CorrectAnswer-A
Ans.is'a'i.e.,Association
Firstreadaboutthesetworelatedterms:?
oAssociation--->
Associationmaybedefinedastheconcurrenceof
twovariablesmoreoftenthanwouldbeexpectedbychance.That
mean2variablesexistsimultaneously.
oCorrelation-->Correlationindicatesthedegree(strength)of
associationbetweentwovariables,
i.e.relationshipbetweentwo
quantitativevariables.
oHeighttoweightisanassociationandthestrengthofthis
associationisindicatedbycorrelation.

898.Correlationinheight&weightare
measuredby?
a)Coefficientofvariation
b)Rangeofvariation
c)Correlationcoefficient
d)None
CorrectAnswer-C
Ans.is'c'i.e.,CorrelationCoefficient

899.InaLeftskewedcurve,truestatement
is?
a)Mean=Median
b)Mean<Mode
c)Mean>Mode
d)Mean=Mode
CorrectAnswer-B
Ans.is'b'i.e.,Mean<Mode

900.Trendscanberepresentedby-
a)Linediagram
b)Bardiagram
c)Scatterdiagram
d)None
CorrectAnswer-A
Ans.is'a'i.e.,Linediagram
Linediagram(Linechart/Linegraph)
oItisusedtoshowthetrendofeventswithpassageoftimeand
showshowthefrequencyofaparticulareventorvariablevaryover
time.

901. Mostcommondeviationusedinsocial
medicineis-
a)Mean
b)Range
c)Variance
d)Standarddeviation
CorrectAnswer-D
Ans.is'd'i.e.,Standarddeviation
"Standarddeviationismostcommonandgenerallymostappropriate
measureofdispersion(variation)".


902. Thefrequentlyoccurringvalueinadata
is-
a)Median
b)Mode
c)Standarddeviation
d)Mean
CorrectAnswer-B
Ans.is'b'i.e.,Mode

903.Analysisdoneforexpenditureoflarge
proportionforsmallnumberandvice
versa?

a)ABCanalysis
b)SUSanalysis
c)HMLanalysis
d)VEDanalysis
CorrectAnswer-A
Ans.is'a'i.e.,ABCanalysis

904.Bajajcommitteein1986proposed?
a)Multipurposehealthworker
b)Manpowerandplanning
c)RuralHealthService
d)Integratedhealthservices
CorrectAnswer-B
Ans.is'b'i.e.,Manpowerandplanning

905.UnderRCHprogramme,intervention
doneinselecteddistricts-
a)Immunization
b)TreatmentofSTD
c)ORStherapy
d)VitaminAsupplementation
CorrectAnswer-B
Ans.is'b'i.e..TreatmentofSTD
oInterventionsinallDistricts
oChildsurvivalinterventionsi.e.,immunization,vitaminA(to
preventblindness),oralrehydrationtherapyandpreventionof
deathsduetopneumonia.
Safemotherhoodinterventionse.g.,antenatalcheckup,
immunizationfortetanus,safedelivery,anaemiacontrolprogramme.
ImplementationofTargetFreeApproach
Highqualitytrainingatalllevels
IECactivities.
SpeciallydesignedRCHpackageforurbanslumsandtribal
areas.
Districtsub-projectsunderlocalcapacityenhancement.
RTI/STDclinicsatDistrictHospitals(wherenotavailable).
FacilityforsafeabortionsatPHCsbyprovidingequipment,
contractualdoctorsetc.
EnhancedcommunityparticipationthroughPanchayats,Women's
GroupsandNGOs.
Adolescenthealthandreproductivehygiene.
ojnten.entionsinselectedStates/Distts.
aScreeningandtreatmentofRTI/STDatsub-divisionallevel.

EmergencyobstetriccareatselectedFRUsbyprovidingdrugs.
EssentialobstetriccarebyprovidingdrugsandPHN/StaffNurseat
PHCs.
AdditionalANMatsub-centresintheweakdistrictsforensuring
MCHcare.
Improveddeliveryservicesandemergencycarebyproviding
equipmentkits.IUDinsertionsandANMkitsatsub-centres.
Facilityofreferraltransportforpregnantwomenduringemergency
tothenearestreferralcentrethroughPanchayatinweakdistricts.
aEnhancedcommunityparticipationthroughPanchayats,Women's
GroupsandNGOs.
uAdolescenthealthandreproductivehygiene.
oInterventionsinselectedStates/Distts.
ScreeningandtreatmentofRTI/STDatsub-divisionallevel.
EmergencyobstetriccareatselectedFRUsbyprovidingdrugs.
EssentialobstetriccarebyprovidingdrugsandPHN/Staff
NurseatPHCs.
AdditionalANMatsub-centresintheweakdistrictsforensuring
MCHcare.
Improveddeliveryservicesandemergencycarebyproviding
equipmentkits,IUDinsertionsandANMkitsatsub-centres.
Facilityofreferraltransportforpregnantwomenduringemergency
tothenearestreferralcentrethroughPanchayatinweakdistricts.
EnhancedcommunityparticipationthroughPanchayats,Women's
GroupsandNGOs.
Adolescenthealthandreproductivehygiene.
oInterventionsinselectedStates/Distts.
ScreeningandtreatmentofRTI/STDatsub-divisionallevel.
EmergencyobstetriccareatselectedFRUsbyprovidingdrugs.
EssentialobstetriccarebyprovidingdrugsandPHN/StaffNurseat
PHCs.
AdditionalANMatsub-centresintheweakdistrictsforensuring
MCHcare.
Improveddeliveryservicesandemergencycarebyproviding
equipmentkits,IUDinsertionsandANMkitsatsub-centres.
Facilityofreferraltransportforpregnantwomenduring
emergencytothenearestreferralcentrethroughPanchayatin


weakdistricts.

906.Ujjwalaisfor-
a)Childabuse
b)Childtrafficking
c)Childlabour
d)None
CorrectAnswer-B
Ans.is'b'i.e.,Childtrafficking
oThe'Ministryofwomen&childDevelopment'hasformulateda
newcomprehensiveschemeforpreventionoftrafficking
andrescue,
rehabilitationandreintegrationofvictimsoftraffickingand
commercialsexualexploitation.
oThenewschemehasbeenconceivedprimarilyforthepurpose
preventingtraffickingontheonehandandrescueandrehabilitation
ofvictimsontheother.
oTargetgroupincludes-
i)Women&childrenwhoarevulnerabletotraffickingfor
commercialsexualexploitation.
ii)Women&childrenwhoarevictimsoftraffickingfor
commercialsexualexploitation.

907.NationalLeprosyEradicationProgramme
wasstartedin-
a)1949
b)1955
c)1973
d)1983
CorrectAnswer-D
Ans.is'd'i.e.,1983

908.Directcashtransferschemeto
adolescentgirlsiscoveredunder-
a)IndiraGandhischeme
b)RajivGandhischeme
c)CSSM
d)RCH
CorrectAnswer-B
Ans.is'b'i.e.,RajivGandhiScheme
oDirectcashtransferschemetoadolescentgirlscomesunder
"RajivGandhiSchemeforEmpowermentofAdolescentGirls"

909.Multi-purposeworkerschemeinIndia
wasintroducedfollowingthe
recommendationof?

a)SrivastavaCommittee
b)BhoreCommittee
c)KartarSinghCommittee
d)fsludaliarCommittee
CorrectAnswer-C
Ans.is'c'i.e.,KartarSinghcommittee
HealthPlanninginIndia
Theguidelinesfornationalhealthplanningwereprovidedbya
numberofcommittees.
oThesecommitteeswereappointedbytheGovernmentofIndia
fromtimetotimetoreviewtheexistinghealthsituationand
recommendmeasuresforfurtheraction.

910.Middaymealprogrammecomesunder?
a)MinistryofSocialWelfare
b)Ministryofeducation
c)MinistryofHumanResourcesDevelopments
d)None
CorrectAnswer-B
Ans.is'b'i.e.,Ministryofeducation
ItisalsocalledasNationalProgrammeofNutritionalSupportto
PrimaryEducation.Itwaslaunchedin1995.Mid-daymealshould
provide1/3oftotalenergyand1/2oftotalproteinrequirements.

911.Simplestmeasureofmortality?
a)Crudedeathrate
b)Casefatalityrate
c)Proportionalmortalityrate
d)Specificdeathrate
CorrectAnswer-A
Ans.is'a'i.e.,Crudedeathrate
oThesimplestmeasureofmortalityisthecrudedeathrate.

912.Whatisabsentinbreastmilk?
a)VitK
b)VitC
c)Lactose
d)VitA
CorrectAnswer-A
Ans.is'a'i.e.,VitaminK
oMilksfromthemotherwhosedietissufficientandproperly
balancedwillsupplyallthenecessarynutrientsexceptfluorideand
VitaminD.
oTheironcontentofhumanmilkislow,butmostnormalterm
infantshavesufficientironstoresforthefirst4-6months.Human
milkironiswellabsorbed.Nonetheless,by6monthsthebreast-fed
infant'sdietshouldbesupplementedwithironfortified
complementaryfoods.
oTheVitaminKcontentofhumanmilkislowandmaycause
hemorrhagicdiseaseofnewborn.

913.IMNCItargetgroup-
a)Upto5yrs
b)Upto10yrs
c)Upto15yrs
d)Upto20yrs
CorrectAnswer-A
Ans.is'a'i.e.,Upto5years

914.Abovewhichlevelofheatstressindexit
isnotpossibletoworkcomfortably-
a)20-40
b)40-60
c)60-80
d)80-100
CorrectAnswer-B
Ans.isb'.,40-60
oHSI40-60causessevereheatstrainanditisnotpossibletowork
comfortably.

915.
Studentsreceivehowmuchcereal/day
inmiddaymealprog-

a)50gm
b)100gm
c)150gm
d)75gm
CorrectAnswer-D
Ans.is'd'i.e.,75gm

916.Lowglycemicindexisclassifiedasvaluelessthan:
a)25
b)45
c)55
d)65
CorrectAnswer-C
Conceptofglycemicindexhasutilityinmanagementofdiabetesandobesity.
GIrange
Classification
Example

Mostfruitsandvegetablesexceptpotatoesandwatermelon,
LowGI
55orless pastabeans,lentils
MediumGI
56-69
Sucrose,brownrice,basmatirice
70or
HighGI
Cornflakes,whitebread,candybar
more
Ref:Park22ndedition,page568

917.Nutritionalsupplementfortwoyearold
childunderICDSschemeis-
a)200Calorie
b)300Calorie
c)400Calorie
d)500Calorie
CorrectAnswer-D
Ans.is'd'i.e.,500calories
oUnderICDSSchemesupplementarynutritionisgivento:
Childrenbelow6yrs
Nursingmothers
Expectantmothers
oTheaimistosupplementnutritionalintakefor
1)Eachchild6-72monthsofage--)500caloriesand12-15grams
ofprotein(financialnormofRs6.00perchildperday).
2)Severelymalnourishedchild6-72monthsofage-->800calories
and20-25gramsprotein(financialnormofRs6.00perchildper
day).
3)Eachpregnantandnursingwoman600caloriesand18-20
gramsofprotein(financialnormofRs5.00perbeneficiaryperday).
Undertherevisednutritionalandfeedingnormsforsupplementary
nutrition,Stategovernments/UTshavebeenmandatedtoprovide
morethanonemealtothechildrenwhocometoAWCs,which
includeprovidingamorningsnackintheformof
milk/banana/egg/seasonalfruit/micronutrientfortifiedfoodfollowed
byahotcookedmeal.Forchildrenbelow3yearsofageand
pregnant&lactatingmothers,"takehomeration"istobeprovided.o
Supplementarynutritionisgivenfor300daysayear.


918.Blackdeath
a)Plague
b)Dengue
c)TB
d)Cholera
CorrectAnswer-A
Answer-A.Plague
WhitediseaseAIDS
PovertydiseaseCholera
Hundreddaycough:Pertussis(Whoopingcough)
5dayfever:Trenchfever
8"daydisease:Tetanus
Blacksickness:Kalaazar
Blackdeath:Plague

919.Irregularpupilisseenin?
a)Glaucoma
b)Trauma
c)Occulomotorpulsy
d)Retinaldetachment
CorrectAnswer-B
Ans,B.Trauma
Irregular:-Itisjaggedlookingandoccursmostoftenafterorbital
trauma.

920.Dioptericpowerisrelated-
a)Directlytosquareoffocallength
b)Inverselytofocallength
c)Directlytofocallength
d)Inverselytosquareoffocallength
CorrectAnswer-B
Answer-B.Inverselytofocallength
Opticalpower(alsoreferredtoasdioptricpower,refractivepower,
focusingPower,orconvergencePower)isthedegreetomirror,or
otheropticalsystemconvergesordivergeslight.

921.Visualaxisis
a)Centerofcorneatoretina
b)Objecttofovea
c)Centeroflenstocornea
d)None
CorrectAnswer-B
Ans.Objecttofovea

922.Eyestructurewithmaximumrefractive
power:
March2007

a)Anteriorsurfaceoflens
b)Posteriorsurfaceoflens
c)Anteriorsurfaceofcornea
d)Posteriorsurfaceofcornea
CorrectAnswer-C
Ans.C:Anteriorsurfaceofcornea
Togetherwiththelens,thecornearefractslight,accountingfor
approximatelytwo-thirdsoftheeye'stotalopticalpower.Inhumans,
therefractivepowerofthecorneaisapproximately43dioptres.
Whilethecorneacontributesmostoftheeye'sfocusingpower,its
focusisfixed.
Thecurvatureofthelens,ontheotherhand,canbeadjustedto
"tune"thefocusdependingupontheobject'sdistance.

923.Foster'sfusch'sspotsareseenin
a)Hypermetropiea
b)Myopia
c)Astigmatism
d)None
CorrectAnswer-B
Ans.Myopia

924.Astigmatismisconsideredtobe:
a)Sphericalabberation
b)Curvaturalametropia
c)Axialametropia
d)Indexametropia
CorrectAnswer-A
Ans.Sphericalabberation

925.Lowastigmatismindimlightisdue?
a)Pupilconstriction
b)Pupildilatation
c)Increasedcurvatureoflens
d)Decreasedcurvatureoflens
CorrectAnswer-B
Ans.B.Pupildilatation

926.AwaveinERGisduetoactivityof:
a)Pigmentedepithelium
b)Rodsandcones
c)Ganglioncell
d)Bipolarcell
CorrectAnswer-B
Ans.Rodsandcones

927.2ndPurkinjeimageis?
a)Erectandmovesinsamedirection
b)Invertedandmovesinsamedirection
c)Erectandmovesinoppositedirection
d)Invertedandmovesinoppositedirection
CorrectAnswer-A
Ans.is'a'i.e.,Erectandmovesinsamedirection
1stfromanteriorsurfac:ofcornea-->Erettanilmovesinsame
direction.
2ndfromposteriorsurfaceofcornea-->Erectandmovesinsame
direction.
3rdfromanteriorsurfaceoflens-->Erectandmoveinsame
direction.
4thfromposteriorsurfaceoflens-->Invertedandmovesinopposite
direction

928.Cornealendothelialcellcountis
measuredby?
a)Specularmicroscope
b)Ophthalmoscope
c)Synoptophore
d)Amsler'sgrid
CorrectAnswer-A
Ans.is'a'i.e.,Specularmicroscope
Cornealendotheliumisexaminedwithspecularmicroscope,which
allowsaclearmorphologicalstudyofendothelialcellsincluding
photographicdocumentation.
Thecelldensityofendotheliumisaround3000cells/mm2inyoung
adults,whichdecreaseswithadvancingage.

929.Featuresofvernalconjunctivitisare:
a)Shieldulcer
b)Horner-Tranta'sspots
c)Papillaryhypertrophy
d)All
CorrectAnswer-D
Ans.A,BandC

930.Complicationofvernalkerato
conjunctivitis:
a)Cataract
b)Keratoconus
c)Retinaldetachment
d)Vitreoushemorrhage
CorrectAnswer-B
Ans.Keratoconus

931.Treatmentofvernalkeratoconjunctivitis
includesallexcept:
a)Steroids
b)Chromoglycate
c)Olopatadine
d)Antibiotics
CorrectAnswer-D
Ans.Antibiotics

932.Neonatalconjunctivitisiscausedbyall
except:
a)Gonococcus
b)Chlamydia
c)Aspergillus
d)Pseudomonas
CorrectAnswer-C
Ans.Aspergillus

933.Subconjunctivalcystisseenin?
a)Toxoplasmosis
b)Cysticercosis
c)Leishmaniasis
d)Chaga'sdisease
CorrectAnswer-B
Ans.isbi.e.,Cysticercosis
Parasiticcystsoccursinsubconjunctivalcysticercus,hydatidcyst
andfilarialcyst.

934.Whichbacteriapenetratesintactcornea
?
a)Corynebacterium
b)Pneumococcus
c)Morexella
d)E.coli
CorrectAnswer-A
Ans.is'a'i.e.,Corynebacterium
Bacterialcornealulcer
Bacterialcornealulcer,alsocalledsuppurativekeratitis,isdueto
organismsthatproducetoxinwhichcausetissuenecrosisandpus
formationinthecornealtissue.Purulentkeratitisisnearlyalways
exogenous,duetopyogenicorganism.Aslongasthecorneais
healthy,themajorityofbacteriaareunabletocrossoradheretothe
cornealepithelium.Thereforethetwomainpredisposingfactorsfor
bacterialcornealulcerare:-
1. Damagetocornealepithelium
2. Infectionoftheerodedarea
However,thereareafewspeciesthatarecapableofpenetratingan
intactepithelium:-

1. Neisseriagonorrhoea
2. Neisseriameningitidis
3. Corynebacteriumdiphtheriae
4. Listeriaspecies
5. Haemophilusagyptus
Bacteriawhichcausecornealulcer(Purulentkeratitis)afterepithelial
injuriesare:-
1. Pseudomonas

2. Pneumococcus
3. Streptococcusepidermidis
4. Staphylococcusaureus
5. Morexella
6. Enterobacterias
7. Proteus,Klebsiella)
Althoughtherearespecificpresentationsdependingonthebacteria
involved,thereexistsaseriesofsymptomsandsignscommontoall
thatallowforarapiddiagnosis,andthereforeearlytreatmentof
cornealulcers.Themostimportantsymptomsarepain,lacrimation,
foreignbodysensation,conjunctivalinjection(redeye),photophobia
andblurredvision.
Thereislidswelling,blepharospasmandyellow
whiteareasofulcerwithswollenandoverhangingmarinmaybe
seen.

935.Herpetickeratitisistreatedby
a)Analgesics
b)Atropine
c)Steroids
d)Idoxuridine
CorrectAnswer-D
Ans:Di.e.Idoxuridine
DrugofchoiceforherpetickeratitisisAcyclovirQ(topical)
Otherantiviraldrugsusedare
Idoxuridine
Trifluorothyrnidine
Vidarabine

936.Disciformkeratitisisseen?
a)HSV
b)HIV
c)HBV
d)Rubella
CorrectAnswer-A
Ans.A.HSV

937.Satellitenodulesareseenin
a)Fungalcornealulcer
b)Tuberculosis
c)Sarcoidosis
d)Viralulcer
CorrectAnswer-A
Ai.e.Fungalcornealulcer

938.Mostcommonprotozoancausing
keratitisis
a)Plasmodium
b)Acanthamoeba
c)Toxoplasma
d)W.bancrofti
CorrectAnswer-B
Ans.is`b'i.e.,Acanthamoeba[RefKanski8th/ep.197]
"Acanthamoebakeratitisisthemostcommonkeratitiscausedbya
protozoanespeciallyincontactlensusers".

939.Cornealdystrophy,trueis-
a)Inflammatory
b)Neovascularization
c)Bilateral
d)All
CorrectAnswer-C
Ans.C.Bilateral
CornealdystroPhyisagroupofdisorders,characterizedbyanon-
inflammatory,inherited,bilateralopacityofthecornea.
Thereisnovascularizationofcornea.
Dystrophiesareclassifiedaccordingtotheanatomicalinvolvement.

940.Followingcornealtransplantation,most
commoninfectionoccur?
a)Staphepidermidis
b)Streptococcus
c)Klebsiella
d)Pseudomonas
CorrectAnswer-A
Ans.A.Staphepidermidis
Pneumococcusandstaphylococcusaureushavebeenfoundtobe
thecommonestmicroorganismsinthedevelopedworld,whereas
staphepidermidisisthecommonestindevelopingcountries,for
causinginfectiouskeratitisaftercornealtransplantations.

941.1stsignofanterioruveitis?
a)Keraticprecipitate
b)Aqueousflare
c)Hypopyon
d)Miosis
CorrectAnswer-B
Ans.B.Aqueousflare

942.1stsignofiridocyclitis?
a)Retrolentalflare
b)KP
c)Congestion
d)Trichiasis
CorrectAnswer-A
Ans.A.Retrolentalflare

943.Mostcommoncauseofanterioruveitis?
a)CMV
b)Ankylosingspondylitis
c)Toxoplasma
d)None
CorrectAnswer-A
Ans,A.CMV

944.Iritisinyoungpatientwithjointpain-
a)Gout
b)RA
c)AS
d)Toxoplasma
CorrectAnswer-C
Ans,C.AS

945.Commonestcomplicationofparsplanitis
?
a)Glaucoma
b)Cataract
c)Retinaldetachment
d)Vitreoushemorrhage
CorrectAnswer-B
Ans.B.Cataract
Themostcommoncomplicationofintermediateuveitis(pers
ptanitis)iscystoidmacularedema,whichmaydecreasethevisual
acuity.
2ndmostcommoncomplicationiscomplicatedcataract'
Othercomplicationincludesretinaldetachment.

946.Metamorphopsiaisseenin?
a)Anterioruveitis
b)Posterioruveitis
c)Cataract
d)Glaucoma
CorrectAnswer-B
Ans.B.Posterioruveitis
Matamorphopsiaisaconditioninwhichpatientsperceivedistorted
imagesoftheobject.
ItoccursinPosterioruveitisduetoalterationinretinalcontour'

947.Recurrentanterioruveitiswithincreased
intraoculartensionisseenin?
a)Posnerschlossmansyndrome
b)Fosterkennedysyndrome
c)Vogt-koyanagi-haradasyndrome
d)None
CorrectAnswer-A
Ans.A.Posnerschlossmansyndrome
Glamatocycliticcrisis(posner-Schlossmansyndrome)isa
unilateralrecurrentnon-granulomatousiritisthatisassociatedwith
anelevatedocularpressureduringtheattacks'

948.
Congenitalcataractcommonly
associatedwithvisualdefect?
a)PunctateCataract
b)Bluedotcataract
c)Zonularcataract
d)Fusiformcataract
CorrectAnswer-C
Ans.C.Zonularcataract
Lamellar(Zonular)cataractisthemostcommontypeofcongenital
cataractpresentingwithvisualimpairment.
Itisusuallybilateralandfrequentlycausesserverevisualdefects'

949.Decreasedreadingabilityisseenin?
a)Fusiformcataract
b)Zonularcataract
c)Bluedotcataract
d)Punctatecataract
CorrectAnswer-B
Ans,B.Zonularcataract

950.Lenssubluxatesinhomocystinuria?
a)Inferotemporal
b)Inferonasal
c)Superonasal
d)Superotemporal
CorrectAnswer-B
Ans.B.Inferonasal

951.Secondsightisseenin?
a)Nuclearcataract
b)Corticalcataract
c)Zonularcataract
d)Punctatecataract
CorrectAnswer-A
Ans.A.Nuclearcataract
Whenanuclearcataractdevelops,itcanbringaboutatemporary
improvementinnearvision,called'secondsight'.

952.Congenitalcataractwithvisual
disturbancessurgeryshouldbedone?
a)Immediately
b)After2months
c)After4months
d)After1year
CorrectAnswer-A
Ans.A.Immediately

953.Mostcommontypeofcongenital
cataractis?
a)Capsular
b)Zonular
c)Coralliform
d)Bluedot
CorrectAnswer-D
AnsD.Bluedot
Mostcommontypeofcongenitalcataract=punctate(bluedot)
cataract.
Mostcommontypeofcataractwhichisclinically(visually)significant
+zonularorlamellarcataract.

954.
Treatmentoftraumaticcataractin
children?
a)ECCE+IOL
b)Lensectomy
c)Contactlens
d)Glasses
CorrectAnswer-A
Ans.A.ECCE+IOL
Traumaticcataractinchildrenisacommoncauseofunilateralloss
ofvision.
Penetratinginjuriesareusuallymorecommonthanbluntinjuries.
Atthetimeofpresentationaftertraumatoeye,primaryrepairofthe
cornealorscleralwoundisusuallypreferred,
CataractsurgeryGCCE)withIOLimplantationsisperformedlater
followingcompleteevaluationofdamagetotheintraocular
structuresbyancillarymethodssuchasB-scanultrasonography.

955.Jackinboxscotomaisseenafter
correctionofAphakiaby?
a)IOL
b)Spectacles
c)Contactlens
d)None
CorrectAnswer-B
Jack-in-the-boxphenomenonisseeninthecorrectionofaphakiaby
spectaclesduetotheprismaticeffectsattheedgeofthelens.
Otherdifficultiesinthecorrectionofaphakiabyspectaclesinclude
thefollowing:
Imagemagnificationby25?30%.
PinCushiondistortion-asphericalaberrationduetothick
spectacles.
RestrictedfieldwithJackintheBoxphenomenona/RovingRing
scotoma-aprismaticaberration.
Cosmetically,theeyeslookenlarged(Frogeyes)behindthethick
spectacles.
Physicalinconvenience.
Treatmentofchoice:
correctionisobtainedbyintraocularlens(IOL).

956.Falseaboutphacolyticglaucoma?
a)Duetocontactofiristolens
b)Openangleglaucoma
c)Seeninhypermaturestageofcataract
d)Lensinducedglaucoma
CorrectAnswer-A
Ans.A.Duetocontactofiristolens
Phacolyticglaucomaisanopenangleglaucomainhypermature
stageofcataractduetoblockageoftrabecularmeshworkbyswollen
macrophages.
Glaucomaduetocontactofiristolens(pupillaryblockglaucoma)is
seeninphacomorphicglaucoma.

957.Neovascularglaucomaisseeninall
except?
a)Diabetes
b)CRVO
c)Eale'sdisease
d)Openangleglaucoma
CorrectAnswer-D
Ans.D.Openangleglaucoma
Itisasecondaryangleclosureglaucomawhichresultsdueto
formationofneovascularmembraneovertheirisi.e.,
neovascularizationofiris(rubeosisiridis).

958.Laseriridotomyisdonein?
a)Angleclosureglaucoma
b)Openangleglaucoma
c)Pigmentoryglaucoma
d)None
CorrectAnswer-A
Ans.A.Angleclosureglaucoma
TreatmentofchoiceforPACGisperipherallaseriridotomy.

959.Inacuteangleclosureglaucoma,primary
mechanismofpathogenesisis?
a)Increasedsecretion
b)Increasedabsorptionbutincreasedsecretion
c)Outflowobstruction
d)None
CorrectAnswer-C
Ans,C.Outflowobstruction
Inacuteangleclosureglaucoma,riseinIOPoccursduetoblockage
ofaqueousoutflowbyclosureofanarrowerangleofanterior
chamber.

960.Notariskfactorforangleclosure
glaucoma?
a)Smalleye
b)Hypermetropia
c)Smallcornea
d)Smalllens
CorrectAnswer-D
Ans.D.Smalllens
PredisposingfactorsforPACG:-i)Shallowanteriorchamber,ii)
Shorteye(shortaxiallensth),i1i)Smallercornealdiameter,iv)
Anteriorlocationofiris-lensdiaphragm,v)Hlpermetropiceye,vi)
Largelens(oldercataractous).

961.Bestdrugforopenangleglaucoma?
a)Latanoprost
b)Pilocarpine
c)Physostigmine
d)Apraclonidine
CorrectAnswer-A
Ans.A.Latanoprost
Medicaltherapy:-Totalmedicaltherapyisthetreatmentofchoice
forPOAG.Topicalp-blockers(Timolol,Betoxalol,Levobunolol,
carteolol)arethedrugsofchoice.
Topicalprostaglandinanalogues(Iatanoprost,bimatoprost,
travoprost)arethesecondchoicedrugs.

962.Broadestneuroretinalrimisseenin-
a)Suppole
b)Infpole
c)Nasalpole
d)Temporallobe
CorrectAnswer-B
Ans.,B.Infpole
Theneurorentinalrimistheareaofopticdiscwhichcontainsneural
elementsandislocatedbetweentheedgeoftheopticdiscandthe
physiologicalcup.
TheneuroretinalrinisbroadestInferiorly,followedbySuperior,
NasalandTemporalregionsindecreasingorderofthickness(the
ISNT)rule.

963.Retinitispigmentosaisduetodefectin
whichgene-
a)Scotopsin
b)Rhodopsin
c)Pigmentedepithelium
d)None
CorrectAnswer-B
Ans.B.Rhodopsin
Severaldifferentrhodopsingenemutationshavebeenidentifiedin
thepedigreeswithautosomaldominantretinitispigmentosa.

964.Ealesdiseaseis?
a)Recurrentopticneuritis
b)Recurrentpappilloedema
c)Recurrentperiphelbitisretinae
d)None
CorrectAnswer-C
Ans,C.Recurrentperiphelbitisretinae
Eale'sdiseaseisanidiopathicinflammatoryvenousocclusion
(phelbitis)thatprimarilyeffectstheperipheralretinai.e.,periphelbitis
andischaracterizedbyrecurrentbilateralvitreoushemorrhage.

965.Shaffer'ssignisseenin?
a)Retinitispigmentosa
b)Retinaldetachment
c)CRVO
d)CRAO
CorrectAnswer-B
Ans.B.Retinaldetachment
Vitreousshowpigmentintheanteriorvitreous(tobaccodustingor
shaffersign),withposteriorvitreousdetachement.

966.Depositinretinalmaculardegeneration?
a)Iron
b)Drusen
c)Lipochrome
d)Hemosiderine
CorrectAnswer-B
Ans.B.Drusen
DryformofARMDbeginswithcharacteristicyellowdepositsinthe
maculacalleddrusenbetweentheretinalpigmentepitheliumandtie
underlyingchoroid

967.Vitreoushemorrhageindiabetic
retinopathy?
a)Non-proliferativediabeticretinopathy
b)Proliferativediabeticretinopathy
c)Both
d)None
CorrectAnswer-B
Ans.B.Proliferativediabeticretinopathy

968.Headlightinfogappearanceisseenin?
a)Syphilis
b)Toxoplasmoss
c)Toxocara
d)Herpes
CorrectAnswer-B
Ans.B.Toxoplasmoss
Onfundoscopicexamination,thereisdiffuse"headtightinthe
fogappearance,incongenitaltoxoplasmosis.
Thisisduetocombinationofactiveretinallesioninthecenterwith
depigmentation(theheadlight)andseverevitreousinflammation
(thefog).

969.Salt&pepperfundus?
a)Congtoxoplasmosis
b)Conghistoplasmosis
c)Congenitalsyphilis
d)None
CorrectAnswer-C
Ans.C.Congenitalsyphilis

970.Cattletrackappearence?
a)CRVO
b)CRAO
c)Diabeticretinopathy
d)Syphiliticretinopathy
CorrectAnswer-B
Ans.B.CRAO

971.Eale'sdiseaseis?
a)Retinalhemorrhage
b)Vitreoushemorrhage
c)Conjunctivalhemorrhage
d)Choroidalhemorrhage
CorrectAnswer-B
Ans.B.Vitreoushemorrhage
Eale'sdiseseisanidiopathicinflammatoryvenousocclusionthat
primarilyaffectstheperipheralretinaofyoungadult(20-30yrs)
male.
Itischaracterizedbyrecurrentbilateralvitreoushemorrhage;
therefore,alsoreferredtoasprimaryvitreoushemorrhage.

972.Extraretinalfibrovascularproliferationat
ridgeis?
a)Normal
b)StageIROM
c)StageIIROM
d)StageIIIROM
CorrectAnswer-D
Ans.D.StageIIIROM
DiseaseSeverity(Stage)
PriortothedevelopmentofROPintheprematureinfant,
vascularizationoftheretinaisincompleteor"immature"(Stage0).
Stage1:DemarcationLine:Thislineisthinandflat(intheretina
plane)andseparatestheavascularretinaanteriorlyfromthe
vascularizedretinaposteriorly.
Stage2:Ridge:Theridgearisesfromthedemarcationlineandhas
heightandwidth,whichextendsabovetheplaneoftheretina.The
ridgemaychangefromwhitetopinkandvesselsmayleavethe
planeoftheretinaposteriortotheridgetoenterit.Smallisolated
tuftsofneovasculartissuelyingonthesurfaceoftheretina,
commonlycalled"popcorn"maybeseenposteriortothisridge
structureanddonotconstitutethedegreeoffibrovasculargrowth
thatisanecessaryconditionforstage3.
Stage3:ExtraretinalFibrovascularProliferation:Neovascularization
extendsfromtheridgeintothevitreous.Thisextraretinal
proliferatingtissueiscontinuouswiththeposterioraspectofthe
ridge,causingaraggedappearanceastheproliferationbecomes
moreextensive.
Stage4:PartialRetinalDetachment:Stage4,intheinitial
classificationwasthefinalstageandinitiallyknownasthecicatricial

classificationwasthefinalstageandinitiallyknownasthecicatricial
phase.Itwaslaterdividedintoextrafoveal(stage4A)andfoveal
(stage4B)partialretinaldetachments.Stage4retinaldetachments
aregenerallyconcaveandmostarecircumferentiallyoriented.
Retinaldetachmentsusuallybeginatthepointoffibrovascular
attachmenttothevascularizedretinaandtheextentofdetachment
dependsontheamountofneovascularizationpresent.
Stage5:TotalRetinalDetachment:Retinaldetachmentsare
generallytractionalandusuallyfunnelshaped.Theconfigurationof
thefunnelitselfisusedforsubdivisionofthisstagedependingifthe
anteriorandposteriorportionsareopenornarrowed

973.Allareseenin3'nervepalsy?
a)Mydriasis
b)Lossoflightreflex
c)Lossofabduction
d)Ptosis
CorrectAnswer-C
Ans.C.Lossofabduction

974.Swinginglighttestispositivein?
a)Conjunctivitis
b)Glaucoma
c)Retrobulbarneuritis
d)Keratoconus
CorrectAnswer-C
Ans.C.Retrobulbarneuritis
Swingingflashlightisusedforrelativeefferentpathwaydefect
(RAPD)whichismostcharacteristicofIesionsintheopticnerve.
Thusitispositiveinretrobulbarneuritis.

975.Optictractlesioncauses?
a)Wernicke'shemianopicpupil
b)Amauraticpupil
c)Amauraticpupil
d)None
CorrectAnswer-A
Ans,A.Wernicke'shemianopicpupil
Wernicke'shemianopicpupilisseenincompletelesionofoptictract.

976.Amblyopiaiscausedby?
a)Methylalcohol
b)Penicillin
c)Propranolol
d)None
CorrectAnswer-A
Ans.A.Methylalcohol
Toxicamblyopiaischronicretrobulbarneuritiswhichresultsfromthe
damagetoopticnervebytheexogenouspoisons.
Thetoxicagentsinvolvemaybe:Tobacco,ethylatcohol,ethylene
glycol,Iead,arsenic,cannabisindica,carbondisulphidevarious
drugs

977.VitaminB12deficiencycauses?
a)Centrocaecalscotoma
b)Binasalhemianopia
c)Constrictionofperipheralfield
d)Bitemporalhemianopia
CorrectAnswer-A
Ans.,A.Centrocaecalscotoma
VitaminB12deficiencycausesopticneuritis.
Mostcommonvisualfielddefectinopticneuritisiscentralor
centrocaecalscotoma.

978.Causeofbilateralopticatrophy?
a)Traumatoopticnerve
b)Intracranialneoplasma
c)CRAO
d)Opticneuritis
CorrectAnswer-B
Ans.B.Intracranialneoplasma

979.
Earliestmuscletoinvolveinthyroid
ophthalmopathy?
a)MR
b)LR
c)IR
d)SR
CorrectAnswer-C
Ans.C.IR

980.Whichofthefollowingislongest
extraocularmuscle?
a)SR
b)MR
c)SO
d)IO
CorrectAnswer-C
Ans.C.SO
Thesuperiorobliqueisthelongest,thinnestextraocularmuscle.

981.Treatmentofchoiceforamblyopiais?
a)Convergentexercises
b)Spectacles
c)Surgery
d)Conventionalocclusion
CorrectAnswer-D
Ans.D.Conventionalocclusion
Thetreatmentofamblyopiashouldbeginasearlyaspossible.
Theamblyopiceyefailstodevelopvisionandvisualimpairment
remainsPermanentunlessitistreatedbeforetheageof7years.
Amblyopictherapyworksbestwhenintitiatedinyoungchildren
under3yearsofage.

982.Aftertrauma,Apersoncannotmoveeye
outwordbeyondmidpoint.Thenerve
injuredis?

a)2nd
b)3rd
c)4th
d)6th
CorrectAnswer-D
Ans,D.6th
Alltheextraocularmusclesaresuppliedby3rdcranial(occulomotor)
nerveexceptforsuperiorobliqueandlateralrectus.superioroblique
muscleissuppliedby4thcranial(trochlear)nerve,andlateralrectus
muscleissuppliedby6thcranial(abducent)nene.Besidethese
extraocularmuscles,occulomotor(3rd)nervealsosupplieslevator
palpebraesuperioris,sphinctorpupillae(causespupillary
constriction),andciliarymuscle(causesaccommodation).

983.Angleofsquintismeasuredby?
a)Gonioscopy
b)Prism
c)Retinoscopy
d)Keratometry
CorrectAnswer-B
Ans.B.Prism

984.Mostcommoncauseofophthalmoplegia
?
a)Aneurysm
b)Infection
c)Mystheniagravis
d)None
CorrectAnswer-A
Ans.A.Aneurysm
Themostcommonindentifiableetiologiesareischemia,aneurysm,
tumorandttauma;some20%remainunexplained.

985.Ataxia,nystagmusandophthalmoplegia
isseenin-
a)Mystheniagravis
b)Chronicprogressiveexternalophthalmoplegia
c)3rdnervepalsy
d)None
CorrectAnswer-B
Ans.B.Chronicprogressiveexternalophthalmoplegia
Themostcommonidentifiableetiologiesareischemia,aneurysm,
tumorandtrauma,some20%remainunexplained.

986.Childwithmildsquint.Intrauterine,birth
history,developmentalhistorytilldateall
normal.Cornealreflexnormal.Allother
eyeparametersnormalexcept
exaggeratedepicanthalfold.Diagnosis?

a)Pseudostrabismus
b)Accommodativesquint
c)Exophoria
d)Esophoria
CorrectAnswer-A
Ans,A.Pseudostrabismus
Pseudoesotropia(apparentconvergentsquint-Duetoprominent
epicanthalfold.
Pseudoexotropia(apparentdivergentsquint-Duetohypertelorism.

987.MCorbitaltumor?
a)Nervesheathtumor
b)Hemangioma
c)Lymphoma
d)Meningioma
CorrectAnswer-B
Ans.B.Hemangioma
Mostcommonorbitaltumor-Cavernoushemangioma.
Mostcommonmalignantorbitaltumor?Lymphoma.

988.
Mostcommonorbitaltumorhasits
originfrom?
a)Bloodvessels
b)Nerves
c)Muscle
d)Lymphnode
CorrectAnswer-A
Ans,A.Bloodvessels
Mostcommonorbitaltumorsarebenignvasculartumors-
Cavernoushemangioma.

989.Mostcommonmalignanttumourof
eyelidis?
a)Sebaceousglandcarcinoma
b)Basalcellcarcinoma
c)Squamouscellcarcinoma
d)Malignantmelanoma
CorrectAnswer-B
Ans,b.Basalcellcarcinoma
Basalcellcarcinomaisthemostcommonmalignanttumourofthe
eyelidsandconstitutes85-90%ofallmalignantepithelialeyelid
tumours.

990.Mostcommoncarcinomaofconjunctiva
?
a)SquamouscellCa
b)Basalcellca
c)Melanoma
d)Lymphoma
CorrectAnswer-A
Ans.A.SquamouscellCa
Squamouscellcarcinomaisthemostcommonmalignantlesionof
theconjunctiva"

991.Retinalastrocytomaisseenin?
a)Tuberoussclerosis
b)Sturgewebersyndrome
c)VonHippel-Lindausyndrome
d)None
CorrectAnswer-A
Ans.A.Tuberoussclerosis

992.Fusionofpalpebralandbulbar
conjunctivais-
a)Symblepharon
b)Trichiasis
c)Ectropion
d)Tylosis
CorrectAnswer-A
Ans.A.Symblepharon
Adhesionofthelidstoeyeballiscalledsymblepharon.
Itoccursduetofusionofpalpebralconjunctiva(coveringinner
surfaceoflid)tobulburconjunctiva(coveringoutersurfaceof
eyeball).

993.Lossofeyelashesis?
a)Tylosis
b)Madarosis
c)Trichiasis
d)Ectropion
CorrectAnswer-B
Ans.B.Madarosis
Madarosisrefertolossofeyelashesandsometimesalsoeyebrows

994.Madarosisisseenin?
a)Addison'sdisease
b)Hypothyroidism
c)Acromegaly
d)None
CorrectAnswer-B
Ans.B.Hypothyroidism
MadarosismaYbeseenin-
Rubella
CongenitalSyphilis
Congenitalleberamaurosis
MayousBattendisease
Thioridazinetoxicity

995.Abnormallyeccentricplacedpupilis
called?
a)Polycoria
b)Corectopia
c)Corectopia
d)Ectopialentis
CorrectAnswer-B
Ans.B.Corectopia

996.Massagingofnasolacrimalductisdone
in?
a)Acutedacryocystitis
b)Congenitaldacryocystitis
c)Conjunctivitis
d)None
CorrectAnswer-B
Ans.B.Congenitaldacryocystitis
Massageoverlacrimalsacisthemainstayoftreatmentincongenital
dacryocystitis.

997.Mostcommoncauseofintermittent
proptosis?
a)Orbitalvarix
b)Thyroidophthalmopathy
c)Neuroblastoma
d)Retinoblastoma
CorrectAnswer-A
Ans.A.Orbitalvarix
Intetmittentproptosis:-Proptosisdevelopingintermittentlyand
rapidlyinoneeyewhenvenousstasisisinducedbyforwardbending
orloweringthehead,turningtheheadforcibly,hyperextensionofthe
neck,coughing,forcedexpirationwithorwithoutcompressionofthe
nostrils,orpressureonjugularveins.Themostimportantcasueis
orbitalvarix(varicocele).

998.Bilateralptosisisseeninallexcept?
a)Hyperthyroidism
b)Congenital
c)Trauma
d)Myotonicdystrophy
CorrectAnswer-C
Ans,C.Trauma

999.Unilaterallacrimalglanddestructionmay
becausedby?
a)Inferiororbitalfissurefracture
b)Fractureofroofoforbit
c)Fractureoflateralwall
d)Fractureofsphenoid
CorrectAnswer-B
Ans.B.Fractureofroofoforbit
Damagetothesuperiororbitalstructures,infractureofroofoforbit,
cancausediplopia,ptosis,opticneuropathy,ptosis,neuroPathyand
lacrimalglandinjury.

1000.Alkalicauses?
a)Symblepharon
b)Papilloedema
c)Opticneuritis
d)Retinaldetachment
CorrectAnswer-A
Ans,A.Symblepharon

1001.Expulsivehemorrhageincataract
surgeryisfrom?
a)Vortexvein
b)Ciliaryartery
c)Choroidalvein
d)None
CorrectAnswer-B
Ans,B.Ciliaryartery
Expulsivehemorrhageaftercataractextractionorglaucomasurgery
=Ruptureofciliaryartery.
ExpulsivehemorrhageinRetinal/Vitrousoperation=Directtrauma
tochoroidalorvortexvein.

1002.Coloboma,mostcommonsite?
a)Superotemporal
b)Inferonasal
c)Inferotemporal
d)Superonasal
CorrectAnswer-B
Ans.B.Inferonasal
Acolobomaisaholeinoneofthestructuresofeye,suchasiris,
retina,choroidoropticdisc.
Theeyedevelopsintheembryo,fromtheopticcupandoptic
fissure.Theopticfissurefusesat5-7weeksdevelopment.
Failureofthisfusionleadstoagapinoculartissueknowas
coloboma,typicallylocatedintheinferonasalquadrant.

1003.Irisclobomaismostcommonin?
a)Inferotemporal
b)Soperotemporal
c)Inferonasal
d)Soperonasal
CorrectAnswer-C
Ans.C.Inferonasal
Allclobomas(includingiris)aremostlyinferonasal.

1004.Enucleationisindicatedin?
a)Acutecongestiveglaucoma
b)Panophthalmitis
c)Retinoblastoma
d)None
CorrectAnswer-C
Ans,C.Retinoblastoma
IndicationsofEnucleation
Absolute=Retinoblastoma,malignantmelanoma.
Relative=Painfulblindeye,mutilatingocularinjury,anterior
staphyloma,phthisisDalEi,endophthalmitis,congenital
anophthalmiaorseveremicrophthalmia.

1005.Cornealtattooingisdoneby?
a)Goldchloride
b)Silverchloride
c)Titaniumchloride
d)Aluminiumchloride
CorrectAnswer-A
Ans.A.Goldchloride
FortattooingIndianblackinkgoldchlorideorplatinummaybeused.

1006.Staphlyomainvolvement?
a)Iriswithconjunctiva
b)Conjunctivawithcornea
c)Choroidwithretina
d)Iriswithcornea
CorrectAnswer-D
Ans.D.Iriswithcornea
Staphylomaisanabnormalprotrusionofuvealtissue(irisorciliary
bodyorchoroid)throughaweekandthinportionofcorneaorsclera.
So,staphylomaislinedinternallybyuvealtissue(irisorciliarybody
orchoroid)andexternallybyweakcorneaorsclera.

1007.65yearoldpersonwithhearingloss
withnormalspeechdiscriminationis
sufferingfrom?

a)Noiseinducedhearingloss
b)Presbycusis
c)Ototoxicdrug
d)NOHL
CorrectAnswer-B
Ans.is'B'i.e.,Presbycusis
Informationinthisquestionare:-(i)Oldage(65years),(ii)Hearing
loss,and(iii)Preservedspeechdiscrimnation.
DiagnosisisPresbycusis.
Presbycusis
Presbycusisreferstosensorineuralhearinglossinelderly
individuals
Characteristically,presbycusisinvolvesbilateralhigh
frequencyhearinglossassociatedwithdifficultyinspeech
discriminationandcentralauditoryprocessinginformation.
Fourpathologicaltypesofpresbycusishavebeenidentified:-
SensorypresbycusisThereisepithelialatrophywithlossofsensory
haircellsandsupportingcellsintheorganofcorti.This
processstartsinthebasalturnofcochleaandslowlyprogress
towardstheapex.Higherfrequenciesareaffectedbutspeech
discriminationispreserved.
Neuralpresbycusis:-Thereisatrophyofnervecellsinthecochlea
andcentralneuralpathways.Atrophyoccursthroughoutthe
cochlea,withthebasilarregiononlyslightlymorepredisposedthan

theremainderofthecochlea.Therefore,noprecipitousdropin
thresholdonaudiometryisobserved.Speechdiscriminationispoor.
Metabolic(strial)presbycusis;;-Thereisatrophyofstriavascularis.
Atrophyresultsinhearinglossrepresentedbyflataudiogram,but
speechdiscriminationispreserved.
Mechanical(cochlearconductive)presbycusis:-Thereisthickening
andsecondarystiffeningofthebasilarmembraneofthecochlea.
Thethickeningismoresevereinthebasalturnofcochleawherethe
basilarmembraneisnarrow.Thiscorrelateswithagraduallysloping
highfrequencysensorineuralhearingloss.Speechdiscriminationis
average.

1008.Athleticsyndromeischaracterizedby:
a)IncreasedamplitudeofQRS
b)Tachycardia
c)DecreasedQTinterval
d)U-waves
CorrectAnswer-A
TheanswerisA(IncreasedamplitudeofQRScomplex):
AthleticHeartSyndrome
AthleticHeartSyndromeisabenignconditionconsistingof
physiologicadaptationstotheincreasedcardiacworkloadof
exerciseintrainedathletes.
Itrepresentsaconstellationofclinicalfindingsthataretheresultof
normalphysiologicadaptationtostrenuousphysicalactivity.
Inresponsetotheincreasedphysicaldemand,theleftventricles
dilateandwallthicknessincreases.Themasstovolumeratio,
however,doesnotchange.
Physicalexamination
Decreasedbodyfatandincreasedmusclemass(generallyvery
physicallyfit)
Pulseslowandoftenirregular(sinusbradycardiaorbradycardiawith
first-andsecond-degreeblocks)
GradeIorIImid-systolicmurmurs(benignfunctionalejection
murmurresolveswithValsalvamaneuver)
Thirdandfourthheartsoundsverycommon(benignfillingsounds)
Bloodpressuretypicallyremainsnormal
Electrocardiogramrhythm
Rhythm
-Sinusbradycardiaof40to55beats/minwhileatrest

-Sinuspausesofmorethan2.0secondsduetoincreasedvagal
tone
-Wanderingatrialpacemakerfoundonlyindynamicathletes
-Firstdegreeatrioventricularblockpresentonlyatrest;P-Rinterval
normalizeswithexercise
-Seconddegreeatrioventricularblockpresentonlyatrest:MobitzI
(wenckebachblock)commoninmarathonrunners;MobitzIIrarein
athlete'sheart.
Voltage:TORSvoltage(Amplitude)
-Leftventricularhypertrophyfoundin85%ofOlympicmarathon
runners
-Rightventricularhypertrophycommonindynamicathletesbut
rarelyseeninsedentarycontrolsandstaticathletessedentary
controlandstaticathletes
Repolarization
-S-TsegmentelevationwithpeakedTwavesnormalizeswith
exertion
-S-Tsegmentdepressionmayberarelyfoundinathletes
T-waveinversioninlateralleadsassociatedwithinterventricular
septalhypertrophyinstaticathletes(canbeanormalfindingin
dynamicathletes)
Chestradiography
-Theheartisglobularinappearance,particularlyinendurance
athletes.
-Cardiomegaly(cardiothoracicratio>0.50)

1009.TallTwavesonECGareseenin:
a)Hyperkalemia
b)Hypokalemia
c)Hypercalcemia
d)Hypocalcemia
CorrectAnswer-A
AnswerisA(Hyperkalemia)
HyperkalemiaistypicallyassociatedwithTallpeakednarrowbased
frntedTwave.


1010.-30?to-60?leftaxisdeviationisseen
in
a)Leftventricularhypertrophy
b)Rightventricularhypertrophy
c)Aorticstenosis
d)Leftatrialenlargement
CorrectAnswer-A
Answer-A.Leftventricularhypertrophy
Normally,QRSaxisrangesfrom-30?to90?C
Anaxismorenegativethan-30?isreferredtoasleftaxisdeviation
andanaxismorepositivethan+100?iscalledrightaxisdeviation
Leftaxisdeviation
Axismorenegativethan-30?
Associatedwith
Leftventricularhypertrophy
Leftanteriorfascicularblock
InferiorM.I.

1011.Kussmaul'ssignisclassically
describedin:
a)RestrictiveCardiomyopathy
b)PericardialTamponade
c)Constrictivepericarditis
d)RightVentricularInfarct
CorrectAnswer-C
AnswerisC(Constrictivepericarditis)
Kussmaul'ssignisclassicallydescribedinassociationwith
ConstrictivePericarditis.
Kussmaul'ssignreferstoparadoxicalelevationofJVP/CVPduring
inspiration(Inhealthypersonsvenouspressurefallsduring
inspirationbecausepressuresintherightheartdecreaseas
intrathoracicpressuresfall)TheKussmaul'ssignisclassically
describedinassociationwithConstrictivepericarditis.Kussmaul's
signishoweveralsoseeninassociationwithRightVentricular
Infarction,RestrictiveCardiomyopathy,PulmonaryEmbolismand
AdvancedSystolicSevereHeartFailure


1012.Jawtightnessistypicallyseenin:
a)PAN
b)Coarctationofaorta
c)Giantcellarteritis
d)Wegner'sGranulomatosis
CorrectAnswer-C
AnswerisC(GiantCellArteritis):
JawClaudication(lawTightness)isatypicalmanifestationof
TemporalarteritisorGiantcellarteritis.

1013.The9montholdchildofadiabetic
motherpresentswithtachypneaand
hepatomegaly.Echocardiographyofthe
heartshowednormalcardiac
morphologywithasymmetricseptal
hypertrophy.Whichofthefollowingyou
willgivetotreatthischild:

a)Digoxin
b)Frusemide
c)Propranolol
d)Isoptin
CorrectAnswer-C
AnswerisC(Propranolol)
Thesymptomsofthepatientandechocardiographicfindingof
asymmetricalseptalhypertrophyalmostconfirmsthediagnosisof
Hypertrophiccardiomyopathy.BetaBlockersshouldbetheinitial
drugQinsymptomaticindividuals
ManagementofHOCM

Avoidanceofstrenuousphysicalactivity
BetaBlockersshouldbetheinitialdrugQinsymptomaticindividuals.
Theyreduce:
-Heartrate

-Bloodpressure


-Stiffnessofleftventricle
-Fatalarrythmias
CalciumchannelBlockersQ(verapamilanddiltiazem)arealternative
drugs.
-Theyreduce-stiffnessofventricle
-Elevateddiastolicpressures
Amiadaronemaybeusedtoreducearrythmias.
Surgicalmyomectomy

1014.Paradoxicalsplittingofsecondheart
soundisseenin?
a)RBBB
b)ASD
c)LBBB
d)VSD
CorrectAnswer-C
AnswerisC(LBBB)
LeftBundleBranchBlock(LBBB)istypicallyassociatedwith
ReversedorParadoxicalSplittingofS2
ParadoxicalsplittingofsecondheartsoundiscausedbydelayedA2
orearlyP2.LeftBundleBranchBlock(LBBB)isassociatedwith
delayedAorticclosure(delayedA2)duetodelayedelectrical
activationoftheleftventricle.
ASDandRBBBareassociatedwithawidephysiological(non-
paradoxical)splitofsecondheartsoundduetodelayedpulmonic
closure(DelayedP2)whileVSDisassociatedwithawide
physiological(non-paradoxical)splitsecondheartsoundfromearly
aorticclosure(EarlyA2).


1015.Whichisincreasedinplasmaofchronic
heartdiseasepts
a)BNP
b)Endothelin1
c)Cortisol
d)None
CorrectAnswer-A
Answer-A.BNP
Theplasmaconcentrationsofbothhormonesareincreasedin
patientswithasymptomaticandsymptomaticleftventricular
dysfunction,permittingtheiruseindiagnosis.
BNPlevelsaresimpleandobjectivemeasuresofcardiacfunction.
Thesemeasurescanbeusedtodiagnooseheartfailureincluding
diastolicfunction.

1016.Inasthmadiagnosisisby
a)FEVi
b)Measurementoftidalvolume
c)Endexpiratoryflowrate
d)Totallungcapacity
CorrectAnswer-A
Answer-A.FEVi
Asthmaisclassifiedaccordingtothefrequencyofsymptoms,forced
expiratoryvolumeinonesecond(FEV1),andpeakexpiratoryflow
rate.
Spirometryisrecommendedtoaidindiagnosisandmanagement.It
isthesinglebesttestforasthma.IftheFEV1measuredbythis
techniqueimprovesmorethan12%andincreasesbyatleast200
millilitersfollowingadministrationofabronchodilatorsuchas
salbutamol,thisissupportiveofthediagnosis.

1017.Apersonhasasthmaattacksmorethan
onceduringdayandonceduringnight
a)Mildintermittentasthma
b)Mildpersistentasthma
c)Moderateasthma
d)Severeasthma
CorrectAnswer-D
Answer-D.Severeasthma
Severeasthma
Symptoms-Throughoutday
Nightawakenings-Daily
Short-actingB2-agonistuseforsymptomcontrol-Severaltimesper
day
Interferencewithnormalactivity-Extremelimitation
FEV1-<60%
FEV1/FVC=>5%reduced

1018.Antibodyusedinthetreatmentof
BronchialAsthmais:
a)Omalizumab
b)Rituximab
c)Daclizumab
d)Transtusuzumab
CorrectAnswer-A
AnswerisA(Omalizumab):
OmalizumabisarecombinantIgEAntibodyapprovedforusein
treatmentofmoderateandseverepersistentasthma
OmalizumabisarecombinantIgEblockingantibodythat
neutralizescirculatingIgE.
ItpreventscirculatingIgEfrombindingtoreceptorsonthesurfaceof
BasophilsandMastcellsandthusinhibitsIgEmediatedreactions.
Omalizumabisindicatedfortreatmentof'moderatetosevere'
persistentasthmainpatientswhoreacttoperennialallergens
(AllergicAsthma).
TreatmentwithOmalizumabhasshowntoreducethenumberof
exacerbationsinpatientswith
severeasthmaandmayimproveasthmacontrol.
Howeverthistreatmentisveryexpensiveandisonlysuitablefor
highlyselectedpatientswhoarenotcontrolledonmaximaldosesof
inhalertherapyandhaveahighcirculatingIgE(withinaspecified
range).
Omalizumabisusuallygivenasasubcutaneousinjectionfor2to4
weeksandmaybeusedinadultsandadolescentsmorethan12
yearsofage.


1019.Achildpresentswithrecurrent
pulmonaryinfectionsforhemoptysis
duetoassociatedbronchiectasiaand
onimagingcharacterizedbyunilateral
lossoflungvolumewithhyperlucency
onchestradiograph,reductionin
vascularityonCTscanofthechest.The
abdominalorgansarenormallyplace
(d)mostlilkelycauseis

a)Kartagener'ssyndrome
b)Swyer-James-MacLeodsyndrome
c)Mendelson'ssyndrome
d)Immotileciliasyndrome
CorrectAnswer-B
Answer-B.Swyer-James-MacLeodsyndrome
Swyer-James-MacLeodsyndromeorunilateralherlucentlun
ndromeisarareentityassociatedwithpostinfectiousbronchiolitis
obliteransoccurringinchildhood.
Itischaracterizedbyhypoplasiaand/oragenesisofthepulmonary
arteriesresultinginpulmonaryparenchymahypoperfusion,showing
acharacteristicradiologicalpattern,suchastranslucentor
hyperlucentunilaterallung.

1020.AllareimportantpathogenscausingpneumoniainCOPDpatients,
EXCEPT:
a)Haemophilusinfluenzae
b)Pseudomonasaeruginosa
c)Legionellaspp
d)Klebsiellapneumoniae
CorrectAnswer-D
AllareimportantpathogenscausingpneumoniainCOPDpatients
Haemophilusinfluenzae
Pseudomonasaeruginosa
Legionellaspp
S.pneumoniae
Moraxellacatarrhalis
Chlamydiapneumoniae
Klebsiellaisanimportantpathogencausingpneumoniainchronicalcoholism.
Ref:Harrison,E-18,P-2132

1021.Fibrosisofupperlobeisdueto
a)Pneumonia
b)ABPA
c)BronchiectasisinCOPD
d)Rheumatoidarthritis
CorrectAnswer-B
Answer-B.ABPA
Silicosis(Progressivemassivefibrosis
Sarcoidosis
Coalworkerpneumoconiosis
Ankylosingspondylitis
Radiation
Allergicbronchopulmonaryaspergillosis
Tuberculosis
Extrinsicallergicalveolitis

1022.WhatisnotseeninCRF
a)Hypercalcemia
b)Hyperkalemia
c)Hyperphosphatemia
d)Hypocalcemia
CorrectAnswer-A
Answer-A.Hypercalcemia
AbnormalitiesseeninCRF
Acidosis
Hyperkalemia
Anemia
Hypernatremia
Hyperphosphatemia
Hyperlipidemia
Hyponatremia
Hypocalcemia
Uremia

1023.CRFwithanemiabesttreatment:
a)OralIronTherapy
b)ErythropoietinStimulatingAgents
c)Bloodtransfusion
d)AndrogenicSteroids
CorrectAnswer-B
AnswerisB(ErythropoietinStimulatingAgents):
Erythropoiesis-stimulatingagents(ESAs)haveemergedasthe
treatmentofchoiceforanemiainchronicrenaldisease.
Erythropoiesis-stimulatingagents(ESAs)shouldbegiventoall
patientswithchronickidneydisease(CKD)withhaemoglobinlevels
consistentlybelowIIg/dl.
Thisappliesequallyto:
PatientswithCKD(stages1-5)developinganaemia
PatientswithCKDstage5treatedwithhaemodialysis(HD)or
peritonealdialysis(PD)
Transplantpatientswithchronicrenalinsufficiencyandanaemia.
StrategiesfortreatmentofAnemiainChronicRenalFailure
Erythropoiesis-stimulatingagents(ESAs)
Erythropoiesis-stimulatingagents(ESAs)haveemergedasthe
treatmentofchoiceforanemiainchronicrenaldisease.
Theyshouldbegiventoallpatientswithchronickidneydisease
(CKD)withhaemoglobin(Hb)levelsconsistentlybelow11g/dl
[haematocrit(Hct)<33%]
Allchronickidneydisease(CKD)patientswithrenalanaemia
undergoingtreatmentwithanerythropoiesis-stimulatingagent(ESA)
shouldbegivensupplementaryirontomaintainadequatebone
marrowironstores

Intravenousadministrationistheoptimumrouteforthedeliveryof
irontopatientswithCKD,asoralironispoorlyabsorbedinuremic
individuals.

BloodTransfusion
Redbloodcelltransfusionsshouldbeavoided,ifatallpossible,in
patientswithchronickidneydisease(CKD),especiallythose
awaitingkidneytransplantation.
Transfusionsshouldnotbegivenunlesspatientshaveoneormore
ofthefollowing:Symptomaticanaemia(fatigue,angina,dyspnoea)
and/orassociatedriskfactors(diabetes,heartfailure,coronary
arterydisease,arteriopathy)
Acuteworseningofanaemiaduetobloodloss(haemorrhageor
surgery)orhaemolysisSevereresistanceto,orhyporesponsiveness
toESAtherapy,e.g.duetothepresenceofahaematological
diseaseorsevereinflammatorysystemicdisease.
Androgens
PriortotheintroductionofESAtherapy,androgenswerewidely
usedinthetreatmentofrenalanaemia.
Thereisevidencethatandrogensmaypotentiatetheeffectof
exogenouserythropoieticproteinandalsostimulateerythropoiesis
byenhancingerythrocytestemcelldifferentiation
Theriskofliverdiseaseandmalignancy,virilisationandhirsutismin
women,priapisminmenanddisfiguringacneinpatientsofboth
sexesmayoutweighthebenefitsofandrogentherapyinmost
anaemicpatients.
Androgensmaybeaneffectivealternativetherapyincountries
whereESAsarenotavailable


1024.Distalrenaltubularacidosisis
associatedwith:
a)Oxalatestones
b)Citrate
c)Calciumstones
d)Uricacidstones
CorrectAnswer-C
AnswerisC(CalciumStones):
DistalRenalTubularAcidosisisassociatedwithincreasedin
cadenceofCalciumPhosphateStonesAlkalineurine,Hypercalciuria
andlowlevelsofurinarycitrateprecipitatecalciumphosphate
stonesinthekidneyinpatientswithDistalRenalTubularAcidosis
(Typel).
ProximalRenalTubularAcidosis(Type-2)isnotassociatedwith
increasedincidenceofRenalStonesdespiteHypercalciuriabecause
urinarycitratelevelsarenormalorhigh.


1025.ThemostcommonpresentationforIgA
nephropathyis:
a)Nephriticsyndrome
b)Nephriticsyndrome
c)Microscopichematuria
d)Repeatedgrosshematuria
CorrectAnswer-D
AnswerisD(RepeatedGrossHematuria):
ThemostcommonpresentationofIgANephropathyiswithrecurrent
episodesofGross(Macroscopic)Hematuriaduringorimmediately
followinganupperrespiratorytractinfection
'RecurrentattacksofPainlessGrossHematuriarepresentthe
classicclinicalpresentationofIgANephropathy'--Rudolph's
Paediatrics


1026.HIVrenalspecificnephropathyis:
a)FocalSegmentalGlomerulosclerosis
b)MembranoproliferativeGlomerulonephritis
c)MesangioproliferativeGlomerulonephritis
d)MembranousGlomerulonephritis
CorrectAnswer-A
AnswerisA(FocalSegmentalGlomerulosclerosis):
ThemostcharacteristicglomerulopathyinHIVisFocalSegmental
Glomerulosclerosis(FSGS)whichtypicallyrevealscollapseofthe
glomerularcapillarytuftcalledcollapsingglomerulopathy.
HIVassociatedNephropathyisasevererapidlyprogressive
collapsingformofFSGS.


1027.Theinitialtreatmentofchoicefor
secondaryhyperparathyroidisminrenal
osteodystrophyis:

a)Cinacalcet
b)Bisphosphonates
c)Calciumrestriction
d)Phosphatebinders
CorrectAnswer-D
AnswerisD(PhosphateBinders)
Theinitialtreatmentofsecondaryhyperparathyroidisminrenal
osteodystrophyismanagementofhighphosphatelevelsbydietary
restrictionandtheuseofPhosphatebinders
Theobjectivesofmanagementaretomaintainbloodlevelsof
calciumandphosphoroustoasclosetonormalaspossible,to
preventortreatestablishedhyperparathyroidismearlyandto
preventparathyroidhyperplasia.
PhosphateretentionbeginsearlyinthecourseofCKD,perhapsas
earlyasinstage2andparticipatesinthedevelopmentofsecondary
hyperparathyroidism.
Centraltothemanagementofhigh-turnoverbonediseaseis
controllingtheserumphosphatelevels.
Thismaybeachievedbydietaryphosphaterestrictionorbytheuse
ofphosphatebinders.
Phosphate-bindertherapyisrecommendedwhenserumphosphate
concentrationsareelevateddespitepatientcompliancewithdietary
phosphaterestriction.
Calcium-basedphosphatebindersareoftenrecommendedasthe


initialbindertherapy.
HighBoneTurnoverDisease
Boneturnover(theformationandremovalofbone)isincreaseddue
toaprocesscalledsecondaryhyperparathyroidism(SHPT).
Secondaryhyperparathyroidismrepresentsacommondisorderin
patientswithCKD.
Itdevelopsasaresultofhyperphosphatemia,hypocalcemiaand
impairedrenalvitaminDsynthesiswithreductioninserumcalcitriol
levels


1028.Maximumurinarycatheterinduced
infection
a)E.coli
b)Pseudomonas
c)Staphylococcusepidermidis
d)Proteus
CorrectAnswer-A
Answer-A.E.coli
Catheterassociatedurinarytractinfectionsrepresentthemost
commontypeofnosocomialinfection.
Mostcommonorganismcausingcatheterassociatedurinarytract
infectionsareE.coli.

1029.Mostcommonsymptomof
genitourinaryTB
a)Renalcolic
b)Increasedfrequency
c)Hematuria
d)Painfulmicturition
CorrectAnswer-B
Answer-B.Increasedfrequency
Theclinicalmanifestationsarevariable.
Theonsetofclinicallyevidentgenitourinarytuberculosisisusually
insidious.
Themostcommonsymptomsare:-
Dysuria,increasedfrequencyofurinationandgrosshematuria

1030.Whatistheuppermostintercostal
spaceusedforhepaticbiopsy:
a)5th
b)7th
c)9th
d)11th
CorrectAnswer-B
AnswerisB7th
Therightsurfaceoftheliverisincontactwiththediaphragm
oppositethe7thto11thribs.
Inneedlebiopsyoftheliverthroughtheintercostalroute,theneedle
maybeinsertedthroughthe6th7th,8th,9thor10thrightintercostal
spaceinthemid-axillaryline.
The8thand9thintercostalspacesaremostcommonlyused.
Insertioninthe6thor7thintercostalspacemayalsobeusedbutis
associatedwithriskofinjurytotheLung.
Needleistypicallyinsertedattheendofexpiration(Attempted
Apnoea).


1031.ThepreferredtestforconfirmingH.
pylorieradicationis:
a)Ureasebreathtest
b)Culture
c)Serologicaltest
d)Biopsyureasetest
CorrectAnswer-A
AnswerisA(UreaseBreathTest)
Thetestofchoicefordocumentingeradicationisureasebreathtest.
AssessmentofsuccessofTreatmentwithEradicationofH.
Pylori
shouldbedoneatleast4weeksaftercompletionofantiH.Pylori
therapy.Non-invasivetestsaretypicallypreferredforassessmentof
Eradication.Thetestofchoicefordocumentingeradicationisurease
breathtest.UreasebreathtestdetectsH.pyloriinfectionby
'bacterialureaseactivity'andremainspositivetillthebacteriahas
notbeeneradicatedwithtreatment.Thusureasebreathtest
becomesnegativeonlyaftereradicationoforganismfollowing
treatmentandnotwithchronicinfection
.

1032.Allofthefollowingareusedfor
treatmentofH.Pylori,except:
a)Gentamycin
b)Clarithromycin
c)Metronidazole
d)Amoxicillin
CorrectAnswer-A
AnswerisA(Gentamycin):
Gentamycinisnotusedinanyofthesuccessfulmulti-drugregimens
againstH.PyloriInfection.


1033.Bestprovocativetestfordiagnosisof
Gastrinomais:
a)Ca++infusiontest
b)Secretininjectiontest
c)ACTHstimulationtest
d)Steroidassay
CorrectAnswer-B
AnswerisB(Secretininjectiontest)
Gastrinomas(ZollingerEllisonSyndrome)arecharacterizedby
pepticulcerationduetohypersecretionofgastrinbyanon-betacell
tumor.Secretininjectiontestisthemostvaluableprovocativetestin
identifyingpatientswithZES.


1034.Wilson'sdiseaseischaracterizedby-
a)Increasedserumceruloplasmin
b)Decreasedcopperexcretioninurine
c)Increasedcopperinliver
d)Autosomaldominant
CorrectAnswer-C
Ans.is'c'i.e.,Increasedcopperinliver
DiagnosisofWilsondisease
Thegoldstandardfordiagnosisisliverbiopsywithquantitative
copperassay-->
concentrationofcopperinaliverbiopsysample>
200mg/gdryweight.
Othertestsare?
Serumceruloplasminlevel-->lowoUrine
copperexcretion-->increased
KFringsoDNA
Haplotypeanalysis.

1035.Allofthefollowingstatementsabout.
Wilson'sdiseasearetrue,EXCEPT-
a)Itisanautosomalrecessivedisorder
b)Serumceruloplasminlevelis<20mg/dl
c)Urinarycopperexcretionis
d)Zincacetateiseffectiveasmaintenancetherapy
CorrectAnswer-C
AnswerisC(Urinarycopperexcretionis<100R/day)
Urinecopperisanimportantdiagnostictoo.Symptomaticpatients
invariablyhaveurinecopperlevels>100p(>1.6umol)per24
hours.
Wilson'sdiseaseisanautosomalrecessivedisorder
ItiscausedbyamutationofageneonchromosomeBQwhich
promotesCuexcretion(ATP7Bgene)
SymptomaticpatientswithWilson'sdiseaseinvariablyhaveurinary
copperexcretionof>100lig,per24hours(>1.6innol/24hr)
Zincisthetreatmentofchoiceformaintainancetherapyin
Wilson'sdisease
ZincisthetreatmentofchoiceinWilson'sdiseasefor
A.Initialtherapyinpatientswithhepatitiswithoutdecompensation(2
A.Maintainancetherapy
B.Presymptomaticpatient
C.Pediatricpatients
D.Pregnantpatients


1036.Diagnosisofcarcinoidtumourisdone
Urinaryestimationof:
a)VMA
b)Metanephrines
c)Catecholamines
d)5HIAA
CorrectAnswer-D
AnswerisD(5HIAA):
CarcinoidTumors
Carcinoidtumorsareassociatedwithelevatedlevelsofmetabolites
ofTryptophan/serotoninwhichinclude5HIAA,5HTand5HTP.
Pheochromocytomas
Pheochromocytomasareassociatedwithelevatedlevelsof
catecholaminesandtheirmetaboliteswhichincludeVanillylmandelic
acid(VMA)andmetanephrines
ThediagnosisofTypicalcarcinoidsyndromeissuggestedby
elevatedlevelsof5HIAAThediagnosisofAtypicalCarcinoid
Syndromeissuggestedbyelevatedlevelsof5HTP.
establishedby
elevatedlevelsof5HIAA.


1037.InZollingerEllisonsyndromewhatis
raised?
a)Insulin
b)VIP
c)Gastrin
d)Glucagon
CorrectAnswer-C
Ans.is'c'i.e.,Gastrin
ZollingerEllisonsyndrome?
Severepepticulcerdiseasesecondarytogastricacid
hypersecretionduetounregulatedgastrinreleasefromanon13cell
endocrinetumour(gastrinoma),definesthecomponentsofZollinger
Ellisonsyndrome.
PathophysiologyofZollingerEllisonsyndrome
ThedrivingforceresponsibleforclinicalmanifestationsofZollinger
EllisonsyndromeishypergastrinemiaoriginatingfromGastrinoma
(autonomusneoplasm,non[3cellneoplasm)
Gastrinoma
Hypergastrinemia
Hyperacidemia
Pepticulcer,erosiveesophagitisanddiarrhoea
OtherimportantcharacteristicofGastrinoma
o
Over80%ofthesetumoursareseeninGastrinomatriangle?
(triangleformedbetweenduodenumandpancreas)mostofthem
areseenintheheadofpancreas.
oAbout2/3'ofthesetumoursaremalignant?.
oAboutonehalfofthesetumoursaremultiple?.

oAboutonefourthofthepatientshavemultipleendocrineneoplasia
(MENI)syndromewithtumoursofparathyroid,pituitaryand
pancreaticisletsbeingpresent.
Remember:
Mostcommonsiteofgastrinoma'sis
Duodenum(50-70%),(Pancreas20-40%)
MostcommonhormonetobesecretedACTH
besidesgastrinis
Mostcommonsiteofpepticulcersproducedisls'partof
Duodenum.
MostvaluableprovocativetestinTheSecretin
injectiontests.identifyingpatientswithZESis
Basalacidoutputisgreaterthan60%ofoutpuBAO>MAO
inducedbymaximalstimulation
Thetermpancreaticendocrinetumourismisnomerbecausethese
tumourscanoccureitheralmostexclusivelyinthepancreasorat
bothpancreaticandextrapancreaticsites

1038.Allfollowingareat-riskgroupadults
meritingHepatitisBvaccinationinlow
endemicareasexcept:

a)Patientsonchronichemodialysis
b)Diabeticsoninsulin
c)Medical/nursingpersonnel
d)Patientswithchronicliverdisease
CorrectAnswer-B
AnswerisB(DiabeticsonInsulin):
Behavioral:Sexuallyactivepersonswhoarenotinalong-term,
mutuallymonogamousrelationship(e.g.,personswithmorethan
onesexpartnerduringtheprevious5months):personsseeking
evaluationortreatmentforasexuallytransmitteddisease(STD):
currentorrecentinjection-drugusers;andmenwhohavesexwith
men.
Occupational:Healthcarepersonnelandpublic-safetyworkerswho
areexposedtobloodorotherpotentiallyinfectiousbodyfluids.
Medical:Personswithend-stagerenaldisease,includingpatients
receivinghemodialysis;personwithHIVinfection;andpersonswith
chronicliverdisease.
Other:Householdcontactsandsexpartnersofpersonswithchronic
HBVinfection;clientsandstaffmembersofinstitutionsforpersons
withdevelopmentaldisabilities;andinternationaltravellersto
countrieswithhighorintermediateprevalenceofchronicHBV
infection.
HepatitisBvaccinationisrecommendedforalladultsinthefollowing
settings:STDtreatmentfacilities;HIVtestingandtreatmentfacilities;

facilitiesprovidingdrug-abusetreatmentandpreventionservices;
healthcaresettingstargetingservicestoinjection--drugusersor
menwhohavesexwithmen;correctionalfacilities;end-stagerenal
diseaseprogramsandfacilitiesforchronichemodialysispatients;
andinstitutionsandnonresidentialday-carefacilitiesforpersons
withdevelopmentaldisabilities.
Administermissingdosestocompletea3-doseseriesofhepatitisB
vaccinetothosepersonsnotvaccinatedornotcompletely
vaccinated.Theseseconddoseshouldbeadministered1month
afterthefirstdose;thethirddoseshouldbegivenatleast2months
aftertheseconddose(andatleast4monthsafterthefirstdose).If
thecombinedhepatitisAandhepatitisBvaccine(Twinrix)isused,
administer3dosesat0,1,and6months;alternatively,a4-dose
Twinrixschedule,administeredondays0,7,and21to30,followed
byaboosterdoseatmonth12maybeused.
Adultpatientsreceivinghemodialysisorwithother
immunocompromisingconditionsshouldreceive1doseof40pg/mL
(RecombivaxHB)administeredona3-dosescheduleor2dosesof
20Ug/mL(Engerix-B)administeredsimultaneouslyona4-dose
scheduleat0,1,2,and6months.

1039.SchillingtestisAbnormalin:
a)Intrinsicfactordeficiency
b)Amylasedeficiency
c)Lipasedeficiency
d)Pancreaticendocrineinsufficiency
CorrectAnswer-A
AnswerisA(Intrinsicfactordeficiency)
Shilling'stestistypicallydonetodeterminethecauseofcobalamine
malabsorption(VitaminB12malabsorption)VitaminB12absorption
(Schilling)testistypicallyusedtodiagnoseconditionsinwhich
intrinsicfactor(IF)maybeabsent,suchasperniciousanemiaor
gastricatrophy.AdiagnosisofIntrinsicfactordeficiencycanbe
establishedifradiolabelledcobalamine(VitaminB12)appearsin
urineafteradministrationofIntrinsicfactor.
Sincecobalamineabsorptionrequiresmultiplestepsincluding
gastric,pancreaticandilealprocesses,theSchillingtestcanalso
beusedtoassesstheintegrityoftheseotherorgans.
DietaryvitaminB12isboundinthestomachtoanendogenous
proteincalledRprotein.PancreaticenzymesdegradetheRprotein
intheproximalsmallbowelandloweritsaffinityforvitaminB12
resultingintherapidtransferofB12toIF;TheIF-B12Complex
continuestotheterminalileum,whereitbindstospecificreceptors
onthesurfaceoftheepithelialcells.
Thuslackofintrinsicfactor,lackofsufficientpancreaticenzymes
(pancreaticexocrinedysfunction)orpresenceofterminalheal
mucosaldiseasemayallresultinabnormalvitaminB12excretion.




1040.Secretorydiarrheaisnotseenin:
a)Phenolphthalein
b)Celiacdisease
c)Cholera
d)Addison'sDisease
CorrectAnswer-B
AnswerisB(CeliacDisease)
CeliacDiseaseisassociatedwithSteatorrhealdiarrhoeafrom
mucosa!malabsorption.

SecretoryDiarrhea
CertainBacterialInfection
VibrioCholera
EnterotoxigenicE.Coli
NonOsmoticStimulantLaxatives
HormoneProducingEndocrineTumors
Carcinoid,
VlPomas,
Gastrinomas,
MedullaryCarcinomaThyroid(Calcitonin)
Bileacids(endogenouslaxatives)
Bowelresection/diseaseorfistula
Addison'sDisease
CongenitalElectrolyteAbsorptiondefects
ChronicAlcoholIngestion
DiabeticDiarrhea

SteatorrhealDiarrhea
Intraluminalmaldigestion
Pancreaticexocrineinsufficiency,
Bacterialovergrowth,
Bariatricsurgery,
Liverdisease
Mucosalmalabsorption
Celiacsprue,
Whipple'sdisease,
Infections,
Abetalipoproteinemia,
Ischemia
Postmucosalobstruction
(1?or2?lymphaticobstruction)

1041.Anominalaphasiaisduetodefectin
a)Leftinferiorfrontal
b)Parietal
c)Temporaloccipital
d)Cerebellum
CorrectAnswer-C
Answer-C.Temporaloccipital
Anomiacanbegeneticorcausedbydamagetovariouspartsofthe
parietallobeorthetemporallobeofthebrainbyanaccidentor
stroke,orabraintumor.

1042.Unabletoconsolidatelongterm
memory.Whichlobeofthebrainis
involved

a)Frontal
b)Parietal
c)Temporal
d)Occipital
CorrectAnswer-C
Answer-C.Parietal
Medialtemporallobeistheareaofbrainresponsiblefor
consolidation,i.e.processingofshorttermmemoryintolongterm
memory.

1043.Gerstmannssyndromeallexcept
a)Acalculia
b)Agraphia
c)Aphasia
d)Agnosia
CorrectAnswer-C
Answer-C.Aphasia
Gerstmannsyndromeconsistsof:

1. Agraphia
2. Acalculia
3. Fingeragnosia
4. Leftrightdisorientation

1044.Lateralmedullarysyndromeiscaused
bythrombosisof:
a)Anteriorinferiorcerebralartery
b)Posteriorinferiorcerebellarartery
c)Vertebralartery
d)bandc
CorrectAnswer-D
AnswerisC>B(Vertebralartery>Posteriorinferiorcerebellar
artery):
`Mostcasesresultfromipsilateralvertebralarteryocclusion;inthe
reminderocclusionofposteriorinferiorcerebellararteryis
responsible'?Harrison
VesselocclusionthatresultinLateralMedullarysyndrome:
-Vertebral(mostcommon)
-Posteriorinferiorcerebellar(2ndmostcommon)
-Superior,middleorInferiorlateralmedullaryarteries
Vertebralartery
Thevertebralarteryconsistsoffoursegments.Thefourthsegment
coursesupwardtojointtheothervertebralarterytoformthebasilar
artery.
Onlythissegmentgivesrisetobranchesthatsupplythebrainstem
andcerebellum.
Embolicocclusionorthrombosisofthefourthsegmentis
responsibleforthissyndromeQ
Posteriorinferiorcerebellarartery
Posteriorinferiorcerebellararteryinitsproximalpartsuppliesthe
lateralmedullaandinitsdistalbranchestheinferiorsurfaceof

cerebellum.

1045.Pontinehemorrhagemostcommon
causeis
a)Hypertension
b)Diabetes
c)Trauma
d)Aneurysmalrupture
CorrectAnswer-A
Answer-A.Hypertension
Mostcommoncauseofpontinehemorrhageishypertension.

1046.Anadulthypertensivemalepresented
withsuddenonsetsevereheadache
andvomiting.Onexamination,thereis
markedneckrigidityandnofocal
neurologicaldeficitwasfound.The
symptomsaremostlikelydueto:

a)Intracerebralparenchymalhemorrhage
b)Ischemicstroke
c)Meningitis
d)Subarachnoidhemorrhage
CorrectAnswer-D
Ans.d.Subarachnoidhemorrhage
Themostlikelydiagnosishereisasubarachnoidbleed(SAH).
Themostcommoncauseistraumaandismanagedconservatively.
Thesecondmostcommoncauseisaruptureofaberryaneurysm.
Thetypicalpresentationofasubarachnoidhemorrhageincludesa
`thunderclap'headache.
Meningiticfeaturesofneckstiffnessandphotophobiaoften
developoverhours.
Computedtomography(CT)istheinvestigationofchoice.
LumbarpunctureshouldbeperformediftheCTscanfailsto
establishthediagnosisofSAH;itshowsaxanthochromicblood
picture.
Delayedischemicneurologicaldeficit(DIND)isattributedto
vasospasmofthecerebralvasculaturetypicallydeveloping3?10
daysfollowingictus.Itisthemaincauseofapooroutcome.

Endovasculartreatment("coiling")isgenerallypreferredover
craniotomyandclippingforaneurysmsamenabletothisapproach.

1047.Predominantlysensoryneuropathy
is/arecausedby:
a)Cisplatin
b)Pyridoxineexcess
c)Suramin
d)aandb
CorrectAnswer-D
AnswerisAandB(CisplatinandPyridoxineexcess):
CisplatinandPyridoxineareassociatedwithpredominantlysensory
neuropathies.


1048.Dailytemperaturevariationin
remmitentfeveris
a)<0.5C
b)>1?C
c)<1.0C
d)>2C
CorrectAnswer-B
Answer-B.>1?C
Continuousfever
Temperatureremainsabovenormalthroughoutthedayanddoes
notfluctuatemorethan1?Cin24hours.Intermittentfever
Thetemperatureelevationispresentonlyforacertainperiod,later
backtonormale.g.malaria,kalaazarsepticaemia.

1049.Hyperthermia
a)Temperature>41.5
b)>40withautonomicdysfunctuion
c)Nochangeinhypothalamicthermostat
d)Failureofthermoregulation
CorrectAnswer-D
ANswer-D.Failureofthermoregulation
Hyperthermiaisdefinedaselevationofcorebodytemperature
abovethenormaldiurnalrangeof36to37.5?Cduetofailureof
thermoregulationatthelevelofhypothalamus.
Hyperthermiaisnotsynonymouswiththemorecommonsign
offever,whichisinducedbycytokineactivationduringinflammation,
andregulatedatthelevelofthehypothalamus.

1050.Myastheniagravisisassociatedwith
a)AntibodiesagainstAchreceptors
b)Decreasedmyosin
c)AbsenttroponinC
d)Increasedmyoneuraljunctiontransmission
CorrectAnswer-A
Answer-A.AntibodiesagainstAchreceptors
Thecharacteristicpathologicalfeatureofmyastheniagravisis
presenceofantibodiesagainstacetylcholinereceptors.
Theseautoantibodiesagainsttheacetylcholinereceptorsleadto
lossoffunctionalacetylcholinereceptorsattheneuromuscular
junction.

1051.EatonLambertsyndromeisseenwith-
a)Cabreast
b)Caliver
c)Calung
d)CNStumors
CorrectAnswer-C
Answer-C.Calung
Paraneoplasticsyndrome
Associatedwith
Smallcellcarcinomalung
Certainautoimmunediseases

1052.Lesionofglobuspalliduscauses
a)Chorea
b)Athetosis
c)Hemibalismus
d)Flexiondystonia
CorrectAnswer-B
Answer-B.Athetosis
Athetosis-Globuspallidus(mainly)andSubthalamicnucleus.
Lesionattheglobuspallidusand
striatum
causeathetosis,whichischaracterizedby
continuous,slowwrithingmovements.
Chorea:rapid,involuntarydancingmovementsMostcommonly,
thelesionisinthestriatum.
Ballism:involuntaryflailing,intenseandviolentmovements.The
movementsareoflargeamplitudeandpredominantlyinvolves
proximalmuscles.
Thelesioninthesubthalamicnucleus.
Athetosis:continuous,slowwrithingmovementsLesionatglobus
pallidusandstriatum.

1053.Mostcommontumorassociatedwith
NF1
a)Opticglioma
b)2ndnerveschwannoma
c)Astrocytoma
d)Bilateralacousticneuroma
CorrectAnswer-A
Answer-A.Opticglioma
"Opticpathwaygliomas"arethepredominanttypeofneoplasm
associatedwithneurofibromatosistypeIbutothercentralnervous
systemandnonCNStumourscanoccur.

1054.CSFpictureinviralmeningitis
a)Lymphocyticpleocytosis
b)WBCcount>1500/mL
c)Sugarisreduced
d)Proteinisdecreased
CorrectAnswer-A
Answer-A.Lymphocyticpleocytosis
Normal Bacterial
Viral
Opening
7-18
>30
NormalorMildlyIncreased
Pressure
Clear,
Appearance
Turbid
Clear
Colorless
Protein
23-38
Increased
NormaltoDecreased
(mg/dl)
2/3rds
Glucose
Serum
Decreased
Normal
(mmol/L)
Glucose
Positive160-90%of
GramStain Negative
Negative
Cases)
Glucose
CSF:Serum 0.6
<0.4
>0.6
Ratio
WhiteCell
cells
PredominatelyNeutrophils PredominatelyLymphocytes
Count

1055.InherpesencephalitisA/E
a)Focalsymptomscommon
b)Temporallobeinvolved
c)MRIisdiagnostic
d)EEGnotdiagnostic
CorrectAnswer-D
Answer-D.EEGnotdiagnostic
Diagnosis
MostsensitiveandspecificinvestigationforHSV-1encephaliticis
MRI
Incontrast,cranialCTscanshaveonly50%sensitivityandthattoo
earlyinthedisease.EEGfindingsinHSV-1encephalitis

1056.Whichofthefollowingisnotseenin
HereditarySpherocytosis
a)DirectCoomb'sPositive
b)IncreasedOsmoticFragility
c)Splenomegaly
d)Gallstones
CorrectAnswer-A
Ans.is'a'i.e.,DirectCoomb'spositive
HereditarySpherocytosis
Membranecytoskeletonthatliescloselyopposedtotheinternal
surfaceoftheplasmamembrane,isresponsibleforelasticityand
maintenanceofRBCshape.
Membraneskeletonconsists:?
SpectrinThechiefproteincomponent
responsibleforbiconcaveshape.
Ankyrinandband4-2Bindsspectrintoband3
Band3Atransmembraneiontransport
protein.
Band4.1BindsspectrintoglycophorinA,
atransmembraneprotein.
Hereditaryspherocytosisisanautosomaldominantdisorder
characterizedbyintrinsicdefectsinredcellmembrane.Thisresults
inproductionofredcellsthataresphere(spherocytes)ratherthan
biconcave.
Themutationmostcommonlyinvolvesthegenecodingforankyrin,
followedbyBand-3(anionictransportchannel),spectrin,andBand
4.2(alsocalledpalladin).


Alsoknow
Mostcommon,defectinhereditaryelliptocytosisisinspectrin
PathogensisofHereditaryspherocytosis
Lossofmembranecytoskeletonproteins(ankyrin,spectrin,Band3,
4.2)resultsinreducedmembranestability.Reducedmembrane
stabilityleadstospontaneouslossofmembranefragmentsduring
exposuretoshearstressesinthecirculation.Thelossofmembrane
relativetocytoplasmforcesthecellstoassumethesmallest
possiblediameterforagivenvolumecellsbecome
microspherocytes.
Becauseoftheirspheroidalshapeandreducedmembraneplasticity,
redcellsbecomelessdeformableandaretrappedintospleenas
theyareunabletopassthroughtheinterendothelialfenestrationsof
thevenoussinusoids.Inthesplenicsinusoides,redcellsare
phagocytosedbyREcellsExtravascularhemolysis.
ClinicalfeaturesofHereditoryspherocytosis
Theclinicalfeaturesarethoseofextravascularhemolysis:
AnemiaMildtomoderate
Jaundice(Mainlyindirectbilirubin)Splenomegaly
GallstonesElevatedexcretionof
bilirubinpromotesformationofpigmentstone.
LegulcerRareclinical
manifestation.
AplasticcrisisTriggeredbyparvo-
virusinfection.
Laboratoryfindings
Spherocytosis-->Peripheralsmearshowsmicrospherocyteswhich
aresmallRBCswithoutcentralpallor(Normallycentral1/3palloris
presentinredcells).
MCV4
MCHCr
Increasedunconjugatebilirubin
Urineurobilinogen1'
Stools
tercobilinogen
Reticulocytosis
-4Asseenwithanytypeofhemolyticanemia.
Hemoglobin1
SerumHeptoglobin-->
Nonnaltodecreased.

Increasedosmoticfragilityonpinktest.
C
oomb'stestisusedtodistinguishhereditaryspherocytosisfrom
autoimmunehemolyticanemias.
Autoimmunehemolyticanemiasarecoomb'spositive_whereas
hereditaryspherocytosisiscoomb'snegative.


1057.HaemoglobinFisraisedin:
a)Juvenilechronicmyeloidleukemia
b)Hereditaryspherocytosis
c)Congenitalredcellaplasia
d)Mysthaniagravis
CorrectAnswer-A
AnswerisA(JuvenileCML)
FetalHaemoglobinlevels(HbF)areincreasedinmostcasesof
JuvenileCML?
CausesofRaisedHbFlevels(InterpretationofDiagnosticTest
8th/411,412)
Haemoglobinopathies
-thalassemiamajor
-thalassemiaminor
-Sicklecelldisease
HereditaryPersistanceofHbF
Anemia:
-NonHereditaryrefractorynormoblasticanemia
-Perniciousanemia
-Aplasticanemia
LeukemiaspeciallyJuvenileMyeloidLeukemia
Multiplemyeloma
Molarpregnancy
PatientswithTrisomy13orTrisomy21(Down'ssyndrome)
Somechronicviralinfections(egCMV,EBV)

1058.Whichofthefollowingprovide
protectionagainstmalariaallexcept
a)Duffybloodgroup
b)Sicklecellanemia
c)Thalassemia
d)PNH
CorrectAnswer-D
Answer-D.PNH
Redcellsurfaceantigenthatofferprotectionagainstmalaria
Duffybloodgroupsystem
ABO(H)bloodgroupsystem
Glycophorins
Gerbichantigen
ComplementreceptortypeI
Knopsbloodgroup
Abnormalitiesoftheredcellcytoskeletonwhichmayoffer
protectioninclude
South-eastasiaovalocytosisHereditaryelliptocytosis
HereditaryspherocytosisSicklecellHbA/S

1059.ChemotherapeuticAgentofChoicefor
thetreatmentofCMLis:
a)Imatinib
b)Vincristine
c)Cyclophosphamide
d)Methotrexate
CorrectAnswer-A
AnswerisA(Imatinib):
TyrosineKinaseInhibitors(Imatinib)arethechemotherapeutic
agentsforchoiceinthemanagementofCML.TyrosineKinase
inhibitorstargetthe'constitutivelyactivetyrosinekinase'implicated
inthepathogenesisofCML.Althoughtheydonotcurethedisease,
theseagentsareabletoachievelongtermcontrolofCMLinthe
majorityofpatients.Mostrecenttexts(IncludingHarrisons)
recommendTyrosineKinaseInhibitors(Imatinib)astheinitial
treatmentofchoicefornewlydiagnosedCMLreservingAllogeneic
StemCellTransplantation(SCT)forthosewhodevelopImatinib
Resistance.


1060.Thromboticthrombocytopenicpurpura
isasyndromecharacterizedby:
a)Thrombocytosis,anemia,neurologicabnormalities,progressive
renalfailureandfever.
b)Thrombocytopenia,anemia,neurologicabnormalities,
progressivehepaticfailureandfever
c)Thrombocytosis,anemianeurologicabnormalities,progressive
renalfailureandfever
d)Thrombocytopenia,anemia,neurologicalabnormalities,
progressiverenalfailureandfever
CorrectAnswer-D
AnswerisD(Thrombocytopenia,anemia,neurological
abnormalities,progressiverenalfailureandfever)
ThromboticThrombocytopenicPurpura(TTP)ischaracterized
clinicallybythePentadofMicroangiopathicHemolyticAnemia,
Thrombocytopenia.DecreasedRenalFunction.Disturbed
NeurologicalfunctionandFever.


1061.Allthefollowingaretrueaboutmultiple
myelomaexcept:
a)Osteolyticbonedisease
b)t(8-14)translocation
c)Lightchainproliferation
d)Bence-Jonesproteinsinurine
CorrectAnswer-B
AnswerisB(t(8-14)translocation)
Avarietyofchromosomalalterationshavebeenfoundinpatients
withMultiplemyeloma.Themostcommontranslocationist(11;14)
(q13;q32).13q14deletionsand17p13deletionsand11q
abnormalitiespredominate.Translocation1(8-14)hasnotbeen
mentioned.
CompleteImmunoglobulinchaincomprisesofbothheavychains
andlightchains.ButinMultiplemyelomathereisexcessproduction
oflightchainse
overheavychains.
TheselightchainsareeliminatedinurineasBenceJonesproteinQ
ProteincastinurinearethusmadeupoflightchainsonlyQ
(not
completeimmunoglobulinchains).
Bonelesionsinmultiplemyelomaarelyticinnatureandarerarely
associatedwithosteoblasticnewboneformation.'-Harrison.
BonelesionsinMMarecausedbytheproliferationoftumorcells
andactivationofosteoclaststhatdestroythebone.'Bonepainisthe
mostcommonsymptominMMaffecting70%ofpatientsQ'-Harrison


1062.Theimmunoglobulinmostcommonly
involvedinMultipleMyelomais:
a)IgG
b)IgM
c)IgA
d)IgD
CorrectAnswer-A
AnswerisA(IgG)
TheMcomponentinMultipleMyelomacanbemadeupofthe
immunoglobulinsIgG,IgM,IgD,IgA,andIgE;lightchainsalone;or
heavychainsalone.IgGMyelomaisthemostcommonformof
MultipleMyelomawhileIgD(2%)andIgE(Rare)aretheleast
common.
Distributionofimmunoglobulintypesinpatientswithmultiple
myeloma

Typeofprotein
Percentage(%)
IgG
52
IgA
22
IgM
12
IgD
2
IgE
Rare

1063.InitialDrugofchoiceforsuspected
caseofacuteadrenalinsufficiencyis:
a)Norepinephrine
b)Hydrocortisone
c)Dexamethasone
d)Fludrocortisones
CorrectAnswer-C
AnswerisC(Dexamethasone):
Thetreatmentofchoiceforacuteadrenalinsufficiencyis
GlucocorticoidReplacementTherapy.Incaseswherethediagnosis
ofacuteadrenalinsufficiencyissuspected(notconfirmed)
Dexamethasoneispreferredastheinitialsteroidofchoicebecause
Dexamethasonedoesnotcompetewiththecortisolassay.
Cosyntropinstimulationtestingmaybeperformedwhilethepatient
isontreatment.


1064.Apregnancywomanisdiagnosedto
sufferingfromGraves'disease.The
mostappropriatetherapyforherwould
be:

a)Radioiodinetherapy
b)Totalthyroidectomy
c)Carbimazoleparenteral
d)Propylthiouraciloral
CorrectAnswer-D
AnswerisD(Propylthiouracil):
Propylthiouracil(PTU)isnotassociatedwithanincreasedriskof
congenitalmalformationsandisconsideredthedrugofchoicefor
treatinghyperthyroidismisPregnancy.
HyperthyroidisminPregnancy
MaternalHyperthyroidisminPregnancyisusuallyduetoGrave's
Disease.TRAbcrossestheplacentaandifmotheristhyrotoxicit
mustbeassumedthatthefoetusissimilarlyaffected
ThetreatmentofchoiceforthyrotoxicosisinPregnancyistherapy
withsafeAntithyroidDrugs
Thionamides(Carbimazole/Propylthiouracil)areequallyeffectivein
controllingGrave'sHyperthyroidisminPregnancyandare
consideredthedrugsofchoice.
AmongstCarbimazole(Methimazole)andPropylthiouracil,
Propylthiouracilistypicallythepreferredagent(Traditionaldrugof
choice)
RadioactiveIodineTherapyiscontraindicatedinpregnancyasit
maydestroythefetalthyroidThyroidectomy(Surgery)israrely


requiredduringPregnancy.Whenindicatedpreoperativetreatment
withantithyroiddrugsandiodineisundertakenandsurgeryis
performedduringthesecondtrimester

Propylthiouracil
EffectiveincontrollingGrave'sHyperthyroidisminPregnancy
Notassociatedwithincreasedriskofcongenitalmalformation
(AplasiaCutisCongenitahasnotreportedwiththeuseof
Propylthiouracil))
ConsideredthedrugofchoicefortreatmentofHyperthyroidismin
pregnancy
ConsideredthedrugofchoiceformothersduringBreastfeeding
(TransferredtothemilkonetenthasmuchasCarbimazole)
EffectiveincontrollingGrave'sHyperthyroidisminPregnancy
AplasiaCutisCongenitaisararedisorderreportedinneonatesof
motherswhoreceivedMethimazole(Carbimazole)during
pregnancy.
(Consensus:Insufficientdatatoestablishadirectcausalrelationship)
ConsideredasaneffectivealternativewherePropylthiouracilisnot
availableorcannotbeusedforanyreason
Maybeusedinmothersduringbreastfeedingatalowdose
(TransferredtomilkmorethanPropylthiouracilbutusuallydoesnot
adverselyaffecttheinfant'sthyroidfunction)


1065.Richner-Hanhartsyndromeis
a)Autosomaldominant
b)Occularandcutaneousfeatures
c)Associatedwithabnormalityinlipidmetabolism
d)Normalmentalfunction
CorrectAnswer-B
Answer-B.Occularandcutaneousfeatures
Rareautosomalrecessivedisorderoftyrosinemetabolismdueto
deficiencyofthecytosolicfractionofhepatictyrosineamino
transferase.
Occursduetodeficiencyof"Tyrosineaminotransferase".
Mentalretardation.
Thepatienthashighurinarytyrosinelevelsalongwithhighplasma
tyrosinelevels.
Thesepatientsrespondsdramaticallytodietaryrestrictionofthe
aminoacidsphenylalanineandtyrosine

1066.Investigationofchoicein
pheochromocytomais:
a)CTscan
b)Urinarycatecholamines
c)MIBGscan
d)MRIScan
CorrectAnswer-B
AnswerisB(UrinaryCatecholamines):
Pheochromocytomassynthesizeandstorecatecholamineswhich
includenorepinephrine,epinephrineanddopamine.The
investigationofchoicefordiagnosisofPheochromocytomasis
determinationofelevatedlevelsofcatecholaminesandtheir
methylatedmetabolites(metanephrines)intheplasmaandurine.
CTscan,MRIandMBIGScintographyareallusefuldiagnostic
modalitiesforlocalizationofpheochromocytomaoncethediagnosis
isestablished.


1067.Thepredominantsymptom/signof
pheochromocytomais:
a)Sweating
b)Weightloss
c)Orthostatichypotension
d)Episodichypertension
CorrectAnswer-D
AnswerisD(EpisodicHypertension):
ThepredominantmanifestationofPheochromocytomais
HypertensionwhichclassicallypresentsasEpisodicHypertension
(SustainedHypertensionandOrthostaticHypotensionmayalsobe
seen).
'ThedominantsignisHypertension.Classicallypatientshave
episodichypertension,butsustainedhypertensionisalsocommon'-
Harrison


1068.Primaryhyperparathyroidismis
suggestedbyallofthefollowing,
except:

a)Increasedserumcalcium
b)Lowurinarycalcium
c)IncreasedPTH
d)IncreasedC-AMP
CorrectAnswer-B
AnswerisB(LowUrinaryCalcium):
PrimaryHyperparathyroidismisassociatedwithnormalorincreased
urinarycalciumlevels.
IncreasedPTHandIncreasedSerumcalciuminassociationwith
highlevelsofurinarycalciumsuggestadiagnosisofPrimary
Hyperparathyroidism
IncreasedPTHandIncreasedSerumcalciuminassociationwithlow
levelsofurinarycalciumsuggestadiagnosisofFamilial
HvpocalciuricHvpercalcemia(FHH)

Disorder
S-
S-
PTHrP Urine
Urinary
Ca2? PHOS
Calcium
Ca2+
Intact
Creatinine
PTH
Clearance
ratio

PrimaryHPTH 1'
sl,(or 1'(or ,I,
U-Ca2'>
>0.02
N) N)
100mg/24h
Familial
l'
.I(or 1'(or ,I,
U-Ca2+
<0.01
benign
N) N)
<100mg/24h
hypercalcemia
Thisclearanceratioiscalculatedfromsimultaneousfastingserum

Thisclearanceratioiscalculatedfromsimultaneousfastingserum
andurineCaandcreatininemeasurements.Theurinesamplecan
befromaspotora24hcollection.Theclearanceratioiscalculated
asfollows:
UrineCa(mg/24h)xplasmacreatinine(mg/dl)/plasmaCa(mg/dL)x
urinecreatinine(mg/24h)

1069.ThemostclassicalsymptomofVIPOMA
is:
a)Gallstones
b)Secretorydiarrhea
c)Steatorrhea
d)Flushing
CorrectAnswer-B
AnswerisB(SecretoryDiarrhoea)
TheprinciplefeatureofVIPOMAislargevolumesecretory
Diarrhoea.
Diarrhoeaissecretoryinnature,persistsduringfastingandisalmost
alwaysgreaterthan>ILitreperday(>3Litresperdayin70
percent).Astoolvolumelessthan700mlperdayisproposedto
excludethediagnosis.Mostpatientsdonothaveaccompanying
Steatorrhea.

VIPOMAS(Verner-MorrisonSyndrome/PancreaticCholera/WDHA
Syndrome)
VIPomasaretumoursthatsecretelargeamountsofVasoactive
IntestinalPeptide(VIP)
VIPisanimportantneurotransmitterubiquitouslypresentintheCNS
andGIT
ThemostcommonlocationofVlPomasisthePancreas
Mostcommonsitewithinthepancreasisthepancreatictail
UsuallySolitary
UsuallyMalignant(37-68%havehepaticmetastasisatdiagnosis
VIP
StimulatesSmallIntestinalchloridesecretion
Stimulatessmoothmusclecontractility


Inhibitsacidsecretion
Hasvasodilatoryeffects

TypicalFeatures
(WDHA)
WateryDiarrhoea(LargeVolumeSecretory
Diarrhoealeadingto
dehydration)
Hypokalemia(Diarrhoeasevereenoughto
causehypokalemia)
Achlorhydria(Hypochlorhydriafrom
increasedsmallintestinal
chloridesecretion)
TheprincipleSymptomsarelargevolume
diarrhoea(100%)
severeenoughtocausehypokalemia(80-
100%),dehydration
(83%),hypochlorhydria(54-76%)and
flushing(20%)
Mostpatientsdonothaveaccompanying
Steatorrhea

OtherFeatures
IncreasedstoolVolumeduetoincreasedsecretionofsodiumand
potassiumwhichwiththeanionaccountforosmolalityofthestool
Hyperglycemia(25-50%
Hypercalcemia(25-50%)
ThediagnosisrequiresdemonstrationofanelevatedplasmaVIP
levelandthepresenceoflargevolumesecretorydiarrhea.


1070.Fabry'sdiseaseaffects
a)ER
b)Lysosome
c)Mitochondria
d)Cellmembrane
CorrectAnswer-B
Answer-B.Lysosome
Fabrydisease,alsocalledAnderson-Fabrydisease,isthesecond
mostprevalentlysosomalstoragedisorderafterGaucherdisease.
ItisanX-linkedinbornerroroftheglycosphingolipidmetabolic
pathway.Thisresultsinaccumulationofglobotriaosylceramide
(Gb3)withinlysosomesinawidevarietyofcells,therebyleadingto
theproteanmanifestationsofthedisease.

1071.Granulomatousconditioncausing
hypercalcemiaincludeallofthe
following,except:

a)TB
b)Sarciodosis
c)Berylliosis
d)SLE
CorrectAnswer-D
AnswerisD(SLE):
SLEisnotclassifiedasagranulomatousdiseaseandisarare
causeofHypercalcemia(DisseminatedSLE).Sarcoidosis,
Tuberculosis(TB)andBerylliosisaretypicalGranulomatous
disorderscausinghypercalcemia.
GranulomatouscausesofHypercalcemia
InfectiveCauses
Tuberculosis
Berylliosis
Histoplasmosis
Coccidoimycosis
Pneumocystis
GranulomatousLeprosy
Cat-ScratchDisease
Non-InfectiveCause
Sarcoidosis(Mostcommon)
Wegner'sGranulomatosis
InflammatoryBowelDisease
Histiocytosis-X

ForeignbodyGranulomas
AlmosteverysinglediseaseassociatedwithGranulomaformation
hasbeenreportedtocauseHypercalcemia

1072.Hypophosphatemiaisseenin:
a)Pseudohypoparathyroidism
b)Hyperparathyroidism
c)Hyperthyroidism
d)Hypoparathyroidism
CorrectAnswer-B
AnswerisB(Hyperparathyroidism):
Hyperparathyroidismistypicallyassociatedwithhypophosphatemia.
PrimaryHyperparathyroidismisassociatedwithHypophosphatemia
andHypercalcemiawhileSecondaryHyperparathyroidismis
associatedwithHypophosphatemiaandHypocalcemia.
Hyperthyroidismistypicalassociatedwithnormalphosphatelevels
HypoparathyroidismandPseudohypoparathyroidismareassociated
withHyperphosphatemia

1073.Calciumhomeostasisdisturbanceis
seenin
a)Malignanthyperthermia
b)DMD
c)Tibialmusculardystrophy
d)Limbgirdlemuscledystrophy
CorrectAnswer-A
Answer-A.Malignanthyperthermia
Malignanthyperthermiaisapharmacogeneticconditioncauseddue
tomutationofthe"Ryanodinereceptorgene".
Ryanodinereceptorgenecontrolsthelevelof"cytosoliccalcium"
andthereforeskeletalmusclecontraction.

1074.Tetanyisseenin
a)Respiratoryalkalosis
b)Respiratoryacidosis
c)Metabolicacidosis
d)Hyperkalemia
CorrectAnswer-A
Answer-A.Respiratoryalkalosis
Inalkalosistetanyoccursbecauseofthedecreasedconcentrationof
freeionizedcalcium.
Itisthefreeionizedcalciumthatisphysiologicallymoreimportant.
[RefHarrison18th/ep.362,360;Guyton10thiep.342]

1075.Hypomagnesemiaisnotseenin
a)Barterssyndrome
b)Diabetesmellitus
c)Diarrhea
d)Gitelmansyndrome
CorrectAnswer-A
Answer-A.Barterssyndrome
HypomagnesemiamayoccurinBarter'ssyndromebutusuallythe
serummagnesiumlevelisnormalinBarterssyndrome.

1076.FeaturesofSLEincludeallofthe
followingexcept:
a)Recurentabortion
b)Sterility
c)Coomb'spositivehemolyticanemia
d)Psychosis
CorrectAnswer-B
AnswerisB(Sterility):
SystemicLupusErythematosus.
RecurrentAbortionsinSLEmaybeseenasamanifestationof
AntiphospholipidAntibodysyndrome.Smallproportionofpatients
withSLEmayhaveaCoomb'sPositiveHaemolyticanemia.
PsychosisisaknownneurologicalmanifestationofSLE.


1077.Antibodiesmostcommonlyseenin
druginducedlupusare:
a)AntidsDNAAntibodies
b)AntiSmAntibodies
c)Anti-RoAntibodies
d)AntihistoneAntibodies
CorrectAnswer-D
AnswerisD(AntihistoneAntibodies):
Themostcommonlyusedmarkerfordruginducedlupusis
AntihistoneAntibodies.
DrugInducedLupusischaracterizedserologicallybythepresence
ofAnti-HistoneAntibodiesandtheabsenceofantibodiesagainst
doublestrandedDNA(dsDNAAntibodyNegative;Anti-Histone
AntibodyPositive).Anti-dsDNA
SerologyinDrugInducedLupusErythematosus
AlmostallpatientswithDrugInducedLupuswilltestpositivefor
AntinuclearAntibodies(PositiveANATest)
ThespectrumofAntinuclearantibodiesinDrugInducedLupus
includes
PositiveAntihistoneantibodies(mostcommon;notspecific;also
seeninSLE)
PositiveautoantibodiesagainstsinglestrandedDNA(common;not
specific;alsoseeninSLE)
Negative(absenceof)autoantibodiesagainstdoublestranded
DNA(dsDNA)
ThepresenceofautoantibodiesagainstdsDNAstronglysuggestsa
diagnosisofSLE
DrugInducedLupusistypicallyassociatedwithaHomogeneous

ANApatternduetothepresenceofAntihistoneAntibodies
Antibodiesareseeninlessthan5percentofpatientswithDrug
InducedLupus
ThepresenceofAntihistoneantibodiesaloneisnotaspecifictest
fordiagnosisofDrugInducedLupusasAntibodiestohistonesmay
alsobeseeninupto50to80percentofpatientswithidiopathic
SLE.
Note:HypocomplementemiaisuncommoninDrugInducedLupus
butnotinSLE.


1078.PrimarySjogrenssyndrometrueis
a)Canbeseeninchildren
b)IncreasedcomplementC4leadstothymoma
c)Associatedwithrheumatoidarthritis
d)Salivaryglandenlargement
CorrectAnswer-D
Answer-D.Salivaryglandenlargement
Sjogrensyndromeisachronicdiseasecharacterizedbydryeyes
(keratoconjuctivitissicca)anddrymouth(xerostomia)resultingfrom
immunologicalmediateddestructionofthelacrimalandsalivary
glands.
Itoccursintwoforms
1. Primaryform(SICCASYNDROME)Occursasanisolated
disorder.
2. SecondryformWhenitoccursinassociationwithother
autoimmunedisorder.Itismorecommon.
Autoimmunediseasesassociatedwithsjogrensyndrome
Symptomsresultfrominflammatorydestructionoftheexocrine
glands.
1. Keratoconjunctivitis
2. Xerostomia
3. Parotidglandenlargement

1079.ProphylaxisforHIVisoptimally
effectiveifstartedupto.....hrsof
exposure

a)1
b)2
c)4
d)12
CorrectAnswer-A
Answer-A.1
goalistostartwithinonetotwohoursorearlierafterexposureoften
usingastarterpackwithappropriatedrugsasimmediatelyavailable.
Themediantimetoinitiationofpostexposureprophylaxisis1.8hrs.
Thecentrefordiseasecontrolandprevention(CDC)
recommendationsaretoofferprophylaxisupto24-36hrsafter
exposure,forlongertimelapses,therecommendationsistoseek
advicefromanexpert

1080.Oddpair
a)Erythemamarginatum-rheumaticfever
b)Erythemagyrensripens-malignancy
c)Necroticacralerythema-HCV
d)Erythemachronicummigransmalignancy
CorrectAnswer-D
Answer-D.Erythemachronicummigransmalignancy
"Erythemagyratumrepens"isarareandcharacteristicrashstrongly
associatedwithmalignancy.
Erythemachromicumnigrans
Ergthemamarginatum
Itisacharacteristiccutaneousmanifestationofrheumaticfever.

1081.ANCAisNOTassociatedwithwhichof
thefollowingdiseases:
a)Wegener'sgranulomatosis
b)Henochschonleinpurpura
c)MicroscopicPAN
d)ChurgStrausssyndrome
CorrectAnswer-B
AnswerisB(H.S.Purpura):
H.S.purpuraisnotassociatedwithanyantinuclearcytoplasmic
antibody(ANCA).ItisanexampleofANCAnegativevasculitis.
ANCA(AntineutrophiliccytoplasmicAntibodies)arcAntibodies
directedagainstcertainproteinsincytoplasmicgranulesof
Neutrophil&monocytes.
ThesearetwomajorcategoriesofANCAbasedondifferenttargets
fortheantibodies.
ANCA(Antineutrophiccytoplasmicantibodies):
ANCAisof2types
C-ANCA
(Cytoplasmicproteinase3qisthetargetantigen)
WegenersGranulomatosisq(90-95%)
P-ANCA
(perinuclearmyeloperoxidaseQisthemajortargetantigen)
MicroscopicPAN(microscopicpolyangitis)
Churg-Strausssyndrome
Crescentericglomerulonephritise
Goodpasteur'ssyndrome

1082.Yellow-nailsyndromeconsistsof
a)Kneejointeffusionandlymphedema,associatedwith
discolorednails
b)Pericardialandlymphedema,associatedwithdiscolorednails
c)Peritonealeffusionandlymphedema,associatedwith
discolorednails
d)Pleuraleffusionandlymphedema,associatedwithdiscolored
nails
CorrectAnswer-D
Answer-D.Pleuraleffusionandlymphedema,associatedwith
discolorednails
Yellownailsyndromeisararedisorderofthenail,whichisusually
accompaniedby
Lymphoedema
Itmayalsobeassociatedwith:
Recurrentpleuraleffusions
Bronchiectasis

1083.Nottobegiveninmalignantmalariais-
a)Quinolone
b)Quinine
c)Doxycycline
d)Artesunate
CorrectAnswer-A
Answer-A.Quinolone
Artemisininderivative:Artesunate
QuinineorQuinidine
Plusoneofthefollowing:Doxycycline,Tetracyclineand
Clindamycin

1084.Muir?Torresyndromeshows
a)Sebaceouskeratomas
b)Lischnodules
c)Intestinalpolyp
d)Hyperelasticjoints
CorrectAnswer-A
Answer-A.Sebaceouskeratomas
Muir-Torresyndromeisanautosomalskinconditionofgenetic
origincharacterizedbytumorsofthesebaceousglandor
keratoacanthomathatareassociatedwithvisceralmalignant
disease
Cutaneouscharacteristic
Adenoma
Epithelioma
Carcinoma
Multiplekeratoacanthomas

1085.DOCforlisteriameningitis-
a)Ampicillin
b)Cefotaxime
c)Cefotriaxone
d)Ciprofloxacin
CorrectAnswer-A
Answer-A.Ampicillin
Theantibioticofchoiceforlisteriainfectionisampicillinorpenicillin
G.

1086.Kawasakidiseaseisassociatedwithall
ofthefollowingclinicalfeaturesexcept
a)Truncalrash
b)Posteriorcervicallymphadenopathy
c)Thrombocytopenia
d)Pericarditis
CorrectAnswer-C
AnswerisC(Thrombocytopenia):
Kawasakidiseaseisassociatedwiththrombocytosisandnot
thrombocytopenia.
CharacteristiclaboratoryfindingsTreatmentof
ChoicePrognosisinclude
IncreasedESRHighdose
intravenousPrognosisforuneventfulrecoveryis
Thrombocytosiseimmunoglobulinse
excellente


1087.Firstsymptomofleprosy
a)Decreasedvibration&positionsense
b)Decreasedpain
c)Decreasedtemperature
d)Decreasedlighttouch
CorrectAnswer-B
Answer-B.Decreasedpain
In90%ofpatientsthefirstsignofthediseaseisafeelingof
numberswhichmaypreceedesskinlesionsbyanumberofyears.
Temperatureisthefirstsensationlostfollowedbylighttourchpain
andthendeeppressure.
[RefHarrison18thiep.1363-1364]

1088.LupusPernioisseenin:
a)Tuberculosis
b)SLE
c)PAN
d)Sarcoidosis
CorrectAnswer-D
AnswerisD(Sarcoidosis):
PernioisatypicalcutaneousmanifestationofSarcoidosis.
LupusPernioisthemosttypicalandeasilyrecognizableskinlesions
ofSarcoidosis.Itischaracterizedbythepresenceofviolaceous,
purpleblueshinyswollenlesionsoverthebridgeofnose,beneath
theeyesandoverthecheeks.Thisspecificcomplexofinvolvement
ofthebridgeofnose,theareabeneaththeeyesandthecheeksis
considereddiagnosticforachronicformofSarcoidosis.


1089.Migraineisdueto
a)Dilatationofcranialarteries
b)Constrictionofcranialarteries
c)Corticalspreadingdepression
d)Meningialinflammation
CorrectAnswer-C
Answer-C.Corticalspreadingdepression
Corticalspreadingdepressionisaselfpropagatingwaveofneuronal
andglialdepolarizationthatspreadsacrossthecerebralcortex.
Theactivationoftrigeminalafferentsbycorticalspreading
depressioninturncausesinflammatorychangesinthepain-
sensitivemeningesthatgeneratetheheadacheofmigrainethrough
centralandperipheralreflexmechanisms.

1090.HowmuchlengthisincreasedinZ
plastywhenitisdoneat60degrees?
a)25%
b)50%
c)75%
d)100%
CorrectAnswer-C
Ansis'c'i.e.75%
Z-plastyisaverycommoninterpositionsurgicaltechniqueutilizedinplasticand
reconstructivesurgerytorevisescars.
Ingeneral,thegreatertheangle,thegreaterthegaininwoundlength.
TissueLengtheningwithZ-Plasty
TypeofZ-Plasty
IncreaseinLengthofCentralLimb
(%)
Simple45-degree
50
Simple60-degree
75
Simple90-degree
100
Four-flapwith60-degree
150
angles
Double-opposing
75
Five-flap
125
Or
Anglescomparedtogaininlengthareasfollows:
30-degreeangleresultsina25%gaininlength
45-degreeangleresultsina50%gaininlength
60-degreeangleresultsina75%gaininlength
75-degreeangleresultsina100%gaininlength
90-degreeangleresultsina125%gaininlength

1091..Zplastyidealangle
a)90?
b)45?
c)60?
d)75?
CorrectAnswer-C
Answer-C.60?
The60degreeZ-plasty(ie,classicZ-plasty)ismostcommonlyused
becauseitprovidestheoptimalbalancebetweenlengtheningand
easeofclosure.

1092.Allofthefollowingareriskfactorsfor
deepveinthrombosis(DVT)except-
a)Durationofsurgerymorethanthirtyminutes
b)Obesity
c)Agelessthanfortyyears
d)Useoftheoestrogen-progesteronecontraceptivepills
CorrectAnswer-C
Ans.is'c'i.e.,Agelessthan40years

1093.Lymphedemaprecoxallaretrue
except
a)U/L
b)Morecommoninmen
c)Affectsthelegs
d)2-35yrsofage
CorrectAnswer-B
Answer-B.Morecommoninmen
Primarylymphedemawithageofonsetb/wages1yearand35
years.
MCformsofprimarylymphedema.
Female:Male-10:1
Swellinginvolvesfootandcalf.
Usuallyunilateral

1094.RTAwithmultiplefracturesinitial
treatmentwouldbe-
a)Managementofshock
b)Splintingoflimbs
c)Airwaymanagement
d)Cervicalspineprotection
CorrectAnswer-C
Answer-C.Airwaymanagement
Managementoftraumabeginswithprimarysurvey.
TheATLS(AdvancedTraumaLifeSupport)definesprimarysurvey
asassessmentofthe`A,B,C'i.e.,Airwaywithcervicalspine
protection,Breathingandcirculation.

1095.Bestskindisinfectantforcentralline
insertionis:
a)Povioneiodine
b)Alcohol
c)Cetrimide
d)Chlorhexidine
CorrectAnswer-D
Ansis'd'i.e.Chlorhexidine
"Chlorhexidineisnowthedisinfectantrecommendedforallcatheter
placementproceduresandforroutinesitecleansingduringdressing
changes."-ComprehensiveHospitalMedicine:AnEvidence-Based
AndSystemsApproachByMarkV.Williams,ScottA.Flanders,
p320
"Useofantisepticsolutionforskindisinfectionatthecatheter
insertionsitehelpspreventcatheter-relatedinfection.Chlorhexidine-
basedsolutionsappeartobesuperiortobothaqueousandalcohol-
basedpovidone-iodineinreducingtheriskforcathetercolonization
andcatheter-relatedbloodstreaminfection.Ifthereisa
contraindicationtochlorhexidine,tinctureofiodine,aniodophoror
70percentalcoholcanbeusedasalternatives"-uptodate.com

1096.Withblunttraumaalloverbodythe
amountofN2&nitrogenendproducts
lost/day

a)35gm
b)45gm
c)55gm
d)65gm
CorrectAnswer-A
Answer-A.35gm
Afterinjury,theinitialsystemicproteolysis,mediatedprimarilyby
glucocorticoidesincreasesurinarynitrogenexcretiontolevelsin
excessof30gm/day.

1097.Thefollowingstatementaboutkeloidis
true?
a)Theydonotextendintonormalskin
b)Localrecurrenceiscommonafterexcision
c)Theyoftenundergomalignantchange
d)Theyaremorecommoninwhitesthaninblacks
CorrectAnswer-B
Ans.is'b'i.e.,Localrecurrenceiscommonafterexcision
Akeloidscarisdefinedasexcessivescartissuethatextends
beyondtheboundariesoftheoriginalincisionorwound.
Features-
Itcontinuestogetworseevenafter1yearanduptoafewyears.
Severeitchingispresent
Marginistender
Vascular,red,erythematous(immaturebloodvessels)
Extendstonormaltissues,hasaclaw-likeprocess.Hencethe
name.
Riskfactors-
Blackrace
Tuberculosispatients
Incisionoverthesternum,earlobe
Equalinbothsexes
Hereditaryandfamilial
Vaccinationsites,injectionsites
Treatment-
Injectionofsteroidpreparationsuchastriamcinoloneacetate
(Kenacort)hasbeenfoundtobeextremelyuseful.

Itflattensthekeloid.Intrakeloidalexcisionandskingraftingare
tobetriedlast.
Recurrenceiscommon.(Anyformofexcisionhasahighchanceof
recurrence)
Careshouldbetakennottoextendtheincisionontothenormal
surroundingtissues.
Siliconeapplication
Topicalretinoids

1098.Indicationsforemergencythoracotomy
areallofthefollowingexcept
a)Majortracheobronchialinjuries
b)Cardiactamponade
c)Penetratinginjuriestoanteriorchest
d)TensionPneumothorax
CorrectAnswer-D
Answer-D.TensionPneumothorax
Emergencythoracotomyisindicatedafterchesttraumain
followingconditions:
Cardiacarrest(resuscitativethoractomy)
Massivehemoththorax(>1500mLofbloodthroughthechesttube
acutelyor>200-300ml/hrafterinitialdrainage.
Penetratinginjuriesoftheanterioraspectofthechestwithcardiac
tamponade.
Largeopenwoundsofthethoraciccage.
Majorthoracicvascularinjuriesinthepresenceofhemodynamic
instability

1099.InvertedChampagnebottleappearance
isseenin
a)Varicoseveins
b)DVT
c)Lipodermatosclerosis
d)Venousulceration
CorrectAnswer-C
Answer-C.Lipodermatosclerosis
Themostcommonlyrecognizedformoflipodermatosclerosis(LDS),
chronicLDSpresencewithindurationandhyperpigmentationofthe
skininvolvingtheoneorbothofthelowerlegsinacharacteristic
"invertedchampagnebottle"appearance.
Associatedwithvenousinsufficiency,LDSismostcommonin
middle-agedwomen.

1100.10cmtumoronanteriorsurfaceof
thigh,whatisdonetoknowto
diagnosis

a)Incisionbiopsy
b)Excisionbiopsy
c)FNAC
d)USG
CorrectAnswer-A
Answer-A.Incisionbiopsy
Incisionalbiopsyisindicatedwhentissuesamplesarenotobtained
byFNACorcoreneedlebiopsyasindeeptumorsandforsuperficial
softtissuetumors>3cm.
Excisionalbiopsyisindicatedforeasilyaccesibleextremityor
truncallesions<3cm.

1101.Bestmethodtotreatalargeport-wine
hemangiomais?
a)Radiotherapy
b)Tatooing
c)Excisionwithskingrafting
d)PulseddyeLaser
CorrectAnswer-D
PulseddyeLaserREF:Roxburgh'scommonSkinDiseases17th
editionpage194&205,ClinicalDermatologybyJohnHunter,John
Savin&MarkDahl3'editionpage275
"SelectivePhotothermolysisorPulseddyelaseristhetreatmentof
choiceforPortwinehemangioma"
Port-winestainsarecrimsonblotchesinwhichthereismarked
capillarydilatationcomparedtoacapillaryangioma,whichisared
noduleorplaquecontainingproliferatingendothelialcells.Thelatter
tendtoflattenanddisappearatpuberty.Largeronesmaycause
problemsfrombleedingand/orerosion.Cavernoushaemangiomata
arelargerandcompressible,containinglargevascularspaces.
Occasionallyaport-winestainofthetrigeminalareaisassociated
withavascularmalformationoftheleptomeningesonthesameside,
whichmaycauseepilepsyorhemiparesis(theSturgeWeber
syndrome),orwithglaucoma.
Excellentresultshavebeenobtainedwithcarefulandtime-
consumingtreatmentwitha585-nmflashlamp-pumpedpulseddye
laser.Treatmentsessionscanbegininbabiesandanaesthesiais
notalwaysnecessary.Ifatrialpatchissatisfactory,40-50pulses
canbedeliveredinasessionandtheprocedurecanberepeatedat
3-monthlyintervals.


1102.Flashburn,tender,red&painful,which
ofthefollowingtypeistheburn-
a)Scaldedburn
b)Firstdegreeburn
c)Seconddegreeburn
d)Fourthdegreeburn
CorrectAnswer-B
Answer-B.Firstdegreeburn
Firstdegreeburn(alsok/aSuperficialorEpidermalburn)
Theseburnsinvolveonlytheepidermis
Theydonotblister,
Areerythematousbecauseofdermalvasodilation
Blanchtotouch(i.e.showcapillaryrefilling)
Arequitepainful
Healwithoutscarringin5to10days.
Theyresultinpainandreddeningoftheepidermis(outerlayerof
theskin).
Theclinicalfearuresareblisteringand/orlossoftheepidermis.The
underlyingdermisispinkandmoist.Thecapillaryreturniscleariy
visiblewhenblanched

1103.VeinusedinbypassSurgery:
a)Greatsaphenousvein
b)Shortsaphenousvein
c)Femoralvein
d)Brachialvein
CorrectAnswer-A
Inpatientswithocclusivecoronarydiseasecausedby
atherosclerosis,thediseasedarterialsegmentcanbebypassedby
insertingagraftconsistingofaportionofthegreatsaphenousvein.
Thevenoussegmentisreversedsothatitsvalvesdonotobstruct
thearterialflow.Followingremovalofthegreatsaphenousveinat
thedonorsite,thesuperficialvenousbloodascendsthelowerlimb
bypassingthroughperforatingveinsandenteringthedeepveins.
Thegreatsaphenousveincanalsobeusedtobypassobstructions
ofthebrachialorfemoralarteries.

1104.Bestgasusedforcreating
pneumoperitoniumatlaparoscopyis:
a)N2
b)02
c)CO2
d)N20Goniometerisused
CorrectAnswer-C
CO2
CO,isthegasusedtocreatepneumoperitoneumduring
laparoscopy.
Otheroptionis-N20:Butitisexpensive,lesssolubleinblood
and
supportscombustion.
Alsoknow:
Instrumentusedforcreatingpneumoperitoneumisveressneedle.
FlowRateofCO,forcreatingpneumoperitoneum200-2000ml/min
&
pressurebetween15-25mmofHg.

1105.Inafemaleabdominalintestinal
perforationoperatedhasserous
dischargeon5thdaywithwoundgap.
Whatisyourdiagnosis

a)Wounddehiscence
b)Enterocutaneousfistula
c)Scroma
d)Peritonitis
CorrectAnswer-A
Answer-A.Wounddehiscence
Dehisencemostoftendevelops7to10dayspostoperativelybut
mayoccuranytimeaftersurgery,from1tomorethan20days."-
Sabiston

1106.Peritonitis
a)Neutrophils>250mm3
b)WBC<100/m1
c)Ascitislactatelevel<25mg/dl
d)AscitisfluidpH>735
CorrectAnswer-A
Answer-A.Neutrophils>250mm3
Morethan250neutrophills/cummofascitisfluidsuggestanacute
inflammatoryprocess,themostcommonofwhichisspontaneous
bacterialperitonitis.
Anasciteslactatelevelofmorethan25mg/diwasfoundtobe100%
sensitiveandspecificinpredictingactivespontaneousbacterial
peritonitis.

1107.Ingastrectomyfollowingoccursexcept
-
a)Calciumdeficiency
b)Steatorrhoea
c)Fe.deficiency
d)Fluidloss
CorrectAnswer-D
Answeris'd'i.e.Fluidloss
Postgastrectomycomplications
AnemiaasaresultofvitaminB12orironmalabsorptionand
osteoporosis.
Irondeficiencyanemiadevelopsbecauseremovalofthestomach
oftenleadstoamarkeddecreaseintheproductionofgastricacid.
Osteoporosisdevelopsasaresultofpoorcalciumabsorption,
anotherproblemthatoccursaftergastricsurgery.
Dumpingsyndrome
Diarrhoea-maybeduetodifferentreasons.associatedwith
dumpingsyndromepost-vagotomydiarrhoeaassociatedwithfat
malabsorption.
Fatmalabsorptionleadingtosteatorrhoeaoccursduetoacid
inactivationofpancreaticenzymesorpoorlycoordinatedmixingof
food&digestivejuices.Fatmalabsorptionleadstomalabsorptionof
fatsolublevitaminsi.e.A,D,E&K.
Pushingfoodfromyourstomachtoyoursmallboweltooquickly
(dumpingsyndrome)
Acidreflux
Chestinfections,includingbronchitisandpneumonia
Internalbleeding

Nauseaandvomiting
Stomachacidleakingintoyouresophagus,causingscarringand
narrowing(stricture)
Vitamindeficiencies
Weightloss

1108.Treatmentofchoiceinanorectal
carcinoma
a)Chemoradiation
b)APRCombinedsurgeryandradiotherapy
c)Chemotherapyalone
d)All
CorrectAnswer-B
Answer-B
Abdominal-perinealresectionwithcolostomywas
thepreferredsurgicalprocedureformostmajorcancersof
theanalcanal,

1109.Rectalprolapsesurgeryis
a)Rectalmucosalstapling
b)Mucosalresection
c)Placation/wiring
d)Rectopexy
CorrectAnswer-D
Answer-D.Rectopexy
Abdominalapproach

1. Reductionofperinealherniaandclosureofced-de-sac
(Moschowitz'soperation)
2. Fixationofrectumeitherwithaprostheticsling(Ripsteinandwells
rectopexy)orbySutureRectopexy.
3. Resectionofredundantsigmoidcolon?rectalfixation(Resection
rectopexy).

1110.Crohn'sdisease
a)Continousinvolvement
b)Sinus&fistula
c)Mesentericlymphadenitis
d)Studulcer
CorrectAnswer-B
Answer-B.Sinus&fistula
Clinicalfeatures-
Intermittentmilddiarrhea,fever,abdominalpain(MC)
Rightlowerquadrantmass,weightloss,anemia
Sometimesmimicsappendicitisorbowelperforation
Analcomplaints(fissure,fistula,abscess)?frequent
Fat/vitaminmalabsorptionpresent

1111.Stepladderpatternofgasshadowis
seenin
a)Duodenalobstruction
b)Intestinalobstruction
c)Gastricoutletobstruction
d)Sigmoidvolvulus
CorrectAnswer-B
Answer-B.Intestinalobstruction
Stepladdersign
representstheappearanceofgas-fluiddistended
smallbowelloopsthatappeartobestackedontopofeachother,
typicallyobservedonerectabdominalradiographsinthesetting
ofsmallbowelobstruction.

1112.Whatisnotseeninshortbowel
syndrome
a)Hypergastrinemia&highgastricsecretionisseen
b)Diarrhea,dehydrationandmalnutrition
c)Hirsutism
d)ChronicTPNdependence
CorrectAnswer-C
Answer-C.Hirsutism
Resectionofjejunumisbettertoleratedthanresectionofileum,as
thecapacityforbilesaltandvitaminB12absorptionisspecifictothe
ileum
Malabsorptionaftermassivesmallbowelresectionisexacerbated
byacharacteristichypergastrinemiaassociatedgastricacid
hypersecretionthatpersistsfor1to2yearspostoperatively
Short-bowelsyndromeisadisorderclinicallydefinedby
malabsorption,diarrhea,steatorrhea,fluidandelectrolyte
disturbances,andmalnutrition.

1113.EsophagealmanometryisusefulallthisconditionsEXCEPT:
a)Achalasia
b)Diffuseesophagealspasm
c)ToassesstheperistalticintegritypriortothesurgeryforGERD
d)Malignancy
CorrectAnswer-D
Esophagealmanometry,ormotilitytesting,entailspositioningapressuresensingcatheter
withintheesophagus.
Manometryisusedtodiagnose
1.Motilitydisorders(achalasia,diffuseesophagealspasm)
2.Toassessperistalticintegritypriortothesurgeryforrefluxdisease.
Esophagealmalignancyisnotdiagnosedwithesophagealmanometry.UpperGI
endoscopyistheeffectivemethodformalignancyandbiopsycanbetaken.
Ref:Harrison,Edition-18,Page-2430

1114.Ogilvie'ssyndromemostcommonly
involves
a)Stomach
b)Colon
c)Gallbladder
d)Smallintestine
CorrectAnswer-B
Answer-B.Colon
Ogilviesyndrome,oracutecolonicpseudo-obstruction(ACPO),isa
clinicaldisorderwiththesigns,symptoms,andradiographic
appearanceofacutelargebowelobstructionwithnoevidenceofthe
actualphysicalcauseoftheobstruction.Thecolonmaybecome
massivelydilated;ifnotdecompressed,thepatientrisksperforation,
peritonitis,anddeath.
Pseudo-obstructionmostcommonlyoccursinhospitalizedpatients
andisassociatedwiththeuseofnarcotics,bedrest,andco-morbid
disease.
Thisconditiondescribesanobstruction,usuallyofthecolon,that
occursintheabsenceofamechanicalcauseoracuteintra-
abdominaldisease.
Abdominalradiographsshowevidenceofcolonicobstruction,with
markedcecaldistensionbeingacommonfeature.

1115.Spigelianherniais
a)Throughlineaalba
b)Throughlateralborderofrectusabdominis
c)Throughmedialwallofiinguinalcanal
d)Throughlateralwallofinguinalcanal
CorrectAnswer-B
Answer-B.Throughlateralborderofrectusabdominis
Spigelianherniascanoccuranywherealongthelengthofthe
spigelianlineorzone--anaponeuroticbandofvariablewidthatthe
lateralborderoftherectusabdominis.Themostfrequentlocationof
theserarehemiasisatorslightlyabovethelevelofthearcuateline.
Spigelianherniaoccursthroughthelinearsemilunariswhich
correspondstothelateralmarginoftherectusabdominis.
Aspigelianherniaoccursthroughthespigelianfascia,whichis
composedoftheaponeuroticlayerbetweentherectusmuscle
mediallyandthesemilunarlinelaterally.

1116.MostcommontypeofIntussusception
is-
a)Ileocolic
b)ileoileal
c)Colo-colic
d)Caeco-colic
CorrectAnswer-A
Ans.is'a'i.e.,Ileocolic
Ileo-ileo-colic(-12)
Ileoileal(-5%)
Colocolic(-2%)
Multiple(1%)
Retrograde
Themostcommontypeofintussusceptionisileocolic(also
knownasileocecal)(90%).

1117.24dayneonatewithprojectilevomiting
&failuretogainweight.whatisthe
diagnosis

a)CHPS
b)NEC
c)Duodenalatresia
d)Hirschsprung'sdisease
CorrectAnswer-A
Answer-A.CHPS
Projectilevomitingin4thweekisquitesuggestiveofCHPS.In
duodenalatresiathevomiting(usuallybilious)isrightfromthe1st
dayoflife.NECandHirschsprung'sdiseasehavedifferentclinical
presentation.

1118.NottrueaboutBarrett'sesophagus
a)Metaplasiaofcells
b)PredisposestoSCC
c)Precancerouscondition
d)Intestinaltypeisthemostcommontype
CorrectAnswer-B
Answer-B.PredisposestoSCC
Barrett'sesophagusispremalignantconditionforadenocarcinma
esophagusandnotSCC.

1119.Gastrotomyis
a)Openthestomachclosedaftertubeinsertion
b)Openingthestomach
c)Resectingtheterminalpartofstomach
d)Resectingtheproximalpartofstomach
CorrectAnswer-B
Answer-B.Openingthestomach
Gastrostomyreferstoasurgicalopeningintothestomach.;creation
ofanartificialexternalopeningintothestomachfornutritional
supportorgastrointestinalcompression.Typicallythiswouldinclude
anincisioninthepatient'sepigastriumaspartofaformaloperation.

1120.Parrotbeakappearanceisseenin
a)Volvulus
b)Intussuption
c)Rectalatresia
d)CAcolon
CorrectAnswer-A
Answer-A.Volvulus
Barium/gastrograffinenema-demonstratesthepointofobstruction,
pathognomonic"Birdbeckdeformity"orparrotbeakdeformity.
Contrastenemaiscontraindicatedifgangreneissuspected.
"Parrotbeaked"clawingofneilsisalsoseeninchroniccocaine
abuse.

1121.Raspberrytumorisseenin
a)Umbilicalfistula
b)Meckel'sdiverticulum
c)Umbilicaladenoma
d)Umbilicalgranuloma
CorrectAnswer-C
Answer-C.Umbilicaladenoma
Anumbilicaladenoma
isalsoknownas"Raspberrytumor".
Itisduetoapartiallyunobliteratedvitellointestinalduct.
Itisseenininfants.
Theprolapsingmucosagivestheraspberryappearanceand
bleedsontouch.
Treatment:
Aligatureistiedarounditsbase,anditfallsoffafterafewdays.
Recurrenceistreatedwithsurgery.

1122.Aftermastectomy,breast
reconstructionisdoneby
a)Deltopectoral
b)latissimusdorsi
c)Serratusanterior
d)Trapezius
CorrectAnswer-B
Answer-B.latissimusdorsi
Iftheskinatthemastectomysiteispoor(e.g.following
radiotherapy)orifalargervolumeoftissueisrequired,a
musculocutaneousflapcanbeconstructedeitherfromthe
latissimusdorsimuscle(anLDflap)orusingthe
transversusabdominismuscle(aTRAMflapas).Thelattergivesan
excellentcosmeticresultinexperiencedhandsbutisalengthy
procedureandrequirescarefulpatientselection.

1123.PeaudeorangeappearanceofCa
breast,whatisthestage
a)T4b
b)T4a
c)T3a
d)T3b
CorrectAnswer-A
Answer-A.T4b
StageI:T1N0M0
StageIla:T0N1M0;T1N1M0;T2N0M0.
StageIlb:T2N1M0;T3N0M0
StageIlla:T0N2M0;T1N2M0;T2N2M0;T3N1M0;T3N2M0
StageIllb:T4N0M0;T4N1M0;T4N2M0
StageIllc:AnyTN3M0
StageIV:AnyT,anyN,M
Earlybreastcancer___StageIand11J1N1,1-2N1;13NOLocally
advancedbreastcancer
(LABC)--Stage111AIMetastaticbreast
cancer--Stage
IV
Primarytumor(T)
TX-Primarytumorcannotbeassessed
T0-Noevidenceofprimarytumor
Tis-Carcinomainsitu
Tis(DCIS)-Ductalcarcinomainsitu

Tis(Paget)-PagetdiseaseofthenippleNOTassociatedwith
invasivecarcinomaand/orcarcinomainsitu(DCIS)intheunderlying
breastparenchyma.Carcinomasinthebreastparenchyma
associatedwithPagetdiseasearecategorizedonthebasisofthe
sizeandcharacteristicsoftheparenchymaldisease,althoughthe
presenceofPagetdiseaseshouldstillbenoted
T1-Tumor20mmingreatestdimension
T1mi-Tumor1mmingreatestdimension
T1a-Tumor>1mmbut5mmingreatestdimension(roundany
measurement>1.0-1.9mmto2mm)
T1b-Tumor>5mmbut10mmingreatestdimension
T1c-Tumor>10mmbut20mmingreatestdimension
T2-Tumor>20mmbut50mmingreatestdimension
T3-Tumor>50mmingreatestdimension
T4-Tumorofanysizewithdirectextensiontothechestwalland/or
totheskin(ulcerationorskinnodules),notincludinginvasionof
dermisalone
T4a-Extensiontochestwall,notincludingonlypectoralismuscle
adherence/invasion
T4b-Ulcerationand/oripsilateralsatellitenodulesand/oredema
(includingpeaud'orange)oftheskin,whichdonotmeetthecriteria
forinflammatorycarcinoma
T4c-BothT4aandT4b
T4d-Inflammatorycarcinoma


1124.
WhatistrueaboutHER2/neu
overexpressioninCabreast:

a)Goodprognosis
b)Respondswelltotaxanes
c)Respondswelltomonoclonalantibodies
d)Seenonlyinbreastcancer
CorrectAnswer-C
Ansis'c'i.e.Respondswelltomonoclonalantibodies
TheHER2receptor(previouslycalledHER2/neu,orERBB-2
receptor)belongstotheepidermalgrowthfactorreceptor(EGFR)
familyofreceptors,whicharecriticalintheactivationofsubcellular
signaltransductionpathwayscontrollingepithelialcellgrowthand
differentiationandpossiblyangiogenesis.
AmplificationofHER2oroverexpressionofitsproteinproductis
observedin18to20percentofhumanbreastcancers.
HER2overexpressionisalsonotedinothertumorssuchas
esophagogastrictumors,lung,ovary&headandnecksquamous
cellca.(Inallofthesesites,HER2overexpressionhasbeen
identifiedasanegativeprognosticfactor.)
FollowingpointsaretobenotedaboutHER2overexpressionin
breastCa:
PrognosticvalueofHER2--HER2overexpressionisapoor
prognosticmarker.HER2overexpressionisassociatedwithhigh
ratesofdiseaserecurrenceanddeathintheabsenceofadjuvant
systemictherapy.
PredictivevalueofHER2--HER2statuspredictsresponseto
specifictherapies:
PatientswithhighlevelsofHER2expressionbenefitfromtreatment

withagentsthattargetHER2,suchastrastuzumab(amonoclonal
antibody)andlapatinib.
HER2statusappearstopredictresistanceorsensitivitytodifferent
typesofchemotherapeuticagents,includinganthracyclinesand
taxanes.
WomenwhosetumorsoverexpressHER2appeartoderivegreater
benefitfromanthracycline-basedadjuvanttherapythanfrom
adjuvanttherapythatisalkylatingagent-based,suchasCMF(cycl
ophosphamide,methotrexate,fluorouracil).
RelationshipbetweenHER2overexpressionandtaxanesisstill
understudywithvariousstudiesgivingconflictingreports.
HER-2positivityisassociatedwithresistancetoendocrine
therapies.
ScoringofHER-2ImmunohistochemistryAssays
Score
HER-2Status
StainingPattern
0
Nostainingormembranestainingin
Negative
<10%oftumorcells?
1+
Negative
?Afaintbarelyperceptible
membranestainingisdetectedin
>10%
oftumorcells.Thecellsareonly
stainedinpartofthemembrane?.
2+
Equivocal
?Weaktomoderatecomplete
membranestainingisseenin>10%
of
tumorcellsor<30%withstrong
staininga

3+
Positive
?Strongcompletemembrane
stainingisseenin>30%oftumor
cells?

1125.Notacomponentoftripletestin
detectionofCabreast-
a)Breastselfexamination
b)USG/mammography
c)FNAC/trucutbiopsy
d)Clinicalexamination
CorrectAnswer-A
Answer-A.Breastselfexamination
Tripleassessmentincludesexaminationbyaclinician.Self
examinationisnotapartoftripleassessment.
BaileyandLovewrites--"Inanypatientwhopresentswithabreast
lumporothersymptomssuspiciousofcarcinoma,thediagnosis
shouldbemadebyacombinationofclinicalassessment,
radiologicalimagingandatissuesampletakenforeithercytological
orhistologicalanalysis,thesocalledtripleassessment.Thepositive
predictivevalue(PPV)ofthiscombinationshouldexceed99.9%."

1126.Treatmentforhydroureter
a)Antibioticprophylaxisalone
b)Immediateureterolithotomy
c)Endoscopicureteralstenting
d)Urinaryalkalization
CorrectAnswer-C
Answer-C.Endoscopicureteralstenting
Hydroureterismainlycausedbyintrinsicandextrinsicobstructionof
ureter.Causesare-Calculi(ureteric/VVJ)-intrinsicstrictureand
Retroperitonealfibrosis-extrinsic.

1127.Diversionofurineisbestdoneat
a)Ileum
b)Jejunum
c)Caecum
d)Colon
CorrectAnswer-A
Answer-A.Ileum
Ilealsegmentisthebestforurinarydiversionaftercystectomy.
Stillbettermethodis"Continentcutaneousdiversion"method.
Butthebestmethodis"Orthotopicneobladders".

1128.Ureterosigmoistomy
a)Hyperchloremicwithhypokalemicacidosis
b)Hyperkalemia
c)Metabolicalkalosis
d)Hyponatremia
CorrectAnswer-A
Answer-A.Hyperchloremicwithhypokalemicacidosis
Thereishyperchloremicmetabolicacidosiswithhypokalemia.

1129.Injurytopeniswhichofthefollowing
preventsextravasationofblood?
a)Bucksfascia
b)Fasciaofcamper
c)Fasciatransversalis
d)None
CorrectAnswer-A
Answer-A.Bucksfascia
SuperficialtotunicaalbugineathereisBuck'sfacia(deeplayerof
superficialfasciaofpenis),aprolongationofcolle'sfascia
(membranouslayerofsuperficialfasciaofperineum).
IfBuck'sfaciaremainsintact,hematomaisrestrictedtopenileshaft
onlycausingegg-plantdeformity.
IfBuck'sfasciaisdisrupted,hematomacanextendtoscrotum,
perineumandsuprapubicregions.

1130.Phimosisisassociatedwith
a)Paraphimosis
b)Meatalstenosis
c)Balanoposthitis
d)Hypospadias
CorrectAnswer-C
Answer-C.Balanoposthitis
Phismosiscauseddueto
chronicinfection
Congenital
Acquired-trauma,PenisCaandBalanitis
Pathologicalphimosis(asopposedtothenaturalnon-retractabilityof
theforeskininchildhood)israreandthecausesarevaried.Some
casesmayarisefrombalanitis(inflammationoftheglanspenis).
Phimosismayoccurafterothertypesofchronicinflammation(such
asbalanoposthitis),repeatedcatheterization,orforcibleforeskin
retraction.
Phimosismayalsoariseinuntreateddiabetics

1131.Circumcisioniscontraindicatedin
a)Balanitis
b)Hypospadias
c)Paraphimosis
d)Exostrophyofbladder
CorrectAnswer-B
Answer-B.Hypospadias
Circumcisionisnotdoneinpatientswithhypospadiasasthe
prepucecanlaterbeusedinsurgicalrepair.
Circucisionismostlydoneforculturalreasons.
Themedicalindicationsforcircumcisionare:
Phimosis&Paraphimosis
Recurrentbalanoposthitis(i.e.inflammationoftheforeskin)
Recurrenturinarytractinfection

1132.Whichisnottrueaboutcancerof
tongue
a)Adenocarcinomamostcommon
b)Lateralsurfaceinvolved
c)Deepcervicallymphnodesnotinvolved
d)Tobaccoisthecause
CorrectAnswer-A
Answer-A.Adenocarcinomamostcommon
SCCisthemostcommontypeofmalignancy,butleiomyosarcomas
andrhabdomyosarcomasarealsoencountered(rarely).
Tumorsonthetonguemayoccuronanysurfacebutaremost
commonlyseenonthelateralandventralsurfaces.
Theregionallymphaticsoftheoralcavityaretothesubmandibular
spaceandtheuppercervicallymphnodes.
Riskfactors-tobaccoandalcohol.

1133.Epulisarisesfrom-
a)Enamel
b)Rootofteeth
c)Gingiva
d)Pulp
CorrectAnswer-C
Answeris'c'i.e.Gingiva
Epulisliterallymeans'uponthegum'.Thusitisaswellingsituated
onthegum.
Itcanoriginatefromthemucousmembrane,theperiosteumorthe
bonegivingrisetodifferentvarietiesofEpulis.

1134.Kasaioperation
a)Biliaryatresia
b)Choledochalcyst
c)Hepatocellularcarcinoma
d)Primarybiliarycirrhosis
CorrectAnswer-A
Answer-A.Biliaryatresia
Kasaioperationisalsoknownashepatoportoenterostomy.
BiliaryatresiaiscurrentlyMCindicationforpediatricliver
transplantation.

1135.Inapatientofacutecholecystitis,
referredpaintotheshoulderisk/a
a)Murphy'ssign
b)GrayTurnersign
c)Boa'ssign
d)Cullen'ssign
CorrectAnswer-C
Ansis'c'i.e.Boas'sign
Boas'sign:Incasesofacutecholecystitispainradiatestothetipof
therightshoulderandanareaofskinbelowthescapulaisfoundto
behypersensitive.Thisisk/aBoas'sign.Sensitivityisquiteless.
AlsoKnow:
Murphy'ssign:Seeninacutecholecystitis.Murphy'ssigniselicited
byaskingthepatienttobreatheoutandthengentlyplacingthehand
belowthecostalmarginontherightsideatthemid-clavicularline
(theapproximatelocationofthegallbladder).Thepatientisthen
instructedtoinspire(breathein).Normally,duringinspiration,the
abdominalcontentsarepusheddownwardasthediaphragmmoves
down(andlungsexpand).Ifthepatientstopsbreathingin(asthe
gallbladderistenderand,inmovingdownward,comesincontact
withtheexaminer's.fingers)andwinceswitha'catch'inbreath,the
testisconsideredpositive.Inorderforthetesttobeconsidered
positive,thesamemaneuvermustnotelicitpainwhenperformedon
theleftside.
GreyTurner&Cullen'ssign:positiveinseverenecrotizing
pancreatitis.GreyTurnerssignisbluishdiscolourationseenatthe
flanks.Bluishdiscolourationaroundtheumbilicusisknownas


Cullen'ssign.

1136.Falseabouthepaticadenoma
a)Benignlesion
b)OCPuse
c)Olderfemales
d)Coldonisotopescan
CorrectAnswer-C
Answer-C.Olderfemales
Hepaticadenomasarebenignsolidneoplasmsofliver.
MCseeninyoungerfemales(20-40yearsofage)
Usuallysolitary
Riskfactor-Prior/currentuseofestrogens(OCP)

1137.Bismuthclassificationinwhichclass,
hepaticductconfluenceisinvolved
a)TypeI
b)TypeII
c)TypeIII
d)TypeIV
CorrectAnswer-B
Answer-B.TypeII
Bismuth-Corletteclassificationisusedtoclassify
cholangiocarcinoma
TypeI-Commonhepaticductinvolvement.
TypeII-CHD+bifurcation/confluenceofhepaticducts.TypeIII-a)
Extensiontorightsecondaryintrahepaticduct.b)Extensiontoleft
secondaryintrahepaticduct.
TypeIV-Involvingbothrightandleftsecondaryintrahepaticducts.

1138.Parathyroidglandsareremovedby
surgery,forrecurrenceinvestigationof
choice

a)Technetiumscan
b)SPECT
c)MRI
d)Neckultrasound
CorrectAnswer-A
Answer-A.Technetiumscan
Preoperativelocalisationtestsforparathyroidglandsare
1. Tc99mlabeledsestamibiscan(>80%sensitive)
2. Singlepositronemissioncomputedtomography(SPECT).
3. CTandMRIscan.
4. Neckultrasound.

1139.Whichisnotacontraindicationfor
pancreaticoduo-denectomy
a)Metastasis
b)Portalveininvolvement
c)StageIIICAheadofpancrease
d)Invasionofsuperiormesentricvein
CorrectAnswer-B
Answer-B.Portalveininvolvement
Tumor(T)
TX-Primarycan'tbeasscessed.
To-Noevidenceofprimarytumor.
T1-Cainsitu.
T1-Limitedtopancreasand2cm.
T2-Limitedtopancreasand>2cm.
T3-Extendsbeyondpancreasbutnoinvolvementofceliacaxis
orsuperiormesentricartery.
T4-InvolvesceliacaxisorSup-mesentericartery(unresectable
primary).
Regionallymphnodes(N)
NX-RegionalLI\l,canotbeassessed.
N1-NoregionalLNmetastasis.
N2-RegionalLNmetastasis.
Regionallymphnodes(N)
Mx-Distantmetastasiscannotbeassessed.
Mo-Nodistantmetastasis.
M1-Distantmetastasis.

1140.Mostcommoncauseofchronic
pancreatitis
a)Chronicalcoholism
b)Tropicalpancreatitis
c)Pancreasdivisium
d)Gallstonedisease
CorrectAnswer-A
Answer-A.Chronicalcoholism
"Worldwide,alcoholconsumptionandabuseisassociatedwith
chronicpancreatitisinupto70%ofcases"

1141.Mostcommoncauseofacuteparotitis-
a)StaphylococcusAureus
b)StreptococcusPneumonia
c)Klebsiella
d)StreptococcusViridans
CorrectAnswer-A
Answer-A.StaphylococcusAureus
MCorganismisStaphylococcusAureus>>streptococcusviridans
>>pneumococcus.
Acutebacterialparotitis:ismostoftencausedby
abacterialinfectionofStaphylococcusaureusbutmay
becausedbyanycommensalbacteria.
UsuallyascendinginfectionStaphylococcusaureus
Ref-ManipalManualofSurgery5thedition

1142.Pre-operativeprophylaxisfor
pheochromocytoma
a)Alphablockadeafterbetablocker
b)Betablockadeafteralphablocker
c)Alphablocker
d)Betablocker
CorrectAnswer-B
Answer-B.Betablockadeafteralphablocker
Betablockerssuchaspropanalolatdosesof10to40mgevery6to
8hoursareoftenneededinpatientswhohavepersistant
tachycardiaandarrhythmias.n-blockersshouldonlybeaddedafter
adequatea-blockadeandrehydration.
a-adrenergicagonistsmaybeneededinimmediatepostoperative
periodtopreventpostoperativehypotensionandcardiovascular
collapse.

1143.VIPomaisassociatedwithwhich
syndrome:
a)ZollingerEllison
b)VernerMorrison
c)Carcinoidsyndrome
d)Cushing'ssyndrome
CorrectAnswer-B
Ansis'b'i.e.VernerMorrison
VlPomasareendocrinetumorsthatsecreteexcessiveamountsof
vasoactiveintestinalpeptide(VIP),
ExcessiveVIPcausesadistinctsyndromecharacterizedbylarge-
volumediarrhea,hypokalemia,anddehydration.Thissyndromealso
iscalledVerner-Morrisonsyndrome,pancreaticcholera,andWDHA
syndromeforwaterydiarrhea,hypokalemia,andachlorhydria.
Theprincipalsymptomsarelarge-volumediarrhea(100%)severe
enoughtocausehypokalemia(80-100%),dehydration(83%),
hypochlorhydria(54-76%),andflushing(20%).
Inadults80-90%ofVlPomasarepancreaticinlocation,withtherest
duetoVIP-secretingpheochromocytomas,

1144.Childwithpolytraumacametocasualty,
doseofpackedcell
a)10ml/kg
b)20ml/kg
c)30ml/kg
d)40ml/kg
CorrectAnswer-A
Answer-A
Usualdosageofpackedredcellsinpediatricpopulationsis10-15
ml/kg,butcanbeincreasedupto20ml/kgincaseswherehigher
increaseinHCtisneeded,asintrauma.
Colloid10ml/kg
Crystalloid20ml/kg
Requiring>20ml/KgofPRBCsinthe1sthourofresuscitation
PediatricMTGPack(50Kg)
?4UnitsofPRBCs
?2Unitsofthawedplasma
?1Unitofapheresisplatelets

1145.PolytraumapatientcametoEMS,dose
ofcrystalloidgivenshouldbe
a)500mlRLbolusthenregulatedbyindicators
b)2000mlbolus
c)1000mlbolusthenregulatedbyclinicalindicators
d)250mlbolus
CorrectAnswer-C
Answer-C.1000mlbolusthenregulatedbyclinicalindicators
Fluidresuscitationbeginswith1000mlbolusofRLforadultand20
ml/kgforachild.Responsetotherapyismonitoredbyclinical
indicatorsasbloodpressure,skinperfusion,urinaryoutputand
mentalstatus.

1146.Mostcommondiaphragmaticherniais:
a)Bochdalekhernia
b)Morgagnihernia
c)Paraesophagelhernia
d)Noneoftheabove
CorrectAnswer-A
Diaphragmaticherniasareofvarioustypes.Themostcommonisaposterolateral
(Bochdalek)hernia,whichoccursasaresultofadefectintheposteriordiaphragminthe
regionofthetenthoreleventhribs.

1147.ApolytraumaCTscan,CTbrainshows
alesionwithconcavemargin
a)EDH
b)SDH
c)Contusion
d)Diffuseaxonalinjury
CorrectAnswer-B
Answer-B.SDH
SDH-OnheadCT,theclotisbrightormixeddensity,cresent
shaped(Lunate),mayhavealessdistinctborderanddoesnotcross
midline.
Contusion-ThecontusedareaappearsbrightonCTScan.

1148.Mostcommoncauseofchylothoraxis?
a)Trauma
b)Lymphoma
c)Left-sidedheartfailure
d)Infections
CorrectAnswer-B
Answer-B.Lymphoma
Chylothoraxisapleuralcollectionofamilkylymphaticfluid
containingmicroglobulesoflipid.
Itresultsfromlymphformedinthedigestivesystemcalledchyle
accumulatinginthepleuralcavityduetoeitherdisruptionor
obstructionofthethoracicduct.
Thetotalvolumeoffluidmaynotbelarge,butchylothoraxisalways
significantbecauseitimpliesobstructionofthemajorlymphducts,
usuallybyintrathoraciccancerlikeprimaryorsecondarymediastinal
neoplasm,suchaslymphoma.

1149.Thecommonestsiteofaspirationofa
foreignbodyinthesupinepositionis
intothe:

a)Rightupperlobeapical
b)Rightlowerlobeapical
c)Leftbasal
d)Rightmedial
CorrectAnswer-B
Insupinepositionandwiththepatientonbacksuperior
segmentofRLListhemostdependentsegment.

1150.Apersoninhaledpeanuttwodays
back&nowunabletocoughitout
whereisthepossiblelocation

a)Rightsuperiorlobe
b)Rtmiddlelobe
c)Rtlowerlobe
d)Ltlowerlobe
CorrectAnswer-C
Answer-C.Rtlowerlobe
Themostcommonanatomiclocationforaforeignbodyistheright
mainstembronchusortherightlowerlobe.

1151.InMarfan'ssyndrome,Aorticaneurysm
occursmostcommonlyin:
a)Ascendingaorta
b)Descendingaorta
c)Abdominalaorta
d)Archofaorta
CorrectAnswer-A
TheanswerisA(AscendingAorta)
AorticaneurysmsinMarfan'ssyndromeoccurmostfrequentlyinthe
ascendingaorta.
CardiovascularlesionsinMarfan'ssyndrome
Cardiovascularlesionsarethemostlife-threateningfeaturesof
Marfan'ssyndrome.
Mitralvalveprolapse(MVP)
Lossofconnectivetissuesupportinmitralvalvesleafletsmakes
themsoftandblowycreatingtheso-called'floppyvalve'.
Mitralregurgitationfrequentlyresults.
DilatationofAscendingAortae
Themediaundergoescysticnecrosis(cysticmedionecrosis).
Lossofmedialsupportresultsinprogressivedilatationofaortic
valveringandrootoftheaorta
SevereaorticincompetenceAorticdissections

1152.Herniamostcommonlystrangulatesin
a)Indirect
b)Direct
c)Spigelian
d)Incisional
CorrectAnswer-A
Answer-A.Indirect
"Moststrangulatedherniasareindirectinguinalhernias,however,
femoralherniashavethehighestrateofstrangulation(15-20%)ofall
hernias".

1153.AllofthefollowingstatementsregardingSickleCellAnemiaistrue,
EXCEPT:
a)Patientsrequirefrequentbloodtransfusions
b)Patientsusuallypresentsbeforetheageof6months
c)ThereisapositivecorrelationbetweenHBSandpolymerization
ofHBS
d)Reccurentinfectionsisthemostcommoncauseofdeath
CorrectAnswer-B
Sicklecellanemiaisanautosomalrecessivedisorder,causedbyanaminoacid
substitutionofvalineforglutamineinthesixthpositiononthebeta-globinchain.Onsetof
thediseasestartsduringthefirstyearoflifeespeciallyafter6monthsofage,when
hemoglobinFlevelsfalls,asasignalissenttoswitchfromproductionofgammaglobinto
betaglobin.
HemoglobinSisunstableandpolymerizesduringhypoxemiaandacidosis,leadingto
sicklingofredbloodcells.Patientsdevelopsjaundice,pigmentedgallstones,
spleenomegaly,andpoorlyhealingulcersoverthelowertibia.Acutepainfulepisodescan
occurduetoacutevaso-occlusionbyclustersofsickledredcellsduringinfection,
dehydration,orhypoxia.Commonsitesofacutepainfulepisodesincludethebonesandthe
chest.
Ref:CurrentMedicalDiagnosisandTreatment2012,Chapter13;MedicalAssisting:
AdministrativeandClinicalCompetenciesByLucilleKeir,6thEdition,Page471

1154.Whatstimulatesthegonadsinmaleat8
weektosecrettestosterone-
a)Inhibinfromcorpusluteum
b)GnRHfromhypothalamusofbaby
c)PlacentalHCG
d)Allofabove
CorrectAnswer-C
Ans.is'c'i.e.,PlacentalHCG
Duringembryogenesis,Inmaleembryo,at6-7weekofgestatin,
secretionoffirstanti-mullerianhormone(alsok/omullerianinhibiting
substance)causesregressionofmullerianduct.
Thenat8-12week,underinfluenceofPlacentalHCG,testesstarted
producingtestosterone.

1155.XXbabypresentingwithpenis&
scrotumcause-
a)Highleveloftestosteroneinmaternalblood
b)Klinfiltersyndrome
c)Turnersyndrome
d)Noneofabove
CorrectAnswer-A
Answer-A.Highleveloftestosteroneinmaternalblood
Thisisacaseoffemalepseudohermaphroditismi.e.XXkaryotype
withvirilizedexternalgenitalia.
Maternalandrogen(testosterone)isthecauseofthis.
Femalepseudohermaphroditism
GenotypeisXX.Internalgonadisovarybutexternalgenitaliaare
virilized(maledifferentiation).
Congenitaladrenalhyperplasia(CAH)dueto21-hydroxylase
deficiencyisthecommonestcause.
Othercausesarematernalvirilizingtumor
(arrhenoblastoma),maternalandrogenadministration,CAHdueto
11-betahydroxylasedeficiency,fetalplacentalaromatase
deficiency,andWNT-4genemutation.

1156.Malepseudohermaphrodismmost
commoncause-
a)Congenitaladrenalhyperplasia
b)Adrenocorticaltumor
c)Chromosomalabnormalities
d)Cytogeneticabnormalities
CorrectAnswer-D
Answer-D.Cytogeneticabnormalities
Mostcommoncauseoffemalepseudohermaphroditism-CAHdue
to21hydroxylasedeficiency.
Mostcommoncauseofmalepseudohermaphroditism-Gonadal
dysgenesisanddefectinandrogenaction.

1157.Causesofmentalretardationin
congenitalhypothyroidismis-
a)Decreasecerebralgrowth
b)DecreasemyelinationofCNSneuron
c)Decreasegrowthhormonebypitutaryglands
d)Decreaeproductionofneurotransmitters
CorrectAnswer-A
Answer-A.Decreasecerebralgrowth
is'a'i.e.,Decreasecerebralgrowth[Ref:Ghai7h/ep.481;Jornalof
AmericalPhysiologicalReview]
Thyroidhormonehasmajoreffectonbraininutero&neonatal
period,deficiencycausediminshaxonalgrowth,dendritic
arborisation,delayproliferation&migrationofgranulecellsoit
decreasecerebralgrowth.

1158.Dosageofintravenousfluidfor2month
oldchildindiarrhoeawithsevere
dehydration-

a)100ml/Kgin6hour
b)50ml/Kgin6hour
c)75ml/Kgin6hour
d)80ml/Kgin6hour
CorrectAnswer-A
Answer-A.100ml/Kgin6hour
Sototalfluidtobegiven100ml/Kgin6hour.

1159.Rehydrationtherapyina2yearold
severelydehydratedchildis-
a)30ml/kgin1hour,70mlin5hours
b)30ml/kgin30min,70ml/kgin2'/2hours
c)20ml/kgin30min,80ml/kgin2'/2
d)75ml/kgin4hours
CorrectAnswer-B
Answer-B.30ml/kgin30min,70ml/kgin2'/2hours
12Month-5year30ml/Kgin30min70ml/Kgin2.30hour

1160.WhichisMCgeneticcauseofmental
retardation-
a)Tuberoussclerosis
b)Cri-du-chatsyndrome
c)Fragile-x-syndrome
d)Angel'ssyndrome
CorrectAnswer-C
Ans.is'c'i.e.,Fragile-X-syndrome
oDown'ssyndromeisthemostcommongeneticcauseofmental
retardation,andfragile-XissecondtoDown's.
Alsoknow
oMostcommoninheritedcauseofmentalretardationisfragile-X-
syndrome(becausedown'ssyndromeiscongenitalcausebutnot
inherited).

1161.Barrbodyisabsentinfemalehaving?
a)46XXgenome
b)45X0genome
c)47XXX
d)Allofabove
CorrectAnswer-B
Ans.is'b'i.e.,45X0genome
Barrbody(Sex-chromatin)
oItisadenselystaininginactivatedcondensed'X'chromosomethat
ispresentineachsomaticcellsoffemale.
oItisfoundinthenucleus.
oItisusedasatestofgeneticfemalenessitispossibleto
determinethegeneticsexofanindividualaccordingastowhether
thereisachromatinmasspresentontheinnersurfaceofthe
nuclearmembraneofcellswithrestingorintermitentnuclei.
Rememberfollowingfactandthequestionwillseemveryeasy.
Chromatidbody(Barrbodyorsexchromatin)isderivedfromoneof
thetwoX-chromosomeswhichbecomesinactivated.
ThenumerofBarrbodiesisthusonelessthanthenumberofX-
chromosomes.
Note-
oBarrbodyisfoundinfemaleBut-
oKleinefeltersyndromeismalewithBarrbody.
oTurnersyndromeisfemalewithoutBarrbody.

1162.Downsyndrome-allareseenexcept-
a)t(14;21)
b)t(11:14)
c)Trisomy21
d)t(15:21)
CorrectAnswer-B
Ans.is'b'i.e.,t(11:14)
In95%ofcasesofDownsyndrome-trisomyof21:-
Extrachromosomeisofmaternalinorigin.
I%havemosaicwithsomeallhave46chromosome.
4%haverobertsoniantranslocation.
t(13:21)
t(14:21)
t(15:21)
Veryrarelylongarmofchromosome21istriplicate(Partialtrisomy).

1163.Simiancreaseisnotseenin-
a)Down
b)Trisomy13
c)Atopicdermatitis
d)Noonansyndrome
CorrectAnswer-C
Answer-C.Atopicdermatitis
Simiancrease
Singletransversepalmercrease
Resemblesnon-human-simiansoknownassimiancrease.
Itisseenin:
1. Downsyndrome
2. Fetalalcoholsyndrome
3. Cri-duchatsyndrome(Chr.5)
4. Klinefelter(XXY)
5. Noonan(Chr.12)
6. Patau(Chr.13)
7. Edward(Chr.18)

1164.Rubellacausesallexcept
a)Microcephaly
b)VSD
c)Conductiondefect
d)All
CorrectAnswer-B
Ans.is'b'i.e.,VSD
Organofcorti:Sensorineuralhearingloss(Mostcommonclinical
findingofcongenitalrubella).
Heart:PDA(MostcommonCHDincongenitalrubella),PS,VSD,
ASD
TOF,pulmonaryarteryhypoplasia.
Eye:Cataract,retinopathy,microphthalmia,myopia,glaucoma.
CNS:Mentalretardation,spasticdiplegia,microcephaly,seizures,
behavioraldisorders.
Other:IDDM,thyroiddisorders,inguinalhernia,cryptorchidism,
interstitialpneumonia.

1165.TBinchildren,mostcommonis-
a)Abscess
b)Consolidation
c)Hilaradenopathy
d)CNStuberculosis
CorrectAnswer-C
Answer-C.Hilaradenopathy
"MostcommonformofprimaryTBinchildrenispulmonaryTB.This
couldbeintheformofhilarlymphadenopathywithorwithoutlung
parenchymalinvolvement".

1166.LEOPARDsyndromeincludesallexcept
-
a)Growthretardation
b)ECGchanges
c)Hypertelorism
d)Hypergonadism
CorrectAnswer-D
Answer-D.Hypergonadism
LEOPARDsyndrome
Lentigines
Electrocardiographicconductionabnormalities
Ocularhypertelorism
Pulmonarystenosis
Abnormalgenitalia
Retardedgrowth
Deafness

1167.GBSinachildtreatment-
a)IVIg
b)Ventilation
c)Plasmapharesis
d)Allofabove
CorrectAnswer-D
Answer-D.Allofabove
TreatmentofGuillain-barresyndrome-
Selflimitedinmajorityofcases.
Intravenousimmunoglobinshowsgoodresponse.
Plasmapharesis-Removalofautoantibodies.
Assistedventilation-Ifpatienthadrespiratorymuscleparalysis.
Physiotherapy.

1168.Doseofi.v.adrenalineinterminfantis
duringneonatalresuscitation-
a)0.1-0.3ml/kgin1:1000
b)0.3-0.5ml/kgin1:1000
c)0.1-0.3ml/kgin1:10,000
d)0.3-0.5ml/kgin1:10,000
CorrectAnswer-C
Ans.isci.e.,0.1-0.3mUkgin1:10,000
Doseoradrenlaine?
0.1ml/kgto0.3inlikgdiluted(1:I0,000)
Routs:(1)Intravenous(umbilicalvein)or
(2)Endotracheal
Indication?
HR<60/minafter30sec.ofpositivepressureventilation&chest
compression

1169.Picarefersto-
a)Icesucking
b)Thumbsucking
c)Foreignobjectbeingputinthemouth
d)Noneofabove
CorrectAnswer-C
Answer-C.Foreignobjectbeingputinthemouth
Picainvolvesrepeatedorchronicingestionofnon-nutritive
substances,whichincludeplaster,charcoal,clay,wool,ashes,paint
&earth.

1170.Maximumconcentrationofdextrose
thatcanbegiventhroughperipheral
vascularlineinneonate-

a)5
b)10
c)12.5
d)25
CorrectAnswer-C
Ans.is'c'i.e.,12.5
oHypoglycemiainneonatesisbloodsugar<40mg/dl
oCommoninpreteen,IUGR,infantofdiabeticmothers
oTreatedbybreastfeed,formulafeed&I.V.dextrosesolution
Insymptomatichypoglycemia2ml/g10%DxIVbolusgiventhen
increasedextrosecontentsofintravenousfluid.
Don'tgive>12.5%dextroseinfusionthroughperipheralline
becauseofriskofthrombophlebitis.(Prefercentralline)

1171.CPRwith2candidateisdoneatrateof
(ininfant)-
a)15:2
b)30:2
c)1:3
d)1:5
CorrectAnswer-A
Answer-A.15:2
Ininfantchildrenwithtworesuscitator=15:2
Inadultwithtworesuscitator-30:2
Insingleresuscitatorchestcompresionventilationratioinallage
groupi.e.Infant,children&adult=30:

1172.Neonatallupus-
a)Heartblock
b)Thrombocytopenia
c)Cutaneouslesion
d)Allofabove
CorrectAnswer-D
Answer-D.Allofabove
Neonatallupus
Ageofonsetnewbornto6month
Skinlesioninclude:
Annularerythematousscalyplaque.
Seenonsunexposed=head,neck
Maybeassociatedwithheartblock/thrombocytopenia
Diagnosedby:
ANA
AntiROantibodies

1173.Mostcommoncerebellartumorin
children?
a)Astrocytoma
b)Medulloblastoma
c)Ependymoma
d)DNET
CorrectAnswer-A
Ans.is'a'i.e.,Astrocytoma
oThemostcommoncerebellartumorinadultsismetastasis,
whereasinchildrenthemostcommontumorisanastrocytoma.

1174.Allofthefollowingarefeaturesof
juvenileCMLexcept-
a)Thrombocytopenia
b)FetalHbisincreased
c)Philadelphiachromosomeispositive
d)Lymphadenopathy
CorrectAnswer-C
Ans.is'c'i.e.,Philadelphiachromosomeispositive
JuvenileCMLismostlyseeninchildrenbelow2yearsofage.
Philadelphiachromosomeisnegativeandleukocytecountisless
than100'000/mm3
.
FeaturesAdult
CMLJuvenileCML
Age10-12
years<2years
Bleedingmanifestation
AbsentFrequent
Thrombocytopenia
UncommonFrequent
Rash
AbsentFrequent
Lymphadenopathy
RareFrequent
Splenomegaly
MarkedVariable
WBCcountatdiagnosis>
100'000<100'000
WBCtype

GranulocyteMonocyte
Normoblastic
UnusualCommon
HbF
NormalIncreased
Immunoglobulins
NormalIncreased
Muramidaselevels
NormalIncreased
ResponsetoBusulphan
GoodPoor
Survival2.5-3
years9months

1175.Mostcommonbenigntumoursduring
infancyis-
a)Lymphangioma
b)Hemangioma
c)Cystichygroma
d)Lipoma
CorrectAnswer-B
Ans.is'b'i.e.,Hemangioma
oHemangiomas,themostcommonbenigntumorsofinfancy,occur
in10%ofterminfants.

1176.Opsoclonus-Myoclonusisa
phenomenonseenin?
a)Wilmstumor
b)Neuroblastoma
c)Meningioma
d)Corticaltuberculoma
CorrectAnswer-B
Ans.is'b'i.e.,Neuroblastoma
Opsoclonusisadisorderofeyemovementcharacterizedby
involuntary,chaoticsaccadesthatoccurinalldirectionsofgaze.Itis
frequentlyassociatedwithmyoclonusandataxia.
Opsoclonus-myoclonusmaybecancer-relatedoridiopathic.
Whenthecauseisparaneoplastic,thetumorsinvolvedareusually
cancerofthelungandbreastinadultsandneuroblastomain
children.

1177.Enzymeusedastreatmentforsickle
cellanemia
a)Chymotrypsin
b)Glucose6-phosphatase
c)Trypsin
d)Noneofabove
CorrectAnswer-A
Answer-A.Chymotrypsin
ChymotrypsisusedinSicklecelldisease.

1178.Achildisabletobuildblocksof5
Cubesdevelopmentalageis-
a)12months
b)15months
c)18months
d)24months
CorrectAnswer-C
Ans.is'c'i.e.,18month
Bang2cube-8month
Towerof2cubes-15month
4cubes-18month
6cubes-22month

1179.Achildisabletosayshortsentencesof
6words-
a)2years
b)3years
c)4years
d)5years
CorrectAnswer-C
Ans.is'c'i.e.,4years
o2wordsentences=19month
o6wordsentences=48month
o10wordsentences=60month

1180.Childstartsmonosyllablesspeechin
whichage
a)4months
b)6months
c)8months
d)10months
CorrectAnswer-B
Answer-B.6months
3months-Startscooing
6months-Producesmonosyllablesoundslikeda,ma
9months-Producesbisyllablesoundlikebaba,mania

1181.Uppersegmenttolowersegmentratio
in3yragechildis-
a)12
b)1.3
c)1.4
d)1.6
CorrectAnswer-B
Ans.is'b'i.e.,1.3
oThelowersegmentextendsfromthesymphysispubistotheheels.
oThelowersegmentgrowsrapidlyafterbirthascomparedtoupper
segmentgivingrisetothegradualreductionintheupper
segment/lowersegmentratiowiththeprogressionofage.
AgeUppersegment:Lowersegmentratio
Atbirth1.7:1.0
3years1.3-1.0
At7years1.0:1.0
Thereafter1.0:1.1

1182.Firstpermanentteethtoeruptis:
September2008
a)Firstpremolar
b)Secondpremolar
c)Firstmolar
d)Secondmolar
CorrectAnswer-C
Ans.C:Firstmolar
Attheageofabout6-7year,firstpermanentmolarteetherupts
behindthesecondtemporarymolar.
Attheageof9,thereare12permanentteeths.Attheageof11,
thereare20permanentteethsAttheageof14,thereare28
permanentteeths

1183.Delayeddentitionisseeninall/except?
a)Downsyndrome
b)Congenitalhypothyroidism
c)Rickets
d)Allofabove
CorrectAnswer-D
Ans.is'd'i.e.,Allofabove
Causesofdelayintootheruption:
1)Impactedteeth4)Cong
hypothyroidsm
2)Downsyndrome5)Gaucher
Cleidocranialdysplasia6)Osteopetrosis

1184.5yearoldchildbedwettingRxof
choice?
a)Notreatment
b)Imipramin
c)Desmopressin
d)Motivationaltherapy
CorrectAnswer-A
Ans.is'a'i.e.,Notreatment
oNotreatmentisgiventochildrenbelow6yearsofagebecauseof
highspontaneouscurerate.o
After6yearstreatmentinclude.
t)Behavioraltherapy:Thisisthetreatmentofchoice.
it)Pharmacologicaltreatment:Itisusedwhennon-pharmacological
(behavioral)therapyfails.Desmopressinisthedrugofchoice.Other
drugsusedareimpramineandoxybutinin.

1185.Nocturnalenuresisbestt/tis
a)Positivereinforcement
b)Punishment
c)Bedalarm
d)Desmopressin
CorrectAnswer-A
Answer-A.Positivereinforcement
Consistentdrybedtrainingwithpositivereinforcementhasa
successrateof85%andbedandpadalarmsystemshavea
successrateofapproximately75%withrelapseratethatarelower
thanthosewithpharmacotherapy.

1186.Whatisthelarche-
a)Pubertalbreastenlargementinboys
b)Breastenlargementinpregnancy
c)Hormonerelatedbreastenlargementingirls
d)Posthormonaltherapybreastenlargementinpostmenopausal
females
CorrectAnswer-C
Ans.is'c'i.e.,Hormonerelatedbreastenlargementingirls
Thelarche
oDefinition:-Beginingofsecondary(Postnatal)breast
developmentatonsetofpubertyingirls.
Tannerstage2breastdevelopment
Usuallyafter8yearsofage
Becauseofrisinglevelofestradiol
Breastdevelopmentduringpubertyinmaletermedas
gynaecomastianotthelarche.

1187.InPrecociouspubertyagelimitofgirls
is-
a)8year
b)10year
c)9year
d)11year
CorrectAnswer-A
Answer-A.8year
Precociouspubertyinagirlistheappearanceofanyofthe
secondarysexualcharacteristicsbeforetheageof8yearsorthe
occurrenceofmenarchebeforetheageof10years.
Precociouspubertyinboysistheonsetofsecondarysexual
characteristicsbeforetheageofnineyears.

1188.Diarrhoeainachildof12month,dose
ofZincis?
a)1mg/10-14day
b)10mg/10-14day
c)15mg/10-14day
d)20mg/10-14day
CorrectAnswer-D
Ans.is'd'i.e.,20mg/10-14days
AccordingtoWHOguidlines
DoseofZinc
2month-6month=10mg/dayx14days>6months=20mg/day
x14days

1189.Seleniumdeficiencyisseenin-
a)Keshandisease
b)Wilsondisease
c)Acrodermatitisenteropathica
d)Noneofabove
CorrectAnswer-A
Ans.is'a'i.e.,Keshandisease
oKeshandisease--->DisorderofSe
metabolism
oWilson'sdisease--->DisorderofCu
metabolism
oAcrodermatitisenteropathica-->DisorderofZn
metabolism

1190.Malnourishedchildminimumweight
gain
a)5gm/kg/day
b)10gm/kg/day
c)15gm/kg/day
d)20gm/kg/day
CorrectAnswer-A
Answer-A.5gm/kg/day
Inmalnourishchildifthere
>10gm/kg/day=goodweightgain
5-10gm/kg/day=moderateweightgain
<5gm/kg/day=poorweightgain

1191.Pretermbabyhave
a)Continueextramedullaryhematopoesis
b)Greaterriskofhypothermia
c)Greaterriskofhypoglycemia
d)Allofabove
CorrectAnswer-D
Answer-D.Allofabove
Pretermbaby(<37weekofgestation).Havehighriskfor
RDS(HMD)
Bronchopulmonarydysplasia(BPD)
PDA
Anemia(early&late)
IVH
Sepsis
NEC
Jaundice
Hypoglycemia,hypokalemia,hypothermia
IVH
ROP(Retinopathyofprematurity).

1192.Lowglucoselevelinpremature
a)Increasedbraintobodyratio
b)Decreasedglycogenstores
c)Decreasedactionofpyruvatecarboxylase
d)Allofabove
CorrectAnswer-D
Answer-D.Allofabove
Hypoglycemiaiscommoninpretem&IUGRbabiesb/callthree
reason

1. Increasedbraintobodyratio.
2. Decreaseactionofpyruvatecarboxylase.
3. Decreaseglycogenstore.

1193.TemperatureofNICUis
a)20-22?C
b)22-26?C
c)26-30?C
d)30-35?C
CorrectAnswer-B
Answer-B.22-26?C
StandardsforNICUthermalinvornment
Specify72-76?F(22-26?C)asacceptablerangeforairtemperature.

1194.Anemiaofprematurity-Trueis
a)Marginalreticulocytosis
b)<10gmcriteria(Hblevel)
c)10ml/kgpackedcell
d)Microcytichypochromictype
CorrectAnswer-B
Answer-B.<10gmcriteria(Hblevel)
Normocytic,normochromic,hyporegenerativeanemiawithEPO
level.
AOPusuallyresolvespontaneouslywithin3-6months.
Aetiology
1. RBCproduction(Reti.count)
2. RBClifespan
3. Bloodloss
Lowhemoglobin(below10gm%)
Reticulocytecountislow.
Treatment:
Requirebloodtransfusionifneeded.
Observeifneonateisasymptomatic.

1195.Ababyisbornat27weeksofgestation
requiredmechanicalventilationfornext
4weeks&02fornext1week.He
maintainedatroomtemperature
subsequently.Aspernew
Bronchopulmonarydysplasiadefinition,
hehaswhichofthefollowing?

a)MildBPD
b)ModerateBPD
c)SevereBPD
d)NoBPD
CorrectAnswer-A
Mild
SupplementO2(for28days)and
<32weeksGABreathingroomairat36weeksatbirthcorrectedGA
oratdischarge(whichevercomesfirst).
.32weeksGABreathingroomairby56daysatbirthpostnatalage
oratdischarge(whichevercomesfirst).
?Thebabyinquestionfallsincategory<32weeksgestationageat
birth.
?Simplylookingatquestion,answerseemstobeseverBPDas
mechanicalventilation(positivepressureventilation)hasincluded
onlyindiagnosticcriteriaofseverBPD.
?But,Thisbabywasbornat27weeksofgestationandrequired
mechanicalventilationfor4moreweeks,i.e.upto31weeks
correctedgestationalage.Afterthathemaintainedatroomair.
Thus,at36weekscorrectedgestaionalage,babyisbreathingat

Thus,at36weekscorrectedgestaionalage,babyisbreathingat
roomair-->diagnosticcriteriaofmildBPD.

1196.HyperglycemiainNeonateifblood
sugarisabove?
a)150mg/dl
b)125mg/dl
c)180mg/dl
d)100mg/dl
CorrectAnswer-B
Ans.is'b'i.e.,125mg/dl
oNoestablisheddefinitionofneonatalhyperglycemiaandupper
safelimitofbloodglucosehasbeendetermined
oVariousresearcheshassuggested
Wholebloodglucose>125mg/dl
Plasmaglucose>150mg/dl

1197.Bloodtransfusiontoneonaterateof
a)1-5ml/min-5ml/min.
b)5-10ml/min
c)10-15ml/min
d)15-20ml/min
CorrectAnswer-B
Answer-B.5-10ml/min
A)Exchangetransfusion
Forterminfant=80-160ml/kg
Forpreterm=100-200ml/kg
B)Top-uptransfusion
DesiredHb(glds)-actualhbxkgx3(usually10-20ml/kg).
Rateoftransfusion5-10ml/min.

1198.Macrosomiais/areassociatedwith:
a)Gestationaldiabetesmellitus
b)Maternalobesity
c)Hypothyroidism
d)A&B
CorrectAnswer-D
Ans.isaandbi.e.GestationaldiabetesandMaternalobesity
Macrosomiaisthetermusedtodescribealargefetus.
Therecommendeddefinitionisfetal(neonatal)weightexceeding
twostandarddeviationsorabove90thcentilefortheappropriate
normalpopulation.
AccordingtoACOG:birthweightof>4500gmiscalledas
macrosomia.
InIndiancontextBirthweightof>4000gmiscalledasmacrosomia.

1199.Inchildren,CHF(congestionheart
failure)isdiagnosedby
a)RaisedJVP
b)Pedaledema
c)Tenderhepatomegaly
d)Basalcrepts
CorrectAnswer-C
Answer-C.Tenderhepatomegaly
Heartfailureininfants&childrenresultssomedegreeof
hepatemegalywhichisusuallytender&sometimessplenomegaly.
Peripheraledemaisagedependent:
Ininfants,edemausuallyeye&overflank.
Olderchildern&adolescentshowsbothperiorbitaledema&pedal
edemaandoccurslate.

1200.LargePDAleadsto?
a)Endocardialvalvulitis
b)Eisenmengersyndrome
c)CHF
d)Allofabove
CorrectAnswer-D
Ans.is'd'i.e.,Allofabove
Patentductusarteriosus(PDA)
SmallPDAmaynotcauseanycomplicationbutlargedefectif
untreadmayleadsto:
PulmonaryhypertensionLefttoRightshuntleadstotoomuch
circulationofbloodinlungwhichleadstopulmonaryhypertension.
Eisenmenger'ssyndrome-Largestandingpulmonaryhypertension
leadstopermanentlungdamageandcausesRighttoLeftshunt.
Endocarditis.
ArrhythmiaEnlargementofheartduetoPDAincreaseriskof
arrhythmias

1201.ASDisassociatedwithallexcept?
a)Infectiveendocarditis
b)Stroke
c)Arrhythemia
d)Pulmonaryhypertension
CorrectAnswer-A
Ans.is'a'i.e.,Infectiveendocarditis
oASDisusuallysubtleandnothavingsignificantproblemexcept
mildgrowthdisturbance.
oOstiumsecundunASDisM.C.typeofASD.
oOstiumsecundumASDusuallyassociatedwithmitralvalve
prolopseorstenosis(Lutembacher'sSyndrome).oComplication
usuallydevelopin4thclecadeandinclude:
I)Pulmonaryhypertension
2)Rt.sidedheartfailure
3)Stroke
4)Eisenmenger'ssyndrome
InfectiveendocarditisisveryrareinASDandnotrequireany
antibioticprophylaxis.

1202.AcuteInfectiveEndocarditiswith
abscessformationismostcommonly
associatedwith

a)Listeria
b)Staphylococcus
c)Streptococcus
d)Enterococcus
CorrectAnswer-B
AnswerisB(Staphylococcus)
AcuteInfectiveEndocarditiswithabscessformationismost
commonlyassociatedwithstaphylococcus.
`Themostcommonorganismcausingacuteinfectiveendocarditis
overallisstaphylococcusaureus.Staphylococcusaureus
endocarditisisparticularlyvirulentandassociatedwithannularand
myocardialabscessformationandahighermortality'


1203.Whichofthefollowingstatementsregardingfetalcirculationiscorrect?
a)Theliverandheartofthefetusreceivebloodwithveryhigh
oxygensaturation
b)PO2offetalbloodleavingtheplacentaisslightlygreaterthan
maternalmixedvenousPO2
c)Thepresenceoffetalhemoglobinshiftstheoxyhemoglobin
dissociationtotheright
d)Theforamenovaleclosesduringthethirdtrimesterunlessthe
fetushasanatrialseptaldefect
CorrectAnswer-A
Sincetheliverissuppliedbyumbilicalvenousbloodfromtheplacenta,andtheheart
andheadreceivebloodbeforeithasmixedwithsignificantamountsofdesaturated
blood,theseimportantorgansreceivebloodthatisrelativelyhighinsaturated
oxyhemoglobin.

Thehighrateofbloodflowattheplacentaandthesignificantresistanceoftheplacentato
diffusionofoxygenresultinbloodintheumbilicalveinthathasalowerPO2(30mmHg)
thanthematernalmixedvenousblood.
However,theleftshiftinfetaloxyhemoglobinconcentrationandtheBohreffectbothactto
increasethetransportofoxygentofetaltissues.
Anumberofsignificantdifferencesincirculatingpatternsarepresentinthefetus.
Theforamenovaleremainsopenuntilafterbirthandasignificantportionofinferior
venacavaflowisshuntedthroughittotheleft.
1/3ofbloodentersdirectlyfromrightatriumtoleftatrium(bypassingrightventricle)through
foramenoval
Themajorportionofrightventricularoutputisshuntedthroughtheductusarteriosustothe
aorta,notthelungs.
Theneteffectoftheseshuntsinthepresenceofhighfetalpulmonaryvascularresistanceis
verylowfetalpulmonarybloodflow.

Atbirth,thesepatternsnormallyarequicklychangedtoex-uteropatternswithhigh
pulmonaryperfusion.
Ref:BarrettK.E.,BarmanS.M.,BoitanoS.,BrooksH.L.(2012).Chapter33.Circulation
throughSpecialRegions.InK.E.Barrett,S.M.Barman,S.Boitano,H.L.Brooks(Eds),
Ganong'sReviewofMedicalPhysiology,24e.

1204.Doublearchaortaisseenin
a)CATCH22
b)Digeorgesyndrome
c)Sphrintzenvelocardiofacialsyndrome
d)Noneofabove
CorrectAnswer-A
Answer-A.CATCH22
CATCH22standsfor
Cardiacdefect-doublearchaorta.VSD,pulmonaryatresia.
Cleftpalate
Abnormalfacies
Hypocalcemia
Thymichypoplasia

1205.Decorticatechild-Flasestatementis?
a)AcuteBraininjurys
b)Susthalamic,CT&frontallobelesion
c)Moredangerousthandecerebratelesion
d)Flexionofarm&extensionoflowerlimb
CorrectAnswer-C
Ans.is'c'i.e.,Moredangerousthandecerebratelesion
DecorticalPosture
oAlsoknownasflexorposturingorMummybaby
oArmsflexed/bentoverchest,handfisted,legextended&rotated
inward
oDamagetoareaincerebralhemisphere,internalcapsule,
thalamus&upperpartofbrain.
oDecorticatepostureisominoussignofseverebraindamage.
Decerebrateposture
oAlsoknownasextensorposturing
oExtensionofupperlimb&lowerlimb(ELBOWEXTENDED)
oIndicatesbrainstemdamage(Belowlevelofrednucleus)
oDecerebratepostureismoreominousthandecorticalposture

1206.Treatmentofsimplefebrileconvulsion
isbasedon
a)Controloffever
b)Rectaldiazepam
c)CSFfinding
d)Bloodreports
CorrectAnswer-A
Answer-A.Controloffever
Promptreductionoftemperaturebyhydrotherapy(sponging)and
antipyretics(paracetamole,ibuprofen)isthemostimportant
measure.Ifseizureslastsformorethan5minutes,diazepam(rectal
orIV)istheanticonvulsantofchoice.Phenobarbitoneisan
alternative

1207.Whatiscorrectaboutfebrileseizures
a)NormalEEG
b)Focaldeficits
c)Repeatedseizure
d)AbnormalEEG
CorrectAnswer-A
Answer-A.NormalEEG
Spontaneousremmissionoccurswithnopostictalneurological
deficitandEEGchangesfewdaysaftertheseizureisnormal.
Recurrentfebrileseizuresoccurin30-50%ofcases.
Morethan90%offebrileseizuresaregeneralized.
Acuterespiratoryillnessaremostcommonlyassociatedwithfebrile
seizures.

1208.Chronicconstipationinchildrenisseen
inallA/E?
a)Hirschspringdisease
b)Jejunalpolyp
c)Hypothyroidism
d)Stricture
CorrectAnswer-B
Ans.is'b'i.e.,Jejunalpolyp
Organiccausesofconstipationare:
i)Intestinal:
Hirschprungdisease,Anal/rectalstenosis,Anal
fissure,anteriorlydisplacedanalopening,strictures.
ii)Drugs:Narcotics,vincristine,Psychotropics.
iii)Metabolic/endocrine:Cysticfibrosis,hypothyroidism,
Panhypopituitrism
iv)Neuromuscular:Cerebralpalsy,Psychomotorretardation,
spinalcordlesions,Myotonicdystophy,Neuropathyormyopathyof
GIT
v)Othercauses:Lowfibrediet,Milkproteinallergy.

1209.3monthinfantswithabdominal
palpablemass&nonbiliousvomiting-
a)Hypertrophicbiliarystenosis
b)Hypertrophicpyloricstenosis
c)Tracheoesophagealfistula
d)Duodenalatresia
CorrectAnswer-B
Ans.is'b'i.e.,Hypertrophicpyloricstenosis
Hypertophicpyloricstenosis
MostcommoncauseofnonbiliousvomitingisHypertrophicpyloric
stenosis.
Male>female.
Vomitingstartswith3weekofage.
Palpablemassisseeninepigastricregion.
Visibleperistalsisisseensoonafterfeeding.
ConfirmedbyUSGabdomen.
Contraststudyshows:
.Shouldersign
.Doubletractsign.
Treatmentsurgery=Ramstedtprocedure.

1210.Causesofconjugated
hyperbilirubinemiais?
a)Rotorsyndrome
b)Breastmilkjaundice
c)Criglernajjar
d)Gilbertsyndrome
CorrectAnswer-A
Ans.is'a'i.e.,Rotorsyndrome
Conjugatedhyperbilirubinemiaisseenwhen?
i)Impairedsecretionofconjugatedbilirubinintobile->Dubin
Johnsonsyndrome,Rotorsyndrome.
ii)Impairedbileflow-->Obstructivejaundice,primarybiliary
cirrhosis,Neonatalcholestasis,e.g.Extrahepaticbiliary
atresia/neonateidiopathichepatitis,Choledocalcyst,Sclerosing
cholangitis,Carolidisease,Metabolic
(Tyrosinemia,Wolman
disease,Niemanpickdisease,Galactosemia,Fructosemia).

1211.Causeofneonatalhyperbilirubinemia?
a)Inefficienterythropoiesis
b)RBChemolysis
c)Immatureliverenzyme
d)Allofabove
CorrectAnswer-D
Answer-D.Allofabove
ImpairedbileflowObstructivejaundice,primarybiliarycirrhosis,
Neonatalcholestasis,e.g.Extrahepaticbiliaryatresia/neonate
idiopathichepatitis,Choledocalcyst,Sclerosingcholangitis,Caroli
disease,Metabolic(Tyrosinemia,Wolmandisease,Niemanpick
disease,Galactosemia,Fructosemia).

1212.InDiapragmaticHernia.Mostcommon
anomalyisseenin
a)Cardiovascularanomalies
b)Urinarytract
c)Craniofacialanomaly
d)Skullanomaly
CorrectAnswer-A
Answer-A.Cardiovascularanomalies
DiaphragmaticHerniaisdefinedasacommunicationbetween
abdominal&thoraciccavitieswithorwithoutabdominalcontentsin
thorax.
Femalesareaffectedmorethanmales.
Morecommononleftsideandisposterolateral.
Associatedanomaliesmaybeseen30%ofcasesCNSlesions,
esophagealatresia,omphaloceleandCVSlesions.Mostcasesare
sporadic.
CardiacanomalisisMCanomalis.

1213.Hirschsprungdiseaseisconfirmedby?
a)Rectalbiopsy
b)Per/Rectalexamination
c)Rectalmanometry
d)X-rayabdomen
CorrectAnswer-A
Ans.is'a'i.e.,Rectalbiopsy
oRectalsuctionbiopsyisprocedureofchoice.

1214.Mostcommonpresentationofwilm's
tumor?
a)Hematuria
b)Asymptomaticabdominalmass
c)Abdominalpain
d)Headache
CorrectAnswer-B
Ans.is`b'i.e.,Asymptomaticabdominalmass
Wilm'stumor(Nephroblastoma)
Mostcommonmalignanttumorofkidney.
80%oftumorpresentbelow5yearofage.
Presention:?
Asymptomaticabdominalmass(M.C.).
Haematuria(10-25%).
Hypertension(25%).
Abdominalpain(30%).
Fever(20%).

1215.Posteriorurethralvalve-trueA/E?
a)Palpablebladder
b)Hydronephrosis
c)Painfulstressincontinence
d)RecurrentUTI
CorrectAnswer-C
Ans.is`c'i.e.,Painfulstressincontinence
Posteriorurethralvalve
Mostcommoncauseofsevereobstructiveuropathyinchildren.
30%ofchildrenexperiencedendstagerenaldisease/CRF
Dilatedprostaticurethra.
Hypertrophyofbladdermuscle
Vesicoureticrefluxseenin50%ofcases.
Backpressurechange:
Hydronephrosis
Distendedbladder
Thinurinarystream
RecurentUTIbecauseofurinarystasis

1216.Nottrueaboutchronicpyelonephritisin
children-
a)AssociatedwithUretericreflux
b)AssociatedwithIntrarenalreflux
c)Associatedwithrenalscarring
d)Malesaremoreaffectedthanfemale
CorrectAnswer-D
Ans.is'd'i.e.,Malesaremoreaffectedthanfemale
o3basicformsofUTI-(1)Pyelonephritis(2)Cystitis(3)
Asymptomaticbacteremia
ChronicPyelonephritis:-
Characterisedbyrenalinflammation&fibrostsinducedbyrecurrent
orpersistentrenalinfection,vesicouretericrefluxorothercausesof
UTI.
Progressiverenalscarringwhichleadstoendstagerenal
diseaes.
Inreflexnephropathy,Intrarenalreflexofinfectedurineinduces
renalinjurywhichhealsbyrenalscarring.
Being-straight,short&wideurethra,recurrentUTIismorecommon
infemale,soCPNisalsotwiceascommonisfemalethanmale.

1217.Mousyurineinachildduetodefectin
phenylalamineto
a)Tyrosine
b)Homogentisicacid
c)Phenylacetate
d)Phenylpyruvate
CorrectAnswer-A
Answer-A.Tyrosine
Phenylketonuria:
Autosomalrecessive
Deficiencyofphenylalaninehydroxylase.
Defectinconversionofphenylalaninetotyrosine.
Thisleadstoincreaselevelofphenylalanine.
Thisincreasephenlylalanineconvertedintophenylpyruvateand
phenylacetate.
Thisphenylacetategivesmousyormustyodourinurine/body.

1218.Highestcurerateisof-
a)Wilm'sTumor
b)Retinoblastoma
c)Rhabdomyosarcoma
d)All
CorrectAnswer-B
Ans.is'b'i.e.,Retinoblastoma
Tumor
5yearssurvivalrate
Retinoblastoma
97%
Wilms
tumor
88%
Rhabdomyosarcoma
61%

1219.Mostcommonsiteofextramedullary
relapseofALLin6yearoldis
a)Testes
b)Liver
c)Lung
d)None
CorrectAnswer-A
Answer-A.Testes
ThecommonsitesofrelapseofALLaftercompleteremissionare:
Bonemarrow(mc),CNS(2mc)andtestis.

1220.Proximalhumerusfracturewhichhas
maximumchancesofavascular
necrosis

a)Onepart
b)Twopart
c)Threepart
d)Fourpart
CorrectAnswer-D
Answer-D.Fourpart
Neer'sclassifiedproximalhumerusfractureinto4types,based
onthesefoursegments:
1.Onepartfracture:-Fracturewithminimaldisplacement.
2.Twopartfracture:-Onesegmentisdisplacedinrelationtoother.
Importanttwopartfracturesare:
(i)surgicalneckfracture(mostcommon)(ii)GTfracture,(iii)LT
fracture,(iv)anatomicalneckfracture.
3.Threepartfracture:-Twosegmentsaredisplacedinrelationto
othertwo.
4.Fourpartfracture:-Allfourmajorfragmentsaredisplaced.
ChancesofAVNofhumeralheadarehigherinfourpartfracture.

1221.Mostcommontypeofshoulder
dislocationis:
March2011

a)Posterior
b)Anterior
c)Inferior
d)Superior
CorrectAnswer-B
Ans.B:Anterior
Shoulderjointisthecommonestjointinthebodytodislocate
Anteriorshoulderdislocationismuchmorecommonthanposterior
dislocation
Shoulderdislocation:
MCjointtodislocateinbody:Shoulder
MCtypeofshoulderdislocation:Subcoracoid/inferior
Raresttypeofshoulderdislocation:Posterior
Posteriortypeofshoulderdislocationisassociatedwith:Epilepticfits
Bankart'slesionisrelatedwithavulsionofglenoidallabrum
HillSach'slesionisfoundonhumeralhead
Testfordislocationofshoulderjoint:
-Duga'stest,
-Hamiltonrulertest,
-Callawaystest
Kocher'smanouevreisdonetoreduceadislocatedshoulder

1222.Mostcommonmuscledamagedin
rotatorcuff
a)Supraspinatus
b)Infraspinatus
c)Subscapularis
d)Teresminor
CorrectAnswer-A
Answer-A.Supraspinatus
Duringinjurytorotatorcuff,tendonsareaffected,notthemuscle.
Supraspinatustendonisaffectedmostfrequently.

1223.Trueaboutsupracondylarfractureof
humerus
a)Commoninadults
b)Extensiontypemostcommon
c)Flexiontypeismostcommon
d)None
CorrectAnswer-B
Answer-B.Extensiontypemostcommon
Supracondylarhumeralfracturesarethemostcommonelbow
fracturesinchildren.Mostcommonagegroupaffectedis5-8years.
Boysareaffectedmorethangirls.Leftsideismorecommonthan
right.
Extensiontypeismostcommon

1224.Themalunionofsupracondylarfracture
ofthehumerusmostcommonlyleads
to:

a)Flexiondeformity
b)Cubitusvarus
c)Cubitusvalgus
d)Extensiondeformity
CorrectAnswer-B
Bi.e.CabitusVarus
Malunion:-Itisthecommonestcomplicationofsupracondylar
fractureandresultsincubitusvarus(Gunstockdeformity).Cubitus
valgusisrareandmayoccuroccasionallyinposterolateral
displacement.

1225.TheMOSTcommontypeofdislocationofelbowjointis:
a)Posterior
b)Posterolateral
c)Posteromedial
d)Lateral
CorrectAnswer-A
Inadults,theelbowisthesecondmostfrequentlydislocatedmajorjoint,aftertheshoulder.
Itisthemostcommonlydislocatedjointinchildren.Morethan90%ofallelbowdislocations
areposteriordislocations.
Thisinjuryentailsdisengagementofthecoronoidprocessoftheulnafromthetrochleaof
thehumeruswithmovementposteriorly.
Themechanismofinjuryistypicallyafallontoanoutstretchedhandwiththeelbowin
extensionuponimpact.

1226.Whatisseenonx-raywithposterior
elbowdislocation
a)Coronoidprocessposteriortohumerus
b)Coronoidprocessanteriortohumerus
c)Coronoidprocessbelowhumerus
d)None
CorrectAnswer-A
Answer-A.Coronoidprocessposteriortohumerus
APview:Greatersuperimpositionofdistalhumeruswithproximal
ulnaandolecranon(normally,onlyterminalpartofhumerusis
superimposed).

1227.

Inposteriordislocationofelbow,most
prominentpart

a)Coronoid
b)Radialhead
c)Olecranon
d)None
CorrectAnswer-C
Answer-C.Olecranon
ThereisexaggratedprominenceoftheOlecranon

1228.Mostcommoncomplicationoflateral
condylehumerusfracture
a)Malunion
b)Nonunion
c)VIC
d)Mediannerveinjury
CorrectAnswer-B
Answer-B.Nonunion
Lateralspur(Lateralcondylarspurorlateralcondylarovergrowth)is
oneofthemostcommoncomplication.Delayedunionornon-union
canoccuriffractureisundetectedorleftuntreated.
Cubitusvalgusisacommoncomplication
Tardyulnarnervepalsyseenafterseveralyears.
Rarelyavascularnecrosisandmyositisossificans.

1229.PulledElbowis:
a)Disarticulationofelbow
b)Disarticulationofelbow
c)Subluxationofproximalradioulnarjoint
d)Noneoftheabove
CorrectAnswer-C
Ci.e.subluxationofproximalradioulnarjoint
Ifayoungchildisliftedbythewrist,theheadoftheradiusmaybe
pulledpartlyoutoftheannularligament,i.e.,subluxationofthehead
oftheradius.
Itoccurswhenforearmispronated,elbowisextendedand
longitudinaltractionisapplied
tothehandorwrist,e.g.,lifting,
spinningorswingingachildwithwristorhand.Pulledelbowmost
commonlyoccursbetweentheageof2-5years

1230.Oneofthecommonfracturesthatoccur
duringboxingbyhittingwithaclosed
fistis?

a)Monteggiafracturedislocation
b)Galeazzifracturedislocation
c)Bennett'sfracturedislocation
d)Smith'sfracture
CorrectAnswer-C
Ans.is'c'i.e.,Bennett'sfracturedislocation
ThecommonmechanismofinjuryforBenett'sfractureisanaxial
blowdirectedagainstthepartiallyflexedmetacarpal,inmostcases
during'fistfights'.
Benett'sfracture
Benett'sfractureisanintra-articularfracturedislocationofthe
palmarbaseoffirstmetacarpalboneofthethumbwitheither
subluxationordislocationoffirstcarpometacarpaljoint,i.e.
trapezometacarpaljoint.
Thecommonmechanismofinjuryisan
axialblowdirectedagainstthepartiallyflexedmetacarpal,inmost
casesduring"Fistfights".Patientcomplainsofpain,swellingand
tendernessoverthebaseofthethumb.Movementsofthumbare
restricted.
DisplacingforceinBennett'sfractures
FollowingarethedeformingforcesinBennett'sfracture:-
i)Atthedistalfragment,itistheadductorpollicis.
ii)
Attheproximalfragment,itistheabductorpollicislongus.
Baseofthethumbmetacarpalispulleddorsallyandmediallybythe
abductorpollicislongus,whilethedistalattachmentofadductor


pollicisfurtherleversthebaseintoabduction.

1231.MostcommoncomplicationofCones
#
a)Malunion
b)Avascularnecrosis
c)Fingerstiffness
d)RuptureofEPLtendon
CorrectAnswer-C
Answer-C.Fingerstiffness
Thecomplicationrateis55%,withthemostcommoncomplication
beingsomedegreeofresidualfingerandwriststiffness(39%).
Mostcommoncomplicationofcolle'sfracture-Fingerstiffness

1232.Mostcommoncomplicationoffracture
neckoffemur
a)Malunion
b)AVN
c)Nonunion
d)Arthritis
CorrectAnswer-B
Answer-B.AVN
AVNisthemostcommoncomplicationoffemoralneckfracture.
Non-unionisthesecondmostcommoncomplicationoffemoralneck
fracture.

1233.Whichofthefollowingdescribesgrade
2fractureneckfemur?
a)Incompletefracture,medialtrabeculaeintact
b)Completefracturewithundisplacedneck
c)Completefracturewithischemichead
d)Moderatedisplacementofneck,vascularitydamaged
CorrectAnswer-B
Answer-B.Completefracturewithundisplacedneck
GardenstageI:undisplacedincomplete,includingvalgusimpacted
fractures
medialgroupoffemoralnecktrabeculaemaydemonstratea
greenstickfracture
GardenstageII:undisplacedcomplete
nodisturbanceofthemedialtrabeculae

1234.Treatmentofchoiceforfractureneck
femurina40yearsoldfemale
a)Multiplescrewfixation
b)Bipolarhemiarthroplasty
c)THR
d)None
CorrectAnswer-A
Answer-A.Multiplescrewfixation
Placementofmultiplescrewsacrossthefracturedfemoralneckis
thetreatmentofchoiceforfemoralneckfractures,andmaybe
performedfollowingeitherclosedoropenreductionusingastandard
lateralapproachoramorelimitedpercutaneoustechnique.

1235.Talusissuppliedby
a)Anteriortibialartery
b)Posteriortibialartery
c)Dorsalpedisartery
d)All
CorrectAnswer-D
Answer-D.All
Extraosseousbloodsupply

1. Posteriortibialartery:-Deltoidbranch,arteryofthetarsalcanal.
2. Anteriortibialartery:-SuperiorneckbranchfromDorsalpedisartery,
Arteryoftarsalcanal.
3. Paronealartery:-Arteryoftarsalsinus.

1236.Commonestligamentinjuredinankle
injury?
a)Anteriortalofibularligament
b)Calcaneofibularligament
c)Posteriortalofibularligament
d)Springligament
CorrectAnswer-A
Ans.is'a'i.e.,Anteriortalofibularligament
Theankleisoneofthemostcommonsitesforacute
musculoskeletalinjuries.Sprainsconstitute85%ofallankleinjuries,
and85%ofthoseinvolvealateralinversionmechanism.
InversionSprain-Inversionanklesprainsoccurwhenthefootturns
inorouttoanabnormaldegreerelativetotheankle.Themost
commonmechanismofananklesprainisacombinationof
plantarflexionandinversionwherethefootispointingdownwardand
inward.
Thelateralligamentsareinvolvedinaninversionanklesprainand
hencemostcommonlydamaged.Theseligamentsareonthe
outsideoftheankle,whichincludestheanteriortalofibular(ATFL),
calcaneofibular(CFL)andposteriortalofibularligaments(PTFL).
InjurytotheATFListhemostcommon.WhenboththeATFLand
CFLareinjuredtogether,ankleinstabilitywillbemorenoticeable.
ThePTFListhestrongestofthethreeligamentsandisrarelyinjured
inaninversionsprain.

1237.Posteriorscallopingofvertebraeisnot
seen
a)Neurofibromatosis
b)Astrocytoma
c)Aorticaneurysm
d)Ependymoma
CorrectAnswer-C
Answer-C.Aorticaneurysm
Posteriorscallopingistheconcavitytotheposterioraspectofthe
vertebralbody.
AnteriorScallopingistheconcavitytotheanterioraspectofthe
vertebralbody.

1238.Incervicalspineinjury,firsttobedone
a)Trunhead
b)Maintainairway
c)Immobilizationofspine
d)None
CorrectAnswer-B
Answer-B.Maintainairway
"Initialcareattheaccidentscenemaybecriticaltosurvival.Thefirst
stepsaretoestablishanairway,maintainoxygenationand
immobilizethecervicalspine".

1239.Inscoliosisdegreeofdeformityis
calculatedby
a)Cobbsmethod
b)Hamburgermethod
c)Haldanemethod
d)Milwaukeemethod
CorrectAnswer-A
Answer-A.Cobbsmethod
Cobbangleisameasurementofthedegreeofside-to-sidespinal
curvature,whichisadeformityyoumayknowasscoliosis.

1240.Partialanteriordislocationofone
segmentofthespineoveranotheris
a)Spondylosis
b)Spondylolisthesis
c)Kyphosis
d)Scoliosis
CorrectAnswer-B
Answer-B.Spondylolisthesis
Displacement(partial)ofonevertebraeoverotheriscalled
Spondylolisthesis.
Spondylolisthesisisadefectintheparsinterarticularis.

1241.InEMG/NCVstudy,H.reflexcorrelates
with
a)L3radiculopathy
b)L4radiculopathy
c)L5radiculopathy
d)SIradiculopathy
CorrectAnswer-D
Answer-D.SIradiculopathy
H-ReflexhasitsutilityininvestigatingpatientswithS1radiculopathy.

1242.Inaxillarynerveparalysis,allthe
followingaretrueexcept
a)Deltoidmuscleiswasted
b)Extensionofshoulderwitharmabductedto90degreesis
impossible
c)Smallareaofnumbnessispresentovertheshoulderregion
d)Patientcannotinitiateabduction
CorrectAnswer-D
Answer-D.Patientcannotinitiateabduction
Initial15?ofabductioniscausedbysupraspinatuswhichissupplied
bysuprascapularnerve(notaxillarynerve).
"Attheinitiationofabductionfromneutralposition,thesupraspinatus
ismoreimportantthandeltoid,whereasdeltoid(middleportion)isof
greaterimportanceforelevationofarmatthehigherangleof
abduction,suchas60?".

1243.Tardyulnarnervepalsyisseenin
a)Medialcondyle#humerus
b)Lateralcondyle#humerus
c)Humerusshaftfracture
d)Fractureshaftradius
CorrectAnswer-B
Ans.is'b'i.e.,Lateralcondyle#humerus
Causesoftardyulnarnervepalsyare:-

1. Malunitedlateralcondylehumerusfracture(cubitusvalgus)
2. Displacedmedialepicondylehumerusfracture
3. Cubitusvarusdeformity(duetosupracondylarfracturehumerus)
4. Elbowdislocation
5. Contusionsofulnarnerve
6. Shallowulnargroove
7. Hypoplasiaofhumeraltrochlea
8. Jointdeformityafterprolongedarthritisofelbow

1244.Cubitaltunnelsyndromeinvolves:
March2013(c,f)
a)Radialnerve
b)Ulnarnerve
c)Mediannerve
d)Axillarynerve
CorrectAnswer-B
Ans.Bi.e.Ulnarnerve
Whentheulnarnervecompressionoccursattheelbow,itiscalled
"cubitaltunnelsyndrome."

1245.Carpaltunnelsyndromeallarepresent
except
a)Ulnarnervedysfunction
b)Tinelsign
c)Phalenssign
d)Pain&paraesthesiaofwrist
CorrectAnswer-A
Answer-A.Ulnarnervedysfunction
Thesymptomsoftenfirstappearduringnight,sincemanypeople
sleepwithflexedwrists.(Flexiondecreasesthespaceincarpal
tunnelwhichresultsinincreasedpressureovermediannerve).
Sensorysymptomscanoftenbereproducedbypercussingover
mediannerve(Tina'ssign)orbyholdingthewristfullyflexedfora
minute(Phalen'stest).
Asthediseaseprogresses,clumsinessofhandandimpairmentof
digitalfunctiondevelop.

1246.Fairbanktriangleisseenin
a)CDH
b)Congenitalcoxavara
c)Perthe'sdisease
d)SCFE
CorrectAnswer-B
Answer-B.Congenitalcoxavara
Theepiphysealplatemaybetoovertical.
Theremaybeaseparatetriangleofboneintheinferiorportionof
themetaphysis,calledFairbank'striangle
[RefEbnezar4th/ep.410]

1247.Dysplastichipinachild,investigation
ofchoice
a)X-ray
b)MRI
c)USG
d)CTScan
CorrectAnswer-C
Answer-C.USG
UltrasonographyistheinvestigationofchoiceforDDH.
Itvisualizesthecartilageandallowsdynamictestingofthehipjoint.

1248.PrimarypathologyinCDH
a)Largeheadoffemur
b)Shallowacetabulum
c)Excessiveretroversion
d)Evertedlimbus
CorrectAnswer-B
Answer-B.Shallowacetabulum
Followingchangesareseenindislocatedhip:

1. Thefemoralheadisdislocatedupwardandlaterally.Itsbony
nucleusappearslateanditsdevelopmentisretarded,therefore
headissmall.
2. Femoralneckisexcessivelyanterverted.
3. Acetabulumisshallow,withasteepslopingroof(Thisisconsidered
tobetheprimarypathology).

1249.InneglectedcasesofCTEV,jointfused
are
a)Calcaneocuboid,talonavicularandtalocalcaneal
b)Tibiotalar,calcaneocuboidandtalonavicular
c)Anklejoint,calcaneocuboidandtalonavicular
d)Noneoftheabove
CorrectAnswer-A
Answer-A.Calcaneocuboid,talonavicularandtalocalcaneal
JointsfusedintriplearthrodesisforCTEVarei)Subtalar
(tolocalcaneal)joint,ii)Calcaneocuboidjoint,iii)Tolonavicularjoint.

1250.Perthe'sdiseaseisOsteochondritisof
theepiphysisofthe:
March2013(g)

a)Capitulum
b)Lunate
c)Femoralhead
d)Calacanealtuberosity
CorrectAnswer-C
Ans.Ci.e.Femoralhead
Osteochondritis
Perthesdisease:
-Osteochondritisoffemoralhead
-Adductionisunaffected
-IOCforPerthesdisease:MRI
Osteochondritisoflunate:Kienbock'sdisease
Osteochondritisoftibialtubercle:OschGood'sSchlatter'sdisease
Osteochondritisofcalcaneum:Sever'sdisease

1251.Osteonerosisisseeninallexcept
a)Fractureneckfemur
b)Sicklecellanemia
c)Perthe'sdisease
d)Paget'sdisease
CorrectAnswer-D
Answer-D.Paget'sdisease
Storagedisorders:-Gaucher'sdisease
Caissondisease:-Dysbaricosteonecrosis
Hemoglobinopathy&Coagulationdisorder:-Sickelcelldisease,
Familialthrombophilia,Hypofibrinolysis,Hypolipoproteinemia.
Congenitaldisorders:-Perthe'sdisease,Slippedcapitalfemoral
epiphysis.
Hematologicalmalignancies:-Leukemia,lymphoma,Polycythemia.
Hyperlipedemia:-Nephroticsyndrome

1252.Bestdiagnosticmodalitytodiagnose
avascularnecrosisis:
March2007

a)MRIscan
b)CTscan
c)X-ray
d)USG
CorrectAnswer-A
Ans.A:MRIScan
Avascularnecrosis/osteonecrosis/aseptic(bone)necrosis/
ischemicbonenecrosisisadiseaseresultingfromthetemporaryor
permanentlossofthebloodsupplytothebones.Withoutblood,the
bonetissuediesandcausesthebonetocollapse.Therearemany
causesofavascularnecrosissuchas:
Alcoholism
Excessivesteroiduse
Posttrauma
Caissondisease(decompressionsickness)
Vascularcompression
Vasculitis
Thrombosis
Damagefromradiation
Bisphosphonates(particularlythemandible)
Sicklecellanaemia
Gaucher'sDisease
Idiopathic(nocauseisfound).
RheumatoidarthritisandlupusarealsocommoncausesofAVN.
Avascularnecrosismostcommonlyaffectstheheadoffemur.Other

Avascularnecrosismostcommonlyaffectstheheadoffemur.Other
commonsitesincludethetalus,scaphoidandthejaw.Avascular
necrosisusuallyaffectspeoplebetween30and50yearsofage.
Whenitoccursinchildrenatthefemoralhead,itisknownasLegg-
Calve-Perthessyndrome.
Itismostoftenlydiagnoseclinically.
BecauseearlyX-raysareusuallynormalintheearlystageofthe
disease,bonescintigraphyandMRIarethediagnosticmodalityof
choicesincebothcandetectminimalchangesatearlystagesofthe
disease.
Lateradiographicsignsincludearadiolucencyareafollowingthe
collapseofsubchondralbone(crescentsign)andringedregionsof
radiodensity

1253.Afterchronicuseofsteroidssevere
paininrighthipwithimmobilityisdue
to

a)Avascularnecrosis
b)Perthesdisease
c)Hipdislocation
d)Osteoarthritis
CorrectAnswer-A
Answer-A.Avascularnecrosis
Paininhipandlimitatationofmovement(immobility)ofterchronic
useofsteroidssuggestthediagnosisofAVNoffemoralhead.

1254.Inelbow,osteochondritisusually
involves
a)Olecranon
b)Trochlea
c)Radialhead
d)Capitulum
CorrectAnswer-D
Answer-D.Capitulum
Knee-Lateralsurfaceofthemedialfemoralcondyle
Elbow-Capitulumofhumerus
Hip-Femoralhead
Ankle-Talus

1255.Mostcommonorganismcausing
infectionafteropenfracture
a)Pseudomonas
b)Staphylococcusaureus
c)Klebsiella
d)Gonococcus
CorrectAnswer-A
Answer-A.Pseudomonas
staphylococcusaureuswasthemostcommonorganismcausing
infectioninopenfractures.Morerecentstudieshaveshownthat
gramnegativeorganismssuchaspseudomonasaeruginosaand
E.coliarebecomingmorecommon

1256.Septicarthritisisdiagnosedby
a)X-ray
b)Jointaspiration
c)USG
d)MRI
CorrectAnswer-B
Answer-B.Jointaspiration
Quickestandbestmethodofdiagnosisofsepticarthritisisaspiration
ofjoint.

1257.Sequestrumisbestdefinedas
a)Apieceofdeadbone
b)Apieceofdeadbonesurroundedbyinfectedtissue
c)Apieceofbonewithpoorvascularity
d)None
CorrectAnswer-B
Answer-B.Apieceofdeadbonesurroundedbyinfectedtissue
Sequestrumisapieceofdeadbone,surroundedbyinfected
granulationtissue.

1258.Cloacaearepresentin
a)Sequestrum
b)Involucrum
c)Normalbone
d)Myositis
CorrectAnswer-B
Answer-B.Involucrum
Involucrumisreactivenewboneoverlyingasequestrum.Theremay
besomeholesintheinvolucrumforpustodrainout.Theseholes
arecalledcloaca.

1259.ComplicationofjointTB
a)Fibrousankylosis
b)Bonyankylosis
c)Normalhealing
d)None
CorrectAnswer-A
Answer-A.Fibrousankylosis
Theoutcomeoftuberculararthritisisfibrousankylosis,exceptin
spine.Spineistheonlysitewheretuberculosishealswithbony
ankylosis.

1260.FalseaboutPott'sspine
a)Commonestatdorsolumbarjunction
b)Alwayshealsbychemotherapy
c)Backpainisanearlysymptom
d)Thereisdiscspacenarrowingonx-ray
CorrectAnswer-B
Answer-B.Alwayshealsbychemotherapy
Chemotherapyisthemainstayoftreatment.Butitisnoteffective
always,surgeryisoftenrequiredinsomecases.
Commonestsiteofpott'sspineisdorsolumbarjunction.
Backpainistheearliestsymptomandnarrowingofdiscspaceisthe
earliestradiologicalsign.

1261.Apparentlengtheningisseeninwhich
stageofTBHip
a)StageI
b)StageII
c)StageIII
d)None
CorrectAnswer-A
Answer-A.StageI
Stageofsynovitis(Stage1):-Threeiseffusioninthehipjointwhich
demandsthehiptobeinapositionofmaximumcapacity.This
positionisacquiredbyflexion,abductionandexternalrotation.
Becauseofabductiondeformity,thereisapparentlengthening.

1262.Tuberculosisspine;mostcommonsite
is
a)Sacral
b)Cevical
c)Dorsolumbar
d)Lumbosacral
CorrectAnswer-C
Answer-C.Dorsolumbar
ThemostcommonsiteisDorsolumbarregion.Lowerdorsal
(thoracic)regionisthemostcommonsegmentinvolvedfollowedby
lumbarsegment.Thetuberculosisofspineisalsocalledpott's
diseaseortubercularspondylytis.

1263.Anterolateraldecompressionisdone
for
a)Spinaltuberculosis
b)ChestTB
c)HandTB
d)FootTB
CorrectAnswer-A
Answer-A.Spinaltuberculosis
Anteriordecompressioncanbecausedby:-

1. Anteriorapproach:-Calledanteriordecompression.Itisthemost
preferredprocedure.
2. Anterolateralapproach:-Calledanterolateraldecompression.

1264.Tumorwithmaximumbonematrix
a)Osteoidosteoma
b)Chondrosarcoma
c)Enchondroma
d)None
CorrectAnswer-A
Answer-A.Osteoidosteoma
Dense,homogenousmineralization(calcification)istypicalof
osteoidmatrix,formedbybenignandmalignantboneforming
lesions.

1265.Whichofthefollowingistrueabout
Giantcelltumor
a)Usuallypresentsasalyticlesionwithscleroticrim
b)Alwaysbeingn
c)Epiphysealorigin
d)Seeninagelessthan15years
CorrectAnswer-C
Answer-C.Epiphysealorigin
GCTisanosteolytictumorarisingfromtheepiphysisandis
commonbetweentheageof20-40years.
Thecommonestsitesarelowerendoffemurandupperendoftibia.
Othercommonsitesarelowerendradiusandupperendof
humerus.Itmayalsooccurinthespineandsacrum.
Theradiologicalfeaturesare:-
1. Asolitarymaybeloculated,lyticlesion.
2. Eccentriclocation,oftensubchondral.
3. Expansionoftheoverlyingcortx(expansilelesion).
4. Soap-bubble'appearance

1266.Striatedvertebraisseenin
a)TBspine
b)Haemangioma
c)Chordoma
d)Metastasis
CorrectAnswer-B
Answer-B.Haemangioma
Haemangiomaofthevertebrahasatypicalradiographicpicturein
theformoflossofhorizontalstriationsandprominenceofvertebral
striations.ThereisPolkadotappearanceonCT.
Intheskull,hemangiomagenerallyaffectsthecalveriumandisseen
asanexpansilelyticlesionwhichhasasunburstappearancewith
striationradiatingfromthecentre.
Ahemangiomamaybeidentifiedduetoassociatedphlebolithandit
maycauselocalgigantismoftheinvovledarea.

1267.Metastasisnotfoundin
a)Femur
b)Humerus
c)Fibula
d)Spine
CorrectAnswer-C
Answer-C.Fibula
Metastaticbonediseaseisthecommonestmalignancyofbonesand
ismuchmorecommonthanprimarybonetumors.
Thecommonestsitesforbonemetastasesarevertebrae(most
common),pelvis,theproximalhalfofthefemurandthehumerus.
Extremitiesdistaltoelbowandkneeareleastcommonlyinvolved
sites.
Spreadisusuallyviathebloodstream;occasionally,visceraltumors
spreaddirectlyintoadjacentbonese.g.,thepelvisandribs.

1268.Periostealreactionsisseenin
a)Osteomyelitis
b)Syphilis
c)Tumor
d)All
CorrectAnswer-D
Answer-D.All
Infection:-Osteomyelitis,Brodie'sabscess,syphilis
Neoplasms
Benign:Osteoidosteoma
Malignant:Ewing'ssarcoma,osteosarcoma
Eosinophilicgranuloma
Healedstressfracture
Hypertrophicpulmonaryosteoarthropathy

1269.Volkmannscontracture,whicharteryis
involved
a)Radial
b)Brachial
c)Ulnar
d)Interosseus
CorrectAnswer-B
Answer-B.Brachial
Injurytobrachialarterymaycausenerveandmuscleischemia
(Volkmann'sischemiaofflexorcompartment)ormayresultin
postischemicswellingduetoedemaorhemorrhage,thereby
causingcompartmentsyndrome,whichcancause,ifnottreated
immediatealy,Volkmann'sischaemiccontracturelateron.

1270.Castsyndromeisdueto
a)Aboveelbowcast
b)Belowelbowcast
c)HipSpica
d)Abovekneecast
CorrectAnswer-C
Answer-C.HipSpica
Castsyndrome(Superiormesentricarterysyndrome)isgastric
dilatationwithpartialorcompleteobstructionoftheduodenum.
Itismostfrequentlyseeninorthopaedicspatientswhohavehad
spinalsurgeryorwhoareinhipspicaorbodycasts.

1271.Heterotopicossificationoccursin
a)Bone
b)Joint
c)Softtissue
d)None
CorrectAnswer-C
Answer-C.Softtissue
Heterotopicossificationistheprocessbywhichbonetissueis
formedinsofttissueoutsidetheskeleton.

1272.Felonmostcommoncomplication
a)Osteomyelitis
b)Subungualhematoma
c)Infectivearthritis
d)None
CorrectAnswer-A
Answer-A.Osteomyelitis
Felonistheinfectionofdistalpulpspace.Nexttoacuteparanychia,
thisisthemostcommonhandinfection.Itusuallyfollowsapinprick,
withindexfingerandthumbbeingthecommonunfortunatevictim.
Thepatientinitiallycomplainsofdullpainandswelling.
Complicationsareosteomyelitis(mostcommon),skinnecrosis,
osteonecrosisofdigitsandrarelytenosynovitisorinfectivearthritis
ofDIPjoint.

1273.Felon/Whitlowis:
September2012
a)Midpalamrspaceinfection
b)Terminalpulpspaceinfection
c)Infectionoftheulnarbursa
d)Infectionoftheradialbursa
CorrectAnswer-B
Ans.Bi.e.Terminalpulpspaceinfection
apainfulabscessofthedeeptissuesofthepalmarsurfaceofthe
fingertipthatistypicallycausedbybacterialinfection(aswitha
staphylococcus)andismarkedbyswellingandpain

1274.Tensionbandwiringisindicatedinfractureofwhichofthefollowing?
a)Fracturehumerus
b)Olecranon
c)Fracturetibia
d)Fracturespine
CorrectAnswer-B
Tensionbandwiringisindicatedinthetreatmentoftwotypesofolecranonfracture.First
typeisacleanbreakwithseparationofthefragments,andsecondtypeiscomminuted
fractureoftheolecranonwithdisplcedfragments.
Acrackintheolecranonwithoutdisplacementistreatedbyimmobilisingtheelbowinan
aboveelbowplasterslabin30degreesofflexion.

1275.Mostcommonboneforwhichnailingis
done
a)Radius
b)Ulna
c)Tibia
d)Humerus
CorrectAnswer-C
Answer-C.Tibia
Mostcommonbonesforwhichintramedullarynailingisdoneare
Tibiaandfemur.

1276.Knucklebendersplintisusedfor:
September2009
a)Ulnarnervepalsy
b)Radialnervepalsy
c)Mediannervepalsy
d)Axillarynervepalsy
CorrectAnswer-A
Ans.A:UlnarNervePalsy
Cock-upsplint?Radialnervepalsy
Knuckle-bendersplint?Ulnarnervepalsy

1277.Whyfractureshaftfemurisearly
stabilised
a)Topreventbloodloss
b)ARDS
c)Nonunion
d)Compartmentsyndrome
CorrectAnswer-A
Answer-A.Topreventbloodloss
Fractureshaftfemurcancauseupto2Lofbloodlossandsevere
hypotension,ifnotimmoblizedearly.
thefractureshaftfemurshouldbeimmoblizedearly.Thiscanbe
temporaryimmobilizationbyTTsplintfollowedbysurgeryorcanbe
byimmediatesurgery.

1278.Drugusedinosteoarthritis
a)Methotrexate
b)Glucosamine
c)Sulfasalizine
d)All
CorrectAnswer-B
Answer-B.Glucosamine
Chondroitinsufate
Diseasemodifyinganti-osteoarthritisdrugs(DMAOAD)Diacerin(IL-
1antagonist),Licofelone(combinedCOX-LOXinhibitors)
Steroids(inacuteexacerbations).

1279.Pannusisseenin
a)OA
b)RA
c)Gout
d)None
CorrectAnswer-B
Answer-B.RA
Pannusisseeninrheumatoidarthritis.
Itisneoplasm-likegrowthofinflammedsynovialtissuethatleadsto
destructionofjointstructures.
Thearticularcartilageatpannusinterfaceappeartoundergo
chondrolysis.
Pannusappeartodevelopwithinandaroundthesynovium,
subsequentlycreepingintoandoverthearticularcartilageand
envelopingitandtheunderlyingboneinadeathlikegrasp.

1280.Sausagedigitsisseenin
a)Lymearthritis
b)Osteoarthritis
c)Psoriaticarthritis
d)None
CorrectAnswer-C
Answer-C.Psoriaticarthritis
Sausagedigitreferstodiffusefusiformswellingofthedigitdueto
softtissueinflammationfromunderlyingarthritisordactylitis.
Causesare:-
Psoriaticarthritis
Osteomyelitis
Sicklecellanemia
Sarcoidosis
Tuberculardactylitis(spinaventosa)

1281.Greenstickfractureis
a)Fractureinadults
b)Completefracture
c)Incompletefracture
d)Fracturespine
CorrectAnswer-C
Answer-C.Incompletefracture
Agreenstickfractureisanincompletetransversefracturepattern
seeninchildren.

1282.Resorptionofdistalphalanxisseenin
a)Scleroderma
b)Hyperparathyroidism
c)Reiter'ssyndrome
d)All
CorrectAnswer-D
Answer-D.All
Acro-osteolysisisthetermusedtodescriberesorptionofthe
distalphalangealtufts.Causesare:-

1. Scleroderma
2. Trauma&thermalinjury
3. Hyperparathyroidism
4. Epidermolysisbullosa
5. Arthropathy(RA,Psoriasis)
6. Neuropathy(diabetes,syringomyelia)
7. Raynaud'sdisease
8. Reiter'ssyndrome

1283.Severelyanaemicpregnantpatientin
cardiacfailure.Choiceoftransfusion?
a)Platelets
b)Packedcells
c)Wholeblood
d)Exchangetransfusion
CorrectAnswer-B
Ans.B.Packedcells

1284.Hydropsfetalisisdueto?
a)Rhmismatch
b)Hyperproteinemia
c)Placentalhypoplasia
d)Alloftheabove
CorrectAnswer-A
Ans.A.Rhmismatch

1285.Caputsuccedaneumindicatesthat
fetuswasalivetill?
a)Immediatelyafterbirth
b)Till2-3daysafterbirth
c)2-3weeksafterbirth
d)2-3monthsafterbirth
CorrectAnswer-A
Ans.A.Immediatelyafterbirth
CaPutsuccedaneum
Formationofswellingduetostagnationoffluidinthelayersofscalp
beneaththegirdleofcontact.
Boggydiffuseswelling,notlimitedtomidline.
Disappearsspontaneouslywithin24hours.
Usuallyoccursafterruptureofmembranes.

1286.Caputsuccedeumissaidtooccurin
baby?
a)Within24hrs
b)2-3days
c)2-3weeks
d)2-3months
CorrectAnswer-A
Ans.A.Within24hrs

1287.Mostofectopicpregnanciesareat
ampullaas?
a)Itisthenarrowestpart
b)Tubalmovementsareleasthere
c)Salpingitisproducesleastcryptshere
d)Plicaearemostnumeroushere
CorrectAnswer-D
Ans,D.Plicaearemostnumeroushere

1288.Besttodiagnoseunrupturedectopic
pregnancy?
a)Scopy
b)UPT
c)USG
d)Culdocentesis
CorrectAnswer-A
Ans,A.Scopy

1289.Definitivetreatmentforpreeclampsia?
a)Deliveryofbaby
b)Antihypertensivedrugs
c)Rest
d)Diet
CorrectAnswer-A
Ans,A.Deliveryofbaby

1290.Whichisleastinjuredingynaecological
procedures?
a)Ureteratpelvicbrim
b)Renalpelvis
c)Urinarybladder
d)Ureteratinfundibulopelvicligament
CorrectAnswer-B
Ans.B.Renalpelvis
Urinarybladder&pelvicureterarevulnerabletoinjuryduring
gynecologicalsurgery.

1291.Whichofthefollowingcannotbe
treatedbylaparoscopy-
a)Ectopicpregnancy
b)Sterilization
c)Nondescentofuterus
d)Genitalprolapsed
CorrectAnswer-C
Ans,C.Nondescentofuterus

1292.Mostcommondegenerationoffibroids
?
a)Calcareous
b)Hyaline
c)Red
d)Cystic
CorrectAnswer-B
AnsB.Hyaline
Mostcommondegenerationoverall_hyaline

1293.DefinitivemanagementofAdenomyosis
is?
a)GNRHanalogue
b)Danazole
c)LH
d)Hysterectomy
CorrectAnswer-D
Ans,D.Hysterectomy
Hysterectomyisthetreatmentofchoice.
Localresectioncanbetriedinyoungerwomeninwhomitis
localized.
MedicaltreatmentoPtionsareNSAIDS&hormonaltherapy,though
notmucheffective
GnRH,danazole,MirenaIUCDformenorrhagea&pain.

1294.Bartholin'scysttreatmentofchoice-
a)Excision
b)Antibiotics
c)Marsupialisation
d)Drainage
CorrectAnswer-C
Ans.C.Marsupialisation

1295.Hydronephrosisisseeninwhichstage
ofCacervix?
a)2a
b)2b
c)3a
d)3b
CorrectAnswer-D
Ans,D.3b

1296.Endometrialcarcinomainvolving
cervix,stageis?
a)1
b)2
c)3
d)4
CorrectAnswer-B
Ans,B.2

1297.Diagnosisofadenomyosisismadeby?
a)Histopathology
b)Ultrasound
c)MRI
d)Laproscopy
CorrectAnswer-D
Ans,D.Laproscopy

1298.Initialdrugforovariancancer?
a)Cisplatin
b)Doxorubicin
c)Ifosfamide
d)Methotrexate
CorrectAnswer-A
Ans,A,Cisplatin

1299.Fallopiantubetuberculosis?
a)MostcommontypeofgenitalTB
b)Sizeofthetubesisunchanged
c)Isasymptomatic
d)Primaryfocusofinfectionisalwaysinfallopiantubes
CorrectAnswer-A:C
Ans,A>C.Isasymptomatic>MostcommontypeofgenitalTB
MostcommontypeofgenitalTB>IsasymPtomatic

1300.TButerusallistrueexcept?
a)Mostlysecondary
b)Increaseincidenceofectopicpregnancy
c)Involvementofendosalpinx
d)Mostcommonisascendinginfection
CorrectAnswer-D
Ans,D.Mostcommonisascendinginfection

1301.Goldstandarddiagnostictechniquefor
diagnosisofendometriosis?
a)Laproscopy
b)Ca125level
c)Ultrasound
d)MRI
CorrectAnswer-A
Ans,A.Laproscopy
Investigationsforendometrioses
Laproscopyisconsideredasgoldstandard.
Usedasbothdiagnosticaswellastherapeutictechnique.
1. CA-f25israised>35u/ml
2. Ultrasound
3. CT&MRI
4. ColorDoPPler
5. CYstoscoPY

1302.Mostcommonsiteofendometriosis-
a)Ovary
b)FT
c)Colon
d)LSCSScar
CorrectAnswer-A
Ans,A.Ovary

1303.Asherman'ssyndromefalseis?
a)Associatedwithmenstrualirregularities
b)Progesteronechallengetestispositive
c)Synechiaeformationinuterus
d)MaybesecondarytoTB
CorrectAnswer-B
Ans,B.Progesteronechallengetestispositive

1304.
35yearoldwithhistoryofrepeated
D&C.Shenowhassecondary
amenorrhea.Whatisyourdiagnosis?
a)Hypothyroidism
b)Kallmansyndrome
c)Sheehan'ssyndrome
d)Asherman'ssyndrome
CorrectAnswer-D
Ans,D.Asherman'ssyndrome

1305.Emergencycontraceptiveshouldmust
bestartedwithinhowmuchtimeafter
unprotectedintercourse?

a)24hrs
b)48hrs
c)72hrs
d)96hrs
CorrectAnswer-C
Ans,C.72hrs
Morningaftnpill:ethinyl-estrediol2.5mg,premarin15mg,thedrug
istakenorallytwicedailyfor5days.
Beginningsoonaftere4tosurebutnotlatcrthan72hrs."

1306.I-pillisusedwhen?
a)Accidentalsexualexposure
b)OCPforgotten
c)Ofchoiceinyoung
d)Alloftheabove
CorrectAnswer-A
Ans.A.Accidentalsexualexposure
I-pill
l-pillisanemergencycontraceptivePillcontaininglevonorgestrel.
AsingledoseofIpillprovidesasafeandeasywaytopreventanunintended
pregnancy,afterunprotectedsexorcontraceptivefailure.
Itshouldbetakei-pillassoonaspossible,preferablywithin12
hoursandnolaterthan72hoursofunprotectedintercourse.
Singledosetablettobetakenorallyafterameal.

1307.Cholestasisofpregnancyfalseis?
a)Bilirubinlevel>2mg%
b)Mostcommoncauseofjaundiceinpregnancy
c)Oestrogenisinvolved
d)Manifestationsusuallyappearinlasttrimester
CorrectAnswer-B
Ans.B.Mostcommoncauseofjaundiceinpregnancy

1308.Leastfailurerateisof?
a)OCpills
b)IUDs
c)Condom
d)DMPA
CorrectAnswer-A
Ans.A.OCpills
Oralcontraceptivepillshaveleastchancesofpregnancyasthey
haveminimumfailurerate(evaluatedbypearlindex).

1309.FailurerateofPomeroy'smethodof
tuballigationis?
a)0.2%
b)0.4%
c)0.6%
d)0.8%
CorrectAnswer-B
Ans,B.0.4%
Thefailurerateis0.4%anditkmainlyduetospontaneous
canalization".

1310.Maximumsuccessafterreversalof
tuballigation?
a)Cauterization
b)Pomeroy'stechnique
c)Clipmethod
d)Fimbriectomy
CorrectAnswer-C
Ans,C.Clipmethod
TheFalopesialisticringdestroys2-3cmfallopiantube.TheHulka&
Filshieclipsdestroyasmallersegment(3-4mm)'thuspreservingthe
potentialofsuccessfulreversalofsurgery.Thefailureratevaries
between.2and15%"

1311.If300microgramantiDisgivento
mother,amountofblooditwill
neutralise?

a)30m1
b)40m1
c)50m1
d)60m1
CorrectAnswer-A
Ans,A.30m1

1312.Tdapvaccineisgiveinbetweenwhich
weeksofpregnancy?
a)10-16weeks
b)17-22weeks
c)22-26weeks
d)27-36weeks
CorrectAnswer-D
Ans,D.27-36weeks
Womenshouldgetadulttetanus,diphtheriaandacellularpertussis
vaccine(Tdap)duringeachpregnancy.Ideally,thevaccineshould
begivenbetween27and36weekesofpregnancy".

1313.Leastdiameterofinletofgynecoid
pelvisis?
a)Transverse
b)Oblique
c)Diagonalconjugate
d)Obstetricconjugate
CorrectAnswer-D
Ans,D.Obstetricconjugate

1314.Whichoffollowingmostcommonly
clinicallyused?
a)Diagonalconjugate
b)Antpostdiameterofinlet
c)Transversediameterofoutlet
d)Obliquediameterofpelvis
CorrectAnswer-A
Ans,A.Diagonalconjugate
MostcommonlyusedclinicalconjugateisDiagonalconjugate.

1315.DefiniteuseforPGE2isallexcept?
a)Contraception
b)Induceslabour
c)Therapeuticabortion
d)KeepspatencyofPDA
CorrectAnswer-A
Ans.A.Contraception

1316.Allofthefollowingoccursbecauseof
prostaglandinuseexcept?
a)Excesswaterretention
b)Flushes
c)Increasedmotilityofbowel
d)Nausea
CorrectAnswer-A
Ans,A.Excesswaterretention
Disadvantages&sideeffectsofprostaglandins

1. Cost
2. Nausea,vomiting,diarrhea,pyrexia,bronchospasm,tachycardia&
chills
3. Cervicallacerations(PGF-2alpha)
4. Tachysystole(hyperstimulation)ofuterus
5. Riskofuterineruptureincaseofuterinescar.

1317.Metherginisgivenforprophylaxisof?
a)Anaemia
b)Cardiacdisease
c)Renaldisease
d)Lungdisease
CorrectAnswer-A
Ans,A.Anaemia
Methergin(methyl-ergo-novine)isasemisyntheticergotderivative
derivedfromlysergicacid.
Indications
1. Prophylactic:Activemanagementof3rdstageoflabourtoprevent
excessbleedingfollowingdelivery(note-bleedingcausesanemia).
2. Therapeutic:Tostopatonicuterinebleeding.

1318.HRTisgivenin?
a)Symptomaticpostmenopausalwomen
b)Followinghysterectomy
c)Gonadaldysgenesis
d)Alloftheabove
CorrectAnswer-D
Ans,D.Alloftheabove
IndicationsofHRT

1. Symptomaticwomensufferingfromoestrogendeficiency.
2. Highriskcasesofmenopausalcomplicationssuchascardiovascular
disease,osteoporosis,stroke,Alzheimerdisease&coloniccancer
(prophylactic).
3. Prematuremenopause,spontaneousorfollowingsurgery
(prophylactic).
4. Gonadaldysgenesisinadolescents.(therapeutic)

1319.Apregnantwomaninfirsttrimesterhas
fourfoldriseinIgGagainst
toxoplasmosis.itindicates?

a)Protectiveantibodies
b)Acuteinfection
c)Chronicinfection
d)Noneoftheabove
CorrectAnswer-B
Ans,B.Acuteinfection
AcuteinfectionisdetectedbydetectingIgMspecificantibodyhigh
titreofIgGantibody&detectionofsero-conversionforIgGfrom
negativetopositive

1320. Drugofchoiceforpneumocystiscarinii
inpregnancy?
a)SMZ/TMP
b)Primaquine
c)Dapsone
d)Pentamidine
CorrectAnswer-A
Ans,A.SMZ/TMP

1321.Maximumchanceoftransmission
duringdelivery?
a)HSV
b)CMV
c)VZV
d)Rubella
CorrectAnswer-A
Ans.A.HSV
"TransplacentalinfectionbyHSVisnotusual.
Thefetusbecomesaffectedbyvirusshedfromthecervixorlower
genitaltractduringvaginaldelivery."

1322.6yearoldsonofpregnantwomenis
sufferingfromchickenpox.Whichof
thefollowingisgiventopregnant
women?

a)Acyclovir
b)Acyclovir+immunoglobulin
c)Onlyimmunoglobulin
d)Vaccination
CorrectAnswer-B
Ans.B.Acyclovir+immunoglobulin
Varicelladuringpregnancy
Theriskofcongenitalmalformationsisnearlyabsentwhenmaternal
infectionoccursafter20weeks.
VariceIavaccineisnotrecommendedinpregnancy.
Varicellazosterimmunoglobulin(VZIG)shouldbegiventoexposed
non-immuneasitreducesthemortality.
Oralacyclovirissafeinpregnancy&reducesthedurationofillness
whengivenwithin24hrsofrashbutitcannotpreventcongenital
infection.

1323.Prophylacticmethergingivenfor?
a)Inductionoflabour
b)Inductionofabortion
c)Tostopexcessbleedingfromuterus
d)Alloftheabove
CorrectAnswer-C
Ans,C.Tostopexcessbleedingfromuterus
Methergin(methyl-ergo-novine)isasemisyntheticergotderivative
derivedfromlysergicacid.
Indications
Prophylactic:-Activemanagementof3rdstageoflabourtoprevent
excessbleedingfollowingdelivery.
Therapeutic:-Tostopatonicuterinebleeding.

1324.DOCformalariainpregnancy?
a)Chloroquin
b)Quinine
c)Primaquin
d)Artesunate
CorrectAnswer-A
Ans,A.Chloroquin
Drugsfortreatmentofmalariainpregnancy-
Malariacanbelifethreateningduringpregnancy.
Chloroquinis1stchoiceofdrug.
Ifresistanttochloroquin,quinineshouldbegivenundersupervision.
Primaquin(forradicalcure)shouldbewithhelduntilthepregnancyis
over.
Artesunateisthe1stchoiceincaseofcomplicatedmalaria.

1325.Foruterineprolapseinpregnancy,Ring
pessarycanbeinsertedupto?
a)12weeks
b)14weeks
c)16weeks
d)18weeks
CorrectAnswer-D
Ans.D.18weeks
PessarytreatmentforProlaPse
Itdoesnotcuretheprolapse,onlygivessymptomaticreliefby
stretchingthehiatusgenitalis,thuspreventingtheuterine&vaginal
descent.
IndicationsofPessarY
EarlYPregnancy-uPto18weeks
PuerPerium
Patientsabsolutelyunfitforsurgery
Patientsunwillingforoperation
Whilewaitingforsurgery
Additionalbenefitslikeimprovementofurinarysymptoms

1326.Notamethodfordeliveryofafter-
comingheadofbreech?
a)Forcepsmethod
b)BurnsandMarshallmethod
c)Malarflexionandshouldertraction
d)Halfhandmethod
CorrectAnswer-D
Ans.D.Halfhandmethod
Methodsofdeliveryofaftercomingheadare
Burns-Marshallmethod
ForcePsdelivery
Malarflexionandshouldertraction(modifiedMauriceau-Smellie-Veit
technique)

1327.Investigationofchoiceinpost
menopausalbleeding?
a)PAPsmear
b)Laproscopy
c)Fractionalcurettage
d)Ultrasound
CorrectAnswer-C
Ans.,C.Fractionalcurettage

1328.PreferredIUDformenorrhagea?
a)NOVAT
b)CuIUD
c)Mirena
d)Gynefix
CorrectAnswer-C
Ans.C.Mirena

1329.Preferredtreatmentformenorrhageain
reproductiveagegroup?
a)NOVAT
b)CuIUD
c)OCPs
d)Hysterectomy
CorrectAnswer-C
Ans.C.OCPs

1330.Drugnotusedcommonlyfor
menorrhagea?
a)Methergin
b)Clomiphene
c)GnRH
d)NSAIDS
CorrectAnswer-A
Ans.A.Methergin

1331.Drugcausingabruptionplacenta?
a)Methadone
b)Cocaine
c)Amphetamine
d)Fluoxetine
CorrectAnswer-B
Ans.B.Cocaine

1332.

ClassicalCsectionindicatedin?
a)CACervix
b)Centralplacentapraevia
c)Failedinduction
d)Fetaldistress
CorrectAnswer-A
Ans.A.CACervix

1333.DefinitiveindicationofLSCS?
a)Mentoant
b)Contractedpelvis
c)Occipitoposterior
d)Vertex
CorrectAnswer-B
Ans,B.Contractedpelvis

1334.Afterdeliveryuptowhichweekis
knownaspuerperium?
a)2weeks
b)4weeks
c)6weeks
d)8weeks
CorrectAnswer-C
Ans.C.6weeks
Immediatepuerperium:24h
Earlypuerperium:upto1week
Remotepuerperium:upto6weeks

1335.Vacuumdeliveryproduces?
a)Chingon
b)Cephalhematoma
c)Both
d)None
CorrectAnswer-C
Ans,C.Both

1336.45yroldfemalepatientunderwent
hysterectomy,on7thpostopday
complaintsaboutcontinuousdribbling
ofurineandfever.Micturitionwasnot
voluntary,whatdiagnosis?

a)Vesicovaginalfistula
b)Ureterovaginalfistula
c)Vesicouterinefistula
d)Urethravaginalfistula
CorrectAnswer-C
Ans,C.Vesicouterinefistula
Fetalcomplicationsofvacuumdelivery
SuPerficialscalloPabrasions
Sub-aPoneurotichaemorrhage
Retinalhaemorrhage

1337.Placentaprevia,falseis?
a)MostcommoncauseofAPH
b)Painfulvaginalbleeding
c)Usgistheinvestigationofchoice
d)Increasedmaternalageisariskfactor
CorrectAnswer-A
Ans.A.MostcommoncauseofAPH

1338.ExclusivelyFetalbloodlossoccursin?
a)Vasaprevia
b)Placentapraevia
c)Polyhydramnios
d)Oligohydramnios
CorrectAnswer-A
Ans.A.Vasaprevia

1339.
Prolapsedofuterusinnulliparous
women,treatmentis?
a)Slingusedinvolvingrectussheath
b)Anteriorcolporrhaphy
c)Posteriorcolporrhaphy
d)Manchesteroperation
CorrectAnswer-A
Ans.,A.Slingusedinvolvingrectussheath

1340.Allarecausesofanovulatory
amenorrhoeaexcept?
a)PCOD
b)Hyperprolactemia
c)Gonadaldysgenesis
d)Drugs
CorrectAnswer-C
Ans.C.Gonadaldysgenesis
Causesofanovulatoryamenorrhea
PCOD
Hyperprolactinaemia
Weightloss,stress,exercise
Drugs
Chestwallstimulation

1341. Whatisfalseaboutpostmenopausal
state?
a)LowLH
b)Lowestrogen
c)HighFSH
d)Highandrogen
CorrectAnswer-A
Ans.A.LowLH
Hormonalchangesinpost-menoPausalstate
FSHlevelisincreased
Oestrogenlevelisdecreased&mostofitissynthesisedperipherally
byconversionofandrogentooestrogen
Androgenlevelisslightlyincreased

1342.Inapostmenopausalfemale,which
hormoneincreases?
a)FSH
b)Estrogen
c)GH
d)Noneoftheabove
CorrectAnswer-A
Ans,A.FSH

1343.Roleoflactobacilliinvaginalsecretions
a)TomaintainalkalinepH
b)TomaintainacidicpH
c)Nutrition
d)None
CorrectAnswer-B
Ans.,B.TomaintainacidicpH
TheimportanceofDoderlein'sbacillusisthatitspresenceis
associatedwithproductionoflacticacidcontainedinthevaignaand
thisacidityinhibitsthegrowthofotherorganisms."
Doderlein'sbacillusistheonlyorganismwhichwillgrowatthepHof
4-4.5(normalpHofvagina).

1344.Hegarsignallaretrueexcept?
a)Bimanualpalpationmethod
b)Difficultinobese
c)Canbedoneat14weeks
d)Presentin2/3rdofcases
CorrectAnswer-C
Ans.C.Canbedoneat14weeks
Hegar'ssign:Presentin2/3rdofcases.Demonstratedbetween6-10
weeks,alittleearlierinmultipara.Thissignisbasedonthe2facts
(1)upperpartofthebodyofuterusisenlargedbygrowinguterus
andlowerpartisemptyandsoftandcervixiscomparativelyfilm.
Thereforeonbimannualexamination(2fingersintheanteriorfornix
andabdominalfingersbehindtheuterus),abdominalandvaginal
fingersseemtoapposebelowthebodyoftheuterus.

1345.Hegarsignisseeninhowmanyweeks
?
a)6-10weeks
b)10-14weeks
c)14-18weeks
d)18-22weeks
CorrectAnswer-A
Ans.A.6-10weeks

1346.Palmersignisrelatedto?
a)Contractionofuterus
b)Duskyhueofantvaginalwall
c)Bluishdiscolourationofantvaginalwall
d)Increasedpulsationsfeltthroughlateralfornix
CorrectAnswer-A
Ans,A,Contractionofuterus
Palmersign:-regularandrhythmiccontractionscanbeelicitedduring
bimanualexaminationasearlyas4-8weeks.

1347.Newborncanbegivenbreastmilkafter
howmuchtimefollowingnormal
delivery?

a)Halfhour
b)1hours
c)2hours
d)3hours
CorrectAnswer-A
AnsA.Halfhour
Ahealthybabyisputtothebreastimmediatelyoratmost%to1hr
followingnormaldelivery.
Followingcaesareansectionsaperiodof4-6hoursmaybesufficient
forthemothertofeedherbaby.

1348.4monthamenorrhoeawithincreased
FSH,LH&decreasedestrogenina35
yrsold?

a)Prematuremenopause
b)Menopause
c)Latemenopause
d)Perimenopause
CorrectAnswer-A
Ans,A.Prematuremenopause
Prematuremenopauseisdefinedasovarianfailureoccurring2SD
inyearsbeforethemeanmenopausalageinapopulation.
Itisclinicallydefinedassecondaryamenorrheaforatleast3months
withraisedFSHlevel,raisedFSH:LHratio&lowE2levelina
womenunder40yrsofage."

1349.35yroldwith4monthsamenorrhea
withincreasedFSH,decreased
estrogen.Whatisthediagnosis?

a)Prematureovarianfailure
b)PCOD
c)Pituitaryfailure
d)Hypothalamicfailure
CorrectAnswer-A
Ans.A.Prematureovarianfailure
ItisacaseofprematuremenoPause(prematureovarianfailure)'

1350.Contractionstresstestfalseis?
a)Oxytocinnotused
b)Invasivemethod
c)Detectsfetalwellbeing
d)Negativetestisassociatedwithgoodfetaloutcome
CorrectAnswer-A
Ans,A.Oxytocinnotused
Contractionstresstest(CST)(syn:Oxytocinchallengetest):
Itisaninvasivetesttoassesfetalrespiratorywellbeingduring
pregnancy.
ItdetectsalterationinFHRinresponsetouterinecontraction
inducedbyoxytocinindicatinghypoxia.
Interpretations
Negativetestindicatedgoodoutcome

1351.NST,whatisseenexcept?
a)Variability
b)Acceleration
c)Timeperiod
d)Oxytocin
CorrectAnswer-D
Ans,D.Oxytocin
Thenon-stresstest(NST)measuresfetalheartrate,whichis
monitoredwithanexternaltransducerforatleast20minutes.
Duringfetalmovement,tracingisobservedforheartrate
acceleration.
Testispositiveiftwoormorefetalheartrateaccelerationsoccursin
20minuteperiod.

1352.Menstrualregulationeffectiveupto?
a)14days
b)21days
c)4weeks
d)6weeks
CorrectAnswer-A
Ans,A.14days
Menstrualregulation(lnduction/Aspiration)
Aspirationofendometrialcavitywithinl4daysofmissedperiodina
womanwithpreviousnormalcycle.
DoneasanOPDprocedure.
Helpstodetectfailedabortion,molarpregnancyorectopic
Pregnancy.
Contraindicatedinadvancedpregnancy&inpresenceoflocalpelvic
inflammation.

1353.pHofvaginainpregnantwomanis
usually?
a)4.0
b)4.5
c)5
d)>5
CorrectAnswer-A
Ans.A.4.0
Thevaginalacidityisduetolacticacid.
ThenormalpHinthehealthywomenofthechildbearingagegroup
is4.5

1354.VaginalpHbeforepubertyis?
a)7
b)6
c)4.5
d)5
CorrectAnswer-A
Ans.A.7

1355.Decidualreactionisduetowhich
hormone?
a)Progesterone
b)Estrogen
c)LH
d)FSH
CorrectAnswer-A
Ans.A.Progesterone
Increasedstructural&secretoryactivityoftheendometriumthat
broughtaboutinresponsetoprogesteronefollowingimplantationis
knownasDecidualreaction".

1356.Inhibinlevelsarecheckedonwhichday
ofmenstrualcycle?
a)Day3
b)Day4
c)Day5
d)Day6
CorrectAnswer-A
Ans.A.Day3
Ovarianreservetestisdesignedtoassessboththenumberof
immatureeggsintheovariesandtheirquality,whichgivesan
indicationofwoman'spotentialfertility.
Itworksbydetectingthelevelsofthreefemalehormones,
usingabloodsampletakenondaythreeofmenstrualcycle:

1. Folliclestimulatinghormone(FSH)
2. Anti-mullerianhormone(AMH)
3. Inhibin-B

1357.Besttestforovulation?
a)Serumestrogen
b)Serumprogesterone
c)Both
d)None
CorrectAnswer-B
Ans.B.Serumprogesterone
Plasmaconcentrationofprogesteronerisesafterovulation&
reachespeakof15ng/mlatmidlutealphase&thendeclinesasthe
corpusluteumdegenerates".

1358.Cardiacoutputincreasesmaximumat
whichweek?
a)26-28wks
b)30-32wks
c)32-34wks
d)34-36wks
CorrectAnswer-C
Ans.C.32-34wks
Cardiacoutput:
Startstoincreasefrom5thweekofpregnancy,reachesitspeak40-
50%atabout30-34weeks.

1359.Inpregnancyplasmavolumeincreased
maximumatwhatgestationalage?
a)10wks
b)20wks
c)25wks
d)30wks
CorrectAnswer-D
Ans,D.30wks
Plasmavolumeisincreased.Startingtoincreaseat6weeks&
reachingmaxupto50%at30weeks.Totalplasmavolume
increasestotheextentof1.25litres.

1360.5monthpregnantfemale,whichofthe
followingistrue?
a)50%havesoftsystolicmurmur
b)Cardiacoutputisreduced
c)Systemicvascularresistanceisincreased
d)IncreaseinCVP
CorrectAnswer-A
Ans.A.50%havesoftsystolicmurmur
Anatomicalchangesduringpregnancy
Heartispushedupwards&outward.Apexbeatisshiftedin4th
intercostalsspace.
Asystolicmurmurcanbeheardinapicalorpulmonaryarea.
Mammarymurmurisacontinuoushissingmurmuraudibleover
tricuspidareainleft2nd&3rdiintercostalsspace.
ECGshowsleftaxisdeviation.53andrarely54canbeheard.

1361.Spinnbarkeitismaximumshownat
whichphase?
a)Menstrualphase
b)Ovulatory
c)Postovulatory
d)Prefollicular
CorrectAnswer-B
Ans.B.Ovulatory
Spinnbarkeittest(ThreadtestorFerntest)
AsPecimenofcervicalmucuswhenseenunderlowpower
microscope,showsacharacteristicfernformation.
Donetoseetheestrogenicactivityinovulatoryphaseofmenstrual
cycle.

1362.Rarestpresentationis?
a)Cephalic
b)Breech
c)Shoulder
d)Vertex
CorrectAnswer-C
Ans.C.Shoulder

1363.Fertileperiodoffemaleismeasuredby
?
a)LH
b)FSH
c)Estrogen
d)Oxytocin
CorrectAnswer-A
Ans,A.LH
"LHsurgefromtheanteriorpituitaryglandoccurs24hourspriorto
ovulation.Radioimmunoassaysofthemorningsampleofurine&
bloodgivesresultsin3hours.NotonlydoestheLHsurgehelpin
predictingovulation,buttheapproximatetimeofovulationcanbe
gauged&coitusaroundthistimecanimprovethechancesof
conception."

1364.Externalversionisdoneafter?
a)34weeks
b)36weeks
c)38weeks
d)40weeks
CorrectAnswer-B
Ans.B.36weeks
Themaneuveriscarriedoutafter36weeksinlabour-delivery
complex."

1365.Leastcommonpresentationoftwins?
a)Bothvertex
b)Bothbreech
c)Bothtransverse
d)Firstvertexand2ndtransverse
CorrectAnswer-C
Ans.C.Bothtransverse
Lie-Presentationintwinpregnancy:
Themostcommonlieinthefetusesislongitudinal(90%).
Thecombinationofpresentationoffetusesare:
Bothvertex(50%)
Firstbreechsecondvertex(10%)
Firstvertexandsecondtransverse(rare)
Firstyertexandsecondbreech(30o/o)
Bothbreech(10%)
Bothtransverse(rarest).

1366.

Presentingpartintransverselie?
a)Shoulder
b)Face
c)Vertex
d)Brow
CorrectAnswer-A
Ans.A.Shoulder
'Whenthelongaxisofthefetusliesperpendicularothematernal
spineorcentraliseduterineaxis,itiscalledtransverselie.
Butmorecommonly,fetalaxisliesobliquetothematernalspine&is
thencalledobliquelie.Ineitheroftheconditionsshoulderusually
presentsoverthecervicalopeningduringlabour&bothare
collectivelycalledshoulderpresentations."


1367.Whichofthefollowinglayercontains
abundantdesmosomes?
a)A
b)B
c)C
d)D
CorrectAnswer-C
Ans.C.C
Stratumspinosumorpricklecelllayercontainsabundant
desmosomesmarkedwiththelegend'C'intheimage.
Pictureshowslayersofepidermis
A Stratumcorneum
B Stratumgranulosum
C Stratumspinosum
D Stratumbasale
"Theepidermiscanbedividedintoanumberoflayersfromdeepto
superficialasfollows:basallayer(stratumbasale),spinousorprickle
celllayer(.stratumspinosum),granularlayer(stratumgranulosum),
clearlayer(stratumlucidum)andcornifiedlayer(stratumcorneum."-
"Thepricklecelllayer(stratumspinosum)consistsofseverallayers
ofcloselypackedkeratinocytesthatinterdigitatewitheachotherby

meansofnumerouscellsurfaceprojections.Thecellsareanchored
toeachotherbydesmosomesthatprovidetensilestrengthand
cohesiontothelayer.Thesesuprabasalcellsarecommittedto
terminaldifferentiationandgraduallymoveupwardstowardsthe
cornifiedlayerasmorecellsareproducedinthebasallayer.When
skinisprocessedforroutinelightmicroscopy,thecellstendtoshrink
awayfromeachotherexceptwheretheyarejoinedby
desmosomes,whichgivesthemtheircharacteristicspiny
appearance.Pricklecellcytoplasmcontainsprominentbundlesof
keratinfilaments,(mostlyK1andKIOkeratinproteins)arranged
concentricallyaroundaeuchromaticnucleus,andattachedtothe
denseplaquesqt.desmosomes.Thecytoplasmalsocontains
melanosomes,eithersinglyoraggregatedwithinmembrane-bound
organelles(compoundmelanosomes).Langerhanscellsandthe
occasionalassociatedlymphocytearetheonlynon-keratinocytes
presentinthepricklecelllayer."
LayersofEpidermis
Alsoknownasstratumgerminativum?
Itcontainsmitoticallyactivekeratinocytescontaining
Stratum
housekeepingorganelles?(RER,golgicomplex,
basale
mitochondria,lysosomes,ribosomes)
Giverisetosuperficiallayer?
Spinelikeappearance?ofcellmarginsinhistological
sections
Stratum
Thesespinesareabundantdesmosomes?,calcium
spinosum
dependentcellsurfacemodificationsthatpromote
adhesionofepidermalcells&resistancetomechanical
stresses.
Characterizedbybuildupofcomponentsnecessaryfor
theprocessofprogrammedcelldeath&formationof
Stratum
superficialwaterimpermeablebarrier?.
granulosum Mostapparentstructurewithinthesecellsisbasophilic
containkeratohyalinegranules?
Stratum
Clearlayer,seenonlyinthickskin?
lucidum
Formedofcornifiedorhornycells(largestcellof
Stratum
corneum

corneum
epidermis)andhavehighestconcentration

1368.Miliariaarisesfromobstructionof?
a)Eccrinesweatglands
b)Apocrinesweatglands
c)Sebaceousglands
d)Ectopicsebaceousglands
CorrectAnswer-A
Ans.is'a'i.e.,Eccrinesweatglands
Milliria
Occurasaresultofeitherobliterationordisruptionoftheeccrine
sweatduct.
Threeforms:-
1. Miliariacrystallina-Clear,thin-walledvesicles,1-2mmindiameter,
withoutaninflammatoryareola,areusuallysymptomlessand
developincrops,mainlyonthetrunk.
2. Miliariarubra-erythematouspapulesespeciallyinareasoffriction
withclothing,andinflexures,produceintenseprickingsensation.
3. Miliariaprofunda-Thisnearlyalwaysfollowsrepeatedattacksof
miliariarubra,oComplications:-Secondaryinfectionand
disturbanceofheatregulation

1369.NottrueaboutSkintag?
a)Associatedwithseborrhoeickeratosis
b)Pedunculated
c)Mostcommonsiteisneckandaxilla
d)Premalignant
CorrectAnswer-D
Ans.D.Premalignant
Skintags(softwarts;achrochordon)
Acommonbenignlesioncomposedofloosefibroustissueand
occurringmainlyontheneckandmajorflexuresasasmallsoft
PedunculatedProtrusion.
Verycommon,particularlyinwomenatthemenopauseorlater.
Theyarefrequentlyfoundtogetherwithseborrhoeickeratoses.
Treatment:Cauteryandcryotherapy

1370.Spidertelengectaciafalseis?
a)Morecommoninmales
b)Canbecausedbytrauma
c)Lighttherapyfortreatment
d)Maybeassociatedwithliverdisease
CorrectAnswer-A
Ans.A.Morecommoninmales
Spidertelangiectasia(arterialspider,spidernevus,nevus
aranus)
Seenin2/3rdofpregnantfemalesandusuallydisappearsafter
delivery.
Estrogenissaidtobeinvolvedinpathogenesis.
Whenmultiple,liverdiseaseshouldberuledout.
Centralradiatingbodywithradiatingvesselsgivesalookofspider.
Lesionusuallyoverupperhalfofthebody.
Treatmentbydiathermyandexcision.

1371.Whichofthefollowingisuntrue
regardingpiebaldism?
a)autosomaldominantcondition
b)amelanoticskinassociatedwithawhiteforelock
c)Islandsofnormalorhypermelanoticskin
d)Usuallyimproveswithage
CorrectAnswer-D
Ans.D.Usuallyimproveswithage
Piebaldism
Piebaldismisarareautosomaldominantconditioncharacterizedby
stableareasofvitiligo-likeamelanoticskinassociatedwithawhite
forelock.
Presentatbirthandusuallyremainunchangedthroughoutlife.
Mostcommonisafrontalmedianorparamedianpatch,associated
withameshofwhitehair(whiteForelock).
Often,whitepatchesoccurontheupperchest,abdomenandlimbs,
bilaterallybutnotnecessarilysymmetrically
Thehandsandfeet,aswellastheback,remainnormallypigmented
Islandsofnormalorhypermelanoticskinoccurinthewhiteareas,or
lessoftenonnormalskin.

1372.Allarefeaturesofatopicdermatitis,except:
a)Dennie-Morganfold
b)Hertoghe'ssign
c)Darier'sSign
d)Hyperlinearityofpalms
CorrectAnswer-C
Itisseeninurticariapigmentosa.

1373.WhichofthefollowingisnotaNEVUS
ofmelanocyte?
a)Mongolianspot
b)NevusofIto
c)Nevusofota
d)Beckernevus
CorrectAnswer-D
Ans.D.Beckernevus
Beckernevusisappendagealneviwhereasotherthreeare
melanocyitcnevi.
Melanocyticneviare:-
Dysplasticnevus
BIuenevus
Nevusofito&nevusofota
Spitznevus
Giantpigmentednevus
Mongolianspot
Intramucosalneyus

1374.Koebnerphenomenonseenin?
a)Psoriasis
b)Lichenplanus
c)Warts
d)Alltheabove
CorrectAnswer-D
Ans.is'd'i.e.,Alltheabove

1375.Ichypurplepapulefollowedby
hayperpigmentationonresolution,is
seenin?

a)Addison'sdisease
b)DM
c)Hypothyroidism
d)Lichenplanus
CorrectAnswer-D
Ans.D.Lichenplanus
LPischaracterizedbyshiny,violaceous,flat-toppedpolygonal
papuleswhichretaintheskinlines.
Whitelines,knownasWickham'sstriae,maytraversethesurfaceof
thepapules.
Linearlesionsoftenappearalongscratchmarksorinscars
(Koebnerphenomenon).
Inmostcases,thepapuleseventuallyflattenaftera6monthsto2
yrs,oftentobereplacedbyanareaofhyperpigmentation.

1376.Pemphigusvulgarisischaracterizedbyall,except:
a)PositiveNikolsky'ssign
b)Oralerosions
c)Subepidermalbulla
d)Tzancksmearshowingacantholyticcells
CorrectAnswer-C
Pemphigusvulgarisisanintraepidermalblisteringdisease.

1377.LossofIntercellularcohesionbetween
keratinocytesiscalledas?
a)Acanthosis
b)Acantholysis
c)Keratinolysis
d)Spongiosis
CorrectAnswer-B
Ans.B.Acantholysis
Separationofepidermalkeratinocytesduetolossofintercellular
bridgeisreferredtoasacantholysis.
Acantholysisisseenintheepidermis(especiallyinbasallayer).

1378.Tzankcellis?
a)Keratinocyte
b)Fibrobalst
c)Neutrophil
d)Lymphocyte
CorrectAnswer-A
Ans.A.Keratinocyte
Tzankcellsareacanthoylyticcellsi.e.largeroundedkeratinocytes
withsrelativelylargenucleuswithcondensedorhazycytoplasm.

1379.Trueaboutlepromatousleprosy?
a)Only3cutaneouslesions
b)Lepromintesthighlypositive
c)Thickenednerveroots
d)ENLin>50%cases
CorrectAnswer-C
Ans.C.Thickenednerveroots
Nerveinvolvementinlepromatousleprosycausesthickeningof
nerve.
Lepromintestisnegative.
Therearemorethan10macules/patches/plaques.

1380.Dermatophytosisisnot?
a)Scaly
b)Itchy
c)Superficial
d)Subdermal
CorrectAnswer-D
Ans.D.Subdermal

1381.Oculoorogenitalulcersareafeatureof
?
a)Behcetdisease
b)lichenplanus
c)SLE
d)Psoriasis
CorrectAnswer-A
Ans.A.Behcetdisease
Behcet'sdiseaseisamultisystemdiseasethatisdefinedbythe
presenceoforalaphthosiswithatleasttwoofthefollowing:genital
aphthaesynovitis,posterioruveitis,cutaneouspustularvasculitisor
meningoencephalitis,intheabsenceofIBDorautoimmune
Diseases.

1382.Allofthefollowingarefeatureof
dermatomyositis,Except:
a)SalmonPatch
b)Gottron'spatch
c)Mechanicfinger
d)Periungualtelengiectasias
CorrectAnswer-A
AnswerisA(SalmonPatch):
Salmonpatchisnotafeatureofdermatomyositis.
Cutaneousfeaturesofdermatomvositis
Cutaneous
Description
feature
Heliotrope
Periocularorfacialerythemaandedemawith
Rash?
pink/purple/blue(heliotrope)hue
(bluepurplediscolorationonupperevelidswith
edema)

Gottren's
Violaceouspapulesovertheknuckles
PapulesQ
Erythemaoftheknuckleswitharaisedviolaceous
(Gottren'ssign) scalyeruption
Ervthematousrashoverotherbodysurfacessuch
`V'sign
asanteriorchest(ofteninaVform)
Shawlsign
Erythematousrashoverotherbodysurfaces
includingtheuppertrunk,neck,back&shoulders
(shawlpattern)

Periungual
Dilatedcapillaryloopsatthebaseoffingernails
Telengiectasiase
Mechanic's

Irregular,thickened,distortedcuticles,withrough

handsQ
andcrackedareasoverthelateralandpalmar
areasoffingerswithirregulardirtyhorizontallines
resemblingmechanic'shand.

Calcinosis
Presenceofhardcalciumdepositsintheskin
Cutise

1383.Hereditaryangioneuroticedemaisdue
to?
a)DeficiencyofC1inhibitor
b)DeficiencyofNADPHoxidase
c)DeficiencyofMPO
d)Deficiencyofproperdin
CorrectAnswer-A
Ans.is'a'i.e.,DeficiencyofC1inhibitor
HereditaryangioneuroticedemaisduetoC1inhibitor(Ciesterase
inhibitor)deficiency.

1384.Phrynodermaisdueto...deficiency-
a)VitaminD
b)Niacin
c)VitaminA
d)Essentialfattyacid
CorrectAnswer-D
Ans.is'd'i.e.,Essentialfattyacid
Invitamin'A'deficiencythereistoadlikeskinalsoknownas
phrynoderma.
oButthisisduetoassociateddeficiencyofessentialfattyacids.

1385.Maximumcumulativedoseof
isotretinoinshouldn'texceedforacne
treatment?

a)30-60mg/kg
b)60-90mg/kg
c)90-120mg/kg
d)120-150mg/kg
CorrectAnswer-D
Ans.,D.120-150mg/kg
Isotretinoinisrecommendedforseverenodulocysticacneandalso
forthepatientswithmilderdiseasewhodon'trespondto
conventionaltreatment.
Treatmentregimensusuallybeginat0.5-1.0mg/kg/dayforthe
durationofbetween16and20weeks.
Cumulativedoseamounttoatotalofatleast12Omg/kg,butthere
isnoaddedbenefitwhen150mg/kgisexceeded.

1386.Exanthemasubitumiscausedby?
a)HHV
b)HPV
c)HIV
d)HCV
CorrectAnswer-A
Ans.A.HHV
HHV-6viruscausesroseolainfantum(exanthemsubitum),themost
commonexanthematicfeverinchildrenundertheageof2years,
withapeakincidencebetween6and9months.

1387.Cocainewasfirstusedaslocal
anaestheticby?
a)Carlkollar
b)Holmerwells
c)Morton
d)None
CorrectAnswer-A
Ans.A.Carlkollar
CocainewasthefirstlocalanestheticusedbyCarlKoller.Itwas
usedforanaesthetizingcornea.

1388.Levelsofetheranesthesiawere
demonstratedbywhom?
a)Morton
b)Guedel
c)Thompson
d)None
CorrectAnswer-B
Ans.B.Guedel
GuedeldescribedfourstagesofetheranesthesiaknownasGuedel
stages.

1389.Infantcircuitforanaesthesia?
a)Bainscircuit
b)Magillcircuit
c)Ayrestpiece
d)Water'scircuit
CorrectAnswer-C
Ans.C.Ayrestpiece

1390.Ayre'sT-pieceiswhichtypeofcircuit
a)TypeA
b)TypeB
c)TypeE
d)TypeD
CorrectAnswer-C
Ans.C.TypeE

1391.Mostreliableindicatortoprevent
oesophagealintubation?
a)Oxygensaturationonpulseoximeter
b)MeasurementofCO2inexhaledair(Etco2)
c)Directvisualizationofpassingtubebeneathvocalcords
d)Auscultationoverchest
CorrectAnswer-B
Ans,B.MeasurementofCO2inexhaledair(Etco2)

1392.Allarefeaturesofdifficultairway
except?
a)Miller'ssign
b)Micrognathiawithmacroglossia
c)TMJankylosis
d)Increasedthyromentaldistance
CorrectAnswer-D
Ans.D.Increasedthyromentaldistance
Decreasedthyromentaldistancepredictsdifficultairway(not
increasedTMdistance).

1393.Ratioof02:N20inEntonoxis?
a)50:50
b)60:40
c)40:60
d)25:75
CorrectAnswer-A
Ans.A.50:50
Entonoxcontainequalamount(50/50)ofN2OandO2.

1394.WhichisthecriticaltemperatureofN20
?
a)-118?C
b)-88?C
c)-26?C
d)-36.5?C
CorrectAnswer-D
Ans.D.-36.5?C
CriticaltemperatureofN2O-36.5
CriticalpressureofN2O-214.7atm

1395.Whichanaestheticagentisneither
metabolisedbylivernorbykidney?
a)Atracurium
b)Vecuronium
c)Pancuronium
d)Rocuronium
CorrectAnswer-A
Ans.A.Atracurium

1396.Dibucainnumberrefersto?
a)Achcholinestraseactivityderangement
b)Potencyofmusclerelaxants
c)Potencyofgeneralanaesthetics
d)None
CorrectAnswer-A
Ans.is'a'i.e.,Achchlinestraseactivityderangement
Dibucainnumber:Dibucain(alocalanaesthetic)inhibits80%of
normalpseudocholinesteraseand20%ofatypical(non-functional)
pseudocholinesterase.Thereforenormaldibucainnumberis70-
80%.Dibucainnumberisusedtomeasuretheactivityofatypical
pseudocholinesterase.

1397.Stagesofanesthesiawereestablished
by
a)Ether
b)N20
c)Halothane
d)Chloroform
CorrectAnswer-A
Ai.e.Ether

1398.Fastinductionandrecoveryisseenin?
a)Methoxyflurane
b)Ether
c)Halothane
d)N2O
CorrectAnswer-D
Ans.D.N2O
Speedofonset&recoveryindecreasingorder(Increasingorderof
B:Gpartitioncoefficientandbloodsolubility).

1399.MACofdesfluraneis?
a)1.15
b)2
c)4
d)6
CorrectAnswer-D
Ans.D.6

1400.WhichcanreplaceN20as02carrier?
a)Argon
b)Xenon
c)Helium
d)None
CorrectAnswer-C
Ans.C.Helium
Heliumcanbeusedtoreplacenitrogen,asthecarriergasfor
oxygen(Helium)toreducetheworkofbreathing.

1401.Allofthefollowingarecorrectaboutketamine,EXCEPT:
a)Itfunctionally"dissociates"thethalamus
b)Itincreasesarterialbloodpressure
c)Itisapotentbronchoconstrictor
d)Itinhibitspolysynapticreflexesinthespinalcord
CorrectAnswer-C
Ketaminefunctionally"dissociates"thethalamus(whichrelayssensoryimpulsesfromthe
reticularactivatingsystemtothecerebralcortex)fromthelimbiccortex(whichisinvolved
withtheawarenessofsensation).
Ketamineincreasesarterialbloodpressure,heartrate,andcardiacoutput.
Racemicketamineisapotentbronchodilator,makingitagoodinductionagentfor
asthmaticpatients.

Itinhibitspolysynapticreflexesinthespinalcordaswellasexcitatoryneurotransmitter
effectsinselectedareasofthebrain.
Ref:ButterworthIVJ.F.,ButterworthIVJ.F.,MackeyD.C.,WasnickJ.D.,MackeyD.C.,
WasnickJ.D.(2013).Chapter9.IntravenousAnesthetics.InJ.F.ButterworthIV,J.F.
ButterworthIV,D.C.Mackey,J.D.Wasnick,D.C.Mackey,J.D.Wasnick(Eds),Morgan&
Mikhail'sClinicalAnesthesiology
,5e.

1402.Allaretrueaboutnitrousoxideexcept?
a)Laughinggas
b)Causesmegaloblasticanemia
c)Causesdiffusionhypoxia
d)Goodmusclerelaxant
CorrectAnswer-D
Ans.is'd'i.e.,Goodmusclerelaxant
NitrousoxideN,O
Itisalsocalledlaughinggas.
Ithasgoodanalgesicbutpoormusclerelaxantactivity.
SecondgaseffectanddiffusionhypoxiaoccurwithN20only.
N20istheonlyanaestheticreportedtoproducehematologictoxicity
andneurotoxicitywithlongtermadministration.
BothtoxicitiesaretheresultoftheinteractionofN20withvitB12.

1403.Inducingagentofchoiceinshock?
a)Isoflurane
b)Desflurane
c)Ketamine
d)Thiopentone
CorrectAnswer-C
Ans.is'c'i.e.,Ketamine
InducingagentofchoiceinAsthma&COPDKetamine.
InhalationalagentofchoiceinAsthma&COPDHalothane.

1404.Benefitofketamine?
a)CausesdecreaseinBP
b)Goodanalgesicaction
c)DecreaseICT
d)DecreaseIOT
CorrectAnswer-B
Ans.B.Goodanalgesicaction
Ketamineisdifferentfrommostotheranaestheticinductionagentsin
thatithassignificantanalgesicaction

1405.Whichofthefollowingisa
sympathomimetic?
a)Propofol
b)Etomidate
c)Ketamine
d)N2O
CorrectAnswer-C
Ans.C.Ketamine
Ketaminehasanindirectsympathomimeticaction.

1406.Followingistrueabouthalothane
except?
a)Volatileliquidwithsweetodour
b)Sensitiseshearttoadrenaline
c)Constrictsbronchii
d)Causesmalignanthyperthermia
CorrectAnswer-C
Ans.is'c'i.e.,Constrictsbronchii
Halothane
Itisavolatileliquidwithsweetodour,nonirritatingand
noninflammable.
Itisapotentanaestheticwithpooranalgesicandmusclerelaxant
properties.
Halothanecausesdirectdepressionofmyocardialcontractilityby
reducingintracellularCa.
ItcausesfallinBPandCO.
Heartratedecreasesduetovagalstimulation.
Ittendstosensitizethehearttoarrhythmogenicactionof
adrenaline
contraindicatedinpheochromocytoma.
Itcausesgreaterdepressionofrespirationandventilationperfusion
mismatch.
Itdilatesthebronchiinhalationagentofchoiceinasthmatics
(intravaneousanaestheticofchoiceinasthmaticsisketamine).
Itisahepatotoxicdrugandcanalsocausemalignanthyperthermia
(Succinylcholineaccentuateit).
Recoveryissmoothandreasonablyquick.
Itcausespostanaestheticshiveringandchills.
Itinhibitsintestinalanduterinecontractionsagentofchoicefor

assistingexternalorinternalversionduringlatepregnancy.
Becauseitsuterinerelaxantactionitiscontraindicatedduring
labour.
Itisparticularlysuitableforinductionandmaintenanceinchildren
andasmaintenanceanaestheticinadults.

1407.Allofthefollowingcausemyocardial
depressionexcept:
a)Halothane
b)Etomidate
c)Thiopentone
d)Ketamine
CorrectAnswer-B
Bi.e.Etomidate
-Etomidatecausesadreno-corticalsuppressionQbyinhibiting
enzymes11/3hydroxylase(mainly)&17ahydroxylaseinvolvedin
cortisolandaldosterone(mineralocorticoid)productionQ.
VitC
supplimentationrestorescortisollevel.
-EtomidateandmidazolamprovidecardiovascularstabilityQ.But
etomidateismostcardiostableagentQthatcausestheleast
hemodynamicdisturbanceofanyoftheintravenousanesthetic
agents.Soitisintravenousanestheticagentofchoiceforpatients
withcardiacdiseaseandaneurysmsurgeryQ.
-
Directmyocardialdepressioniscausedbyhalothane(severe),
nitrousoxide(moderate),iso/sevo/des-flurane(mild),thiopental
(marked),propofol(dosedependent)andketamineQ
(butthisis
maskedbycardiotonicsympatheticstimulatoryaction).Etomidate>
midazolemaremostcardiostableagentsQ.


1408.Midazolamcausesallexcept:
a)Anterogradeamnesia
b)Retrogradeamnesia
c)Causestachyphylaxisduringhighdoseinfusions
d)Decreasedcardiovasculareffectsascomparedtopropofol
CorrectAnswer-B
Ans.b.Retrogradeamnesia
Atthetimeofpeakconcentrationinplasma,hypnoticdosesof
benzodiazepines(midazolam)canbeexpectedtocausevarying
degreesoflightheadedness,lassitude,increasedreactiontime,
motorincoordination,impairmentofmentalandmotorfunctions,
confusion,andanterogradeamnesia."
Midazolam:
ItcausesanterogradeamnesiaQ
Toleranceandtachyphylaxismayoccur,particularlywithlonger-term
infusionsQ(ShaferA.Complicationsofsedationwithmidazolamin
theintensivecareunitandacomparisonwithothersedative
regimens.CritCareMed.1998;26(5):947-56)
Benzodiazepinewithdrawalsyndromehasalsobeenassociatedwith
highdose/long-termmidazolaminfusionsQ
Comparedwithpropofolinfusions,midazolaminfusionshavebeen
associatedwithadecreasedoccurrenceofhypotension?butamore
variabletimecourseforrecoveryoffunctionafterthecessationof
theinfusion.

1409.WhichcantbegivenbyEpidural
anaesthesia?
a)Morphine
b)Remifentanil
c)Alfentanil
d)Fentanyl
CorrectAnswer-B
Ans.B.Remifentanil
Opioidsusedforepiduralanalgesiaaremorphine,Fentanyl,
Tramadol,Buprenorphine,alfentanil,sulfentanil,pentazocine.
Ramifentanilcontainsglycinewhichcancausemotorweakness--?
hencenotsuitableforepiduralanalgesia.

1410.ThedrugforOPDanalgesiais-
a)Morphine
b)Pethidine
c)Fentanyl
d)Alfentanil
CorrectAnswer-D
Ans.is'd'i.e.,Alfentanil
Drugsusefulfordaycaresurgery
oPropofoloSevoflurane,desflurane&isoflurane
oMidazolamoMivacurium
oAlfantanil

1411.Additionofepinephrinetolignocaine?
a)IncreasesesdistributionofLA
b)DecreasesabsorptionofLA
c)DecreasesdurationofLA
d)IncreasesmetabolismofLA
CorrectAnswer-B
Ans.B.DecreasesabsorptionofLA
VasoconstrictorsareusedalongwithLAwhichprolongsdurationof
actionasrateofabsorptionisdecreasedduetovasoconstriction.
Forthesamereason,metabolismofLAisreducedandtoxicityis
decreasedasthereislesserabsorptionofLA,

1412.Maximumconcentrationforepidural
block?
a)Bupivacaine
b)Lidocaine
c)Ropivacaine
d)Chlorprocaine
CorrectAnswer-D
Ans,D.Chlorprocaine

1413.VasoconstricatorL.A.is?
a)Cocaine
b)Procaine
c)Lidocaine
d)Chlorprocaine
CorrectAnswer-A
Ans.A.Cocaine
AllLAsarevasodilatorexceptcocaine.Cocainecauses
vasoconstriction'
"Ropivacaineandbupivacainealsocausevasoconstriction"'

1414.
Allarecontraindicationsofspinal
anaesthesiaExcept?
a)Bleedingdisorder
b)Raisedintracranialtension
c)Hypertension
d)Infectionatinjectionsite
CorrectAnswer-C
Ans,C.Hypertension

1415.Forpreventionofheadacheduring
spinalanaesthesia?
a)Dilutedsolutionoflocalanaestheticshouldbeused
b)Preloadingwithcrystalloids
c)FinerI.P.needleshouldbeused
d)Headendshouldbeelevated
CorrectAnswer-C
Ans,C.FinerI.P.needleshouldbeused
AssinglemostimPortantpredisposingfactorislargeboreneedle
useoffineneedlepreventsPDPH.

1416.Innewborn,chestcompressionshould
bestartedifheartrateis?
a)<120/min
b)<100/min
c)<80/min
d)<60/min
CorrectAnswer-D
Ans,D.<60/min
CompressionfornewbornshouldbestartedifHR<60/min'

1417.Nonionicdyeis
a)Ioxaglate
b)lohexol
c)Iothalamate
d)None
CorrectAnswer-B
Answer-B.Iohexol
Iohexolisanonionicmonomercontrastagent.
Thenon-iodineradicalsatpositions1,3and5(includingtheionic
carboxylradical)werereplacedbylongaminohydrocarbonsthat
provideadequatesolubilitywithoutionizing.
Examplesofthistypeofcompoundareiopromide,iohexol,
iopamidol,ioversol,iopentol,iobitridol,andiomeprol.

1418.HUismeasureof
a)CT
b)MRI
c)PET
d)USG
CorrectAnswer-A
Answer-A.CT
Hounsfield'scontributionismemorializedintheHounsfieldscale,
whichisusedtomeasurethex-rayattenuationinCTscanning.
Waterisarbitrarilyassignedavalueof0Hounsfieldunits(HU).
[RefEssentialradiologyp.86]

1419.Whichofthefollowingtechniquesuses
piezoelectriccrystals?
a)Ultrasonography
b)NMRimaging
c)X-raydiffraction
d)Xeroradiography
CorrectAnswer-A
Ans.Ultrasonography
Ultrasonographyisbasedonpiezoelectriceffect.
MRIisbasedongyeromagneticpropertyofproton(IP).

1420.WhichlookssameonTi&T2onMRI
a)Gallbladder
b)Fat
c)Kidney
d)CSF
CorrectAnswer-B
Answer-B.Fat
Fat(adiposetissue)hashighsignalintensityonbothT1&T2
images.
OtherthreeoptionshavelowsignalintesityonT1andhighsignal
intensityonT2images.

1421.Thescanwithhighestsensitivitytodetectadrenalmetastasisdueto
bronchogeniccarcinomais:
a)ContrastEnhancedCTabdomenwithAdrenalprotocol
b)PETscan
c)MRIscan
d)Radionuclidescan
CorrectAnswer-B
Scanstodifferentiateadrenaladenomafromadrenalmetastasisare:
CECT-
ChemicalshiftMRI
FDG-PET
PETscanhasalmost100%sensitivityfordetectingadrenalmetastasis,soanegative
studyexcludesthepossibilityofadrenalmets.Butadenomascangivefalsepositivetest.
Ref:FundamentalsofDiagnosticRadiology,Edition-4,Page-427.

1422.HypertranslucentchestX-rayisseenin
allexcept
a)Mcleodsyndrome
b)Emphysema
c)Pneumonectomy
d)Polandsyndrome
CorrectAnswer-C
Answer-C.Pneumonectomy
Causesofopaquehemithorax
Technical:-Rotation,Scoliosis.
Pleural:-Pleuraleffusion,Pleuralthickening,Mesothelioma.
Surgical:-Pneumonectomy,Thoracoplasty.
Congenital:-Pulmonaryagenesis.
Mediastinal:-Grosscardiomegaly,tumors.
Pulmonary:-Collapse,Consolidation,fibrosis,Foreignbody.
Diaphragmatichernia

1423.WhatisnotseenonchestX-rayin
pulmonaryarteryhypertension
a)Enlargementofcentralarteries
b)Peripheralprunning
c)Narrowingofcentralarteries
d)None
CorrectAnswer-C
Answer-C.Narrowingofcentralarteries
Characteristicradiologicalfeatureofpulmonaryhypertensionis
enlargementofcentralarterieswithperipheralprunning.
Increasedpulmonaryarterypressureandpulmonaryvascular
resistancecharacterizepulmonaryhypertension.PAHisdefinedas
systolicpressureinthepulmonaryarteryexceeding30mmHg.

1424.Whichchamberenlargementshowsa
doublerightheartborderwithawide
subcarinalangle?

a)Leftatrium
b)Leftventricle
c)Rightatrium
d)Rightventricle
CorrectAnswer-A
Answer-A-Leftatrium
Radiologicalsignsofleftatrialenlargement:
A'double'rightheartborder
Elevationoftheleftmainbronchus
Splayingofthecarina
Enlargementoftheleftatrialappendage
Theprominenceoftheportionoftheleftheartborderatthelevelof
theleftmainbronchus

1425.Corkscrewesophagusisseeninwhich
ofthefollowingconditions?
a)Carcinomaesophagus
b)Scleroderma
c)Achalasiacardia
d)Diffuseesophagusspasm
CorrectAnswer-D
Ans.isdi.e.,DiffuseEsophagealspasm
Radiologicalappearancesofdiffuseesophagealspasmhavebeen
describedas:
curlingesophagus
Corkscrewesophagusor
-pseudodiverticulosis
Diffuseesophagealspasmisamotordisorderofesophagus
characterizedbyrepetitivesimultaneousnon-peristalticcontractions.
Symptomsaresubsternalchestpainand/ordysphagia
Diagnosedbymanometry
Alsoknow
Nutcrackeresophagus
alsoknownas'supersqueezer'esophagus
-itischaracterizedbyextremelyhigh-amplitudeperistaltic
contraction
-Symptomsarepainanddysphagia
Diagnosedbymanometrystudywhichshowsperistalticesophageal
contractionswithpeakamplitudesgreaterthantwostandard
deviationsabovethenormalvalues.

1426.Rattailappearanceincontrast
radiographyisseenin?
a)Achalasiacardia
b)Carcinomaesophagus
c)Corkscrewesophagus
d)Diffuseesophagealspasms
CorrectAnswer-B
CarcinomaesophagusREF:sabiston's18'editionchapter41See
tableinthepreviousquestion
"Rat-tail"appearanceonbariumstudyisseenincarcinoma
esophagus.

1427.ColonisidentifiedonX-ray
a)Haustra
b)Valvulaeconniventes
c)Stringofbeadssign
d)Morenumberofloops
CorrectAnswer-A
Answer-A.Haustra
Haustra(incompletebandacrossthebowelgasshadow)areseenin
colon.

1428.Doubletracksignisseenin
a)Duodenalatresia
b)CHPS
c)Gastriculcer
d)Achalasia
CorrectAnswer-B
Answer-B.CHPS
Double/tripletracksignisseenincongenitalhypertrophicpyloric
stenosis.

1429.Investigationofchoiceforstudying
RenalCorticalmass
a)99TcDTPA
b)53CrStudy
c)99TcDMSA
d)99TcPyrophosphate
CorrectAnswer-C
Ci.e.99-Tc-DMSA
DTPA(Renogram)
DMSA(IsotopeScanning)
-DTPAisfreelyfilteredat -Tc.99DMSAisusedforrenal
glomeruluswithno morphological
tubularreabsorptionor (anatomic)imagine
excretion(i.e.GFR=
-Thiscompoundgetsfixedinrenal
Excretoryfunction)
tubules&images
-DTPAisusefulforevaluating maybeobtainedafter1-2hoursof
perfusionand injection.Lesions
excretoryfunctionofeach
suchastumors&benignlesionsas
kidneyQ
cystsshowfilling
-Indications: defectQ
1.Measurementofrelative
-Usedtoassesscorticalfunctionof
renal
KidneyQanddetect
functionQineachkidney. renalscarringQ.
2.UrinarytractobstructionQ
3.Diagnosisof
Renovascularcauseof
hypertensionQ

4.InvestigationofRenal
transplantQ

1430.RimsigninIVPisseenin
a)PolycysticKidney
b)Hydronephrosis
c)Chronicpyelonephritis
d)Hypernephroma
CorrectAnswer-B
Bi.e.Hydronephrosis
RimSigninNephrogram-
1.Severehydronephrosisofthekidneys
2.Acutecompletearterialobstruction

1431.Investigationofchoiceformultiple
sclerosis
a)CT
b)MRI
c)USG
d)PET
CorrectAnswer-B
Answer-B.MRI
MRIistheinvestigationofchoicefordemyelinatingdisorder,e.g.
multiplesclerosis.

1432.Investigationofchoicefor
intramedullarySOLis-
a)MRI
b)USG
c)CT
d)X-ray
CorrectAnswer-A
Answer-A.MRI
Investigationofchoiceforintramedullaryspaceoccupylesionis
MRI.

1433.Investigationofchoiceforscreeningof
proximalinternalcarotidarterystenosis
is:

a)DopplerflowUSG
b)CTsubstractionangiography
c)MRI
d)Angiography(DSA)
CorrectAnswer-A
AnswerisA(DopplerflowUSG):
'StenosisattheoriginoftheinternalcarotidArterycanbeidentified
andquantifiedreliablybyultrasonographythatcombinesBmode
ultrasoundimagewithaDopplerultrasoundassessmentofflow
velocity.'


1434."Sunrayappearance"onX-raysis
suggestiveof:
a)AChondrosarcoma
b)Ametastatictumourinthebone
c)AnOsteogenicsarcoma
d)AnEwing'ssarcoma
CorrectAnswer-C
Ci.e.OsteogenicSarcoma
Sunrayappearanceisclassicallyseeninosteosarcoma.Itmayalso
occurinmetastases,Ewingssarcoma,Haemangoma,Meningioma
andtuberculosis.

1435.Bonewithinboneappearanceisseen
in?
a)CML
b)Osteoporosis
c)Osteopetrosis
d)Boneinfarct
CorrectAnswer-C
Ci.e.Osteopetrosis
Inosteopetrosis,thereisreducedosteoclasticboneresorption
resultingindiffusesymmetricalskeletonsclerosis.Alsok/amarble
bonediseased/titsstonelikequalityofbones;howeverthebones
areabnormallybrittle&fracturelikeapieceofchalk.Itcanpresent
radiologicallyas?
SclerosisofallbonesmoreprominentatbaseofskullQ.
Sclerosisofvertebralendplate1/tcharacteristicsandwitchorbroad
stripped(rugbyjerseyspine)QBoneinboneappearanceQ
d/t
scleroticfociwithinthebone.

1436.WhitelineofFrenkelisseenin:
September2008
a)Osteoporosis
b)Osteomalacia
c)Scurvy
d)Beri-Beri
CorrectAnswer-C
Ans.C:Scurvy
Clinicalsymptomsandsignsofinfantilescurvy,inorderoffrequency
areirritability,tendernessandweaknessoflowerextremities,a
scorbuticrosaryofthelegs,bleedingofthegums(usuallywhere
teethhaveerupted),andfever.
Radiographicfindingsofscurvy:
Densemetaphysealline-Thisisduetoanintensificationofthezone
ofpreparatorycalcification,resultingfrommatrixformationfailing,is
hasbeenreferredtoasthewhitelineofFrenkel,butisnon-specific,
asitisalsoseeninhealingrickets,andleadorphosphorus
poisoning.
Groundglassosteoporosis:Thisappearsattheendoftheshaftwith
blurringordisappearanceoftrabecularmarkings.
Haloossificationcentre:AlsocalledtheWimberger'sring,itisthe
sameeffectthatproducesthewhitelineofFrenkel,affectingthe
epiphysealossificationcenter.
Cornersign-Seenduetosubepiphysealinfarction,orseparationof
theepiphysisfromthemetaphysis.
Lateralspurs-Thesemetaphysealspursprojectatrightanglestothe
axisoftheshaft,theymaybeseenduetomushroomingofepiphysis
onthemetaphysis,ormayrepresentearliestcalcificationof

periosteumelevatedbyasubperiostealhemorrhage.
Subperiostealhematomas-Theseoccurattheendoflongbones,
seenafterabout2weeksofonsetofclinicalsymptoms;itisnotthe
periostealhemorrhagethatcalcifies,buttheelevatedperiosteum,
secondarytoresumptionofboneformation.
Metaphysealfractures-Subperiostealcomminutedfracturesatthe
endoflongbones,extendingonlypartiallythroughthewidthofthe
bone.
Atrophyscurvyline-AradiolucentzoneontheshaftsideofFrenkel's
whiteline,ithasbeenreferredtoastheTrummerfeldZone.

1437.VonRosen'sviewisfor
a)CDH
b)Perthe'sdisease
c)CTEV
d)None
CorrectAnswer-A
Answer-A.CDH
InVonRosen'sviewisusedinDDH/CDH.

1438.Popcorncalcificationisseenin:
a)Pulmonaryhamartoma
b)Fungalinfection
c)Metastasis
d)Tuberculosis
CorrectAnswer-A
Ans.Pulmonaryhamartoma
Popcorncalcification
oPopcorncalcificationisaclusterofsharplydefined,irregularly
lobulated,calcification,usuallyinapulmonarynodule.
oPopcorncalcificationischaracteristicofhamartomaonchestX-ray
examination.
oItmayalsobeseeninmediastinallymphnodesofacute
histoplasmosis.
Egg-shellcalcification
oEgg-shellcalcificationmeansperipheralrimcalcificationoflymph
nodes:
oItisseenin:
NSilicosis(mostcommoncause)
(vi)
Histoplasmosis(vii)Tuberculosis
(ii)Coalworker
pneumoconiosis(v)Sarcoidosis(viii)
Coccidiodomycosis
(iii)Lymphomafollowingradiotherapy(vi)Progressivemassive
fibrosis

1439.Nuchaltranslucencyisusedin
a)Headscan
b)MRIneck
c)ANCUSG
d)Anthropometry
CorrectAnswer-C
Answer-C.ANCUSG
Nuchaltranslucencyisusedforscreeningofdownsyndromein
antenatalUSG.

1440.Half-lifeofIodine131is
a)8hours
b)8days
c)8weeks
d)8months
CorrectAnswer-B
Answer-B.8days
Iodine-131Halflife8days
Iodine-123-3Halflife13hours
Iodine-132Halflife2.3hours

1441.CalcificationofIntervertebralDiscis
seenin
a)Gout
b)Rheumatoid
c)Alkaptonuria
d)Psoriasis
CorrectAnswer-C
Ci.e.Alkaptonuria
Features Disease
FishMouthVertebrae
-SickelCellAnemiaQ
HomocystinuriaQ
CodFishVertebra
-Osteomalacia,Osteoporosis,
(Biconcavevertebra)
Hyperparathyroid
Ruggerjerseyspine
-CRFinducedosteomalaciaQ
(sclerosisofupper&
-Osteopterosis(marbelbone
lowerspineborders)
disease)
Calcificationof
-Alkaptonuria(m.c.)
Intervertebraldisc
PictureFramevertebrae -
Paget'sdisease
Vertebraeplana
-EosinophilicgranulomaQ

1442.ThemaximumDALYlossisforthe
followingdisease-
a)Schizophrenia
b)Unipolardepression
c)Bipolardepression
d)Mania
CorrectAnswer-B
Ans.B.Unipolardepression
AmongstthepsychiatricdisordersmaximumDALYlossiscausedby
majordepression.

1443.Etheromaniasreferto?
a)Acutepsychosispostetheranaesthesia
b)Etheraddiction
c)Excessiveetherusedruganaesthesia
d)None
CorrectAnswer-B
Ans,B.Etheraddiction
Etheraddictionoretheromaniaistheaddictiontoinhalationor
drinkingofdiethylether.

1444.Theterm"Dementiaprecox"was
coinedby?
a)Freud
b)Bleuler
c)Kraepelin
d)Schneider
CorrectAnswer-C
Ans.C.Kraepelin

1445.Theterm'id'wascoinedby?
a)Freud
b)Skinner
c)Wayker
d)Blueler
CorrectAnswer-A
Ans,A.Freud
Structuraltheoryofmind(theid,egoandsuperego)wasgivenby
sigmundfreud.

1446.
Subcorticaldementiaisseeninall
except?
a)Parkinsonism
b)Alzheimer'sdisease
c)Wilson'sdisease
d)Huntingtonschorea
CorrectAnswer-B
Ans.B.Alzheimer'sdisease
Subcorticaldementiaisseeninparkinsonism,Huntington'sdisease,
wilson'sdisease,progressivesupranuclearpalsy,idiopathicbasal
ganglioncalcification(Fahr'sdisease),thalamiclesions,multiple
sclerosis,HIVassociateddementiaandmultisYstematroPhY.
Alzheimer'sdiseasecausescorticaldementia.

1447.Formicationisseenwith?
a)Acuteamphetamineintoxication
b)Chronicuseofamhetamine
c)Alcoholwithdrawal
d)Cannabispoisoning
CorrectAnswer-B
Ans.B.Chronicuseofamhetamine
Magnan'ssyndrome(alsocalledformication)isseeninchronic
cocaineandchronicamphetamineabuse.

1448.Hangoverfollowingalcohol
consumptioncanbetreatedwith?
a)Pyridoxine
b)Thiamine
c)Riboflavin
d)Niacin
CorrectAnswer-B
ANs.B.Thiamine
Thiaminehelpspreventthebuildupofglutarateinthebrain,which
maybeassociatedwithpartoftheheadacheassociatedwith
hangover.

1449.Mostcommonlyabusedopioid-
a)Morphine
b)Diacetylmorphine
c)Oxycodine
d)Bupremorphine
CorrectAnswer-B
Ans,B.Diacetylmorphine
Diacetylmorphine(heroin)isthemostcommonlyabusedopioid.

1450.Hallucinationsareproducedby?
a)Amphetamine
b)Morphine
c)Paraxetine
d)Chlorpromazine
CorrectAnswer-A
Ans,A.Amphetamine
Amphetaminepsychosismimicparanoidschizophrenia.Theremay
bedeleusionsandhallucinations.

1451.Anxietyis?
a)Neurosis
b)Psychosis
c)Personalitydisorder
d)None
CorrectAnswer-A
Ans.,A.Neurosis
ImportantneuroticdisordersareAnxietydisorders(Panic),Phobia
(Phobicanxietydisorder),obsessivecompulsiveDisorder&
Dissociativeconversiondisorder.

1452.Negativesymptomofschizophrenia?
a)Hallucination
b)Delusion
c)Ambivalance
d)Motorhyperactivity
CorrectAnswer-C
Ans.is'c'i.e.,Ambivalance
Symptomsofschizophreniacanbedividedinto:?
1. Positivesymptoms:-Positivesymptomsarepsychoticsymptoms
notseeninnormalindividualsbutare"activelyexpressed"inpatient,
i.e.,hallucinations,delusionsandbizarremotoracts.Positive
symptomsaremorecommoninacuteschizophrenia.Theserespond
welltotypicalantipsychotics.
2. Negativesymptoms:-Negativesymptomsarenormallyexpected
behaviours,emotions(feeling),thoughtsanddrivesthattheperson
withschizophreniafailtoexhibit,i.e.,deficitstate(diminutionorloss)
ofnormalfunction.Theprominentnegativesymptonsareflattening
orbluntingofaffect,anhedonia,ambivalence(avolition)asociality
(socialwithdrawal),alogia,apathy,paucityofthoughtandpoverityof
speech.Negativesymptomsaremorecommoninchronic
schizophrenia.Negativesymptomsdonotrespondwelltotypical
antipsychoticsThereforepatientontypicalantipsychoticsmayshow
improvementofpositivesymptomsandpersistentofnegative
symptoms.
Morerecentlyathirdcategoryhasbeenproposed.
3.Disorganizedsymptoms:-Disorganizedspeech/thinking,and
disorganizedbehavior.

1453.Typeofschizophreniawithmental
retardation:
NEET13

a)Von-Goghsyndrome
b)Paranoidschizophrenia
c)Catatonicschizophrenia
d)Pfropfschizophrenia
CorrectAnswer-D
Ans.Pfropfschizophrenia

1454.Erotomaniaisseenin:
NEET13
a)Bipolarmania
b)Unipolarmania
c)Neurosis
d)Obsessivecompulsivedisorder
CorrectAnswer-A
Ans.Bipolarmania

1455.Indepressions,thereisdeficiencyof?
a)5-HT
b)Acetylcholine
c)Dopamine
d)GABA
CorrectAnswer-A
Ans.A.5-HT
Depression=Decreaseinserotoninandnorepinephrine.
SerotoninisthemostimPortantneurotransmitterindepression.
Mania=Increasedofnorepipherine.

1456.Theaminoacidderivedneurotransmitter
usedfortreatingdepressionis?
a)Serotonine
b)Histamine
c)acetylcholine
d)none
CorrectAnswer-A
Ans.A.Serotonine
Thethreemainneurotransmittersinvolvedindepressionare
dopamine,norepinephrineandserotonin(s-HT).
WhenbrainIevelsofoneormoreneurotransmitterarelowor
unbalanced,depressioncanresult.Generally,antidepressantdrugs
workbyincreasingproductionordecreasingthebreakdownofone
ormoreoftheseneurotransmitters.

1457.Suicidaltendenciesaremostcommonly
seenin:
March2003

a)Female
b)Youngerage
c)Severedepression
d)Alloftheabove
CorrectAnswer-C
Ans.Ci.e.Severedepression

1458.Repetitiveirresesistablethoughttodo
something
a)Phobia
b)Obsession
c)Compulsion
d)Anxiety
CorrectAnswer-B
Ans.is'b'i.e.,Obsession
Obsessivecompulsivedisorder(OCD)
OCDisananxietydisorderwhichischaracterizedbyrecurrent,
unwantedthoughts(Obsessions)andrepettivebehaviors
(compulsions).
Anobsessionhasfollowingcharacteristic:-
1. Anidea,impulseorimagewhichintrudesintotheconscious
awarenessrepeatedly.
2. Itisrecognizedasone'sownideas,impulseorimagebutis
perceivedasegoalien(foreigntoone'spersonality).
3. Itisrecognizedasirrationalandabsurd(insightispresent--4
Patientknowsaboutdisorder).
4. Patienttriestoresistagainstitbutisunabletodosowhichleadsto
markeddistressoranxiety.
Acompulsionhasfollowingcharacteristics:-
1. Itisrepetitive,purposefulformofbehavioriscarriedoutbecauseof
strongfeelingofcompulsiontodoso.
2. Itusuallyfollowsanobsession.
3. tsgoalistopreventorreducetheanxietyorstresscausedby
obsessionhoweveritdonotalwayssucceedindoingso.
4. Itisirrationalorexcessiveandnotrealistic.

5. Insightispresent.
ApatientwithOCDmayhaveanobsession,acompulsionorboth
(mostlypatientshaveboth).
ApatientwithOCDrealizestheirrationalityoftheobsessionand
experiencesboththeobsessionandthecompulsionasego-dystonic
(unwantedbehavior).
Thepersonispreoccupiedwithdetailsrules,listorder,organisation
orschedulestotheextentthatthemajorpointofactivityislost.
Thepersonshowsperfectionismthatinterferswithtaskcompletion
(e.g.,unabletocompleteprojectbecausehisownovertlystrict
standardarenotmet).

1459.Repetitivetimesworkdoingfor
premonitionof?
a)Obscession
b)Compulsion
c)Anxiety
d)None
CorrectAnswer-B
Ans.B.Compulsion
Repetitiveirresistablethoughts>Obsession.
Repetitivepurposefulbehavior(workdoing)>compulsion.

1460.Derelation&depersonalisationseenin
whichtypeofdisorder?
a)Dissociativedisorder
b)Personalitydisorders
c)Mania
d)None
CorrectAnswer-A
Ans.A.Dissociativedisorder

1461.Phobiais:
a)Psychosis
b)Fearofanimal
c)Anxiety
d)Neurosis
CorrectAnswer-D
Neurosis

1462.Persistentpreoccupationwithserious
illnessandnormalbodyfunctionis
called?

a)Obsession
b)Somatization
c)Hypochondriasis
d)Conversiondisorder
CorrectAnswer-C
Ans.is'c'i.e.,Hypochondriasis
Hvpochondriasis(hypochondriacaldisorder)
Theprimaryfeatureofhypochondriasisispersistentpreoccupation
withexcessivefearofaserious(e.g.,cancer)orincurable(e.g.,
AIDS)disease,whichisbasedonperson'sowninterpretationof
physicalsymptomsorsensation,i.e.,misinterpretationofphysical
symptomsorsensations,forexampleanoccasionalchangeinheart
ratewillleadapersonwithhypochondriasistofearofheartdisease.
Therefore,Hypochondriasisisabnormalpreoccupationaboutnormal
bodyfunction,i.e.,bodyfunctioningisnormalbutpatientthinksitas
abnormal.
Thepreoccupationwiththepresenceofafearedillnesspersists
inspiteofnormalmedicalassessmentandinvestigations.Patient
goesfromonedoctortoanotherforaconsultation.
Peoplewithhypochondriasisareabletoacknowledgethattheir
fearsareunrealistic(insightispresent),butthisintellectual
realizationisnotenoughtoreducetheiranxiety.
Twoimportantfactsdifferentiatehypochondriasisfromsomatization
disorder:-

1. Patientwithhypochondriasisispreoccupiedwithillness,ontheother
handpatientwithsomatizationdisorderisoccupiedwithsymptoms.
2. Hypochondriasispatientispreoccupiedwithoneillness(usually)
whilepatientwithsomatizationdisorderhasmany(atleast8or
more)symptoms.

1463.Allaretrueregardingsomatization
dosorderexcept:
a)Maintainsickrole
b)4-Painsymptoms
c)1-Sexualsymptom
d)1-Pseudoneurologicalsymptom
CorrectAnswer-A
Ai.e.Maintainsickrole

1464.Schizoidpersonalitydisorderallare
seenexcept?
a)Aloof&detached
b)Pronetofantasy
c)Suspicious
d)Introspective
CorrectAnswer-C
Ans.C.Suspicious
Suspiciousnessisseeninparanoidpersonalitydisorder.

1465.Narcolepsyisduetoabnormalityin?
a)Hypothalamus
b)Neocortex
c)Cerebellum
d)Medullaoblongata
CorrectAnswer-A
Ans.A.Hypothalamus
NarcolepsyisuniqueinthatthosewhosufferfromittyPicalyfall
almostinstantaneouslyintoREMsleep.
Itisthoughtthatnarcolepsyiscausedbyamalfunctioningofthe
hypothalamusinbrain.

1466.MaxdurationoftimespentisinNREM
stage?
a)I
b)II
c)III
d)IV
CorrectAnswer-B
Ans.B.II

1467.NREMSleeptrueis?
a)Teethgrinding
b)Narcolepsy
c)Nightmares
d)Sleepparalysis
CorrectAnswer-A
Ans.A.Teethgrinding
Slowwavesleep(stage3&4ofNREMsleep)disordersrSleep
walking(somnambulism),nightterror(sleepterroror
pavornocturnus),Nocturnalenuresis,Bruxism(teethgrinning),and
sleeptalking(somniloquy).
REMsleepevents/disorder:-Nightmares,nocturnalpenile
tumescence,Narcolepsy.

1468.Bruxismis?
a)Walkingduringsleep
b)Nocturnalenuresis
c)Grindingofteethduringsleep
d)Sleepapnoea
CorrectAnswer-C
Ans.C.Grindingofteethduringsleep

1469.Narcolepsy,nottrue?
a)Cataplexy
b)Sleeparchitecturenormal
c)Lossofmuscletone
d)Hallucination
CorrectAnswer-B
Ans.B.Sleeparchitecturenormal
ThereisdisturbedREMsleep.
Cataplexy(suddenlossofmuscletone)isthemostcommon
accessorysymPtom.
Theremaybehallucinations.

1470.Howtodifferentiatebetween
psychologicalandorganicerectile
dysfunction?

a)Nocturnalpeniletumescence
b)PIPEtherapy
c)Sildenafilinducederection
d)Squeezetechnique
CorrectAnswer-A
Ans.A.Nocturnalpeniletumescence
Oneoftheimportantmethodtodistinguishpsychogenicimpotence
fromorganicimPotenceisnocturnalpeniletumsescence&early
morningerectionwhicharepreservedinpsychogenicimpotencebut
notinorganiccauseofimpotence.

1471.Desensitizationisatypeof?
a)Psychotherapy
b)Psychoanalysis
c)Behavioraltherapy
d)None
CorrectAnswer-C
Ans.C.Behavioraltherapy
Typesofbehavioraltherapyare:-Systemicdesensitization,
therapeuticgradedexPosure,exPosure&responseprevention,
flooding,aversiontherapy,andoperantconditioning.

1472.Stimulantdrugisgiventochildfor?
a)Conductdisorder
b)Speechdevelopmentaldisorder
c)Pervasivedisorder
d)ADHD
CorrectAnswer-D
Ans.D.ADHD
Stimulants(likemethylphenidate,dextramphetamine)arethedrugs
ofchoiceforADHD.

1473.Kleinlevinsyndrome?
a)Insomnia
b)Anxiety
c)Depression
d)Hypersomnia
CorrectAnswer-D
Ans.D.Hypersomnia
Kleinelevinsyadromeorsleepingbeautysyndromeisaneurological
disorderofrecurringperiodsofexcessiveamountsofsleepingand
eating.

1474.AccordingtorecentroundsbyDSM
whatcodeisgiventopsychiatric
diseasesinICD10?

a)E
b)F
c)P
d)G
CorrectAnswer-B
Ans.B.F
ICD-10isWHOclassificationforalldiseasesandhealthproblems
(andnotonlypsychiatricdisorders).
ICD-10usesalphanumericcodemadeofanalphabet?incontrast
DSM-IVusesnumericalcoding)?'F'isformentaldisorders.
Therearel0maincategoriesdenotedbydigits0to9.

1475.Thecurrentagentofchoicefortreatmentofbipolaraffective(manic-
depressive)disorderis:
a)Chlorpromazine
b)Haloperidol
c)Diazepam
d)Lithiumcarbonate
CorrectAnswer-D
Lithiumcarbonateisthecurrentagentofchoice,particularlyduring
themanicphase.Becausetheonsetofactionisslow,concurrent
useofantipsychoticagentssuchaschlorpromazineorhaloperidol
maybenecessarytocontrolmania.
Concurrentuseoftricyclicantidepressantsmaybenecessaryinthe
depressivephase.
Monitoringoflithiumlevelsisnecessarybecauseoftheserious
natureoftheadverseeffects(neurologic,renal,cardiac).
Ref:RopperA.H.,SamuelsM.A.(2009).Chapter57.Depression
andBipolarDisease.InA.H.Ropper,M.A.Samuels(Eds),Adams
andVictor'sPrinciplesofNeurology
,9e.

1476.DSMIVcriterionfordepressionis?
a)1week
b)2weeks
c)3weeks
d)4weeks
CorrectAnswer-B
Ans.B.2weeks
Forthediagnosisofminordepression2-4andformajordepression
>5DSMIVsymptomsarerequiredforatleastforatwoweek
period.


1477.Typeofconnectivetissuepresentinthe
arrowmarkedareais:
a)Looseandirregular
b)Specialized
c)Denseirregular
d)Denseregular
CorrectAnswer-C
Ans.C.Denseirregular
Themarkedareaisreticulardermis.
Itiscomposedofdenseirregularcollagenousconnectivetissue
(mostcommonlytypeIcollagen)containingtheusualarrayof
connectivetissueelements,includingcells,bloodandlymphatic
vessels.
invalidquestionid

This post was last modified on 30 July 2021